Está en la página 1de 192

G.R. No. 185568 March 21, 2012 COMMISSIONER OF INTERNAL REVENUE, Petitioner, vs. PETRON CORPORATION, Respondent.

DECISION SERENO, J.: This is a Petition for Review on Certiorari under Rule 45 of the 1997 Rules of Civil Procedure filed by the Commissioner of Internal Revenue (CIR) assailing the Decision1 dated 03 December 2008 of the Court of Tax Appeals En Banc (CTA En Banc) in CTA EB No. 311. The assailed Decision reversed and set aside the Decision2 dated 04 May 2007 of the Court of Tax Appeals Second Division (CTA Second Division) in CTA Case No. 6423, which ordered respondent Petron Corporation (Petron) to pay deficiency excise taxes for the taxable years 1995 to 1998, together with surcharges and delinquency interests imposed thereon. Respondent Petron is a corporation engaged in the production of petroleum products and is a Board of Investment (BOI) registered enterprise in accordance with the provisions of the Omnibus Investments Code of 1987 (E.O. 226) under Certificate of Registration Nos. 89-1037 and D95136.3 The Facts The CTA En Banc in CTA EB Case No. 311 adopted the findings of fact by the CTA Second Division in CTA Case No. 6423. Considering that there are no factual issues in this case, we likewise adopt the findings of fact by the CTA En Banc, as follows: As culled from the records and as agreed upon by the parties in their Joint Stipulation of Facts and Issues, these are the facts of the case. During the period covering the taxable years 1995 to 1998, petitioner (herein respondent Petron) had been an assignee of several Tax Credit Certificates (TCCs) from various BOI-registered entities for which petitioner utilized in the payment of its excise tax liabilities for the taxable years 1995 to 1998. The transfers and assignments of the said TCCs were approved by the Department of Finances One Stop Shop Inter -Agency Tax Credit and Duty Drawback Center (DOF Center), composed of representatives from the appropriate government agencies, namely, the Department of Finance (DOF), the Board of Investments (BOI), the Bureau of Customs (BOC) and the Bureau of Internal Revenue (BIR). Taking ground on a BOI letter issued on 15 May 1998 which states that hydraulic oil, penetrating oil, diesel fuels and i ndustrial gases are classified as supplies and considered the suppliers thereof as qualified transferees of tax credit, petitioner acknowledged and accepted the transfers of the TCCs from the various BOI-registered entities. Petitioners acceptance and use of the TCCs as payment of its excise tax liabilities for the taxable years 1995 to 1998, had been continuously approved by the DOF as well as the BIRs Collection Program Division through its surrender and subsequent issuance by the Ass istant Commissioner of the Collection Service of the BIR of the Tax Debit Memos (TDMs). On January 30, 2002, respondent [herein petitioner CIR] issued the assailed Assessment against petitioner for deficiency excise taxes for the taxable years 1995 to 1998, in the total amount of P 739,003,036.32, inclusive of surcharges and interests, based on the ground that the TCCs utilized by petitioner in its payment of excise taxes have been cancelled by the DOF for having been fraudulently issued and transferred, pursuant to its EXCOM Resolution No. 03-05-99. Thus, petitioner, through letters dated August 31, 1999 and September 1, 1999, was required by the DOF Center to submit copies of its sales invoices and delivery receipts showing the consummation of the sale transaction to certain TCC transferors. Instead of submitting the documents required by the respondent, on February 27, 2002, petitioner filed its protest letter to the Assessment on the grounds, among others, that: a. The BIR did not comply with the requirements of Revenue Regulations 12-99 in issuing the "assessment" letter dated January 30, 2002, hence, the assessment made against it is void; b. The assignment/transfer of the TCCs to petitioner by the TCC holders was submitted to, examined and approved by the concerned government agencies which processed the assignment in accordance with law and revenue regulations; c. There is no basis for the imposition of the 50% surcharge in the amount of P 159,460,900.00 and interest penalties in the amount of P 260,620,335.32 against it; d. Some of the items included in the assessment are already pending litigation and are subject of the case entitled Commis sioner of Internal Revenue vs. Petron Corporation, C.A. GR SP No. 55330 (CTA Case No. 5657) and hence, should no longer be inc luded in the assessment; and e. The assessment and collection of alleged excise tax deficiencies sought to be collected by the BIR against petitioner through the January 30, 2002 letter are already barred by prescription under Section 203 of the National Internal Revenue Code. On 27 March 2002, respondent, through Assistant Commissioner Edwin R. Abella served a Warrant of Distraint and/or Levy on petitioner to enforce payment of the P 739,003,036.32 tax deficiencies. Respondent allegedly served the Warrant of Distraint and/or Levy against petitioner without first acting on its letter-protest. Thus, construing the Warrant of Distraint and/or Levy as the final adverse decision of the BIR on its protest of the assessment, petitioner filed the instant petition before this Honorable Court [referring to the CTA Second Division] on April 2, 2002. On April 30, 2002, respondent filed his Answer, raising the following as his Special Affirmative Defenses: 6. In a post-audit conducted by the One-Stop Inter-Agency Tax Credit and Duty Drawback Center (Center) of the Department of Finance (DOF), pursuant to the Centers Excom Resolution No. 03 -05-99, it was found that TCCs issued to Alliance Thread Co., Inc., Allstar Spinning, Inc., Diamond Knitting Corp., Fiber Technology Corp., Filstar Textile Industrial Corp., FLB International Fiber Corp., Jantex Philippines, Inc., Jibtex Industrial Corp., Master Colour System Corp. and Spintex International, Inc. were fraudulently obtained and were fraudulently transferred to petitioner. As a result of said findings, the TCCs and the Tax Debit Memos (TDMs) issued by the Center to petitioner against said TCCs were cancelled by the DOF; 7. Prior to the cancellation of the aforesaid TCCs and TDMs, petitioner had utilized the same in the payment of its excise tax liabilities. With such cancellation, the TCCs and TDMs have no value in money or moneys worth and, therefore, the excise tax es for which they were used as payment are now deemed unpaid; 8. The cancellation by the DOF of the aforesaid TCCs and TDMs has the presumption of regularity upon which respondent may validly rely; 9. Petitioner was informed by the DOF of the post-audit conducted on the TCCs and was given the opportunity to submit documents showing that the TCCs were transferred to it in payment of petroleum products allegedly delivered by it to the TCC transferors upon which the TCC transfers were approved, with the admonition that failure to submit the required documents would result in the cancellation of the transfers. Petitioner was also informed of the cancellation of the TCCs and TDMs and the reason for their cancellation;

10. Since petitioner is deemed not to have paid its excise tax liabilities, a pre-assessment notice is not required under Section 228 of the Tax Code; 11. The letter dated January 20, 2002 (should be January 30, 2002), demanding payment of petitioners excise tax liabilities explicitly states the basis for said demand, i.e., the cancellation of the TCCs and TDMs; 12. The government is never estopped from collecting legitimate taxes due to the error committed by its agents (Visayas Cebu Terminal Inc., vs. Commissioner of Internal Revenue, 13 SCRA 257; Atlas Consolidated Mining and Development Corporation vs. Commissioner of Internal Revenue, 102 SCRA 246). The acceptance by the Bureau of Internal Revenue of the TCCs fraudulently obtained and fraudulently transferred to petitioner as payment of its excise tax liabilities turned out to be a mistake after the post-audit was conducted. Hence, said payments were void and the excise taxes may be validly collected from petitioner. 13. As found in the post-audit, petitioner and the TCC transferors committed fraud in the transfer of the TCCs when they made appear (sic) that the transfers were in consideration for the delivery of petroleum products by petitioner to the TCCs transferors, for which reason said transfers were approved by the Center, when in fact there were no such deliveries; 14. Petitioner used the TCCs fraudulently obtained and fraudulently transferred in the payment of excise taxes declared in its excise tax returns with intent to evade tax to the extent of the value represented by the TCCs, thereby rendering the returns fraudulent; 15. Since petitioner wilfully filed fraudulent returns, it is liable for the 50% surcharge and 20% annual interest imposed under Sections 248 and 249 of the Tax Code; 16. Since petitioner wilfully filed fraudulent returns with intent to evade tax, the prescriptive period to collect the tax is ten (10) years from the discovery of the fraud pursuant to Section 222 of the Tax Code; and 17. The case pending in the Court of Appeals (CA-G.R. Sp. No. 55330 [CTA Case No. 5657]), and the case at bar have distinct causes of action. The former involves the invalid transfers of the TCCs to petitioner on the theory that it is not a qualified transferee thereof, while the latter involves the fraudulent procurement of said TCCs and the fraudulent transfers thereof to petitioner. However, on November 12, 2002, respondent filed a Manifestation informing this Court that on May 29, 2002, it had reduced the amount of deficiency excise taxes to P 720,923,224.74 as a result of its verification that some of the TCCs which formed part of the original "Assessment" were already included in a case previously filed with this Court. In effect, the amount of deficiency excise taxes is recomputed as follows: Transferor Alliance Thread Co. Inc. Allstar Spinning, Inc. Diamond Knitting Corporation Fiber Technology Corp. Filstar Textile Corp. FLB International Fiber Corp. Jantex Philippines, Inc. Jibtex Industrial Corp. Master Colour system Corp. Spintex International Inc. Total Basic Tax P 12,078,823.00 37,265,310.00 36,764,587.00 25,300,911.00 40,767,783.00 25,934,695.00 12,036,192.00 15,506,302.00 33,333,536.00 14,912,408.00 P 253,900,547.00 Surcharge P 6,039,411.50 18,632,655.00 18,382,293.50 12,650,455.50 20,383,891.50 12,967,347.50 6,018,096.00 7,753,151.00 16,666,768.00 7,456,204.00 P 126,950,273.50 Interest P 16,147,293.21 49,781,486.95 49,264,758.35 34,295,655.90 54,802,550.16 34,977,257.14 15,812,547.24 20,610,319.52 44,822,167.06 19,558,368.71 P 340,072,404.24 Total P 34,265,527.21 105,679,451.95 104,411,638.85 72,247,022.40 115,954,224.66 73,879,299.64 33,866,835.24 43,869,772.52 94,822,471.06 41,926,980.71 P 720,923,224.74

During the pendency of the case, but after respondent had already submitted his Formal Offer of Evidence for this Courts con sideration, he filed an Urgent Motion to Reopen Case on August 24, 2004 on the ground that additional evidence consisting of documents presented to the Center in support of the TCC transferors claims for tax credit as well as document supporting the applications for approval of the transfer of the TCCs to petitioner, must be presented to prove the fraudulent issuance and transfer of the subject TCCs. Respondent submits that it is imperative on his part to do so considering that, without necessarily admitting that the evidence presented in the case of Pilipinas Shell Petroleum Corporation vs. Commissioner of Internal Revenue, to prove fraud is not clear and convincing, he may suffer the same fate that had befallen upon therein respondent when this Court held, among others, that there is no clear and convincing evidence tha t the Tax Credit Certificates (TCCs) transferred to Shell (for brevity) and used by it in the payment of excise taxes, were fraudulently issued to the TCC transferors and were fraudulently transferred to Shell. An Opposition to Urgent Motion to Reopen Case was filed by petitioner on September 3, 2004 contending that to sustain respondents motion would smack of procedural disorder and spawn a reversion of the proceedings. While litigation is not a game of technicalitie s, it is a truism that every case must be presented in accordance with the prescribed procedure to insure an orderly administration of justice. On October 4, 2004, this Court resolved to grant respondents Motion and allowed respondent to present additional evidence in support of his arguments, but deferred the resolution of respondents original Formal Offer of Evidence until after the respondent has terminated his presentation of evidence. Subsequent to this Courts Resolution, respondent then filed on October 20, 2004, a Request for the Issuance of Subpoena Duces Tecum to the Executive Director of the Center or his duly authorized representative, and on October 21, 2004, a Subpoena Ad Testificandum to Ms. Elizabeth R. Cruz, also of the Center. Petitioner filed a Motion for Reconsideration (Re: Resolution dated October 4, 2004) on October 27, 2004, with respondent filing his Opposition on November 4, 2004, and petitioner subsequently filing its Reply to Opposition on December 20, 2004. Petitioners motion was denied by this Court in a Resolution dated February 28, 2005 for lack of merit. On March 18, 2005, petitioner filed an Urgent Motion to Revert Case to the First Division with respondents Manifestation filed on April 6, 2005 stating that the question of which Division of this Honorable Court shall hear the instant case is an internal matter which is better left to the sound discretion of this Honorable Court without interference by a party litigant. On April 28, 2005, this Court denied the Motion of petitioner for lack of merit.

On November 7, 2005, the Court finally resolved respondents Formal Offer of Evidence filed on May 7, 2004 and Supplementa l Formal Offer of Evidence filed on August 25, 2005. On November 22, 2005, respondent filed a Motion for Partial Reconsideration of the Courts Resolution to admit Exhibits 31 and 31-A on the ground that he already submitted and offered certified true copies of said exhibits, which the Court granted in its Resolution on January 19, 2006. However, on February 10, 2006, respondent filed a Motion to Amend Formal Offer of Evidence praying that he be allowed to amend his formal offer since some exhibits although attached thereto were inadvertently not mentioned in the Formal Offer of Evidence. Petitio ners Opposition was filed on March 14, 2006. This Court granted respondents motion in the Resolution dated April 24, 2006 and considering that the partie s already filed their respective Memoranda, this case was then considered submitted for decision. On May 16, 2006, however, respondent filed an Omnibus Motion praying that this Court take judicial notice of the fact that the TCCs issued by the Center, including the TCCs in this instant case, contained the standard Liability Clause and that the case be consolida ted with CTA Case No. 6136, on the ground that both cases involve the same parties and common questions of law or fact. An Opposition/Comment on O mnibus Motion was filed by petitioner on June 26, 2006, and Reply to Opposition/Comment was filed by respondent on July 17, 2 006. In a Resolution promulgated on September 1, 2006, this Court granted respondents motion only insofar as taking judicial noti ce of the fact that each of the dorsal side of the TCCs contains the subject liability clause, but denied respondents motio n to consolidate considering that C.T.A. Case No. 6136 was already submitted for decision on April 24, 2006. 4 The Ruling of the Court of Tax AppealsSecond Division (CTA Case No. 6423) On 04 May 2007, the CTA Second Division promulgated a Decision in CTA Case No. 6423, the dispositive portion of which reads: WHEREFORE, premises considered, the instant Petition for Review is hereby DENIED for lack of merit. Accordingly, petitioner is ORDERED TO PAY the respondent the reduced amount of SIX HUNDRED MILLION SEVEN HUNDRED SIXTY NINE THOUSAND THREE HUNDRED FIFTY THREE AND 95/100 PESOS (P600,769,353.95), representing petitioners deficiency excise taxes for the taxable ye ars 1995 to 1998, recomputed as follows: Transferor Alliance Thread Co. Inc. Allstar Spinning, Inc. Diamond Knitting Corporation Fiber Technology Corp. Filstar Textile Corp. FLB International Fiber Corp. Jantex Philippines, Inc. Jibtex Industrial Corp. Master Colour system Corp. Spintex International Inc. Total Basic Tax P 12,078,823.00 37,265,310.00 36,764,587.00 25,300,911.00 40,767,783.00 25,934,695.00 12,036,192.00 15,506,302.00 33,333,536.00 14,912,408.00 P 253,900,547.00 25% Surcharge P 3,019,705.75 9,316,327.50 9,191,146.75 6,325,227.75 10,191,945.75 6,483,673.75 3,009,048.00 3,876,575.50 8,333,384.00 3,728,102.00 P 63,475,136.75 20% Interest P 13,456,077.68 41,484,572.46 41,053,965.29 28,579,713.25 45,668,791.80 29,147,714.28 13,177,122.70 17,175,266.27 37,351,805.88 16,298,640.59 P 283,393,670.20 Total P 28,554,606.43 88,066,209.96 87,009,699.04 60,205,852.00 96,628,520.55 61,566,083.03 28,222,362.70 36,558,143.77 79,018,725.88 34,939,150.59 P 600,769,353.95

In addition, petitioner is ORDERED TO PAY the respondent TWENTY FIVE PERCENT (25%) LATE PAYMENT SURCHARGE AND TWENTY PERCENT (20%) DELIQUENCY INTEREST per annum on the amount of SIX HUNDRED MILLION SEVEN HUNDRED SIXTY NINE THOUSAND THREE HUNDRED FIFTY THREE & 95/100 PESOS (P 600,769,353.95), computed from June 27, 2002 until the amount is fully paid. SO ORDERED.5 The CTA Second Division held Petron liable for deficiency excise taxes on the ground that the cancellation by the DOF of the TCCs previously issued to and utilized by respondent to settle its tax liabilities had the effect of nonpayment of the latters excise taxes. These taxes corresponded to the value of the TCCs Petron used for payment. The CTA Second Division ruled that payment can only occur if the instrument used to discharge an obligation represents its stated value.6 It further ruled that Petrons acceptance of the TCCs was considered a contract entered into by respondent with the CIR and subject to post-audit,7 which was considered a suspensive condition governed by Article 1181 of the Civil Code. 8 Further, the CTA Second Division found that the circumstances pertaining to the issuance of the subject TCCs and their transfer to Petron "brim with fraud."9 Hence, the said court concluded that since the TCCs used by Petron were found to be spurious, respondent was deemed to have not paid its excise taxes and ought to be liable to the CIR in the amount of P 600,769,353.95 plus 25% interests and 20% surcharges.10 Petron filed a Motion for Reconsideration11 of the Decision of the CTA Second Division, which denied the motion in a Resolution dated 14 August 2007.12 The court reiterated its conclusion that the TCCs utilized by Petron to pay the latters excise tax liabilities did not resul t in payment after these TCCs were found to be fraudulent in the post-audit by the DOF. The CTA Second Division also affirmed its ruling that Petron was liable for a 25% late payment surcharge and 20% surcharges under Section 248 13 of the National Internal Revenue Code (NIRC) of 1997.14 Aggrieved, Petron appealed the Decision to the CTA En Banc through a Petition for Review, which was docketed as CTA EB No. 311. In its Petition, Petron alleged that the Second Division erred in holding respondent liable to pay the amount of P 600,769,353.95 in deficiency excise taxes with penalties and interests covering the taxable years 1995-1998. Petron prayed that the said Decision be reversed and set aside, and that CIR be enjoined from collecting the contested excise tax deficiency assessment. 15 The CTA En Banc summed up into one issue the grounds relied upon by Petron in its Petition for Review, as follows: Whether or not the Second Division erred in holding petitioner liable for the amount of P 600,769,353.95 as deficiency excise taxes for the years 1995-1998, including surcharges and interest, plus 25% surcharge and 20% delinquency interest per annum from June 27, 2002 until the amount is fully paid.16

The Ruling of the Court of Tax Appeals En Banc (CTA EB Case No. 311) On 03 December 2008, the CTA En Banc promulgated a Decision, which reversed and set aside the CTA Second Division on 04 May 2007. The former absolved Petron from any deficiency excise tax liability for taxable years 1995 to 1998. Its ruling in favor of Petron was anchored on this Courts pronouncements in Pilipinas Shell Petroleum Corp. v. Commissi oner of Internal Revenue (Shell),17 which found that the factual background and legal issues therein were similar to those in the present case. In resolving the issues, the CTA En Banc adopted the main points in Shell, which it quoted at length as basis for deciding the appeal in favor of Petron. The gist of the main points of Shell cited by the said court is as follows: a) The issued TCCs are immediately valid and effective and are not subject to a post-audit as a suspensive condition18 b) A TCC is subject only to the following conditions: i) Post-audit in the event of a computational discrepancy ii) A reduction for any outstanding account with the BIR and/or BOC iii) A revalidation of the TCC if not utilized within one year from issuance or date of utilization 19 c) A transferee of a TCC should only be a BOI-registered firm under the Implementing Rules and Regulations of Executive Order (E.O.) No. 226.20 d) The liability clause in the TCCs provides only for the solidary liability of the transferee relative to its transfer in the event it is a party to the fraud.21 e) A transferee can rely on the Centers approval of the TCCs transfer and subsequent acceptance as payment of the transferees excise tax liability.22 f) A TCC cannot be cancelled by the Center, as it was already cancelled after the transferee had applied it as payment for th e latters excise tax liabilities.23 The CTA En Banc also found that Petron had no participation in or knowledge of the fraudulent issuance and transfer of the subject TCCs. In fact, the parties made a joint stipulation on this matter in CTA Case No. 6423 before the CTA Second Division. 24 In resolving the issue of whether the government is estopped from collecting taxes due to the fault of its agents, the CTA En Banc quoted Shell as follows: While we agree with respondent that the State in the performance of government function is not estopped by the neglect or omission of its agents, and nowhere is this truer than in the field of taxation, yet this principle cannot be applied to work injustice against an innocent party. 25 (Emphasis supplied.) Finally, the CTA En Banc ruled that Petron was considered an innocent transferee of the subject TCCs and may not be prejudiced by a reassessment of excise tax liabilities that respondent has already settled, when due, with the use of the TCCs. 26 Petron is thus considered to have not fraudulently filed its excise tax returns. Consequently, the assessment issued by the CIR against it had no legal basis. 27 The dispositive portion of the assailed 03 December 2008 Decision of the CTA En Banc reads: WHEREFORE, the instant petition for Review is hereby GRANTED. Accordingly, the May 4, 2007 Decision and August 14, 2007 Resolution of the CTA Second Division in CTA Case No. 6423 entitled, "Petron Corporation, petitioner vs. Commissioner of Internal Revenue, respondent", are hereby REVERSED and SET ASIDE. In addition, the demand and collection of the deficiency excise taxes of PETRON in the amount of P 600,769,353.95 excluding penalties and interest covering the taxable years 1995 to 1998 are hereby CANCELLED and SET ASIDE, and respondent-Commissioner of Internal Revenue is hereby ENJOINED from collecting the said amount from PETRON. SO ORDERED.28 The CIR moved for the reconsideration of the CTA En Banc Decision, but the motion was denied in a Resolution dated 14 August 2007.29 The Issues The CIR appealed the Decision of the CTA En Banc by filing a Petition for Review on Certiorari under Rule 45 of the Rules of Court.30 Petitioner assails the Decision by raising the following issues: The court of tax appeals committed reversible error in holding that respondent petron is not liable for its excise tax liabilities from 1995 to 1998. Arguments I The cta en banc erred in finding that respondent petron was not shown to have participated in the fraudulent acts. The finding of the cta second division that the tax credit certificates were fraudulently transferred by the transferor-companies to respondent is supported by substantial evidence. Respondent was involved in the perpetration of fraud in the tccs transfer and utilization. II Respondent cannot validly claim the right of innocent transferee for value. As assignee/transferee of the tccs, respondent merely succeeded to the rights of the tcc assignors/transferors. Accordingly, if the tccs assigned to respondent were void, it did not acquire any valid title over the tccs. III The government is not Estopped from collecting taxes due to the mistakes of its agents. IV Respondent is liable for 25% surcharge and 20% interest per annum pursuant to the provisions of sections 248 and 249 of the NIRC. Moreover, since respondents returns were false, the assessment prescribes in ten (10) years from the discovery of the falsity thereof pursuant to section 22 of the same code.31 The Courts Ruling We DENY the CIRs Petition for lack of merit. Article 21 of E.O. 226 defines a tax credit as follows: ARTICLE 21. "Tax credit" shall mean any of the credits against taxes and/or duties equal to those actually paid or would have been paid to evidence which a tax credit certificate shall be issued by the Secretary of Finance or his representative, or the Board, if so delegated by the Secretary of Finance. The tax credit certificates including those issued by the Board pursuant to laws repealed by this Code but without in any way diminishing the scope of negotiability under their laws of issue are transferable under such conditions as may be determined by the Board after consultation with the Department of Finance. The tax credit certificate shall be used to pay taxes, duties, charges and fees due to the National Government; Provided, That the tax credits issued under this Code shall not form part of the gross income of the grantee/transferee for income tax purposes under Section 29 of the National Internal Revenue Code and are therefore not taxable: Provided, further, That such tax credits shall be valid only for a period of ten (10) years from date of issuance.

Under Article 39 (j) of the Omnibus Investment Code of 1987,32 tax credits are granted to entities registered with the Bureau of Investment (BOI) and are given for taxes and duties paid on raw materials used for the manufacture of their export products. A TCC is defined under Section 1 of Revenue Regulation (RR) No. 5-2000, issued by the BIR on 15 August 2000, as follows: B. Tax Credit Certificate means a certification, duly issued to the taxpayer named therein, by the Commissioner or his duly authorized representative, reduced in a BIR Accountable Form in accordance with the prescribed formalities, acknowledging that the grantee-taxpayer named therein is legally entitled a tax credit, the money value of which may be used in payment or in satisfaction of any of his internal revenue tax liability (except those excluded), or may be converted as a cash refund, or may otherwise be disposed of in the manner and in accordance with the limitations, if any, as may be prescribed by the provisions of these Regulations. RR 5-2000 prescribes the regulations governing the manner of issuance of TCCs and the conditions for their use, revalidation and transfer. Under the said regulation, a TCC may be used by the grantee or its assignee in the payment of its direct internal revenue tax liability.33 It may be transferred in favor of an assignee subject to the following conditions: 1) the TCC transfer must be with prior approval of the Commissioner or the duly authorized representative; 2) the transfer of a TCC should be limited to one transfer only; and 3) the transferee shall strictly use the TCC for the payment of the assignees direct internal revenue tax liability and shall not be convertible to cash.34 A TCC is valid only for 10 years subject to the following rules: (1) it must be utilized within five (5) years from the date of issue; and (2) it must be revalidated thereafter or be otherwise considered invalid.35 The processing of a TCC is entrusted to a specialized agency called the "One-Stop-Shop Inter-Agency Tax Credit and Duty Drawback Center" ("Center"), created on 07 February 1992 under Administrative Order (A.O.) No. 226. Its purpose is to expedite the processing and approval of tax credits and duty drawbacks.36 The Center is composed of a representative from the DOF as its chairperson; and the members thereof are representatives of the Bureau of Investment (BOI), Bureau of Customs (BOC) and Bureau of Internal Revenue (BIR), who are tasked to process the TCC and approve its application as payment of an assignees tax liability. 37 A TCC may be assigned through a Deed of Assignment, which the assignee submits to the Center for its approval. Upon approval of the deed, the Center will issue a DOF Tax Debit Memo (DOF-TDM),38 which will be utilized by the assignee to pay the latters tax liabilities for a specified period. Upon surrender of the TCC and the DOF-TDM, the corresponding Authority to Accept Payment of Excise Taxes (ATAPET) will be issued by the BIR Collection Program Division and will be submitted to the issuing office of the BIR for acceptance by the Assistant Commissioner of Collection Service. This act of the BIR signifies its acceptance of the TCC as payment of the assignees exci se taxes. Thus, it is apparent that a TCC undergoes a stringent process of verification by various specialized government agencies before it is accepted as payment of an assignees tax liability. In the case at bar, the CIR disputes the ruling of the CTA En Banc, which found Petron to have had no participation in the fraudulent procurement and transfer of the TCCs. Petitioner believes that there was substantial evidence to support its allegation of a fraudulent transfer of the TCCs to Petron.39 The CIR further contends that respondent was not a qualified transferee of the TCCs, because the latter did not supply petroleum products to the companies that were the assignors of the subject TCCs.40 The CIR bases its contentions on the DOFs post-audit findings stating that, for the periods covering 1995 to 1998, Petron did not deliver fuel and other petroleum products to the companies (the transferor companies) that had assigned the subject TCCs to respondent. Petitioner further alleges that the findings indicate that the transferor companies could not have had such a high volume of export sales declared to the Center and made the basis for the issuance of the TCCs assigned to Petron.41 Thus, the CIR impugns the CTA En Banc ruling that respondent was a transferee in good faith and for value of the subject TCCs.42 Not finding merit in the CIRs contention, we affirm the ruling of the CTA En Banc finding that Pet ron is a transferee in good faith and for value of the subject TCCs. From the records, we observe that the CIR had no allegation that there was a deviation from the process for the approval of the TCCs, which Petron used as payment to settle its excise tax liabilities for the years 1995 to 1998. The CIR quotes the CTA Second Division and urges us to affirm the latters Decision, which found Petron to have participated in the fraudulent issuance and transfer of the TCCs. However, any merit in the position of petitioner on this issue is negated by the Joint Stipulation it entered into with Petron in the proceedings before the said Division. As correctly noted by the CTA En Banc, herein parties jointly stipulated before the Second Division in CTA Case No. 6423 as follows: 13. That petitioner (Petron) did not participate in the procurement and issuance of the TCCs, which TCCs were transferred to Petron and later utilized by Petron in payment of its excise taxes.43 This stipulation of fact by the CIR amounts to an admission and, having been made by the parties in a stipulation of facts at pretrial, is treated as a judicial admission. Under Section 4, Rule 129 of the Rules of Court, a judicial admission requires no proof.44 The Court cannot lightly set it aside, especially when the opposing party relies upon it and accordingly dispenses with further proof of the fact already admitted. The exception provided in Rule 129, Section 4 is that an admission may be contradicted only by a showing that it was made through a palpable mistake, or that no such admission was made. In this case, however, exception to the rule does not exist. We agree with the pronouncement of the CTA En Banc that Petron has not been shown or proven to have participated in the alleged fraudulent acts involved in the transfer and utilization of the subject TCCs. Petron had the right to rely on the joint stipulation that absolved it from any participation in the alleged fraud pertaining to the issuance and procurement of the subject TCCs. The joint stipulation made by the parties consequently obviated the opportunity of the CIR to present evidence on this matter, as no proof is required for an admission made by a party in the course of the proceedings.45 Thus, the CIR cannot now be allowed to change its stand and renege on that admission. Moreover, a close examination of the arguments proffered by the CIR in their Petition calls for a reevaluation of the sufficiency of evidence in the case. The CIR seeks to persuade this Court to believe that there is substantial evidence to prove that Petron committed a misrepresentation, because the petroleum products were delivered not to the transferor but to other companies. 46 Thus, the TCCs assigned by the transferor companies to Petron were fraudulent. Clearly, a recalibration of the sufficiency of evidence presented by the CIR is needed for a different conclusion to be reached. The fundamental rule is that the scope of our judicial review under Rule 45 of the Rules of Court is confined only to errors of law and does not extend to questions of fact.47 It is basic that where it is the sufficiency of evidence that is being questioned, there is a question of fact. 48 Evidently, the CIR does not point out any specific provision of law that was wrongly interpreted by the CTA En Banc in the latters assailed Decision. Petitioner anchors it contention on the alleged existence of the sufficiency of evidence it had proffered to prove that Petron was involved in the perpetration of fraud in the transfer and utilization of the subject TCCs, an allegation that the CTA En Banc failed to consider. We have consistently held that it is not the function of this Court to analyze or weigh the evidence all over again, unless there is a showing that the findings of the lower court are totally devoid of support or are glaringly erroneous as to constitute palpable error or grave abuse of discretion.49 Such an exception does not obtain in the circumstances of this case.

The CIR claims that Petron was not an innocent transferee for value, because the TCCs assigned to respondent were void. Petitioner based its allegations on the post-audit report of the DOF, which declared that the subject TCCs were obtained through fraud and, thus, had no monetary value.50 The CIR adds that the TCCs were subject to a post-audit by the Center to complete the payment of the excise tax liability to which they were applied. Petitioner further contends that the Liability Clause of the TCCs makes the transferee or assignee solidarily liable with the original grantee for any fraudulent act pertinent to their procurement and transfer. The CIR assails the contrary ruling of the CTA En Banc, which confined the solidary liability only to the original grantee of the TCCs. Thus, petitioner believes that the correct interpretation of the Liability Clause in the TCCs makes Petron and the transferor companies or the original grantee solidarily liable for any fraudulent act or violation of the pertinent laws relating to the transfers of the TCCs. 51 We are not persuaded by the CIRs position on this matter. The Liability Clause of the TCCs reads: Both the TRANSFEROR and the TRANSFEREE shall be jointly and severally liable for any fraudulent act or violation of the pertinent laws, rules and regulations relating to the transfer of this TAX CREDIT CERTIFICATE. The scope of this solidary liability, as stated in the TCCs, was clarified by this Court in Shell, as follows: The above clause to our mind clearly provides only for the solidary liability relative to the transfer of the TCCs from the original grantee to a transferee. There is nothing in the above clause that provides for the liability of the transferee in the event that the validity of the TCC issued to the original grantee by the Center is impugned or where the TCC is declared to have been fraudulently procured by the said original grantee. Thus, the solidary liability, if any, applies only to the sale of the TCC to the transferee by the original grantee. Any fraud or breach of law or rule relating to the issuance of the TCC by the Center to the transferor or the original grantee is the latter's responsibility and liability. The transferee in good faith and for value may not be unjustly prejudiced by the fraud committed by the claimant or transferor in the procurement or issuance of the TCC from the Center. It is not only unjust but well-nigh violative of the constitutional right not to be deprived of one's property without due process of law. Thus, a re-assessment of tax liabilities previously paid through TCCs by a transferee in good faith and for value is utterly confiscatory, more so when surcharges and interests are likewise assessed. A transferee in good faith and for value of a TCC who has relied on the Center's representation of the genuineness and validity of the TCC transferred to it may not be legally required to pay again the tax covered by the TCC which has been belatedly declared null and void, that is, after the TCCs have been fully utilized through settlement of internal revenue tax liabilities. Conversely, when the transferee is party to the fraud as when it did not obtain the TCC for value or was a party to or has knowledge of its fraudulent issuance, said transferee is liable for the taxes and for the fraud committed as provided for by law.52 (Emphasis supplied.) We also find that the post-audit report, on which the CIR based its allegations, does not have the effect of a suspensive condition that would determine the validity of the TCCs. We held in Petron v. CIR (Petron),53 which is on all fours with the instant case, that TCCs are valid and effective from their issuance and are not subject to a post-audit as a suspensive condition for their validity. Our ruling in Petron finds guidance from our earlier ruling in Shell, which categorically states that a TCC is valid and effective upon its issuance and is not subject to a post-audit. The implication on the instant case of the said earlier ruling is that Petron has the right to rely on the validity and effectivity of the TCCs that were assigned to it. In finally determining their effectivity in the settlement of respondents excise tax liabilities, the validity of those TCCs should not depend on the results of the DOFs post-audit findings. We held thus in Petron: As correctly pointed out by Petron, however, the issue about the immediate validity of TCCs and the use thereof in payment of tax liabilities and duties are not matters of first impression for this Court. Taking into consideration the definition and nature of tax credits and TCCs, this Court's Second Division definitively ruled in the aforesaid Pilipinas Shell case that the post audit is not a suspensive condition for the validity of TCCs, thus: Art. 1181 tells us that the condition is suspensive when the acquisition of rights or demandability of the obligation must await the occurrence of the condition. However, Art. 1181 does not apply to the present case since the parties did NOT agree to a suspensive condition. Rather, specific laws, rules, and regulations govern the subject TCCs, not the general provisions of the Civil Code. Among the applicable laws that cover the TCCs are EO 226 or the Omnibus Investments Code, Letter of Instructions No. 1355, EO 765, RP-US Military Agreement, Sec. 106 (c) of the Tariff and Customs Code, Sec. 106 of the NIRC, BIR Revenue Regulations (RRs), and others. Nowhere in the aforementioned laws does the post-audit become necessary for the validity or effectivity of the TCCs. Nowhere in the aforementioned laws is it provided that a TCC is issued subject to a suspensive condition. xxx xxx xxx . . . (T)he TCCs are immediately valid and effective after their issuance. As aptly pointed out in the dissent of Justice Lovell Bautista in CTA EB No. 64, this is clear from the Guidelines and instructions found at the back of each TCC, which provide: 1. This Tax Credit Certificate (TCC) shall entitle the grantee to apply the tax credit against taxes and duties until the amount is fully utilized, in accordance with the pertinent tax and customs laws, rules and regulations. xxx xxx xxx 4. To acknowledge application of payment, the One-Stop-Shop Tax Credit Center shall issue the corresponding Tax Debit Memo (TDM) to the grantee. The authorized Revenue Officer/Customs Collector to which payment/utilization was made shall accomplish the Application of Tax Credit at the back of the certificate and affix his signature on the column provided." The foregoing guidelines cannot be clearer on the validity and effectivity of the TCC to pay or settle tax liabilities of the grantee or transferee, as they do not make the effectivity and validity of the TCC dependent on the outcome of a post-audit. In fact, if we are to sustain the appellate tax court, it would be absurd to make the effectivity of the payment of a TCC dependent on a post-audit since there is no contemplation of the situation wherein there is no post-audit. Does the payment made become effective if no post-audit is conducted? Or does the so-called suspensive condition still apply as no law, rule, or regulation specifies a period when a post-audit should or could be conducted with a prescriptive period? Clearly, a tax payment through a TCC cannot be both effective when made and dependent on a future event for its effectivity. Our system of laws and procedures abhors ambiguity. Moreover, if the TCCs are considered to be subject to post-audit as a suspensive condition, the very purpose of the TCC would be defeated as there would be no guarantee that the TCC would be honored by the government as payment for taxes. No investor would take the risk of utilizing TCCs if these were subject to a post-audit that may invalidate them, without prescribed grounds or limits as to the exercise of said post-audit. The inescapable conclusion is that the TCCs are not subject to post-audit as a suspensive condition, and are thus valid and effective from their issuance.54 In addition, Shell and Petron recognized an exception that holds the transferee/assignee liable if proven to have been a party to the fraud or to have had knowledge of the fraudulent issuance of the subject TCCs. As earlier mentioned, the parties entered into a joint stipulation of facts

stating that Petron did not participate in the procurement or issuance of those TCCs. Thus, we affirm the CTA En Bancs rulin g that respondent was an innocent transferee for value thereof. On the issue of estoppel, petitioner contends that the TCCs, which the Center had continually approved as payment for respondents excise tax liabilities, were subsequently found to be void. Thus, the CIR insists that the government is not estopped from collecting from Petron the excise tax liabilities that had accrued to the latter as a result of the voidance of these TCCs. Petitioner argues that the State should not be prejudiced by the neglect or omission of government employees entrusted with the collection of taxes. 55 We are not persuaded by the CIRs argument. We recognize the well-entrenched principle that estoppel does not apply to the government, especially on matters of taxation. 1wphi1 Taxes are the nations lifeblood through which government agencies continue to operate and with which the State discharges its functions for the welfare of its constituents.56 As an exception, however, this general rule cannot be applied if it would work injustice against an innocent party. 57 Petron, in this case, was not proven to have had any participation in or knowledge of the CIRs allegation of the fraudulent transfer and utilization of the subject TCCs. Respondents status as a transferee in good faith and for value of these TCCs has been established and e ven stipulated upon by petitioner.58 Respondent was thereby provided ample protection from the adverse findings subsequently made by the Center. 59 Given the circumstances, the CIRs invocation of the non-applicability of estoppel in this case is misplaced. On the final issue it raised, the CIR contends that a 25% surcharge and a 20% interest per annum must be imposed upon Petron for respondents excise tax liabilities as mandated under Sections 248 and 249 of the National Internal Revenue Code (NIRC). 60 Petitioner considers the tax returns filed by respondent for the years 1995 to 1998 as fraudulent on the basis of the post-audit finding that the TCCs were void. It argues that the prescriptive period within which to lawfully assess Petron for its tax liabilities has not prescribed under Section 222 (a)61 of the Tax Code. The CIR explains that respondents assessment on 30 January 2002 of respondents deficiency excise tax for the years 1995 to 1998 was well within the ten-year prescription period.62 In the light of the main ruling in this case, we affirm the CTA En Banc Decision finding Petron to be an innocent transferee for value of the subject TCCs. Consequently, the Tax Returns it filed for the years 1995 to 1998 are not considered fraudulent. Hence, the CIR had no legal basis to assess the excise taxes or any penalty surcharge or interest thereon, as respondent had already paid the appropriate excise taxes using the subject TCCs. WHEREFORE, the CIRs Petition is DENIED for lack of merit. The CTA En Banc Decision dated 03 December 2008 in CTA EB No. 311 is hereby AFFIRMED in toto. No pronouncement as to costs. G.R. No. 171251 March 5, 2012 LASCONA LAND CO., INC., Petitioner, vs. COMMISSIONER OF INTERNAL REVENUE, Respondent. DECISION PERALTA, J.: Before this Court is a Petition for Review on Certiorari under Rule 45 of the Rules of Court seeking the reversal of the Decision 1 dated October 25, 2005 and Resolution2 dated January 20, 2006 of the Court of Appeals (CA) in CA-G.R. SP No. 58061 which set aside the Decision3 dated January 4, 2000 and Resolution4 dated March 3, 2000 of the Court of Tax Appeals (CTA) in C.T.A. Case No. 5777 and declared Assessment Notice No. 0000047-93-407 dated March 27, 1998 to be final, executory and demandable. The facts, as culled from the records, are as follows: On March 27, 1998, the Commissioner of Internal Revenue (CIR) issued Assessment Notice No. 0000047-93-4075 against Lascona Land Co., Inc. (Lascona) informing the latter of its alleged deficiency income tax for the year 1993 in the amount of P753,266.56. Consequently, on April 20, 1998, Lascona filed a letter protest, but was denied by Norberto R. Odulio, Officer-in-Charge (OIC), Regional Director, Bureau of Internal Revenue, Revenue Region No. 8, Makati City, in his Letter 6 dated March 3, 1999, which reads, thus: xxxx Subject: LASCONA LAND CO., INC. 1993 Deficiency Income Tax Madam, Anent the 1993 tax case of subject taxpayer, please be informed that while we agree with the arguments advanced in your letter protest, we regret, however, that we cannot give due course to your request to cancel or set aside the assessment notice issued to your client for the reason that the case was not elevated to the Court of Tax Appeals as mandated by the provisions of the last paragraph of Section 228 of the Tax Code. By virtue thereof, the said assessment notice has become final, executory and demandable. In view of the foregoing, please advise your client to pay its 1993 deficiency income tax liability in the amount of P753,266.56. x x x x (Emphasis ours) On April 12, 1999, Lascona appealed the decision before the CTA and was docketed as C.T.A. Case No. 5777. Lascona alleged that the Regional Director erred in ruling that the failure to appeal to the CTA within thirty (30) days from the lapse of the 180-day period rendered the assessment final and executory. The CIR, however, maintained that Lascona's failure to timely file an appeal with the CTA after the lapse of the 180-day reglementary period provided under Section 228 of the National Internal Revenue Code (NIRC) resulted to the finality of the assessment. On January 4, 2000, the CTA, in its Decision,7 nullified the subject assessment. It held that in cases of inaction by the CIR on the protested assessment, Section 228 of the NIRC provided two options for the taxpayer: (1) appeal to the CTA within thirty (30) days from the lapse of the one hundred eighty (180)-day period, or (2) wait until the Commissioner decides on his protest before he elevates the case. The CIR moved for reconsideration. It argued that in declaring the subject assessment as final, executory and demandable, it did so pursuant to Section 3 (3.1.5) of Revenue Regulations No. 12-99 dated September 6, 1999 which reads, thus: If the Commissioner or his duly authorized representative fails to act on the taxpayer's protest within one hundred eighty (180) days from date of submission, by the taxpayer, of the required documents in support of his protest, the taxpayer may appeal to the Court of Tax Appeals within thirty (30) days from the lapse of the said 180-day period; otherwise, the assessment shall become final, executory and demandable. On March 3, 2000, the CTA denied the CIR's motion for reconsideration for lack of merit.8 The CTA held that Revenue Regulations No. 12-99 must conform to Section 228 of the NIRC. It pointed out that the former spoke of an assessment becoming final, executory and demandable by reason of the inaction by the Commissioner, while the latter referred to decisions becoming final, executory and demandable should the taxpayer adversely affected by the decision fail to appeal before the CTA within the prescribed period. Finally, it emphasized that in cases of discrepancy, Section 228 of the NIRC must prevail over the revenue regulations. Dissatisfied, the CIR filed an appeal before the CA.9

In the disputed Decision dated October 25, 2005, the Court of Appeals granted the CIR's petition and set aside the Decision dated January 4, 2000 of the CTA and its Resolution dated March 3, 2000. It further declared that the subject Assessment Notice No. 0000047-93-407 dated March 27, 1998 as final, executory and demandable. Lascona moved for reconsideration, but was denied for lack of merit. Thus, the instant petition, raising the following issues: I THE HONORABLE COURT HAS, IN THE REVISED RULES OF COURT OF TAX APPEALS WHICH IT RECENTLY PROMULGATED, RULED THAT AN APPEAL FROM THE INACTION OF RESPONDENT COMMISSIONER IS NOT MANDATORY. II THE COURT OF APPEALS SERIOUSLY ERRED WHEN IT HELD THAT THE ASSESSMENT HAS BECOME FINAL AND DEMANDABLE BECAUSE, ALLEGEDLY, THE WORD "DECISION" IN THE LAST PARAGRAPH OF SECTION 228 CANNOT BE STRICTLY CONSTRUED AS REFERRING ONLY TO THE DECISION PER SE OF THE COMMISSIONER, BUT SHOULD ALSO BE CONSIDERED SYNONYMOUS WITH AN ASSESSMENT WHICH HAS BEEN PROTESTED, BUT THE PROTEST ON WHICH HAS NOT BEEN ACTED UPON BY THE COMMISSIONER. 10 In a nutshell, the core issue to be resolved is: Whether the subject assessment has become final, executory and demandable due to the failure of petitioner to file an appeal before the CTA within thirty (30) days from the lapse of the One Hundred Eighty (180)-day period pursuant to Section 228 of the NIRC. Petitioner Lascona, invoking Section 3,11 Rule 4 of the Revised Rules of the Court of Tax Appeals, maintains that in case of inaction by the CIR on the protested assessment, it has the option to either: (1) appeal to the CTA within 30 days from the lapse of the 180-day period; or (2) await the final decision of the Commissioner on the disputed assessment even beyond the 180-day period in which case, the taxpayer may appeal such final decision within 30 days from the receipt of the said decision. Corollarily, petitioner posits that when the Commissioner failed to act on its protest within the 180-day period, it had the option to await for the final decision of the Commissioner on the protest, which it did. The petition is meritorious. Section 228 of the NIRC is instructional as to the remedies of a taxpayer in case of the inaction of the Commissioner on the protested assessment, to wit: SEC. 228. Protesting of Assessment. x x x xxxx Within a period to be prescribed by implementing rules and regulations, the taxpayer shall be required to respond to said notice. If the taxpayer fails to respond, the Commissioner or his duly authorized representative shall issue an assessment based on his findings. Such assessment may be protested administratively by filing a request for reconsideration or reinvestigation within thirty (30) days from receipt of the assessment in such form and manner as may be prescribed by implementing rules and regulations. Within sixty (60) days from filing of the protest, all relevant supporting documents shall have been submitted; otherwise, the assessment shall become final. If the protest is denied in whole or in part, or is not acted upon within one hundred eighty (180) days from submission of documents, the taxpayer adversely affected by the decision or inaction may appeal to the Court of Tax Appeals within (30) days from receipt of the said decision, or from the lapse of the one hundred eighty (180)-day period; otherwise the decision shall become final, executory and demandable. (Emphasis supplied). Respondent, however, insists that in case of the inaction by the Commissioner on the protested assessment within the 180-day reglementary period, petitioner should have appealed the inaction to the CTA. Respondent maintains that due to Lascona's failure to file an appeal with the CTA after the lapse of the 180-day period, the assessment became final and executory. We do not agree. In RCBC v. CIR,12 the Court has held that in case the Commissioner failed to act on the disputed assessment within the 180-day period from date of submission of documents, a taxpayer can either: (1) file a petition for review with the Court of Tax Appeals within 30 days after the expiration of the 180-day period; or (2) await the final decision of the Commissioner on the disputed assessments and appeal such final decision to the Court of Tax Appeals within 30 days after receipt of a copy of such decision. 13 This is consistent with Section 3 A (2), Rule 4 of the Revised Rules of the Court of Tax Appeals, 14 to wit: SEC. 3. Cases within the jurisdiction of the Court in Divisions. The Court in Divisions shall exercise: (a) Exclusive original or appellate jurisdiction to review by appeal the following: (1) Decisions of the Commissioner of Internal Revenue in cases involving disputed assessments, refunds of internal revenue taxes, fees or other charges, penalties in relation thereto, or other matters arising under the National Internal Revenue Code or other laws administered by the Bureau of Internal Revenue; (2) Inaction by the Commissioner of Internal Revenue in cases involving disputed assessments, refunds of internal revenue taxes, fees or other charges, penalties in relation thereto, or other matters arising under the National Internal Revenue Code or other laws administered by the Bureau of Internal Revenue, where the National Internal Revenue Code or other applicable law provides a specific period for action: Provided, that in case of disputed assessments, the inaction of the Commissioner of Internal Revenue within the one hundred eighty day-period under Section 228 of the National Internal revenue Code shall be deemed a denial for purposes of allowing the taxpayer to appeal his case to the Court and does not necessarily constitute a formal decision of the Commissioner of Internal Revenue on the tax case; Provided, further, that should the taxpayer opt to await the final decision of the Commissioner of Internal Revenue on the disputed assessments beyond the one hundred eighty day-period abovementioned, the taxpayer may appeal such final decision to the Court under Section 3(a), Rule 8 of these Rules; and Provided, still further, that in the case of claims for refund of taxes erroneously or illegally collected, the taxpayer must file a petition for review with the Court prior to the expiration of the two-year period under Section 229 of the National Internal Revenue Code; (Emphasis ours) In arguing that the assessment became final and executory by the sole reason that petitioner failed to appeal the inaction of the Commissioner within 30 days after the 180-day reglementary period, respondent, in effect, limited the remedy of Lascona, as a taxpayer, under Section 228 of the NIRC to just one, that is - to appeal the inaction of the Commissioner on its protested assessment after the lapse of the 180-day period. This is incorrect. As early as the case of CIR v. Villa,15 it was already established that the word "decisions" in paragraph 1, Section 7 of Republic Act No. 1125, quoted above, has been interpreted to mean the decisions of the Commissioner of Internal Revenue on the protest of the taxpayer against the assessments. Definitely, said word does not signify the assessment itself. We quote what this Court said aptly in a previous case:

In the first place, we believe the respondent court erred in holding that the assessment in question is the respondent Collector's decision or ruling appealable to it, and that consequently, the period of thirty days prescribed by section 11 of Republic Act No. 1125 within which petitioner should have appealed to the respondent court must be counted from its receipt of said assessment. Where a taxpayer questions an assessment and asks the Collector to reconsider or cancel the same because he (the taxpayer) believes he is not liable therefor, the assessment becomes a "disputed assessment" that the Collector must decide, and the taxpayer can appeal to the Court of Tax Appeals only upon receipt of the decision of the Collector on the disputed assessment, . . . 16 Therefore, as in Section 228, when the law provided for the remedy to appeal the inaction of the CIR, it did not intend to limit it to a single remedy of filing of an appeal after the lapse of the 180-day prescribed period. Precisely, when a taxpayer protested an assessment, he naturally expects the CIR to decide either positively or negatively. A taxpayer cannot be prejudiced if he chooses to wait for the final decision of the CIR on the protested assessment. More so, because the law and jurisprudence have always contemplated a scenario where the CIR will decide on the protested assessment. It must be emphasized, however, that in case of the inaction of the CIR on the protested assessment, while we reiterate the taxpayer has two options, either: (1) file a petition for review with the CTA within 30 days after the expiration of the 180-day period; or (2) await the final decision of the Commissioner on the disputed assessment and appeal such final decision to the CTA within 30 days after the receipt of a copy of such decision, these options are mutually exclusive and resort to one bars the application of the other. Accordingly, considering that Lascona opted to await the final decision of the Commissioner on the protested assessment, it then has the right to appeal such final decision to the Court by filing a petition for review within thirty days after receipt of a copy of such decision or ruling, even after the expiration of the 180-day period fixed by law for the Commissioner of Internal Revenue to act on the disputed assessments. 17 Thus, Lascona, when it filed an appeal on April 12, 1999 before the CTA, after its receipt of the Letter 18 dated March 3, 1999 on March 12, 1999, the appeal was timely made as it was filed within 30 days after receipt of the copy of the decision.1wphi1 Finally, the CIR should be reminded that taxpayers cannot be left in quandary by its inaction on the protested assessment. It is imperative that the taxpayers are informed of its action in order that the taxpayer should then at least be able to take recourse to the tax court at the opportune time. As correctly pointed out by the tax court: x x x to adopt the interpretation of the respondent will not only sanction inefficiency, but will likewise condone the Bureau's inaction. This is especially true in the instant case when despite the fact that respondent found petitioner's arguments to be in order, the assessment will become final, executory and demandable for petitioner's failure to appeal before us within the thirty (30) day period. 19 Taxes are the lifeblood of the government and so should be collected without unnecessary hindrance. On the other hand, such collection should be made in accordance with law as any arbitrariness will negate the very reason for government itself. It is therefore necessary to reconcile the apparently conflicting interests of the authorities and the taxpayers so that the real purpose of taxation, which is the promotion of the common good, may be achieved.20 Thus, even as we concede the inevitability and indispensability of taxation, it is a requirement in all democratic regimes that it be exercised reasonably and in accordance with the prescribed procedure. 21 WHEREFORE, the petition is GRANTED. The Decision dated October 25, 2005 and the Resolution dated January 20, 2006 of the Court of Appeals in CA-G.R. SP No. 58061 are REVERSED and SET ASIDE. Accordingly, the Decision dated January 4, 2000 of the Court of Tax Appeals in C.T.A. Case No. 5777 and its Resolution dated March 3, 2000 are REINSTATED. G.R. No. 188497 April 25, 2012 COMMISSIONER OF INTERNAL REVENUE, Petitioner, vs. PILIPINAS SHELL PETROLEUM CORPORATION, Respondent. DECISION VILLARAMA, JR., J.: Petitioner Commissioner of Internal Revenue appeals the Decision 1 dated March 25, 2009 and Resolution2 dated June 24, 2009 of the Court of Tax Appeals (CTA) En Banc in CTA EB No. 415. The CTA dismissed the petition for review filed by petitioner assailing the CTA First Divisions Decision3 dated April 25, 2008 and Resolution4 dated July 10, 2008 which ordered petitioner to refund the excise taxes paid by respondent Pilipinas Shell Petroleum Corporation on petroleum products it sold to international carriers. The facts are not disputed. Respondent is engaged in the business of processing, treating and refining petroleum for the purpose of producing marketable products and the subsequent sale thereof.5 On July 18, 2002, respondent filed with the Large Taxpayers Audit & Investigation Division II of the Bureau of Internal Revenue (BIR) a formal claim for refund or tax credit in the total amount of P28,064,925.15, representing excise taxes it allegedly paid on sales and deliveries of gas and fuel oils to various international carriers during the period October to December 2001. Subsequently, on October 21, 2002, a similar claim for refund or tax credit was filed by respondent with the BIR covering the period January to March 2002 in the amount of P41,614,827.99. Again, on July 3, 2003, respondent filed another formal claim for refund or tax credit in the amount of P30,652,890.55 covering deliveries from April to June 2002.6 Since no action was taken by the petitioner on its claims, respondent filed petitions for review before the CTA on September 19, 2003 and December 23, 2003, docketed as CTA Case Nos. 6775 and 6839, respectively. In its decision on the consolidated cases, the CTAs First Division ruled that respondent is entitled to the refund of excise taxes in the reduced amount of P95,014,283.00. The CTA First Division relied on a previous ruling rendered by the CTA En Banc in the case of "Pilipinas Shell Petroleum Corporation v. Commissioner of Internal Revenue "7 where the CTA also granted respondents claim for refund on the basis of excise tax exemption for petroleum products sold to international carriers of foreign registry for their use or consumption outside the Philippines. Petitioners motion for reconsideration was denied by the CTA First Division. Petitioner elevated the case to the CTA En Banc which upheld the ruling of the First Division. The CTA pointed out the specific exemption mentioned under Section 135 of the National Internal Revenue Code of 1997 (NIRC) of petroleum products sold to international carriers such as respondents clients. It said that this Courts ruling in Maceda v. Macaraig, Jr. 8 is inapplicable because said case only put to rest the issue of whether or not the National Power Corporation (NPC) is subject to tax considering that NPC is a tax-exempt entity mentioned in Sec. 135 (c) of the NIRC (1997), whereas the present case involves the tax exemption of the sale of petroleum under Sec. 135 (a) of the same Code. Further, the CTA said that the ruling in Philippine Acetylene Co., Inc. v. Commissioner of Internal Revenue9 likewise finds no application because the party asking for the refund in said case was the seller-producer based on the exemption granted under the law to the tax-exempt buyers, NPC and Voice of America (VOA), whereas in this case it is the article or product which is exempt from tax and not the international carrier. Petitioner filed a motion for reconsideration which the CTA likewise denied. Hence, this petition anchored on the following grounds:

I SECTION 148 OF THE NATIONAL INTERNAL REVENUE CODE EXPRESSLY SUBJECTS THE PETROLEUM PRODUCTS TO AN EXCISE TAX BEFORE THEY ARE REMOVED FROM THE PLACE OF PRODUCTION. II THE ONLY SPECIFIC PROVISION OF THE LAW WHICH GRANTS TAX CREDIT OR TAX REFUND OF THE EXCISE TAXES PAID REFERS TO THOSE CASES WHERE GOODS LOCALLY PRODUCED OR MANUFACTURED ARE ACTUALLY EXPORTED WHICH IS NOT SO IN THIS CASE. III THE PRINCIPLES LAID DOWN IN MACEDA VS. MACARAIG, JR. AND PHILIPPINE ACETYLENE CO. VS. CIR ARE APPLICABLE TO THIS CASE.10 The Solicitor General argues that the obvious intent of the law is to grant excise tax exemption to international carriers and exempt entities as buyers of petroleum products and not to the manufacturers or producers of said goods. Since the excise taxes are collected from manufacturers or producers before removal of the domestic products from the place of production, respondent paid the subject excise taxes as manufacturer or producer of the petroleum products pursuant to Sec. 148 of the NIRC. Thus, regardless of who the buyer/purchaser is, the excise tax on petroleum products attached to the said goods before their sale or delivery to international carriers, as in fact respondent averred that it paid the excise tax on its petroleum products when it "withdrew petroleum products from its place of production for eventual sale and delivery to various international carriers as well as to other customers."11 Sec. 135 (a) and (c) granting exemption from the payment of excise tax on petroleum products can only be interpreted to mean that the respondent cannot pass on to international carriers and exempt agencies the excise taxes it paid as a manufacturer or producer. As to whether respondent has the right to file a claim for refund or tax credit for the excise taxes it paid for the petroleum products sold to international carriers, the Solicitor General contends that Sec. 130 (D) is explicit on the circumstances under which a taxpayer may claim for a refund of excise taxes paid on manufactured products, which express enumeration did not include those excise taxes paid on petroleum products which were eventually sold to international carriers (expressio unius est exclusio alterius). Further, the Solicitor General asserts that contrary to the conclusion made by the CTA, the principles laid down by this Court in Maceda v. Macaraig, Jr. 12 and Philippine Acetylene Co. v. Commissioner of Internal Revenue13 are applicable to this case. Respondent must shoulder the excise taxes it previously paid on petroleum products which it later sold to international carriers because it cannot pass on the tax burden to the said international carriers which have been granted exemption under Sec. 135 (a) of the NIRC. Considering that respondent failed to prove an express grant of a right to a tax refund, such claim cannot be implied; hence, it must be denied. On the other hand, respondent maintains that since petroleum products sold to qualified international carriers are exempt from excise tax, no taxes should be imposed on the article, to which goods the tax attaches, whether in the hands of the said international carriers or the petroleum manufacturer or producer. As these excise taxes have been erroneously paid taxes, they can be recovered under Sec. 229 of the NIRC. Respondent contends that contrary to petitioners assertion, Sections 204 and 229 authorizes respondent to maintain a suit or proceeding to recover such erroneously paid taxes on the petroleum products sold to tax-exempt international carriers. As to the jurisprudence cited by the petitioner, respondent argues that they are not applicable to the case at bar. It points out that Maceda v. Macaraig, Jr. is an adjudication on the issue of tax exemption of NPC from direct and indirect taxes given the passage of various laws relating thereto. What was put in issue in said case was NPCs right to claim for refund of indirect taxes. Here, respondents claim f or refund is not anchored on the exemption of the buyer from direct and indirect taxes but on the tax exemption of the goods themselves under Sec. 135. Respondent further stressed that in Maceda v. Macaraig, Jr., this Court recognized that if NPC purchases oil from oil companies, NPC is entitled to claim reimbursement from the BIR for that part of the purchase price that represents excise taxes paid by the oil company to the BIR. Philippine Acetylene Co. v. CIR, on the other hand, involved sales tax, which is a tax on the transaction, which this Court held as due from the seller even if such tax cannot be passed on to the buyers who are tax-exempt entities. In this case, the excise tax is a tax on the goods themselves. While indeed it is the manufacturer who has the duty to pay the said tax, by specific provision of law, Sec. 135, the goods are stripped of such tax under the circumstances provided therein. Philippine Acetylene Co., Inc. v. CIR was thus not anchored on an exempting provision of law but merely on the argument that the tax burden cannot be passed on to someone. Respondent further contends that requiring it to shoulder the burden of excise taxes on petroleum products sold to international carriers would effectively defeat the principle of international comity upon which the grant of tax exemption on aviation fuel used in international flights was founded. If the excise taxes paid by respondent are not allowed to be refunded or credited based on the exemption provided in Sec. 135 (a), respondent avers that the manufacturers or oil companies would then be constrained to shift the tax burden to international carriers in the form of addition to the selling price. Respondent cites as an analogous case Commissioner of International Revenue v. Tours Specialists, Inc. 14 which involved the inclusion of hotel room charges remitted by partner foreign tour agents in respondent TSIs gross receipts for purposes of computing the 3% contractors tax. TSI opposed the deficiency assessment invoking, among others, Presidential Decree No. 31, which exempts foreign tourists from paying hotel room tax. This Court upheld the CTA in ruling that while CIR may claim that the 3% contractors tax is imposed upon a differen t incidence, i.e., the gross receipts of the tourist agency which he asserts includes the hotel room charges entrusted to it, the effect would be to impose a tax, and though different, it nonetheless imposes a tax actually on room charges. One way or the other, said the CTA, it would not have the effect of promoting tourism in the Philippines as that would increase the costs or expenses by the addition of a hotel room tax in the overall expenses of said tourists. The instant petition squarely raised the issue of whether respondent as manufacturer or producer of petroleum products is exempt from the payment of excise tax on such petroleum products it sold to international carriers. In the previous cases15 decided by this Court involving excise taxes on petroleum products sold to international carriers, what was only resolved is the question of who is the proper party to claim the refund of excise taxes paid on petroleum products if such tax was either paid by the international carriers themselves or incorporated into the selling price of the petroleum products sold to them. We have ruled in the said cases that the statutory taxpayer, the local manufacturer of the petroleum products who is directly liable for the payment of excise tax on the said goods, is the proper party to seek a tax refund. Thus, a foreign airline company who purchased locally manufactured petroleum products for use in its international flights, as well as a foreign oil company who likewise bought petroleum products from local manufacturers and later sold these to international carriers, have no legal personality to file a claim for tax refund or credit of excise taxes previously paid by the local manufacturers even if the latter passed on to the said buyers the tax burden in the form of additional amount in the price. Excise taxes, as the term is used in the NIRC, refer to taxes applicable to certain specified goods or articles manufactured or produced in the Philippines for domestic sales or consumption or for any other disposition and to things imported into the Philippines. These taxes are imposed in addition to the value-added tax (VAT).16

As to petroleum products, Sec. 148 provides that excise taxes attach to the following refined and manufactured mineral oils and motor fuels as soon as they are in existence as such: (a) Lubricating oils and greases; (b) Processed gas; (c) Waxes and petrolatum; (d) Denatured alcohol to be used for motive power; (e) Naphtha, regular gasoline and other similar products of distillation; (f) Leaded premium gasoline; (g) Aviation turbo jet fuel; (h) Kerosene; (i) Diesel fuel oil, and similar fuel oils having more or less the same generating power; (j) Liquefied petroleum gas; (k) Asphalts; and (l) Bunker fuel oil and similar fuel oils having more or less the same generating capacity. Beginning January 1, 1999, excise taxes levied on locally manufactured petroleum products and indigenous petroleum are required to be paid before their removal from the place of production.17 However, Sec. 135 provides: SEC. 135. Petroleum Products Sold to International Carriers and Exempt Entities or Agencies. Petroleum products sold to the following are exempt from excise tax: (a) International carriers of Philippine or foreign registry on their use or consumption outside the Philippines: Provided, That the petroleum products sold to these international carriers shall be stored in a bonded storage tank and may be disposed of only in accordance with the rules and regulations to be prescribed by the Secretary of Finance, upon recommendation of the Commissioner; (b) Exempt entities or agencies covered by tax treaties, conventions and other international agreements for their use or consumption: Provided, however, That the country of said foreign international carrier or exempt entities or agencies exempts from similar taxes petroleum products sold to Philippine carriers, entities or agencies; and (c) Entities which are by law exempt from direct and indirect taxes. Respondent claims it is entitled to a tax refund because those petroleum products it sold to international carriers are not subject to excise tax, hence the excise taxes it paid upon withdrawal of those products were erroneously or illegally collected and should not have been paid in the first place. Since the excise tax exemption attached to the petroleum products themselves, the manufacturer or producer is under no duty to pay the excise tax thereon. We disagree. Under Chapter II "Exemption or Conditional Tax-Free Removal of Certain Goods" of Title VI, Sections 133, 137, 138, 139 and 140 cover conditional tax-free removal of specified goods or articles, whereas Sections 134 and 135 provide for tax exemptions. While the exemption found in Sec. 134 makes reference to the nature and quality of the goods manufactured (domestic denatured alcohol) without regard to the tax status of the buyer of the said goods, Sec. 135 deals with the tax treatment of a specified article (petroleum products) in relation to its buyer or consumer. Respondents failure to make this important distinction apparently led it to mistakenly assume that the tax exemption under Sec. 135 (a) "attaches to the goods themselves" such that the excise tax should not have been paid in the first place. On July 26, 1996, petitioner Commissioner issued Revenue Regulations 8-9618 ("Excise Taxation of Petroleum Products") which provides: SEC. 4. Time and Manner of Payment of Excise Tax on Petroleum Products, Non-Metallic Minerals and Indigenous Petroleum I. Petroleum Products xxxx a) On locally manufactured petroleum products The specific tax on petroleum products locally manufactured or produced in the Philippines shall be paid by the manufacturer, producer, owner or person having possession of the same, and such tax shall be paid within fifteen (15) days from date of removal from the place of production. (Underscoring supplied.) Thus, if an airline company purchased jet fuel from an unregistered supplier who could not present proof of payment of specific tax, the company is liable to pay the specific tax on the date of purchase.19 Since the excise tax must be paid upon withdrawal from the place of production, respondent cannot anchor its claim for refund on the theory that the excise taxes due thereon should not have been collected or paid in the first place. Sec. 229 of the NIRC allows the recovery of taxes erroneously or illegally collected. An "erroneous or illegal tax" is defined as one levied without statutory authority, or upon property not subject to taxation or by some officer having no authority to levy the tax, or one which is some other similar respect is illegal.20 Respondents locally manufactured petroleum products are clearly subject to excise tax under Sec. 148. Hence, its claim for tax refund may not be predicated on Sec. 229 of the NIRC allowing a refund of erroneous or excess payment of tax. Respondents claim is premised on what it determined as a tax exemption "attaching to the goods themselves," which must be based on a statute granting tax exemption, or "the result of legislative grace." Such a claim is to be construed strictissimi juris against the taxpayer, meaning that the claim cannot be made to rest on vague inference. Where the rule of strict interpretation against the taxpayer is applicable as the claim for refund partakes of the nature of an exemption, the claimant must show that he clearly falls under the exempting statute.21 The exemption from excise tax payment on petroleum products under Sec. 135 (a) is conferred on international carriers who purchased the same for their use or consumption outside the Philippines. The only condition set by law is for these petroleum products to be stored in a bonded storage tank and may be disposed of only in accordance with the rules and regulations to be prescribed by the Secretary of Finance, upon recommendation of the Commissioner. On January 22, 2008, or five years after the sale by respondent of the subject petroleum products, then Secretary of Finance Margarito B. Teves issued Revenue Regulations No. 3-2008 "Amending Certain Provisions of Existing Revenue Regulations on the Granting of Outright Excise Tax Exemption on Removal of Excisable Articles Intended for Export or Sale/Delivery to International Carriers or to Tax-Exempt Entities/Agencies and Prescribing the Provisions for Availing Claims for Product Replenishment." Said issuance recognized the "tax relief to which the taxpayers are entitled" by availing of the following remedies: (a) a claim for excise tax exemption pursuant to Sections 204 and 229 of the NIRC; or (2) a product replenishment. SEC. 2. IMPOSITION OF EXCISE TAX ON REMOVAL OF EXCISABLE ARTICLES FOR EXPORT OR SALE/DELIVERY TO INTERNATIONAL CARRIERS AND OTHER TAX-EXEMPT ENTITIES/AGENCIES. Subject to the subsequent filing of a claim for excise tax credit/refund or product replenishment, all manufacturers of articles subject to excise tax under Title VI of the NIRC of 1997, as amended,

shall pay the excise tax that is otherwise due on every removal thereof from the place of production that is intended for exportation or sale/delivery to international carriers or to tax-exempt entities/agencies: Provided, That in case the said articles are likewise being sold in the domestic market, the applicable excise tax rate shall be the same as the excise tax rate imposed on the domestically sold articles. In the absence of a similar article that is being sold in the domestic market, the applicable excise tax shall be computed based on the value appearing in the manufacturers sworn statement converted to Philippine currency, as may be applicable. x x x x (Emphasis supplied.) In this case, however, the Solicitor General has adopted a position contrary to existing BIR regulations and rulings recognizing the right of oil companies to seek a refund of excise taxes paid on petroleum products they sold to international carriers. It is argued that there is nothing in Sec. 135 (a) which explicitly grants exemption from the payment of excise tax in favor of oil companies selling their petroleum products to international carriers and that the only claim for refund of excise taxes authorized by the NIRC is the payment of excise tax on exported goods, as explicitly provided in Sec. 130 (D), Chapter I under the same Title VI: (D) Credit for Excise Tax on Goods Actually Exported. -- When goods locally produced or manufactured are removed and actually exported without returning to the Philippines, whether so exported in their original state or as ingredients or parts of any manufactured goods or products, any excise tax paid thereon shall be credited or refunded upon submission of the proof of actual exportation and upon receipt of the corresponding foreign exchange payment: Provided, That the excise tax on mineral products, except coal and coke, imposed under Section 151 shall not be creditable or refundable even if the mineral products are actually exported. According to the Solicitor General, Sec. 135 (a) in relation to the other provisions on excise tax and from the nature of indirect taxation, may only be construed as prohibiting the manufacturers-sellers of petroleum products from passing on the tax to international carriers by incorporating previously paid excise taxes into the selling price. In other words, respondent cannot shift the tax burden to international carriers who are allowed to purchase its petroleum products without having to pay the added cost of the excise tax. We agree with the Solicitor General. In Philippine Acetylene Co., Inc. v. Commissioner of Internal Revenue22 this Court held that petitioner manufacturer who sold its oxygen and acetylene gases to NPC, a tax-exempt entity, cannot claim exemption from the payment of sales tax simply because its buyer NPC is exempt from taxation. The Court explained that the percentage tax on sales of merchandise imposed by the Tax Code is due from the manufacturer and not from the buyer. Respondent attempts to distinguish this case from Philippine Acetylene Co., Inc. on grounds that what was involved in the latter is a tax on the transaction (sales) and not excise tax which is a tax on the goods themselves, and that the exemption sought therein was anchored merely on the tax-exempt status of the buyer and not a specific provision of law exempting the goods sold from the excise tax. But as already stated, the language of Sec. 135 indicates that the tax exemption mentioned therein is conferred on specified buyers or consumers of the excisable articles or goods (petroleum products). Unlike Sec. 134 which explicitly exempted the article or goods itself (domestic denatured alcohol) without due regard to the tax status of the buyer or purchaser, Sec. 135 exempts from excise tax petroleum products which were sold to international carriers and other tax-exempt agencies and entities. Considering that the excise taxes attaches to petroleum products "as soon as they are in existence as such," 23 there can be no outright exemption from the payment of excise tax on petroleum products sold to international carriers. The sole basis then of respondents clai m for refund is the express grant of excise tax exemption in favor of international carriers under Sec. 135 (a) for their purchases of locally manufactured petroleum products. Pursuant to our ruling in Philippine Acetylene, a tax exemption being enjoyed by the buyer cannot be the basis of a claim for tax exemption by the manufacturer or seller of the goods for any tax due to it as the manufacturer or seller. The excise tax imposed on petroleum products under Sec. 148 is the direct liability of the manufacturer who cannot thus invoke the excise tax exemption granted to its buyers who are international carriers. In Maceda v. Macaraig, Jr.,24 the Court specifically mentioned excise tax as an example of an indirect tax where the tax burden can be shifted to the buyer: On the other hand, "indirect taxes are taxes primarily paid by persons who can shift the burden upon someone else". For example, the excise and ad valorem taxes that the oil companies pay to the Bureau of Internal Revenue upon removal of petroleum products from its refinery can be shifted to its buyer, like the NPC, by adding them to the "cash" and/or "selling price." An excise tax is basically an indirect tax. Indirect taxes are those that are demanded, in the first instance, from, or are paid by, one person in the expectation and intention that he can shift the burden to someone else. Stated elsewise, indirect taxes are taxes wherein the liability for the payment of the tax falls on one person but the burden thereof can be shifted or passed on to another person, such as when the tax is imposed upon goods before reaching the consumer who ultimately pays for it. When the seller passes on the tax to his buyer, he, in effect, shifts the tax burden, not the liability to pay it, to the purchaser as part of the price of goods sold or services rendered. 25 Further, in Maceda v. Macaraig, Jr., the Court ruled that because of the tax exemptions privileges being enjoyed by NPC under existing laws, the tax burden may not be shifted to it by the oil companies who shall pay for fuel oil taxes on oil they supplied to NPC. Thus: In view of all the foregoing, the Court rules and declares that the oil companies which supply bunker fuel oil to NPC have to pay the taxes imposed upon said bunker fuel oil sold to NPC. By the very nature of indirect taxation, the economic burden of such taxation is expected to be passed on through the channels of commerce to the user or consumer of the goods sold. Because, however, the NPC has been exempted from both direct and indirect taxation, the NPC must be held exempted from absorbing the economic burden of indirect taxation. This means, on the one hand, that the oil companies which wish to sell to NPC absorb all or part of the economic burden of the taxes previously paid to BIR, which they could shift to NPC if NPC did not enjoy exemption from indirect taxes. This means also, on the other hand, that the NPC may refuse to pay that part of the "normal" purchase price of bunker fuel oil which represents all or part of the taxes previously paid by the oil companies to BIR. If NPC nonetheless purchases such oil from the oil companies because to do so may be more convenient and ultimately less costly for NPC than NPC itself importing and hauling and storing the oil from overseas NPC is entitled to be reimbursed by the BIR for that part of the buying price of NPC which verifiably represents the tax already paid by the oil company-vendor to the BIR.26 (Emphasis supplied.) In the case of international air carriers, the tax exemption granted under Sec. 135 (a) is based on "a long-standing international consensus that fuel used for international air services should be tax-exempt." The provisions of the 1944 Convention of International Civil Aviation or the "Chicago Convention", which form binding international law, requires the contracting parties not to charge duty on aviation fuel already on board any aircraft that has arrived in their territory from another contracting state. Between individual countries, the exemption of airlines from national taxes and customs duties on a range of aviation-related goods, including parts, stores and fuel is a standard element of the network of bilateral "Air Service Agreements."27 Later, a Resolution issued by the International Civil Aviation Organization (ICAO) expanded the provision as to similarly exempt from taxes all kinds of fuel taken on board for consumption by an aircraft from a contracting state in the territory of another contracting State departing for the territory of any other State.28 Though initially aimed at establishing uniformity of taxation among parties to the

treaty to prevent double taxation, the tax exemption now generally applies to fuel used in international travel by both domestic and foreign carriers. On April 21, 1978, then President Ferdinand E. Marcos issued Presidential Decree (P.D.) No. 1359: PRESIDENTIAL DECREE No. 1359 AMENDING SECTION 134 OF THE NATIONAL INTERNAL REVENUE CODE OF 1977. WHEREAS, under the present law oil products sold to international carriers are subject to the specific tax; WHEREAS, some countries allow the sale of petroleum products to Philippine Carriers without payment of taxes thereon; WHEREAS, to foster goodwill and better relationship with foreign countries, there is a need to grant similar tax exemption in favor of foreign international carriers; NOW, THEREFORE, I, FERDINAND E. MARCOS, President of the Philippines, by virtue of the powers vested in me by the Constitution, do hereby order and decree the following: Section 1. Section 134 of the National Internal Revenue Code of 1977 is hereby amended to read as follows: "Sec. 134. Articles subject to specific tax. Specific internal revenue taxes apply to things manufactured or produced in the Philippines for domestic sale or consumption and to things imported, but not to anything produced or manufactured here which shall be removed for exportation and is actually exported without returning to the Philippines, whether so exported in its original state or as an ingredient or part of any manufactured article or product. "HOWEVER, PETROLEUM PRODUCTS SOLD TO AN INTERNATIONAL CARRIER FOR ITS USE OR CONSUMPTION OUTSIDE OF THE PHILIPPINES SHALL NOT BE SUBJECT TO SPECIFIC TAX, PROVIDED, THAT THE COUNTRY OF SAID CARRIER EXEMPTS FROM TAX PETROLEUM PRODUCTS SOLD TO PHILIPPINE CARRIERS. "In case of importations the internal revenue tax shall be in addition to the customs duties, if any." Section 2. This Decree shall take effect immediately. Contrary to respondents assertion that the above amendment to the former provision of the 1977 Tax Code supports its position that it was not liable for excise tax on the petroleum products sold to international carriers, we find that no such inference can be drawn from the words used in the amended provision or its introductory part. Founded on the principles of international comity and reciprocity, P.D. No. 1359 granted exemption from payment of excise tax but only to foreign international carriers who are allowed to purchase petroleum products free of specific tax provided the country of said carrier also grants tax exemption to Philippine carriers. Both the earlier amendment in the 1977 Tax Code and the present Sec. 135 of the 1997 NIRC did not exempt the oil companies from the payment of excise tax on petroleum products manufactured and sold by them to international carriers. Because an excise tax is a tax on the manufacturer and not on the purchaser, and there being no express grant under the NIRC of exemption from payment of excise tax to local manufacturers of petroleum products sold to international carriers, and absent any provision in the Code authorizing the refund or crediting of such excise taxes paid, the Court holds that Sec. 135 (a) should be construed as prohibiting the shifting of the burden of the excise tax to the international carriers who buys petroleum products from the local manufacturers. Said provision thus merely allows the international carriers to purchase petroleum products without the excise tax component as an added cost in the price fixed by the manufacturers or distributors/sellers. Consequently, the oil companies which sold such petroleum products to international carriers are not entitled to a refund of excise taxes previously paid on the goods.1wphi1 Time and again, we have held that tax refunds are in the nature of tax exemptions which result to loss of revenue for the government. Upon the person claiming an exemption from tax payments rests the burden of justifying the exemption by words too plain to be mistaken and too categorical to be misinterpreted,29 it is never presumed30 nor be allowed solely on the ground of equity.31 These exemptions, therefore, must not rest on vague, uncertain or indefinite inference, but should be granted only by a clear and unequivocal provision of law on the basis of language too plain to be mistaken. Such exemptions must be strictly construed against the taxpayer, as taxes are the lifeblood of the government. 32 WHEREFORE, the petition for review on certiorari is GRANTED. The Decision dated March 25, 2009 and Resolution dated June 24, 2009 of the Court of Tax Appeals En Banc in CTA EB No. 415 are hereby REVERSED and SET ASIDE. The claims for tax refund or credit filed by respondent Pilipinas Shell Petroleum Corporation are DENIED for lack of basis. No pronouncement as to costs. G.R. No. 190102 July 11, 2012 ACCENTURE, INC., Petitioner, vs. COMMISSIONER OF INTERNAL REVENUE, Respondent. DECISION SERENO, J.: This is a Petition filed under Rule 45 of the 1997 Rules of Civil Procedure, praying for the reversal of the Decision of the Court of Tax Appeals En Banc (CTA En Banc ) dated 22 September 2009 and its subsequent Resolution dated 23 October 2009. 1 Accenture, Inc. (Accenture) is a corporation engaged in the business of providing management consulting, business strategies development, and selling and/or licensing of software.2 It is duly registered with the Bureau of Internal Revenue (BIR) as a Value Added Tax (VAT) taxpayer or enterprise in accordance with Section 236 of the National Internal Revenue Code (Tax Code). 3 On 9 August 2002, Accenture filed its Monthly VAT Return for the period 1 July 2002 to 31 August 2002 (1st period). Its Quarterly VAT Return for the fourth quarter of 2002, which covers the 1st period, was filed on 17 September 2002; and an Amended Quarterly VAT Return, on 21 June 2004.4 The following are reflected in Accentures VAT Return for the fourth quarter of 2002:5 1wphi1 Purchases Domestic Purchases- Capital Goods Domestic Purchases- Goods other than capital Goods Domestic Purchases- Services Total Input Tax Amount P12,312,722.00 P64,789,507.90 P16,455,868.10 Input VAT P1,231,272.20 P6,478,950.79 P1,645,586.81 P9,355,809.80

Zero-rated Sales Total Sales

P316,113,513.34 P335,640,544.74

Accenture filed its Monthly VAT Return for the month of September 2002 on 24 October 2002; and that for October 2002, on 12 November 2002. These returns were amended on 9 January 2003. Accentures Quarterly VAT Return for the first quarter of 2003, which inc luded the period 1 September 2002 to 30 November 2002 (2nd period), was filed on 17 December 2002; and the Amended Quarterly VAT Return, on 18 June 2004. The latter contains the following information:6 Purchases Domestic Purchases- Capital Goods Domestic Purchases- Goods other than capital Goods Domestic Purchases-Services Total Input Tax Zero-rated Sales Total Sales P Amount P80,765,294.10 P132,820,541.70 P63,238,758.00 Input VAT P8,076,529.41 P13,282,054.17 P6,323,875.80 P27,682,459.38 P545,686,639.18 P572,880,982.68

The monthly and quarterly VAT returns of Accenture show that, notwithstanding its application of the input VAT credits earned from its zerorated transactions against its output VAT liabilities, it still had excess or unutilized input VAT credits. These VAT credits are in the amounts of P9,355,809.80 for the 1st period and P27,682,459.38 for the 2nd period, or a total of P37,038,269.18. 7 Out of the P37,038,269.18, only P35,178,844.21 pertained to the allocated input VAT on Accentures "domestic purchases of tax able goods which cannot be directly attributed to its zero-rated sale of services."8 This allocated input VAT was broken down to P8,811,301.66 for the 1st period and P26,367,542.55 for the 2nd period.9 The excess input VAT was not applied to any output VAT that Accenture was liable for in the same quarter when the amount was earnedor to any of the succeeding quarters. Instead, it was carried forward to petitioners 2nd Quarterly VAT Return for 2003. 10 Thus, on 1 July 2004, Accenture filed with the Department of Finance (DoF) an administrative claim for the refund or the issuance of a Tax Credit Certificate (TCC). The DoF did not act on the claim of Accenture. Hence, on 31 August 2004, the latter filed a Petition for Review with the First Division of the Court of Tax Appeals (Division), praying for the issuance of a TCC in its favor in the amount of P35,178,844.21. The Commissioner of Internal Revenue (CIR), in its Answer,11 argued thus: 1. The sale by Accenture of goods and services to its clients are not zero-rated transactions. 2. Claims for refund are construed strictly against the claimant, and Accenture has failed to prove that it is entitled to a refund, because its claim has not been fully substantiated or documented. In a 13 November 2008 Decision,12 the Division denied the Petition of Accenture for failing to prove that the latters sale o f services to the alleged foreign clients qualified for zero percent VAT.13 In resolving the sole issue of whether or not Accenture was entitled to a refund or an issuance of a TCC in the amount of P35,178,844.21,14 the Division ruled that Accenture had failed to present evidence to prove that the foreign clients to which the former rendered services did business outside the Philippines.15 Ruling that Accentures services would qualify for zero-rating under the 1997 National Internal Revenue Code of the Philippines (Tax Code) only if the recipient of the services was doing business outside of the Philippines,16 the Division cited Commissioner of Internal Revenue v. Burmeister and Wain Scandinavian Contractor Mindanao, Inc. (Burmeister)17 as basis. Accenture appealed the Divisions Decision through a Motion for Reconsideration (MR).18 In its MR, it argued that the reliance of the Division on Burmeister was misplaced19 for the following reasons: 1. The issue involved in Burmeister was the entitlement of the applicant to a refund, given that the recipient of its service was doing business in the Philippines; it was not an issue of failure of the applicant to present evidence to prove the fact that the recipient of its services was a foreign corporation doing business outside the Philippines. 20 2. Burmeister emphasized that, to qualify for zero-rating, the recipient of the services should be doing business outside the Philippines, and Accenture had successfully established that.21 3. Having been promulgated on 22 January 2007 or after Accenture filed its Petition with the Division, Burmeister cannot be made to apply to this case.22 Accenture also cited Commissioner of Internal Revenue v. American Express (Amex)23 in support of its position. The MR was denied by the Division in its 12 March 2009 Resolution.24 Accenture appealed to the CTA En Banc. There it argued that prior to the amendment introduced by Republic Act No. (R.A.) 9337, 25 there was no requirement that the services must be rendered to a person engaged in business conducted outside the Philippines to qualify for zero-rating. The CTA En Banc agreed that because the case pertained to the third and the fourth quarters of taxable year 2002, the applicable law was the 1997 Tax Code, and not R.A. 9337.26 Still, it ruled that even though the provision used in Burmeister was Section 102(b)(2) of the earlier 1977 Tax Code, the pronouncement therein requiring recipients of services to be engaged in business outside the Philippines to qualify for zero-rating was applicable to the case at bar, because Section 108(B)(2) of the 1997 Tax Code was a mere reenactment of Section 102(b)(2) of the 1977 Tax Code. The CTA En Banc concluded that Accenture failed to discharge the burden of proving the latters allegation that its clients w ere foreign-based.27 Resolute, Accenture filed a Petition for Review with the CTA En Banc, but the latter affirmed the Divisions Decision and Res olution.28 A subsequent MR was also denied in a Resolution dated 23 October 2009. Hence, the present Petition for Review29 under Rule 45. In a Joint Stipulation of Facts and Issues, the parties and the Division have agreed to submit the following issues for resolution:

1. Whether or not Petitioners sales of goods and services are zero-rated for VAT purposes under Section 108(B)(2)(3) of the 1997 Tax Code. 2. Whether or not petitioners claim for refund/tax credit in the amount of P35,178,884.21 represents unutilized input VAT pa id on its domestic purchases of goods and services for the period commencing from 1 July 2002 until 30 November 2002. 3. Whether or not Petitioner has carried over to the succeeding taxable quarter(s) or year(s) the alleged unutilized input VAT paid on its domestic purchases of goods and services for the period commencing from 1 July 2002 until 30 November 2002, and applied the same fully to its output VAT liability for the said period. 4. Whether or not Petitioner is entitled to the refund of the amount of P35,178,884.21, representing the unutilized input VAT on domestic purchases of goods and services for the period commencing from 1 July 2002 until 30 November 2002, from its sales of services to various foreign clients. 5. Whether or not Petitioners claim for refund/tax credit in the amount of P35,178,884.21, as alleged unutilized input VAT on domestic purchases of goods and services for the period covering 1 July 2002 until 30 November 2002 are duly substantiated by proper documents.30 For consideration in the present Petition are the following issues: 1. Should the recipient of the services be "doing business outside the Philippines" for the transaction to be zero-rated under Section 108(B)(2) of the 1997 Tax Code? 2. Has Accenture successfully proven that its clients are entities doing business outside the Philippines? Recipient of services must be doing business outside the Philippines for the transactions to qualify as zero-rated. Accenture anchors its refund claim on Section 112(A) of the 1997 Tax Code, which allows the refund of unutilized input VAT earned from zerorated or effectively zero-rated sales. The provision reads: SEC. 112. Refunds or Tax Credits of Input Tax. (A) Zero-Rated or Effectively Zero-Rated Sales. - Any VAT-registered person, whose sales are zero-rated or effectively zero-rated may, within two (2) years after the close of the taxable quarter when the sales were made, apply for the issuance of a tax credit certificate or refund of creditable input tax due or paid attributable to such sales, except transitional input tax, to the extent that such input tax has not been applied against output tax: Provided, however, That in the case of zero-rated sales under Section 106(A)(2)(a)(1), (2) and (B) and Section 108 (B)(1) and (2), the acceptable foreign currency exchange proceeds thereof had been duly accounted for in accordance with the rules and regulations of the Bangko Sentral ng Pilipinas (BSP): Provided, further, That where the taxpayer is engaged in zero-rated or effectively zero-rated sale and also in taxable or exempt sale of goods of properties or services, and the amount of creditable input tax due or paid cannot be directly and entirely attributed to any one of the transactions, it shall be allocated proportionately on the basis of the volume of sales. Section 108(B) referred to in the foregoing provision was first seen when Presidential Decree No. (P.D.) 1994 31 amended Title IV of P.D. 1158,32 which is also known as the National Internal Revenue Code of 1977. Several Decisions have referred to this as the 1986 Tax Code, even though it merely amended Title IV of the 1977 Tax Code. Two years thereafter, or on 1 January 1988, Executive Order No. (E.O.) 27333 further amended provisions of Title IV. E.O. 273 by transferring the old Title IV provisions to Title VI and filling in the former title with new provisions that imposed a VAT. The VAT system introduced in E.O. 273 was restructured through Republic Act No. (R.A.) 7716. 34 This law, which was approved on 5 May 1994, widened the tax base. Section 3 thereof reads: SECTION 3. Section 102 of the National Internal Revenue Code, as amended, is hereby further amended to read as follows: "SEC. 102. Value-added tax on sale of services and use or lease of properties. x x x xxx xxx xxx "(b) Transactions subject to zero-rate. The following services performed in the Philippines by VAT-registered persons shall be subject to 0%: "(1) Processing, manufacturing or repacking goods for other persons doing business outside the Philippines which goods are subsequently exported, where the services are paid for in acceptable foreign currency and accounted for in accordance with the rules and regulations of the Bangko Sentral ng Pilipinas (BSP). "(2) Services other than those mentioned in the preceding sub-paragraph, the consideration for which is paid for in acceptable foreign currency and accounted for in accordance with the rules and regulations of the Bangko Sentral ng Pilipinas (BSP)." Essentially, Section 102(b) of the 1977 Tax Codeas amended by P.D. 1994, E.O. 273, and R.A. 7716provides that if the consideration for the services provided by a VAT-registered person is in a foreign currency, then this transaction shall be subjected to zero percent rate. The 1997 Tax Code reproduced Section 102(b) of the 1977 Tax Code in its Section 108(B), to wit: (B) Transactions Subject to Zero Percent (0%) Rate. - The following services performed in the Philippines by VAT- registered persons shall be subject to zero percent (0%) rate. (1) Processing, manufacturing or repacking goods for other persons doing business outside the Philippines which goods are subsequently exported, where the services are paid for in acceptable foreign currency and accounted for in accordance with the rules and regulations of the Bangko Sentral ng Pilipinas (BSP); (2) Services other than those mentioned in the preceding paragraph, the consideration for which is paid for in acceptable foreign currency and accounted for in accordance with the rules and regulations of the Bangko Sentral ng Pilipinas (BSP); x x x. On 1 November 2005, Section 6 of R.A. 9337, which amended the foregoing provision, became effective. It reads: SEC. 6. Section 108 of the same Code, as amended, is hereby further amended to read as follows: "SEC. 108. Value-added Tax on Sale of Services and Use or Lease of Properties. (B) Transactions Subject to Zero Percent (0%) Rate. - The following services performed in the Philippines by VAT-registered persons shall be subject to zero percent (0%) rate: (1) Processing, manufacturing or repacking goods for other persons doing business outside the Philippines which goods are subsequently exported, where the services are paid for in acceptable foreign currency and accounted for in accordance with the rules and regulations of the Bangko Sentral ng Pilipinas (BSP); "(2) Services other than those mentioned in the preceding paragraph rendered to a person engaged in business conducted outside the Philippines or to a nonresident person not engaged in business who is outside the Philippines when the services are performed, the consideration for which is paid for in acceptable foreign currency and accounted for in accordance with the rules and regulations of the Bangko Sentral ng Pilipinas (BSP); x x x." (Emphasis supplied) The meat of Accentures argument is that nowhere does Section 108(B) of the 1997 Tax Code state that services, to be zero-rated, should be rendered to clients doing business outside the Philippines, the requirement introduced by R.A. 9337. 35 Required by Section 108(B), prior to the

amendment, is that the consideration for the services rendered be in foreign currency and in accordance with the rules of the Bangko Sentral ng Pilipinas (BSP). Since Accenture has complied with all the conditions imposed in Section 108(B), it is entitled to the refund prayed for. In support of its claim, Accenture cites Amex, in which this Court supposedly ruled that Section 108(B) reveals a clear intent on the part of the legislators not to impose the condition of being "consumed abroad" in order for the services performed in the Philippines to be zero-rated.36 The Division ruled that this Court, in Amex and Burmeister, did not declare that the requirement that the client must be doing business outside the Philippinescan be disregarded, because this requirement is expressly provided in Article 108(2) of the Tax Code.37 Accenture questions the Divisions application to this case of the pronouncements made in Burmeister. According to petitioner , the provision applied to the present case was Section 102(b) of the 1977 Tax Code, and not Section 108(B) of the 1997 Tax Code, which was the law effective when the subject transactions were entered into and a refund was applied for. In refuting Accentures theory, the CTA En Banc ruled that since Section 108(B) of the 1997 Tax Code was a mere reproduction of Section 102(b) of the 1977 Tax Code, this Courts interpretation of the latter may be used in interpreting the former, viz: In the Burmeister case, the Supreme Court harmonized both Sections 102(b)(1) and 102(b)(2) of the 1977 Tax Code, as amended, pertaining to zero-rated transactions. A parallel approach should be accorded to the renumbered provisions of Sections 108(B)(2) and 108(B)(1) of the 1997 NIRC. This means that Section 108(B)(2) must be read in conjunction with Section 108(B)(1). Section 108(B)(2) requires as follows: a) services other than processing, manufacturing or repacking rendered by VAT registered persons in the Philippines; and b) the transaction paid for in acceptable foreign currency duly accounted for in accordance with BSP rules and regulations. The same provision made reference to Section 108(B)(1) further imposing the requisite c) that the recipient of services must be performing business outside of Philippines. Otherwise, if both the provider and recipient of service are doing business in the Philippines, the sale transaction is subject to regular VAT as explained in the Burmeister case x x x. xxx xxx xxx Clearly, the Supreme Courts pronouncements in the Burmeister case requiring that the recipient of the services must be doing business outside the Philippines as mandated by law govern the instant case.38 Assuming that the foregoing is true, Accenture still argues that the tax appeals courts cannot be allowed to apply to Burmeis ter this Courts interpretation of Section 102(b) of the 1977 Tax Code, because the Petition of Accenture had already been filed before the case was even promulgated on 22 January 2007,39 to wit: x x x. While the Burmeister case forms part of the legal system and assumes the same authority as the statute itself, however, the same cannot be applied retroactively against the Petitioner because to do so will be prejudicial to the latter. 40 The CTA en banc is of the opinion that Accenture cannot invoke the non-retroactivity of the rulings of the Supreme Court, whose interpretation of the law is part of that law as of the date of its enactment.41 We rule that the recipient of the service must be doing business outside the Philippines for the transaction to qualify for zero-rating under Section 108(B) of the Tax Code. This Court upholds the position of the CTA en banc that, because Section 108(B) of the 1997 Tax Code is a verbatim copy of Section 102(b) of the 1977 Tax Code, any interpretation of the latter holds true for the former. Moreover, even though Accentures Petition was filed before Burmeister was promulgated, the pronouncements made in that case may be applied to the present one without violating the rule against retroactive application. When this Court decides a case, it does not pass a new law, but merely interprets a preexisting one.42 When this Court interpreted Section 102(b) of the 1977 Tax Code in Burmeister, this interpretation became part of the law from the moment it became effective. It is elementary that the interpretation of a law by this Court constitutes part of that law from the date it was originally passed, since this Court's construction merely establishes the contemporaneous legislative intent that the interpreted law carried into effect.43 Accenture questions the CTAs application of Burmeister, because the provision interpreted therein was Section 102(b) of the 1977 Tax Code. In support of its position that Section 108 of the 1997 Tax Code does not require that the services be rendered to an entity doing business outside the Philippines, Accenture invokes this Courts pronouncements in Amex. However, a reading of that case wil l readily reveal that the provision applied was Section 102(b) of the 1977 Tax Code, and not Section 108 of the 1997 Tax Code. As previously mentioned, an interpretation of Section 102(b) of the 1977 Tax Code is an interpretation of Section 108 of the 1997 Tax Code, the latter being a mere reproduction of the former. This Court further finds that Accentures reliance on Amex is misplaced. We ruled in Amex that Section 102 of the 1977 Tax Code does not require that the services be consumed abroad to be zero-rated. However, nowhere in that case did this Court discuss the necessary qualification of the recipient of the service, as this matter was never put in question. In fact, the recipient of the service in Amex is a nonresident foreign client. The aforementioned case explains how the credit card system works. The issuance of a credit card allows the holder thereof to obtain, on credit, goods and services from certain establishments. As proof that this credit is extended by the establishment, a credit card draft is issued. Thereafter, the company issuing the credit card will pay for the purchases of the credit card holders by redeeming the drafts. The obligation to collect from the card holders and to bear the lossin case they do not payrests on the issuer of the credit card. The service provided by respondent in Amex consisted of gathering the bills and credit card drafts from establishments located in the Philippines and forwarding them to its parent company's regional operating centers outside the country. It facilitated in the Philippines the collection and payment of receivables belonging to its Hong Kong-based foreign client. The Court explained how the services rendered in Amex were considered to have been performed and consumed in the Philippines, to wit: Consumption is "the use of a thing in a way that thereby exhausts it." Applied to services, the term means the performance or "successful completion of a contractual duty, usually resulting in the performers release from any past or future liability x x x." The services rendered by respondent are performed or successfully completed upon its sending to its foreign client the drafts and bills it has gathered from service establishments here. Its services, having been performed in the Philippines, are therefore also consumed in the Philippines.44 The effect of the place of consumption on the zero-rating of the transaction was not the issue in Burmeister.1wphi1 Instead, this Court addressed the squarely raised issue of whether the recipient of services should be doing business outside the Philippines for the transaction to qualify for zero-rating. We ruled that it should. Thus, another essential condition for qualification for zero-rating under Section 102(b)(2) of the 1977 Tax Code is that the recipient of the business be doing that business outside the Philippines. In clarifying that there is no conflict between this pronouncement and that laid down in Amex, we ruled thus: x x x. As the Court held in Commissioner of Internal Revenue v. American Express International, Inc. (Philippine Branch), the place of payment is immaterial, much less is the place where the output of the service is ultimately used. An essential condition for entitlement to 0% VAT under Section 102 (b) (1) and (2) is that the recipient of the services is a person doing business outside the Philippines. In this case, the recipient of the services is the Consortium, which is doing business not outside, but within the Philippines because it has a 15-year contract to operate and maintain NAPOCORs two 100-megawatt power barges in Mindanao. (Emphasis in the original)45

In Amex we ruled that the place of performance and/or consumption of the service is immaterial. In Burmeister, the Court found that, although the place of the consumption of the service does not affect the entitlement of a transaction to zero-rating, the place where the recipient conducts its business does. Amex does not conflict with Burmeister. In fact, to fully understand how Section 102(b)(2) of the 1977 Tax Code and consequently Section 108(B)(2) of the 1997 Tax Codewas intended to operate, the two aforementioned cases should be taken together. The zero-rating of the services performed by respondent in Amex was affirmed by the Court, because although the services rendered were both performed and consumed in the Philippines, the recipient of the service was still an entity doing business outside the Philippines as required in Burmeister. That the recipient of the service should be doing business outside the Philippines to qualify for zero-rating is the only logical interpretation of Section 102(b)(2) of the 1977 Tax Code, as we explained in Burmeister: This can only be the logical interpretation of Section 102 (b) (2). If the provider and recipient of the "other services" are both doing business in the Philippines, the payment of foreign currency is irrelevant. Otherwise, those subject to the regular VAT under Section 102 (a) can avoid paying the VAT by simply stipulating payment in foreign currency inwardly remitted by the recipient of services. To interpret Section 102 (b) (2) to apply to a payer-recipient of services doing business in the Philippines is to make the payment of the regular VAT under Section 102 (a) dependent on the generosity of the taxpayer. The provider of services can choose to pay the regular VAT or avoid it by stipulating payment in foreign currency inwardly remitted by the payer-recipient. Such interpretation removes Section 102 (a) as a tax measure in the Tax Code, an interpretation this Court cannot sanction. A tax is a mandatory exaction, not a voluntary contribution. xxx xxx xxx Further, when the provider and recipient of services are both doing business in the Philippines, their transaction falls squarely under Section 102 (a) governing domestic sale or exchange of services. Indeed, this is a purely local sale or exchange of services subject to the regular VAT, unless of course the transaction falls under the other provisions of Section 102 (b). Thus, when Section 102 (b) (2) speaks of "services other than those mentioned in the preceding subparagraph," the legislative intent is that only the services are different between subparagraphs 1 and 2. The requirements for zero-rating, including the essential condition that the recipient of services is doing business outside the Philippines, remain the same under both subparagraphs. (Emphasis in the original)46 Lastly, it is worth mentioning that prior to the promulgation of Burmeister, Congress had already clarified the intent behind Sections 102(b)(2) of the 1977 Tax Code and 108(B)(2) of the 1997 Tax Code amending the earlier provision. R.A. 9337 added the following phrase: "rendered to a person engaged in business conducted outside the Philippines or to a nonresident person not engaged in business who is outside the Philippines when the services are performed." Accenture has failed to establish that the recipients of its services do business outside the Philippines. Accenture argues that based on the documentary evidence it presented,47 it was able to establish the following circumstances: 1. The records of the Securities and Exchange Commission (SEC) show t hat Accentures clients have not established any branch office in which to do business in the Philippines. 2. For these services, Accenture bills another corporation, Accenture Participations B.V. (APB), which is likewise a foreign corporation with no "presence in the Philippines." 3. Only those not doing business in the Philippines can be required under BSP rules to pay in acceptable currency for their purchase of goods and services from the Philippines. Thus, in a domestic transaction, where the provider and recipient of services are both doing business in the Philippines, the BSP cannot require any party to make payment in foreign currency. 48 Accenture claims that these documentary pieces of evidence are supported by the Report of Emmanuel Mendoza, the Court-commissioned Independent Certified Public Accountant. He ascertained that Accentures gross billings pertaining to zero -rated sales were all supported by zerorated Official Receipts and Billing Statements. These documents show that these zero-rated sales were paid in foreign exchange currency and duly accounted for in the rules and regulations of the BSP.49 In the CTAs opinion, however, the documents presented by Accenture merely substantiate the existence of the sales, receipt o f foreign currency payments, and inward remittance of the proceeds of these sales duly accounted for in accordance with BSP rules. Petitioner presented no evidence whatsoever that these clients were doing business outside the Philippines. 50 Accenture insists, however, that it was able to establish that it had rendered services to foreign corporations doing business outside the Philippines, unlike in Burmeister, which allegedly involved a foreign corporation doing business in the Philippines. 51 We deny Accentures Petition for a tax refund. The evidence presented by Accenture may have established that its clients are foreign. 1wphi1 This fact does not automatically mean, however, that these clients were doing business outside the Philippines. After all, the Tax Code itself has provisions for a foreign corporation engaged in business within the Philippines and vice versa, to wit: SEC. 22. Definitions - When used in this Title: xxx xxx xxx (H) The term "resident foreign corporation" applies to a foreign corporation engaged in trade or business within the Philippines. (I) The term nonresident foreign corporation applies to a foreign corporation not engaged in trade or business within the Philippines. (Emphasis in the original) Consequently, to come within the purview of Section 108(B)(2), it is not enough that the recipient of the service be proven to be a foreign corporation; rather, it must be specifically proven to be a nonresident foreign corporation. There is no specific criterion as to what constitutes "doing" or "engaging in" or "transacting" business. We ruled thus in Commissioner of Internal Revenue v. British Overseas Airways Corporation:52 x x x. There is no specific criterion as to what constitutes "doing" or "engaging in" or "transacting" business. Each case must be judged in the light of its peculiar environmental circumstances. The term implies a continuity of commercial dealings and arrangements, and contemplates, to that extent, the performance of acts or works or the exercise of some of the functions normally incident to, and in progressive prosecution of commercial gain or for the purpose and object of the business organization. "In order that a foreign corporation may be regarded as doing business within a State, there must be continuity of conduct and intention to establish a continuous business, such as the appointment of a local agent, and not one of a temporary character."53 A taxpayer claiming a tax credit or refund has the burden of proof to establish the factual basis of that claim.1wphi1 Tax refunds, like tax exemptions, are construed strictly against the taxpayer.54 Accenture failed to discharge this burden. It alleged and presented evidence to prove only that its clients were foreign entities. However, as found by both the CTA Division and the CTA En Banc, no evidence was presented by Accenture to prove the fact that the foreign clients to whom petitioner rendered its services were clients doing business outside the Philippines.

As ruled by the CTA En Banc, the Official Receipts, Intercompany Payment Requests, Billing Statements, Memo Invoices-Receivable, Memo Invoices-Payable, and Bank Statements presented by Accenture merely substantiated the existence of sales, receipt of foreign currency payments, and inward remittance of the proceeds of such sales duly accounted for in accordance with BSP rules, all of these were devoid of any evidence that the clients were doing business outside of the Philippines. 55 WHEREFORE, the instant Petition is DENIED. The 22 September 2009 Decision and the 23 October 2009 Resolution of the Court of Tax Appeals En Banc in C.T.A. EB No. 477, dismissing the Petition for the refund of the excess or unutilized input VAT credits of Accenture, Inc., are AFFIRMED.

G.R. No. 189999 June 27, 2012 ANGELES UNIVERSITY FOUNDATION, Petitioner, vs. CITY OF ANGELES, JULIET G. QUINSAAT, in her capacity as Treasurer of Angeles City and ENGR. DONATO N. DIZON, in his capacity as Acting Angeles City Building Official, Respondents. DECISION VILLARAMA, JR., J.: Before us is a petition for review on certiorari under Rule 45 of the 1997 Rules of Civil Procedure, as amended, which seeks to reverse and set aside the Decision1 dated July 28, 2009 and Resolution2 dated October 12, 2009 of the Court of Appeals (CA) in CA-G.R. CV No. 90591. The CA reversed the Decision3 dated September 21, 2007 of the Regional Trial Court of Angeles City, Branch 57 in Civil Case No. 12995 declaring petitioner exempt from the payment of building permit and other fees and ordering respondents to refund the same with interest at the legal rate. The factual antecedents: Petitioner Angeles University Foundation (AUF) is an educational institution established on May 25, 1962 and was converted into a non-stock, non-profit education foundation under the provisions of Republic Act (R.A.) No. 6055 4 on December 4, 1975. Sometime in August 2005, petitioner filed with the Office of the City Building Official an application for a building permit for the construction of an 11-storey building of the Angeles University Foundation Medical Center in its main campus located at MacArthur Highway, Angeles City, Pampanga. Said office issued a Building Permit Fee Assessment in the amount of P126,839.20. An Order of Payment was also issued by the City Planning and Development Office, Zoning Administration Unit requiring petitioner to pay the sum of P238,741.64 as Locational Clearance Fee.5 In separate letters dated November 15, 2005 addressed to respondents City Treasurer Juliet G. Quinsaat and Acting City Building Official Donato N. Dizon, petitioner claimed that it is exempt from the payment of the building permit and locational clearance fees, citing legal opinions rendered by the Department of Justice (DOJ). Petitioner also reminded the respondents that they have previously issued building permits acknowledging such exemption from payment of building permit fees on the construction of petitioners 4 -storey AUF Information Technology Center building and the AUF Professional Schools building on July 27, 2000 and March 15, 2004, respectively. 6 Respondent City Treasurer referred the matter to the Bureau of Local Government Finance (BLGF) of the Department of Finance, which in turn endorsed the query to the DOJ. Then Justice Secretary Raul M. Gonzalez, in his letter-reply dated December 6, 2005, cited previous issuances of his office (Opinion No. 157, s. 1981 and Opinion No. 147, s. 1982) declaring petitioner to be exempt from the payment of building permit fees. Under the 1st Indorsement dated January 6, 2006, BLGF reiterated the aforesaid opinion of the DOJ stating further that "xxx the Department of Finance, thru this Bureau, has no authority to review the resolution or the decision of the DOJ." 7 Petitioner wrote the respondents reiterating its request to reverse the disputed assessments and invoking the DOJ legal opinions which have been affirmed by Secretary Gonzalez. Despite petitioners plea, however, respondents refused to issue the building permits for the construction of the AUF Medical Center in the main campus and renovation of a school building located at Marisol Village. Petitioner then appealed the matter to City Mayor Carmelo F. Lazatin but no written response was received by petitioner. 8 Consequently, petitioner paid under protest9 the following: Medical Center (new construction) Building Permit and Electrical Fee Locational Clearance Fee Fire Code Fee P 217,475.20 283,741.64 144,690.00 Total - P 645,906.84

School Building (renovation) Building Permit and Electrical Fee Locational Clearance Fee Fire Code Fee Petitioner likewise paid the following sums as required by the City Assessors Office: Real Property Tax Basic Fee SEF Locational Clearance Fee P 37,857.20 6,000.57 5,967.74 Total - P 49,825.51 P 86,531.10 43,274.54 1,125.00 Total P130,930.6410 [GRAND TOTAL - P 826,662.99]

By reason of the above payments, petitioner was issued the corresponding Building Permit, Wiring Permit, Electrical Permit and Sanitary Building Permit. On June 9, 2006, petitioner formally requested the respondents to refund the fees it paid under protest. Under letters dated June 15, 2006 and August 7, 2006, respondent City Treasurer denied the claim for refund.11 On August 31, 2006, petitioner filed a Complaint12 before the trial court seeking the refund of P826,662.99 plus interest at the rate of 12% per annum, and also praying for the award of attorneys fees in the amount of P300,000.00 and litigation expenses.

In its Answer,13 respondents asserted that the claim of petitioner cannot be granted because its structures are not among those mentioned in Sec. 209 of the National Building Code as exempted from the building permit fee. Respondents argued that R.A. No. 6055 should be considered repealed on the basis of Sec. 2104 of the National Building Code. Since the disputed assessments are regulatory in nature, they are not taxes from which petitioner is exempt. As to the real property taxes imposed on petitioners property located in Marisol Village, respon dents pointed out that said premises will be used as a school dormitory which cannot be considered as a use exclusively for educational activities. Petitioner countered that the subject building permit are being collected on the basis of Art. 244 of the Implementing Rules and Regulations of the Local Government Code, which impositions are really taxes considering that they are provided under the chapter on "Local Government Taxation" in reference to the "revenue raising power" of local government units (LGUs). Moreover, petitioner contended that, as held in Philippine Airlines, Inc. v. Edu,14 fees may be regarded as taxes depending on the purpose of its exaction. In any case, petitioner pointed out that the Local Government Code of 1991 provides in Sec. 193 that non-stock and non-profit educational institutions like petitioner retained the tax exemptions or incentives which have been granted to them. Under Sec. 8 of R.A. No. 6055 and applicable jurisprudence and DOJ rulings, petitioner is clearly exempt from the payment of building permit fees.15 On September 21, 2007, the trial court rendered judgment in favor of the petitioner and against the respondents. The dispositive portion of the trial courts decision16 reads: WHEREFORE, premises considered, judgment is rendered as follows: a. Plaintiff is exempt from the payment of building permit and other fees Ordering the Defendants to refund the total amount of Eight Hundred Twenty Six Thousand Six Hundred Sixty Two Pesos and 99/100 Centavos (P826,662.99) plus legal interest thereon at the rate of twelve percent (12%) per annum commencing on the date of extra-judicial demand or June 14, 2006, until the aforesaid amount is fully paid. b. Finding the Defendants liable for attorneys fees in the amount of Seventy Thousand Pesos (Php70,000.00), plus litigation expenses. c. Ordering the Defendants to pay the costs of the suit. SO ORDERED.17 Respondents appealed to the CA which reversed the trial court, holding that while petitioner is a tax-free entity, it is not exempt from the payment of regulatory fees. The CA noted that under R.A. No. 6055, petitioner was granted exemption only from income tax derived from its educational activities and real property used exclusively for educational purposes. Regardless of the repealing clause in the National Building Code, the CA held that petitioner is still not exempt because a building permit cannot be considered as the other "charges" mentioned in Sec. 8 of R.A. No. 6055 which refers to impositions in the nature of tax, import duties, assessments and other collections for revenue purposes, following the ejusdem generisrule. The CA further stated that petitioner has not shown that the fees collected were excessive and more than the cost of surveillance, inspection and regulation. And while petitioner may be exempt from the payment of real property tax, petitioner in this case merely alleged that "the subject property is to be used actually, directly and exclusively for educational purposes," declaring merely that such premises is intended to house the sports and other facilities of the university but by reason of the occupancy of informal settlers on the area, it cannot yet utilize the same for its intended use. Thus, the CA concluded that petitioner is not entitled to the refund of building permit and related fees, as well as real property tax it paid under protest. Petitioner filed a motion for reconsideration which was denied by the CA. Hence, this petition raising the following grounds: THE COURT OF APPEALS COMMITTED REVERSIBLE ERROR AND DECIDED A QUESTION OF SUBSTANCE IN A WAY NOT IN ACCORDANCE WITH LAW AND THE APPLICABLE DECISIONS OF THE HONORABLE COURT AND HAS DEPARTED FROM THE ACCEPTED AND USUAL COURSE OF JUDICIAL PROCEEDINGS NECESSITATING THE HONORABLE COURTS EXERCISE OF ITS POWER OF SUPERVISION CONSIDERING THAT: I. IN REVERSING THE TRIAL COURTS DECISION DATED 21 SEPTEMBER 2007, THE COURT OF APPEALS EFFECTIVELY WITHDREW THE PRIVILEGE OF EXEMPTION GRANTED TO NON-STOCK, NON-PROFIT EDUCATIONAL FOUNDATIONS BY VIRTUE OF RA 6055 WHICH WITHDRAWAL IS BEYOND THE AUTHORITY OF THE COURT OF APPEALS TO DO. A. INDEED, RA 6055 REMAINS VALID AND IS IN FULL FORCE AND EFFECT. HENCE, THE COURT OF APPEALS ERRED WHEN IT RULED IN THE QUESTIONED DECISION THAT NON-STOCK, NON-PROFIT EDUCATIONAL FOUNDATIONS ARE NOT EXEMPT. B. THE COURT OF APPEALS APPLICATION OF THE PRINCIPLE OF EJUSDEM GENERIS IN RULING IN THE QUESTIONED DECISION THAT THE TERM "OTHER CHARGES IMPOSED BY THE GOVERNMENT" UNDER SECTION 8 OF RA 6055 DOES NOT INCLUDE BUILDING PERMIT AND OTHER RELATED FEES AND/OR CHARGES IS BASED ON ITS ERRONEOUS AND UNWARRANTED ASSUMPTION THAT THE TAXES, IMPORT DUTIES AND ASSESSMENTS AS PART OF THE PRIVILEGE OF EXEMPTION GRANTED TO NON-STOCK, NON-PROFIT EDUCATIONAL FOUNDATIONS ARE LIMITED TO COLLECTIONS FOR REVENUE PURPOSES. C. EVEN ASSUMING THAT THE BUILDING PERMIT AND OTHER RELATED FEES AND/OR CHARGES ARE NOT INCLUDED IN THE TERM "OTHER CHARGES IMPOSED BY THE GOVERNMENT" UNDER SECTION 8 OF RA 6055, ITS IMPOSITION IS GENERALLY A TAX MEASURE AND THEREFORE, STILL COVERED UNDER THE PRIVILEGE OF EXEMPTION. II. THE COURT OF APPEALS DENIAL OF PETITIONER AUFS EXEMPTION FROM REAL PROPERTY TAXES CONTAINED IN ITS QUESTIONED DECISION AND QUESTIONED RESOLUTION IS CONTRARY TO APPLICABLE LAW AND JURISPRUDENCE. 18 Petitioner stresses that the tax exemption granted to educational stock corporations which have converted into non-profit foundations was broadened to include any other charges imposed by the Government as one of the incentives for such conversion. These incentives necessarily included exemption from payment of building permit and related fees as otherwise there would have been no incentives for educational foundations if the privilege were only limited to exemption from taxation, which is already provided under the Constitution. Petitioner further contends that this Court has consistently held in several cases that the primary purpose of the exaction determines its nature. Thus, a charge of a fixed sum which bears no relation to the cost of inspection and which is payable into the general revenue of the state is a tax rather than an exercise of the police power. The standard set by law in the determination of the amount that may be imposed as license fees is such that is commensurate with the cost of regulation, inspection and licensing. But in this case, the amount representing the building permit and related fees and/or charges is such an exorbitant amount as to warrant a valid imposition; such amount exceeds the probable cost of regulation. Even with the alleged criteria submitted by the respondents (e.g., character of occupancy or use of building/structure, cost of construction, floor area and height), and the construction by petitioner of an 11-storey building, the costs of inspection will not amount to P645,906.84, presumably for the salary of inspectors or employees, the expenses of transportation for inspection and the preparation and reproduction of documents.

Petitioner thus concludes that the disputed fees are substantially and mainly for purposes of revenue rather than regulation, so that even these fees cannot be deemed "charges" mentioned in Sec. 8 of R.A. No. 6055, they should properly be treated as tax from which petitioner is exempt. In their Comment, respondents maintain that petitioner is not exempt from the payment of building permit and related fees since the only exemptions provided in the National Building Code are public buildings and traditional indigenous family dwellings. Inclusio unius est exclusio alterius. Because the law did not include petitioners buildings from those structures exempt from the payment of building permit fee, it is therefore subject to the regulatory fees imposed under the National Building Code. Respondents assert that the CA correctly distinguished a building permit fee from those "other charges" mentioned in Sec. 8 of R.A. No. 6055. As stated by petitioner itself, charges refer to pecuniary liability, as rents, and fees against persons or property. Respondents point out that a building permit is classified under the term "fee." A fee is generally imposed to cover the cost of regulation as activity or privilege and is essentially derived from the exercise of police power; on the other hand, impositions for services rendered by the local government units or for conveniences furnished, are referred to as "service charges". Respondents also disagreed with petitioners contention that the fees imposed and collected are exorbitant and exceeded the probable expenses of regulation. These fees are based on computations and assessments made by the responsible officials of the City Engineer s Office in accordance with the Schedule of Fees and criteria provided in the National Building Code. The bases of assessment cited by petitioner (e.g. salary of employees, expenses of transportation and preparation and reproduction of documents) refer to charges and fees on business and occupation under Sec. 147 of the Local Government Code, which do not apply to building permit fees. The parameters set by the National Building Code can be considered as complying with the reasonable cost of regulation in the assessment and collection of building permit fees. Respondents likewise contend that the presumption of regularity in the performance of official duty applies in this case. Petitioner should have presented evidence to prove its allegations that the amounts collected are exorbitant or unreasonable. For resolution are the following issues: (1) whether petitioner is exempt from the payment of building permit and related fees imposed under the National Building Code; and (2) whether the parcel of land owned by petitioner which has been assessed for real property tax is likewise exempt. R.A. No. 6055 granted tax exemptions to educational institutions like petitioner which converted to non-stock, non-profit educational foundations. Section 8 of said law provides: SECTION 8. The Foundation shall be exempt from the payment of all taxes, import duties, assessments, and other charges imposed by the Government onall income derived from or property, real or personal, used exclusively for the educational activities of the Foundation.(Emphasis supplied.) On February 19, 1977, Presidential Decree (P.D.) No. 1096 was issued adopting the National Building Code of the Philippines. The said Code requires every person, firm or corporation, including any agency or instrumentality of the government to obtain a building permit for any construction, alteration or repair of any building or structure. 19Building permit refers to "a document issued by the Building Official x x x to an owner/applicant to proceed with the construction, installation, addition, alteration, renovation, conversion, repair, moving, demolition or other work activity of a specific project/building/structure or portions thereof after the accompanying principal plans, specifications and other pertinent documents with the duly notarized application are found satisfactory and substantially conforming with the National Building Code of the Philippines x x x and its Implementing Rules and Regulations (IRR)."20 Building permit fees refers to the basic permit fee and other charges imposed under the National Building Code. Exempted from the payment of building permit fees are: (1) public buildings and (2) traditional indigenous family dwellings. 21 Not being expressly included in the enumeration of structures to which the building permit fees do not apply, petitioners claim for exemption rests solely on its interpretation of the term "other charges imposed by the National Government" in the tax exemption clause of R.A. No. 6055. A "charge" is broadly defined as the "price of, or rate for, something," while the word "fee" pertains to a "charge fixed by law for services of public officers or for use of a privilege under control of government."22 As used in the Local Government Code of 1991 (R.A. No. 7160), charges refers to pecuniary liability, as rents or fees against persons or property, while fee means a charge fixed by law or ordinance for the regulation or inspection of a business or activity.23 That "charges" in its ordinary meaning appears to be a general term which could cover a specific "fee" does not support petit ioners position that building permit fees are among those "other charges" from which it was expressly exempted. Note that the "other charges" mentioned in Sec. 8 of R.A. No. 6055 is qualified by the words "imposed by the Government on all x x x property used exclusively for the educational activities of the foundation." Building permit fees are not impositions on property but on the activity subject of government regulation. While it may be argued that the fees relate to particular properties, i.e., buildings and structures, they are actually imposed on certain activities the owner may conduct either to build such structures or to repair, alter, renovate or demolish the same. This is evident from the following provisions of the National Building Code: Section 102. Declaration of Policy It is hereby declared to be the policy of the State to safeguard life, health, property, and public welfare, consistent with theprinciples of sound environmental management and control; and tothis end, make it the purpose of this Code to provide for allbuildings and structures, a framework of minimum standards and requirements to regulate and control their location, site, design quality of materials, construction, use, occupancy, and maintenance. Section 103. Scope and Application (a) The provisions of this Code shall apply to the design,location, sitting, construction, alteration, repair,conversion, use, occupancy, maintenance, moving, demolitionof, and addition to public and private buildings andstructures, except traditional indigenous family dwellingsas defined herein. xxxx Section 301. Building Permits No person, firm or corporation, including any agency orinstrumentality of the government shall erect, construct, alter, repair, move, convert or demolish any building or structure or causethe same to be done without first obtaining a building permittherefor from the Building Official assigned in the place where thesubject building is located or the building work is to be done. (Italics supplied.) That a building permit fee is a regulatory imposition is highlighted by the fact that in processing an application for a building permit, the Building Official shall see to it that the applicant satisfies and conforms with approved standard requirements on zoning and land use, lines and grades, structural design, sanitary and sewerage, environmental health, electrical and mechanical safety as well as with other rules and regulations implementing the National Building Code.24 Thus, ancillary permits such as electrical permit, sanitary permit and zoning clearance must also be secured and the corresponding fees paid before a building permit may be issued. And as can be gleaned from the implementing rules and regulations of the National Building Code, clearances from various government authorities exercising and enforcing regulatory functions affecting buildings/structures, like local government units, may be further required before a building permit may be issued. 25 Since building permit fees are not charges on property, they are not impositions from which petitioner is exempt.

As to petitioners argument that the building permit fees collected by respondents are in reality taxes because the primary p urpose is to raise revenues for the local government unit, the same does not hold water. A charge of a fixed sum which bears no relation at all to the cost of inspection and regulation may be held to be a tax rather than an exercise of the police power.26 In this case, the Secretary of Public Works and Highways who is mandated to prescribe and fix the amount of fees and other charges that the Building Official shall collect in connection with the performance of regulatory functions, 27 has promulgated and issued the Implementing Rules and Regulations28 which provide for the bases of assessment of such fees, as follows: 1. Character of occupancy or use of building 2. Cost of construction " 10,000/sq.m (A,B,C,D,E,G,H,I), 8,000 (F), 6,000 (J) 3. Floor area 4. Height Petitioner failed to demonstrate that the above bases of assessment were arbitrarily determined or unrelated to the activity being regulated. Neither has petitioner adduced evidence to show that the rates of building permit fees imposed and collected by the respondents were unreasonable or in excess of the cost of regulation and inspection. In Chevron Philippines, Inc. v. Bases Conversion Development Authority, 29 this Court explained: In distinguishing tax and regulation as a form of police power, the determining factor is the purpose of the implemented measure. If the purpose is primarily to raise revenue, then it will be deemed a tax even though the measure results in some form of regulation. On the other hand, if the purpose is primarily to regulate, then it is deemed a regulation and an exercise of the police power of the state, even though incidentally, revenue is generated. Thus, in Gerochi v. Department of Energy, the Court stated: "The conservative and pivotal distinction between these two (2) powers rests in the purpose for which the charge is made. If generation of revenue is the primary purpose and regulation is merely incidental, the imposition is a tax; but if regulation is the primary purpose, the fact that revenue is incidentally raised does not make the imposition a tax."30 (Emphasis supplied.) Concededly, in the case of building permit fees imposed by the National Government under the National Building Code, revenue is incidentally generated for the benefit of local government units. Thus: Section 208. Fees Every Building Official shall keep a permanent record and accurate account of all fees and other charges fixed and authorized by the Secretary to be collected and received under this Code. Subject to existing budgetary, accounting and auditing rules and regulations, the Building Official is hereby authorized to retain not more than twenty percent of his collection for the operating expenses of his office. The remaining eighty percent shall be deposited with the provincial, city or municipal treasurer and shall accrue to the General Fund of the province, city or municipality concerned. Petitioners reliance on Sec. 193 of the Local Government Code of 1991 is likewise misplaced. Said provision states: SECTION 193. Withdrawal of Tax Exemption Privileges. -- Unless otherwise provided in this Code, tax exemptions or incentives granted to, or presently enjoyed by all persons, whether natural or juridical, including government-owned or controlled corporations, except local water districts, cooperatives duly registered under R.A. No. 6938, non-stock and non-profit hospitals and educational institutions, are hereby withdrawn upon the effectivity of this Code. (Emphasis supplied.) Considering that exemption from payment of regulatory fees was not among those "incentives" granted to petitioner under R.A. No. 6055, there is no such incentive that is retained under the Local Government Code of 1991. Consequently, no reversible error was committed by the CA in ruling that petitioner is liable to pay the subject building permit and related fees. Now, on petitioners claim that it is exempted from the payment of real property tax assessed against its real property prese ntly occupied by informal settlers. Section 28(3), Article VI of the 1987 Constitution provides: xxxx (3) Charitable institutions, churches and parsonages or convents appurtenant thereto, mosques, non-profit cemeteries, and all lands, buildings, and improvements, actually, directly and exclusively used for religious, charitable or educational purposes shall be exempt from taxation. x x x x (Emphasis supplied.) Section 234(b) of the Local Government Code of 1991 implements the foregoing constitutional provision by declaring that -SECTION 234. Exemptions from Real Property Tax. The following are exempted from payment of the real property tax: xxxx (b) Charitable institutions, churches, parsonages or convents appurtenant thereto, mosques, non-profit or religious cemeteries and all lands, buildings, and improvements actually, directly, and exclusively used for religious, charitable or educational purposes; x x x x (Emphasis supplied.) In Lung Center of the Philippines v. Quezon City,31 this Court held that only portions of the hospital actually, directly and exclusively used for charitable purposes are exempt from real property taxes, while those portions leased to private entities and individuals are not exempt from such taxes. We explained the condition for the tax exemption privilege of charitable and educational institutions, as follows: Under the 1973 and 1987 Constitutions and Rep. Act No. 7160 in order to be entitled to the exemption, the petitioner is burdened to prove, by clear and unequivocal proof, that (a) it is a charitable institution; and (b) its real properties are ACTUALLY, DIRECTLY and EXCLUSIVELY used for charitable purposes. "Exclusive" is defined as possessed and enjoyed to the exclusion of others; debarred from participation or enjoyment; and "exclusively" is defined, "in a manner to exclude; as enjoying a privilege exclusively." If real property is used for one or more commercial purposes, it is not exclusively used for the exempted purposes but is subject to taxation. The words "dominant use" or "principal use" cannot be substituted for the words "used exclusively" without doing violence to the Constitutions and the law. Solely is synonymous with exclusively.1wphi1 What is meant by actual, direct and exclusive use of the property for charitable purposes is the direct and immediate and actual application of the property itself to the purposes for which the charitable institution is organized. It is not the use of the income from the real property that is determinative of whether the property is used for tax-exempt purposes.32 (Emphasis and underscoring supplied.) Petitioner failed to discharge its burden to prove that its real property is actually, directly and exclusively used for educational purposes. While there is no allegation or proof that petitioner leases the land to its present occupants, still there is no compliance with the constitutional and statutory requirement that said real property is actually, directly and exclusively used for educational purposes. The respondents correctly assessed the land for real property taxes for the taxable period during which the land is not being devoted solely to petitioners educa tional activities. Accordingly, the CA did not err in ruling that petitioner is likewise not entitled to a refund of the real property tax it paid under protest.

WHEREFORE, the petition is DENIED. The Decision dated July 28, 2009 and Resolution dated October 12, 2009 of the Court of Appeals in CA-G.R. CV No. 90591 are AFFIRMED. No pronouncement as to costs. G.R. No. 192945 September 5, 2012 CITY OF IRIGA, Petitioner, vs. CAMARINES SUR III ELECTRIC COOPERATIVE, INC. (CASURECO III), Respondent. DECISION PERLAS-BERNABE, J.: The Court reiterates that a franchise tax is a tax levied on the exercise by an entity of the rights or privileges granted to it by the government.1 In the absence of a clear and subsisting legal provision granting it tax exemption, a franchise holder, though non-profit in nature, may validly be assessed franchise tax by a local government unit. Before the Court is a petition filed under Rule 45 of the Revised Rules of Court seeking to set aside the February 11, 2010 Decision2 and July 12, 2010 Resolution3 of the Court of Appeals (CA), which reversed the February 7, 2005 Decision of the Regional Trial Court (RTC) of Iriga City, Branch 36 and ruled that respondent Camarines Sur III Electric Cooperative, Inc. (CASURECO III) is exempt from payment of local franchise tax. The Facts CASURECO III is an electric cooperative duly organized and existing by virtue of Presidential Decree (PD) 269, 4 as amended, and registered with the National Electrification Administration (NEA). It is engaged in the business of electric power distribution to various end-users and consumers within the City of Iriga and the municipalities of Nabua, Bato, Baao, Buhi, Bula and Balatan of the Province of Camarines Sur, otherwise known as the "Rinconada area."5 Sometime in 2003, petitioner City of Iriga required CASURECO III to submit a report of its gross receipts for the period 1997-2002 to serve as the basis for the computation of franchise taxes, fees and other charges.6 The latter complied7 and was subsequently assessed taxes. On January 7, 2004, petitioner made a final demand on CASURECO III to pay the franchise taxes due for the period 1998-2003 and real property taxes due for the period 1995-2003.8 CASURECO III, however, refused to pay said taxes on the ground that it is an electric cooperative provisionally registered with the Cooperative Development Authority (CDA),9 and therefore exempt from the payment of local taxes.10 On March 15, 2004, petitioner filed a complaint for collection of local taxes against CASURECO III before the RTC, citing its power to tax under the Local Government Code (LGC) and the Revenue Code of Iriga City. 11 It alleged that as of December 31, 2003, CASURECO IIIs franchise and real property taxes liability, inclusive of penalties, surcharges and interest, amounted to Seventeen Million Thirty-Seven Thousand Nine Hundred Thirty-Six Pesos and Eighty-Nine Centavos (P 17,037,936.89) and Nine Hundred Sixteen Thousand Five Hundred Thirty-Six Pesos and Fifty Centavos (P 916,536.50), respectively.12 In its Answer, CASURECO III denied liability for the assessed taxes, asserting that the computation of the petitioner was erroneous because it included 1) gross receipts from service areas beyond the latters territorial jurisdiction; 2) taxes that had already prescribed; and 3) taxes during the period when it was still exempt from local government tax by virtue of its then subsisting registration with the CDA. 13 Ruling of the Trial Court In its Decision dated February 7, 2005, the RTC ruled that the real property taxes due for the years 1995-1999 had already prescribed in accordance with Section 19414 of the LGC. However, it found CASURECO III liable for franchise taxes for the years 2000-2003 based on its gross receipts from Iriga City and the Rinconada area on the ground that the "situs of taxation is the place where the privilege is exercised." 15 The dispositive portion of the RTC Decision reads: WHEREFORE, in view of the foregoing, defendant is hereby made liable to pay plaintiff real property taxes and franchise taxes on its receipts, including those from service area covering Nabua, Bato, Baao and Buhi for the years 2000 up to the present. The realty taxes for the years 1995 and 1999 is hereby declared prescribed. The City Assessor is hereby directed to make the proper classification of defendants real property in accordance with Ordinance issued by the City Council. SO ORDERED.16 Only CASURECO III appealed from the RTC Decision, questioning its liability for franchise taxes. Ruling of the Court of Appeals In its assailed Decision, the CA found CASURECO III to be a non-profit entity, not falling within the purview of "businesses enjoying a franchise" pursuant to Section 137 of the LGC. It explained that CASURECO IIIs non -profit nature is diametrically opposed to the concept of a "business," which, as defined under Section 131 of the LGC, is a "trade or commercial activity regularly engaged in as a means of livelihood or with a view to profit." Consequently, it relieved CASURECO III from liability to pay franchise taxes. Petitioner moved for reconsideration, which the CA denied in its July 12, 2010 Resolution for being filed a day late, hence, the instant petition. Issues Before the Court Petitioner raises two issues for resolution, which the Court restates as follows: (1) whether or not an electric cooperative registered under PD 269 but not under RA 693817 is liable for the payment of local franchise taxes; and (2) whether or not the situs of taxation is the place where the franchise holder exercises its franchise regardless of the place where its services or products are delivered. CASURECO III, on the other hand, raises the procedural issue that since the motion for reconsideration of the CA Decision was filed out of time, the same had attained finality. The Courts Ruling The petition is meritorious. Before delving into the substantive issues, the Court notes the procedural lapses extant in the present case. Proper Mode of Appeal from the Decision of the Regional Trial Court involving local taxes RA 9282,18 which took effect on April 23, 2004, expanded the jurisdiction of the Court of Tax Appeals (CTA) to include, among others, the power to review by appeal decisions, orders or resolutions of the Regional Trial Courts in local tax cases originally decided or resolved by them in the exercise of their original or appellate jurisdiction.19 Considering that RA 9282 was already in effect when the RTC rendered its decision on February 7, 2005, CASURECO III should have filed its appeal, not with the CA, but with the CTA Division in accordance with the applicable law and the rules of the CTA. Resort to the CA was, therefore, improper, rendering its decision null and void for want of jurisdiction over the subject matter. A void judgment has no legal or binding

force or efficacy for any purpose or at any place.20 Hence, the fact that petitioner's motion for reconsideration from the CA Decision was belatedly filed is inconsequential, because a void and non-existent decision would never have acquired finality.21 The foregoing procedural lapses would have been sufficient to dismiss the instant petition outright and declare the decision of the RTC final. However, the substantial merits of the case compel us to dispense with these lapses and instead, exercise the Courts power o f judicial review. CASURECO III is not exempt from payment of franchise tax PD 269, which took effect on August 6, 1973, granted electric cooperatives registered with the NEA, like CASURECO III, several tax privileges, one of which is exemption from the payment of "all national government, local government and municipal taxes and fees, including franchise, filing, recordation, license or permit fees or taxes."22 On March 10, 1990, Congress enacted into law RA 6938,23 otherwise known as the "Cooperative Code of the Philippines," and RA 6939 24 creating the CDA. The latter law vested the power to register cooperatives solely on the CDA, while the former provides that electric cooperatives registered with the NEA under PD 269 which opt not to register with the CDA shall not be entitled to the benefits and privileges under the said law. On January 1, 1992, the LGC took effect, and Section 193 thereof withdrew tax exemptions or incentives previously enjoyed by "all persons, whether natural or juridical, including government-owned or controlled corporations, except local water districts, cooperatives duly registered under R.A. No. 6938, non-stock and non-profit hospitals and educational institutions."25 In Philippine Rural Electric Cooperatives Association, Inc. (PHILRECA) v. The Secretary, Department of Interior and Local Government,26 the Court held that the tax privileges granted to electric cooperatives registered with NEA under PD 269 were validly withdrawn and only those registered with the CDA under RA 6938 may continue to enjoy the tax privileges under the Cooperative Code. Therefore, CASURECO III can no longer invoke PD 269 to evade payment of local taxes. Moreover, its provisional registration with the CDA which granted it exemption for the payment of local taxes was extended only until May 4, 1992. Thereafter, it can no longer claim any exemption from the payment of local taxes, including the subject franchise tax.1wphi1 Indisputably, petitioner has the power to impose local taxes. The power of the local government units to impose and collect taxes is derived from the Constitution itself which grants them "the power to create its own sources of revenues and to levy taxes, fees and charges subject to such guidelines and limitation as the Congress may provide."27 This explicit constitutional grant of power to tax is consistent with the basic policy of local autonomy and decentralization of governance. With this power, local government units have the fiscal mechanisms to raise the funds needed to deliver basic services to their constituents and break the culture of dependence on the national government. Thus, consistent with these objectives, the LGC was enacted granting the local government units, like petitioner, the power to impose and collect franchise tax, to wit: SEC. 137. Franchise Tax. - Notwithstanding any exemption granted by any law or other special law, the province may impose a tax on businesses enjoying a franchise, at a rate not exceeding fifty percent (50%) of one percent (1%) of the gross annual receipts for the preceding calendar year based on the incoming receipt, or realized, within its territorial jurisdiction. xxx SEC. 151. Scope of Taxing Powers. - Except as otherwise provided in this Code, the city, may levy the taxes, fees, and charges which the province or municipality may impose: Provided, however, That the taxes, fees and charges levied and collected by highly urbanized and independent component cities shall accrue to them and distributed in accordance with the provisions of this Code. The rates of taxes that the city may levy may exceed the maximum rates allowed for the province or municipality by not more than fifty percent (50%) except the rates of professional and amusement taxes. Taking a different tack, CASURECO III maintains that it is exempt from payment of franchise tax because of its nature as a non-profit cooperative, as contemplated in PD 269,28 and insists that only entities engaged in business, and not non-profit entities like itself, are subject to the said franchise tax. The Court is not persuaded. In National Power Corporation v. City of Cabanatuan,29 the Court declared that "a franchise tax is a tax on the privilege of transacting business in the state and exercising corporate franchises granted by the state."30 It is not levied on the corporation simply for existing as a corporation, upon its property or its income, but on its exercise of the rights or privileges granted to it by the government.31 "It is within this context that the phrase tax on businesses enjoying a franchise in Section 137 of the LGC should be int erpreted and understood."32 Thus, to be liable for local franchise tax, the following requisites should concur: (1) that one has a "franchise" in the sense of a secondary or special franchise; and (2) that it is exercising its rights or privileges under this franchise within the territory of the pertinent local government unit.33 There is a confluence of these requirements in the case at bar. By virtue of PD 269, NEA granted CASURECO III a franchise to operate an electric light and power service for a period of fifty (50) years from June 6, 1979, 34 and it is undisputed that CASURECO III operates within Iriga City and the Rinconada area. It is, therefore, liable to pay franchise tax notwithstanding its non-profit nature. CASURECO III is liable for franchise tax on gross receipts within Iriga City and Rinconada area CASURECO III further argued that its liability to pay franchise tax, if any, should be limited to gross receipts received from the supply of the electricity within the City of Iriga and not those from the Rinconada area. Again, the Court is not convinced. It should be stressed that what the petitioner seeks to collect from CASURECO III is a franchise tax, which as defined, is a tax on the exercise of a privilege. As Section 13735 of the LGC provides, franchise tax shall be based on gross receipts precisely because it is a tax on business, rather than on persons or property.36 Since it partakes of the nature of an excise tax/37 the situs of taxation is the place where the privilege is exercised, in this case in the City of Iriga, where CASURECO III has its principal office and from where it operates, regardless of the place where its services or products are delivered. Hence, franchise tax covers all gross receipts from Iriga City and the Rinconada area. WHEREFORE, the petition is GRANTED. The assailed Decision dated February 11, 2010 and Resolution dated July 12, 2010 of the Court of Appeals are hereby SET ASIDE and the Decision of the Regional Trial Court of lriga City, Branch 36, is REINSTATED.

G.R. No. 167146

October 31, 2006

COMMISSIONER OF INTERNAL REVENUE, petitioner, vs. PHILIPPINE GLOBAL COMMUNICATION, INC., respondent.

DECISION

CHICO-NAZARIO, J.: This is a Petition for Review on Certiorari, under Rule 45 of the Rules of Court, seeking to set aside the en banc Decision of the Court of Tax Appeals (CTA) in CTA EB No. 37 dated 22 February 2005,1 ordering the petitioner to withdraw and cancel Assessment Notice No. 00068880-7333 issued against respondent Philippine Global Communication, Inc. for its 1990 income tax deficiency. The CTA, in its assailed en banc Decision, affirmed the Decision of the First Division of the CTA dated 9 June 20042 and its Resolution dated 22 September 2004 in C.T.A. Case No. 6568. Respondent, a corporation engaged in telecommunications, filed its Annual Income Tax Return for taxable year 1990 on 15 April 1991. On 13 April 1992, the Commissioner of Internal Revenue (CIR) issued Letter of Authority No. 0002307, authorizing the appropriate Bureau of Internal Revenue (BIR) officials to examine the books of account and other accounting records of respondent, in connection with the investigation of respondents 1990 income tax liability. On 22 April 1992, the BIR sent a letter to respondent requesting the latter to present for examination certain records and documents, but respondent failed to present any document. On 21 April 1994, respondent received a Preliminary Assessment Notice dated 13 April 1994 for deficiency income tax in the amount of P118,271,672.00, inclusive of surcharge, interest, and compromise penalty, arising from deductions that were disallowed for failure to pay the withholding tax and interest expenses that were likewise disallowed. On the following day, 22 April 1994, respondent received a Formal Assessment Notice with Assessment Notice No. 000688-80-7333, dated 14 April 1994, for deficiency income tax in the total amount of P118,271,672.00.3 On 6 May 1994, respondent, through its counsel Ponce Enrile Cayetano Reyes and Manalastas Law Offices, filed a formal protest letter against Assessment Notice No. 000688-80-7333. Respondent filed another protest letter on 23 May 1994, through another counsel Siguion Reyna Montecillo & Ongsiako Law Offices. In both letters, respondent requested for the cancellation of the tax assessment, which they alleged was invalid for lack of factual and legal basis.4 On 16 October 2002, more than eight years after the assessment was presumably issued, the Ponce Enrile Cayetano Reyes and Manalastas Law Offices received from the CIR a Final Decision dated 8 October 2002 denying the respondents protest against Assessment Notice No. 000688-80-7333, and affirming the said assessment in toto.5

On 15 November 2002, respondent filed a Petition for Review with the CTA. After due notice and hearing, the CTA rendered a Decision in favor of respondent on 9 June 2004.6 The CTA ruled on the primary issue of prescription and found it unnecessary to decide the issues on the validity and propriety of the assessment. It decided that the protest letters filed by the respondent cannot constitute a request for reinvestigation, hence, they cannot toll the running of the prescriptive period to collect the assessed deficiency income tax.7 Thus, since more than three years had lapsed from the time Assessment Notice No. 000688-80-7333 was issued in 1994, the CIRs right to collect the same has prescribed in conformity with Section 269 of the National Internal Revenue Code of 19778 (Tax Code of 1977). The dispositive portion of this decision reads: WHEREFORE, premises considered, judgment is hereby rendered in favor of the petitioner. Accordingly, respondents Final Decision dated October 8, 2002 is hereby REVERSED and SET ASIDE and respondent is hereby ORDERED to WITHDRAW and CANCEL Assessment Notice No. 000688-80-7333 issued against the petitioner for its 1990 income tax deficiency because respondents right to collect the same has prescribed.9 The CIR moved for reconsideration of the aforesaid Decision but was denied by the CTA in a Resolution dated 22 September 2004.10 Thereafter, the CIR filed a Petition for Review with the CTA en banc, questioning the aforesaid Decision and Resolution. In its en banc Decision, the CTA affirmed the Decision and Resolution in CTA Case No. 6568. The dispositive part reads: WHEREFORE, premises considered, the Petition for Review is hereby DISMISSED for lack of merit. Accordingly, the assailed Decision and Resolution in CTA Case No. 6568 are hereby AFFIRMED in toto.11 Hence, this Petition for Review on Certiorari raising the following grounds: THE COURT OF TAX APPEALS, SITTING EN BANC, COMMITTED REVERSIBLE ERROR IN AFFIRMING THE ASSAILED DECISION AND RESOLUTION IN CTA CASE NO. 6568 DECLARING THAT THE RIGHT OF THE GOVERNMENT TO COLLECT THE DEFICIENCY INCOME TAX FROM RESPONDENT FOR THE YEAR 1990 HAS PRESCRIBED A. THE PRESCRIPTIVE PERIOD WAS INTERUPTED WHEN RESPONDENT FILED TWO LETTERS OF PROTEST DISPUTING IN DETAIL THE DEFICIENCY ASSESSMENT IN QUESTION AND REQUESTING THE CANCELLATION OF SAID ASSESSMENT. THE TWO LETTERS OF PROTEST ARE, BY NATURE, REQUESTS FOR REINVESTIGATION OF THE DISPUTED ASSESSMENT. B. THE REQUESTS FOR REINVESTIGATION OF RESPONDENT WERE GRANTED BY THE BUREAU OF INTERNAL REVENUE.12 This Court finds no merit in this Petition.

The main issue in this case is whether or not CIRs right to collect respondents alleged deficiency income tax is barred by prescription under Section 269(c) of the Tax Code of 1977, which reads: Section 269. Exceptions as to the period of limitation of assessment and collection of taxes. x x x xxxx c. Any internal revenue tax which has been assessed within the period of limitation above-prescribed may be collected by distraint or levy or by a proceeding in court within three years following the assessment of the tax. The law prescribed a period of three years from the date the return was actually filed or from the last date prescribed by law for the filing of such return, whichever came later, within which the BIR may assess a national internal revenue tax.13 However, the law increased the prescriptive period to assess or to begin a court proceeding for the collection without an assessment to ten years when a false or fraudulent return was filed with the intent of evading the tax or when no return was filed at all.14 In such cases, the ten-year period began to run only from the date of discovery by the BIR of the falsity, fraud or omission. If the BIR issued this assessment within the three-year period or the ten-year period, whichever was applicable, the law provided another three years after the assessment for the collection of the tax due thereon through the administrative process of distraint and/or levy or through judicial proceedings.15 The three-year period for collection of the assessed tax began to run on the date the assessment notice had been released, mailed or sent by the BIR.16 The assessment, in this case, was presumably issued on 14 April 1994 since the respondent did not dispute the CIRs claim. Therefore, the BIR had until 13 April 1997. However, as there was no Warrant of Distraint and/or Levy served on the respondents nor any judicial proceedings initiated by the BIR, the earliest attempt of the BIR to collect the tax due based on this assessment was when it filed its Answer in CTA Case No. 6568 on 9 January 2003, which was several years beyond the three-year prescriptive period. Thus, the CIR is now prescribed from collecting the assessed tax. The provisions on prescription in the assessment and collection of national internal revenue taxes became law upon the recommendation of the tax commissioner of the Philippines. The report submitted by the tax commission clearly states that these provisions on prescription should be enacted to benefit and protect taxpayers: Under the former law, the right of the Government to collect the tax does not prescribe. However, in fairness to the taxpayer, the Government should be estopped from collecting the tax where it failed to make the necessary investigation and assessment within 5 years after the filing of the return and where it failed to collect the tax within 5 years from the date of assessment thereof. Just as the government is interested in the stability of its collections, so also are the taxpayers entitled to an assurance that they will not be

subjected to further investigation for tax purposes after the expiration of a reasonable period of time. (Vol. II, Report of the Tax Commission of the Philippines, pp. 321-322).17 In a number of cases, this Court has also clarified that the statute of limitations on the collection of taxes should benefit both the Government and the taxpayers. In these cases, the Court further illustrated the harmful effects that the delay in the assessment and collection of taxes inflicts upon taxpayers. In Collector of Internal Revenue v. Suyoc Consolidated Mining Company,18 Justice Montemayor, in his dissenting opinion, identified the potential loss to the taxpayer if the assessment and collection of taxes are not promptly made. Prescription in the assessment and in the collection of taxes is provided by the Legislature for the benefit of both the Government and the taxpayer; for the Government for the purpose of expediting the collection of taxes, so that the agency charged with the assessment and collection may not tarry too long or indefinitely to the prejudice of the interests of the Government, which needs taxes to run it; and for the taxpayer so that within a reasonable time after filing his return, he may know the amount of the assessment he is required to pay, whether or not such assessment is well founded and reasonable so that he may either pay the amount of the assessment or contest its validity in court x x x. It would surely be prejudicial to the interest of the taxpayer for the Government collecting agency to unduly delay the assessment and the collection because by the time the collecting agency finally gets around to making the assessment or making the collection, the taxpayer may then have lost his papers and books to support his claim and contest that of the Government, and what is more, the tax is in the meantime accumulating interest which the taxpayer eventually has to pay . In Republic of the Philippines v. Ablaza,19 this Court emphatically explained that the statute of limitations of actions for the collection of taxes is justified by the need to protect law-abiding citizens from possible harassment: The law prescribing a limitation of actions for the collection of the income tax is beneficial both to the Government and to its citizens; to the Government because tax officers would be obliged to act promptly in the making of assessment, and to citizens because after the lapse of the period of prescription citizens would have a feeling of security against unscrupulous tax agents who will always find an excuse to inspect the books of taxpayers, not to determine the latters real liability, but to take advantage of every opportunity to molest, peaceful, law-abiding citizens. Without such legal defense taxpayers would furthermore be under obligation to always keep their books and keep them open for inspection subject to harassment by unscrupulous tax agents. The law on prescription being a remedial measure should be interpreted in a way conducive to bringing about the beneficient purpose of affording protection to the taxpayer within the contemplation of the Commission which recommended the approval of the law. And again in the recent case Bank of the Philippine Islands v. Commissioner of Internal Revenue,20 this Court, in confirming these earlier rulings, pronounced that:

Though the statute of limitations on assessment and collection of national internal revenue taxes benefits both the Government and the taxpayer, it principally intends to afford protection to the taxpayer against unreasonable investigation. The indefinite extension of the period for assessment is unreasonable because it deprives the said taxpayer of the assurance that he will no longer be subjected to further investigation for taxes after the expiration of a reasonable period of time. Thus, in Commissioner of Internal Revenue v. B.F. Goodrich,21 this Court affirmed that the law on prescription should be liberally construed in order to protect taxpayers and that, as a corollary, the exceptions to the law on prescription should be strictly construed. The Tax Code of 1977, as amended, provides instances when the running of the statute of limitations on the assessment and collection of national internal revenue taxes could be suspended, even in the absence of a waiver, under Section 271 thereof which reads: Section 224. Suspension of running of statute. The running of the statute of limitation provided in Sections 268 and 269 on the making of assessments and the beginning of distraint or levy or a proceeding in court for collection in respect of any deficiency, shall be suspended for the period during which the Commissioner is prohibited from making the assessment or beginning distraint or levy or a proceeding in court and for sixty days thereafter; when the taxpayer requests for a reinvestigation which is granted by the Commissioner; when the taxpayer cannot be located in the address given by him in the return filed upon which a tax is being assessed or collected x x x. (Emphasis supplied.) Among the exceptions provided by the aforecited section, and invoked by the CIR as a ground for this petition, is the instance when the taxpayer requests for a reinvestigation which is granted by the Commissioner. However, this exception does not apply to this case since the respondent never requested for a reinvestigation. More importantly, the CIR could not have conducted a reinvestigation where, as admitted by the CIR in its Petition, the respondent refused to submit any new evidence. Revenue Regulations No. 12-85, the Procedure Governing Administrative Protests of Assessment of the Bureau of Internal Revenue, issued on 27 November 1985, defines the two types of protest, the request for reconsideration and the request for reinvestigation, and distinguishes one from the other in this manner: Section 6. Protest. - The taxpayer may protest administratively an assessment by filing a written request for reconsideration or reinvestigation specifying the following particulars: xxxx For the purpose of protest herein (a) Request for reconsideration-- refers to a plea for a re-evaluation of an assessment on the basis of existing records without need of additional evidence. It may involve both a question of fact or of law or both.

(b) Request for reinvestigationrefers to a plea for re-evaluation of an assessment on the basis of newly-discovered evidence or additional evidence that a taxpayer intends to present in the investigation. It may also involve a question of fact or law or both. The main difference between these two types of protests lies in the records or evidence to be examined by internal revenue officers, whether these are existing records or newly discovered or additional evidence. A re-evaluation of existing records which results from a request for reconsideration does not toll the running of the prescription period for the collection of an assessed tax. Section 271 distinctly limits the suspension of the running of the statute of limitations to instances when reinvestigation is requested by a taxpayer and is granted by the CIR. The Court provided a clear-cut rationale in the case of Bank of the Philippine Islands v. Commissioner of Internal Revenue22 explaining why a request for reinvestigation, and not a request for reconsideration, interrupts the running of the statute of limitations on the collection of the assessed tax: Undoubtedly, a reinvestigation, which entails the reception and evaluation of additional evidence, will take more time than a reconsideration of a tax assessment, which will be limited to the evidence already at hand; this justifies why the former can suspend the running of the statute of limitations on collection of the assessed tax, while the latter cannot. In the present case, the separate letters of protest dated 6 May 1994 and 23 May 1994 are requests for reconsideration. The CIRs allegation that there was a request for reinvestigation is inconceivable since respondent consistently and categorically refused to submit new evidence and cooperate in any reinvestigation proceedings. This much was admitted in the Decision dated 8 October 2002 issued by then CIR Guillermo Payarno, Jr. In the said conference-hearing, Revenue Officer Alameda basically testified that Philcom, despite repeated demands, failed to submit documentary evidences in support of its claimed deductible expenses. Hence, except for the item of interest expense which was disallowed for being not ordinary and necessary, the rest of the claimed expenses were disallowed for non-withholding. In the same token, Revenue Officer Escober testified that upon his assignment to conduct the re-investigation, he immediately requested the taxpayer to present various accounting records for the year 1990, in addition to other documents in relation to the disallowed items (p.171). This was followed by other requests for submission of documents (pp.199 &217) but these were not heeded by the taxpayer. Essentially, he stated that Philcom did not cooperate in his reinvestigation of the case. In response to the testimonies of the Revenue Officers, Philcom thru Atty. Consunji, emphasized that it was denied due process because of the issuance of the Pre-Assessment Notice and the Assessment Notice on successive dates. x x x Counsel for the taxpayer even questioned the propriety of the conference-hearing inasmuch as the only question to resolved (sic) is the legality of the issuance of the assessment. On the disallowed items, Philcom thru counsel manifested that it has no intention to present documents and/or

evidences allegedly because of the pending legal question on the validity of the assessment.23 Prior to the issuance of Revenue Regulations No. 12-85, which distinguishes a request for reconsideration and a request for reinvestigation, there have been cases wherein these two terms were used interchangeably. But upon closer examination, these cases all involved a reinvestigation that was requested by the taxpayer and granted by the BIR. In Collector of Internal Revenue v. Suyoc Consolidated Mining Company,24 the Court weighed the considerable time spent by the BIR to actually conduct the reinvestigations requested by the taxpayer in deciding that the prescription period was suspended during this time. Because of such requests, several reinvestigations were made and a hearing was even held by the Conference Staff organized in the collection office to consider claims of such nature which, as the record shows, lasted for several months. After inducing petitioner to delay collection as he in fact did, it is most unfair for respondent to now take advantage of such desistance to elude his deficiency income tax liability to the prejudice of the Government invoking the technical ground of prescription. Although the Court used the term "requests for reconsideration" in reference to the letters sent by the taxpayer in the case of Querol v. Collector of Internal Revenue,25 it took into account the reinvestigation conducted soon after these letters were received and the revised assessment that resulted from the reinvestigations. It is true that the Collector revised the original assessment on February 9, 1955; and appellant avers that this revision was invalid in that it was not made within the five-year prescriptive period provided by law (Collector vs. Pineda, 112 Phil. 321). But that fact is that the revised assessment was merely a result of petitioner Querols requests for reconsideration of the original assessment, contained in his letters of December 14, 1951 and May 25, 1953. The records of the Bureau of Internal Revenue show that after receiving the letters, the Bureau conducted a reinvestigation of petitioners tax liabilities, and, in fact, sent a tax examiner to San Fernando, La Union, for that purpose; that because of the examiners report, the Bureau revised the original assessment, x x x. In other words, the reconsideration was granted in part, and the original assessment was altered. Consequently, the period between the petition for reconsideration and the revised assessment should be subtracted from the total prescriptive period (Republic vs. Ablaza, 108 Phil 1105). The Court, in Republic v. Lopez,26 even gave a detailed accounting of the time the BIR spent for each reinvestigation in order to deduct it from the five-year period set at that time in the statute of limitations: It is now a settled ruled in our jurisdiction that the five-year prescriptive period fixed by Section 332(c) of the Internal Revenue Code within which the Government may sue to collect an assessed tax is to be computed from the last revised assessment resulting from a reinvestigation asked for by the taxpayer and (2) that where a taxpayer demands a

reinvestigation, the time employed in reinvestigating should be deducted from the total period of limitation. xxxx The first reinvestigation was granted, and a reduced assessment issued on 29 May 1954, from which date the Government had five years for bringing an action to collect. The second reinvestigation was asked on 16 January 1956, and lasted until it was decided on 22 April 1960, or a period of 4 years, 3 months, and 6 days, during which the limitation period was interrupted. The Court reiterated the ruling in Republic v. Lopez in the case of Commissioner of Internal Revenue v. Sison,27 "that where a taxpayer demands a reinvestigation, the time employed in reinvestigating should be deducted from the total period of limitation." Finally, in Republic v. Arcache,28 the Court enumerated the reasons why the taxpayer is barred from invoking the defense of prescription, one of which was that, "In the first place, it appears obvious that the delay in the collection of his 1946 tax liability was due to his own repeated requests for reinvestigation and similarly repeated requests for extension of time to pay." In this case, the BIR admitted that there was no new or additional evidence presented. Considering that the BIR issued its Preliminary Assessment Notice on 13 April 1994 and its Formal Assessment Notice on 14 April 1994, just one day before the three-year prescription period for issuing the assessment expired on 15 April 1994, it had ample time to make a factually and legally well-founded assessment. Added to the fact that the Final Decision that the CIR issued on 8 October 2002 merely affirmed its earlier findings, whatever examination that the BIR may have conducted cannot possibly outlast the entire three-year prescriptive period provided by law to collect the assessed tax, not to mention the eight years it actually took the BIR to decide the respondents protest. The factual and legal issues involved in the assessment are relatively simple, that is, whether certain income tax deductions should be disallowed, mostly for failure to pay withholding taxes. Thus, there is no reason to suspend the running of the statute of limitations in this case. The distinction between a request for reconsideration and a request for reinvestigation is significant. It bears repetition that a request for reconsideration, unlike a request for reinvestigation, cannot suspend the statute of limitations on the collection of an assessed tax. If both types of protest can effectively interrupt the running of the statute of limitations, an erroneous assessment may never prescribe. If the taxpayer fails to file a protest, then the erroneous assessment would become final and unappealable.29 On the other hand, if the taxpayer does file the protest on a patently erroneous assessment, the statute of limitations would automatically be suspended and the tax thereon may be collected long after it was assessed. Meanwhile the interest on the deficiencies and the surcharges continue to accumulate. And for an unrestricted number of years, the taxpayers remain uncertain and are burdened with the costs of preserving their books and records. This is the predicament that the law on the statute of limitations seeks to prevent.

The Court, in sustaining for the first time the suspension of the running of the statute of limitations in cases where the taxpayer requested for a reinvestigation, gave this justification: A taxpayer may be prevented from setting up the defense of prescription even if he has not previously waived it in writing as when by his repeated requests or positive acts the Government has been, for good reasons, persuaded to postpone collection to make him feel that the demand was not unreasonable or that no harassment or injustice is meant by the Government. xxxx This case has no precedent in this jurisdiction for it is the first time that such has risen, but there are several precedents that may be invoked in American jurisprudence. As Mr. Justice Cardozo has said: "The applicable principle is fundamental and unquestioned. He who prevents a thing from being done may not avail himself of the nonperformance which he himself occasioned, for the law says to him in effect "this is your own act, and therefore you are not damnified." (R.H. Stearns Co. v. U.S., 78 L. ed., 647). (Emphasis supplied.)30 This rationale is not applicable to the present case where the respondent did nothing to prevent the BIR from collecting the tax. It did not present to the BIR any new evidence for its reevaluation. At the earliest opportunity, respondent insisted that the assessment was invalid and made clear to the BIR its refusal to produce documents that the BIR requested. On the other hand, the BIR also communicated to the respondent its unwavering stance that its assessment is correct. Given that both parties were at a deadlock, the next logical step would have been for the BIR to issue a Decision denying the respondents protest and to initiate proceedings for the collection of the assessed tax and, thus, allow the respondent, should it so choose, to contest the assessment before the CTA. Postponing the collection for eight long years could not possibly make the taxpayer feel that the demand was not unreasonable or that no harassment or injustice is meant by the Government. There was no legal, or even a moral, obligation preventing the CIR from collecting the assessed tax. In a similar case, Cordero v. Conda,31 the Court did not suspend the running of the prescription period where the acts of the taxpayer did not prevent the government from collecting the tax. The government also urges that partial payment is "acknowledgement of the tax obligation", hence a "waiver on the defense of prescription." But partial payment would not prevent the government from suing the taxpayer. Because, by such act of payment, the government is not thereby "persuaded to postpone collection to make him feel that the demand was not unreasonable or that no harassment or injustice is meant." Which, as stated in Collector v. Suyoc Consolidated Mining Co., et al., L-11527, November 25, 1958, is the underlying reason behind the rule that prescriptive period is arrested by the taxpayers request for reexamination or reinvestigation even if "he has not previously waived it [prescription] in writing." The Court reminds us, in the case of Commissioner of Internal Revenue v. Algue, Inc., 32 of the need to balance the conflicting interests of the government and the taxpayers.

Taxes are the lifeblood of the government and so should be collected without unnecessary hindrance. On the other hand, such collection should be made in accordance with law as any arbitrariness will negate the very reason for government itself. It is therefore necessary to reconcile the apparently conflicting interest of the authorities and the taxpayers so that the real purpose of taxation, which is the promotion of common good, may be achieved. Thus, the three-year statute of limitations on the collection of an assessed tax provided under Section 269(c) of the Tax Code of 1977, a law enacted to protect the interests of the taxpayer, must be given effect. In providing for exceptions to such rule in Section 271, the law strictly limits the suspension of the running of the prescription period to, among other instances, protests wherein the taxpayer requests for a reinvestigation. In this case, where the taxpayer merely filed two protest letters requesting for a reconsideration, and where the BIR could not have conducted a reinvestigation because no new or additional evidence was submitted, the running of statute of limitations cannot be interrupted. The tax which is the subject of the Decision issued by the CIR on 8 October 2002 affirming the Formal Assessment issued on 14 April 1994 can no longer be the subject of any proceeding for its collection. Consequently, the right of the government to collect the alleged deficiency tax is barred by prescription. IN VIEW OF THE FOREGOING, the instant Petition is DENIED. The assailed en banc Decision of the CTA in CTA EB No. 37 dated 22 February 2005, cancelling Assessment Notice No. 000688-80-7333 issued against Philippine Global Communication, Inc. for its 1990 income tax deficiency for the reason that it is barred by prescription, is hereby AFFIRMED. No costs. G.R. No. 118176 April 12, 2000

PROTECTOR'S SERVICES, INC., petitioner, vs. COURT OF APPEALS AND COMMISSIONER OF INTERNAL REVENUE, respondents.

QUISUMBING, J.: Assailed in this petition for review is the Decision1 of the Court of Appeals dated November 28, 1994, in CA-G.R. SP No.31825. It affirmed the judgment of the Court of Tax Appeals which had dismissed the petition for review of assessments made by the Commissioner of Internal Revenue imposing deficiency percentage taxes on petitioner for the years 1983, 1984 and 1985. The dispositive portion of the CTA's decision states: WHEREFORE, in all the foregoing, this case is hereby DISMISSED for lack of jurisdiction the subject assessments having become final and unappealable.2 The facts are as follows:

Petitioner Protector's Services, Inc. (PSI) is a contractor engaged in recruiting security guards for clients. After an audit investigation conducted by the Bureau of Internal Revenue (BIR), petitioner was assessed for deficiency percentage taxes including surcharges, penalties and interests thereon, as follows: YEAR 1983 1984 1985 AMOUNT DEMAND LETTER NO.

P503,564.59 18-452-83B-87-B2 831,464.30 18-451-84B-87-B2

1,514,047.86 18-450-85B-87-B2

On December 7, 1987, respondent Commissioner sent by registered mail, demand letters for payment of the aforesaid assessments. However, petitioner alleged that on December 10, 1987, it only received Demand Letter Nos. 18-452-83B-87-B2 and 18-451-84B-87-B2 for the years 1983 and 1984, respectively. It denied receiving any notice of deficiency percentage tax for the year 1985. Petitioner sent a protest letter dated January 02, 1988, to the BIR regarding the 1983 and 1984 assessments. The petitioner claimed that its gross receipts subject to percentage taxes should exclude the salaries of the security guards as well as the corresponding employer's share of Social Security System (SSS), State Insurance Fund (SIF) and Medicare contributions.1wphi1.nt Without formally acting on the petitioner's protest, the BIR sent a follow-up letter dated July 12, 1988, ordering the settlement of taxes based on its computation. Additional documentary stamp taxes of two thousand twenty-five (P2,025.00) pesos on petitioner's capitalization for 1983 and 1984, and seven hundred three pesos and forty-one centavos (P703.41) as deficiency expanded withholding tax were included in the amount demanded. The total unsettled tax amounted to two million, eight hundred fifty-one thousand, eight hundred five pesos and sixteen centavos (P2,851,805.16). On July 21, 1988, petitioner paid the P2,025.00 documentary stamp tax and the P703.41 deficiency expanded withholding tax. On the following day, July 22, 1988, petitioner filed its second protest on the 1983 and 1984 percentage taxes, and included, for the first time, its protest against the 1985 assessment. On November 9, 1990, BIR Deputy Commissioner Eufracio Santos sent a letter to the petitioner which denied with finality the latter's protests against the subject assessments, stating thus: . . . [T]hat the salaries paid to the security guards form part of your taxable gross receipts in the determination of the 3% and 4% contractor's tax imposed under Section 191 of the Tax Code prior to its amendment by the provision of Executive Order No. 273. Considering that the security guards are actually your employees and not that of your clients, the salaries corresponding to the services rendered by your employees form part

of your taxable receipts. This contention finds support in the case of Avecilla Building Corporation versus Commissioner, et al., G.R. L-42395, 17 January 1985 and Resty Arbon Singh versus Commissioner, CTA Case No. 1901, 5 December 1970.3 On December 5, 1990, petitioner filed a petition for review before the CTA contending that: 1) Assessments for documentary stamp tax and expanded withholding tax are without basis since they were paid on July 22, 1988. 2) The period for collection of the 1985 percentage tax had prescribed, because PSI denied having received any assessment letter for the same year. 3) Percentage taxes for the three quarters of 1984 were filed as follows: 1st Qtr. April 23, 1984; 2nd Qtr. July 20, 1984, and; 3rd Qtr. October 19, 1984. The three-year prescriptive period to collect percentage taxes for the 1st, 2nd and 3rd quarters had prescribed because the BIR sent an assessment letter only on December 10, 1987. 4) The base amount for computing percentage tax was erroneous because the BIR included in the taxable amount, the salaries of the security guards and the employer's corresponding remittances to SSS, SIF, and Medicare, which amounts were earmarked for other persons, and should not form part of PSI's receipts. The CTA dismissed the petition on the following grounds: (1) The three-year period of limitation for assessment of taxes in 1984 commenced from the date of filing the final return on January 20, 1985, hence assessment made on December 10, 1987, was within said period. (2) Petitioner could not deny receipt of the 1985 assessment on the same date, December 10, 1987, for as supported by testimony of the BIR personnel, all the assessment letters for the years 1983, 1984, and 1985 were included in one envelope and mailed together. (3) Petitioner's protest letter dated January 2, 1988, was filed on January 12, 1988, or thirty-three days from December 10, 1987, hence, the request for reinvestigation was filed out of time. Petitioner appealed to the Court of Appeals, which affirmed the decision of the CTA. Hence, the present petition, wherein petitioner raises the following issues: I. WHETHER THE COURT OF TAX APPEALS HAS JURISDICTION TO ACT ON THE PETITION FOR REVIEW FILED BEFORE IT. II. WHETHER THE ASSESSMENTS AGAINST THE PETITIONER FOR DEFICIENCY PERCENTAGE TAX FOR TAXABLE YEARS 1983 AND 1984 WERE MADE AFTER THE LAPSE OF THE PRESCRIPTIVE PERIOD. III. WHETHER THE PERIOD FOR THE COLLECTION OF TAXES FOR TAXABLE YEARS 1983, 1984, AND 1985 HAS ALREADY PRESCRIBED. IV. WHETHER THE ASSESSMENTS ARE CORRECT.4

As to the first issue, petitioner maintains that the assessments only became final on November 9, 1990, when the CIR denied the request for reconsideration. Consequently, the CTA had jurisdiction over the appeal filed by the petitioner on December 5, 1990. Furthermore, the CTA resolved that the assessments became final after thirty days from receipt of demand letters by the petitioner, without the latter interposing a reconsideration. The pertinent provision of the National Internal Revenue Code of 1977 (NIRC 1977), concerning the period within which to file a protest before the CIR, reads: Sec. 270. Protesting of assessment. When the Commissioner of Internal Revenue or his duly authorized representative finds that proper taxes should be assessed, he shall first notify the taxpayer of his findings. Within a period to be prescribed by implementing regulations, the taxpayer shall be required to respond to said notice. If the taxpayer fails to respond, the Commissioner shall issue an assessment based on his findings. Such assessment may be protested administratively by filing a request for reconsideration or reinvestigation in such form and manner as may be prescribed by the implementing regulations within thirty (30) days from receipt of the assessment; otherwise, the assessment shall become final, and unappealable. If the protest is denied in whole or in part, the individual, association or corporation adversely affected by the decision on the protest may appeal to the Court of Tax Appeals within thirty (30) days from receipt of the said decision; otherwise, the decision shall become final, executory and demandable. We note that indeed on December 10, 1987, petitioner received the BIR's assessment notices. On January 12, 1988, petitioner protested the 1983 and 1984 assessments and requested for a reinvestigation. From December 10, 1987 to January 12, 1988, thirty-three days had lapsed. Thereafter petitioner may no longer dispute the correctness of the assessments. Hence, in our view, the CTA correctly dismissed the appeal for lack of jurisdiction. On the second issue, petitioner argues that the government's right to assess and collect the 1983, 1984 and 1985 taxes had already prescribed. Relying on Batas Pambansa (BP) Blg. 700, which reduced the period of limitation for assessment and collection of internal revenue taxes from five to three years, petitioner asserts that the government was barred from reviewing the 1983 tax starting December 10, 1987, the expiry date of the three-year limit. Petitioner insists that the reckoning period of prescription should start from the date when the quarterly percentage taxes were paid and not when the Final Annual Percentage Tax Return for the year was filed. Moreover, he denies having received the 1985 tax assessment. Petitioner's contentions lack merit. Sections one and three of BP 700, "An Act Amending Sections 318 and 319 of the National Internal Revenue Code, which reduced the period of limitation for assessment and collection of internal revenue taxes from five to three years," provides:

Sec. 1. Section 318 of the National Internal Revenue Code, as amended, is hereby amended to read as follows: Sec. 318. Period of limitation upon assessment and collection. Except as provided in the succeeding sections, internal revenue taxes shall be assessed within three years after the last day prescribed by law for the filing of the return, and no proceeding in court without assessment for the collection of such taxes shall be begun after the expiration of such period: Provided, That in a case where a return is filed beyond the period prescribed by law, the three-year period shall be counted from the day the return was filed. For the purposes of this section, a return filed before the last day prescribed by law for the filing thereof shall be considered as filed on such last day. xxx xxx xxx

Sec. 3. The period of limitation herein prescribed shall apply to assessments of internal revenue taxes beginning taxable year 1984. B.P. 700 was approved on April 5, 1984. The three-year prescriptive period for assessment and collection of revenue taxes applied to taxes paid beginning 1984. Clearly, the tax assessment made on December 10, 1987, for the year 1983 was still covered by the five-year statutory prescriptive period. This rule was emphasized in Revenue Memorandum Circular (RMC) No. 33-84, published on November 12, 1984, which defined the salient features of the application of BP 700, to wit: B. Effectivity of Prescriptive Periods of Assessment and Collection 1 Assessment made on or after April 5, 1984 (date, of approval of BP 700) will still be governed by the original five-year period if the taxes assessed thereby cover taxable years prior to January 1, 1984. (emphasis supplied) Corollarily, assessments made before April 5, 1984 shall still be governed by the original five-year period. However, assessments made on or April 5, 1984 covering taxable years beginning January 1, 1984 shall be under the new three-year period. Should the three-year limitation be reckoned at the time of the quarterly payment of contractor's tax or at the due date of the final annual tax? Sec. 2 of Revenue Regulation No. 6-81, states: Sec. 2. Percentage tax. In general, unless otherwise specifically provided in the Tax Code, every person conducting business on which a percentage tax is imposed under Chapter II Title V of the Tax Code must render quarterly declaration on cumulative basis

of the amount of his sales, receipts or earnings or gross value of output actually removed from the factory or near warehouse, compute and pay the tax due thereon. (a) Quarterly Percentage Return. For each of the first three quarters of the taxable year, the tax so computed shall be decreased by the amount of tax previously paid and by the sum of the tax credits allowed under this Title for the preceding current quarters. The tax due shall be paid not later than twenty (20) days following the close of each of the first three quarters of the taxable year. (b) Final Annual Percentage Tax Return On or before the twentieth day of the second month following the close of the taxable year, a final percentage tax return shall be filed under BIR Form No. __ covering the entire taxable year. If the sum of the total quarterly percentage tax payments made for the first three quarters and total tax credit allowable for the taxable year are not equal to the total tax due on the entire gross sales, receipts or earnings or gross value of the output for that taxable year, the taxpayer shall either: (1) Pay the tax still due; or (2) Credit to the extent allowable under this Title, the amount of excess tax credits shown in the final adjustment return against the quarterly percentage tax liabilities for the succeeding taxable quarters. Only recently in G.R. No. 115712, Commission of Internal Revenue vs. Court of Appeals, February 25, 1999, we held, that the three-year prescriptive period of tax assessment of contractor's tax should be computed at the time of the filing of the "final annual percentage tax return,"5 when it can be finally ascertained if the taxpayer still has an unpaid tax, and not from the tentative quarterly payments. Turning now to petitioner's denial that he received the 1985 assessment, we agree with the factual findings of the CTA that the assessment letter may be presumed to have been received by petitioner. The CTA found as follows: The 1985 assessment which petitioner denied as having been received was negated when the respondent introduced documentary evidence showing that it was mailed by registered mail. It was further buttressed by the testimony of witness Mr. Arnold C. Larroza, Chief Administrative Branch Mailing Section, Rev. Region No. 4B-1, Quezon City that the 1983, 1984 and 1985 assessments were placed in one envelope when it was mailed by registered mail. Presumably, it was received in the regular course of the mail. . . . The facts to be proved to raise this presumption are (a) that the letter was properly addressed with postage prepaid; and (b) that it was mailed. Once these facts are proved, the presumption is that the letter was received by the addressee as soon as it could have

been transmitted to him in the ordinary course of the mails. Such being the case, this Court cannot be made to believe that the 1985 assessment which incidentally has a substantially greater amount involved, was not received by the petitioner. Hence, the same assessment is also considered final and unappealable for failure of the petitioner to protest the same within the reglementary period provided by law.6 In reviewing administrative decisions, the reviewing court cannot re-examine the factual basis and sufficiency of the evidence.7 The findings of fact must be respected, so long as they are supported by substantial evidence.8 As a subsidiary defense, petitioner interposes the third issue claiming that since the CIR failed, until now, to commence the collection of the 1983, 1984, and 1985 deficiency tax, the right to collect had, likewise, prescribed. Petitioner urges us to consider that for the government's failure to institute collection remedies either by judicial action or by distraint and levy, the right to collect the same has prescribed pursuant to Section 219 of the NIRC. Note, however, that Section 271 of the 1986 Tax Code provides for the suspension of running of the statute of limitation of tax collection, as follows: Sec. 271. Suspension of running of statute. The running of the statute of limitations provided in Sections 268 and 269 on the making of assessment and the beginning of distraint or levy or a proceeding in court for collection, in respect of any deficiency, shall be suspended for the period during which the Commissioner is prohibited from making the assessment or beginning distraint or levy or a proceeding in court and for sixty days thereafter; when the taxpayer request for a reinvestigation which is granted by the Commissioner; when the taxpayer cannot be located in the address given by him in the return filed upon which a tax is being assessed or collected: Provided, That, if the taxpayer informs the Commissioner of any change in address, the running of the statute of limitation will not be suspended; when the warrant of distraint and levy is duly served upon the taxpayer, his authorized representative, or a member of his household with sufficient discretion, and no property could be located; and when the taxpayer is out of the Philippines. (Emphasis supplied.) In the instant case, PSI filed a petition before the CTA to prevent the collection of the assessed deficiency tax. When the CTA dismissed the case, petitioner elevated the case before us, hoping for a review in its favor. The actions taken by the petitioner before the CTA and now before us, suspended the running of the statute of limitation. In the old case of Republic of the Philippines vs. Ker and Company, Ltd., 9 we held: Under Section 333 (renumbered to 271 during the instant case) of the Tax Code the running of the prescriptive period to collect deficiency taxes shall be suspended for the period during which the Commissioner of Internal Revenue is prohibited from beginning a distraint and levy or instituting a proceeding in court, and for sixty days thereafter. In the case at bar, the pendency of the taxpayer's appeal in the Court of Tax Appeals and in the Supreme Court had the effect of temporarily staying the hands of the said Commissioner. If the taxpayer's stand that the pendency of the appeal did not stop the running of the period because the Court of Tax Appeals did not have jurisdiction over the

case of taxes is upheld, taxpayers would be encouraged to delay the payment of taxes in the hope of ultimately avoiding the same. Under the circumstances, the running of the prescriptive period was suspended. 10 Finally, petitioner contends that the assessments made by the respondent CIR were erroneous because they included in the gross receipts subject to the contractor's tax the salaries of the security guards and the employer's share in the SSS, SIF and Medicare. Petitioner claims that it did not benefit from those amounts earmarked for other persons or institutions, hence, they must not be taxable. Contractor's tax on gross receipts imposed on business agents including private detective watchman agencies, 11 was a tax on the sale of services or labor, imposed on the exercise of a privilege. 12 The term "gross receipts" means all amounts received by the prime or principal contractor as the total price, undiminished by the amount paid to the subcontractor under a subcontract arrangement. 13 Hence, gross receipts could not be diminished by employer's SSS, SIF and Medicare contributions. 14 Furthermore, it has been consistently ruled by the BIR that the salaries paid to security guards should form part of the gross receipts, subject to tax, to wit: . . . This Office has consistently ruled that salaries of security guards form part of the taxable gross receipts of a security agency for purposes of the 4% [formerly 3%] contractors tax under Section 205 of the Tax Code, as amended. The reason is that the salaries of the security guards are actually the liability of the agency and that the guards are considered their employees; hence, for percentage tax purposes, the salaries of the security guards are includible in its gross receipts. (BIR Ruling No.271-81 citing BIR Ruling No. 69-002). 15 These rulings were made by the CIR in the exercise of his power to "make judgments or opinions in connection with the implementation of the provisions of the internal revenue code." The opinions and rulings of officials of the government called upon to execute or implement administrative laws, command respect and weight. 16 We see no compelling reason in this case to rule otherwise. WHEREFORE, the assailed decision of the Court of Appeals, in CA- G.R. SP 31825, is AFFIRMED. Costs against petitioner.

G.R. Nos. 172045-46

June 16, 2009

COMMISSIONER OF INTERNAL REVENUE, Petitioner, vs. FIRST EXPRESS PAWNSHOP COMPANY, INC., Respondent. DECISION CARPIO, J.:

The Case The Commissioner of Internal Revenue (petitioner) filed this Petition for Review1 to reverse the Court of Tax Appeals Decision2 dated 24 March 2006 in the consolidated cases of C.T.A. EB Nos. 60 and 62. In the assailed decision, the Court of Tax Appeals (CTA) En Banc partially reconsidered the CTA First Divisions Decision3 dated 24 September 2004. The Facts On 28 December 2001, petitioner, through Acting Regional Director Ruperto P. Somera of Revenue Region 6 Manila, issued the following assessment notices against First Express Pawnshop Company, Inc. (respondent): a. Assessment No. 31-1-984 for deficiency income tax of P20,712.58 with compromise penalty of P3,000; b. Assessment No. 31-14-000053-985 for deficiency value-added tax (VAT) of P601,220.18 with compromise penalty of P16,000; c. Assessment No. 31-14-000053-986 for deficiency documentary stamp tax (DST) of P12,328.45 on deposit on subscription with compromise penalty of P2,000; and d. Assessment No. 31-1-000053-987 for deficiency DST of P62,128.87 on pawn tickets with compromise penalty of P8,500. Respondent received the assessment notices on 3 January 2002. On 1 February 2002, respondent filed its written protest on the above assessments. Since petitioner did not act on the protest during the 180-day period,8 respondent filed a petition before the CTA on 28 August 2002.9 Respondent contended that petitioner did not consider the supporting documents on the interest expenses and donations which resulted in the deficiency income tax.10 Respondent maintained that pawnshops are not lending investors whose services are subject to VAT, hence it was not liable for deficiency VAT.11 Respondent also alleged that no deficiency DST was due because Section 18012 of the National Internal Revenue Code (Tax Code) does not cover any document or transaction which relates to respondent. Respondent also argued that the issuance of a pawn ticket did not constitute a pledge under Section 19513 of the Tax Code.14 In its Answer filed before the CTA, petitioner alleged that the assessment was valid and correct and the taxpayer had the burden of proof to impugn its validity or correctness. Petitioner maintained that respondent is subject to 10% VAT based on its gross receipts pursuant to Republic Act No. 7716, or the Expanded Value-Added Tax Law (EVAT). Petitioner also cited BIR Ruling No. 221-91 which provides that pawnshop tickets are subject to DST. 15 On 1 July 2003, respondent paid P27,744.88 as deficiency income tax inclusive of interest.16 After trial on the merits, the CTA First Division ruled, thus:

IN VIEW OF ALL THE FOREGOING, the instant petition is hereby PARTIALLY GRANTED. Assessment No. 31-1-000053-98 for deficiency documentary stamp tax in the amount of Sixty-Two Thousand One Hundred Twenty-Eight Pesos and 87/100 (P62,128.87) and Assessment No. 31-14-000053-98 for deficiency documentary stamp tax on deposits on subscription in the amount of Twelve Thousand Three Hundred Twenty-Eight Pesos and 45/100 (P12,328.45) are CANCELLED and SET ASIDE. However, Assessment No. 31-14-000053-98 is hereby AFFIRMED except the imposition of compromise penalty in the absence of showing that petitioner consented thereto (UST vs. Collector, 104 SCRA 1062; Exquisite Pawnshop Jewelry, Inc. vs. Jaime B. Santiago, et al., supra). Accordingly petitioner is ORDERED to PAY the deficiency value added tax in the amount of Six Hundred One Thousand Two Hundred Twenty Pesos and 18/100 (P601,220.18) inclusive of deficiency interest for the year 1998. In addition, petitioner is ORDERED to PAY 25% surcharge and 20% delinquency interest per annum from February 12, 2002 until fully paid pursuant to Sections 248 and 249 of the 1997 Tax Code. SO ORDERED.17 (Boldfacing in the original) Both parties filed their Motions for Reconsideration which were denied by the CTA First Division for lack of merit. Thereafter, both parties filed their respective Petitions for Review under Section 11 of Republic Act No. 9282 (RA 9282) with the CTA En Banc.18 On 24 March 2006, the CTA En Banc promulgated a Decision affirming respondents liability to pay the VAT and ordering it to pay DST on its pawnshop tickets. However, the CTA En Banc found that respondents deposit on subscription was not subject to DST.19 Aggrieved by the CTA En Bancs Decision which ruled that respondents deposit on subscription was not subject to DST, petitioner elevated the case before this Court. The Ruling of the Court of Tax Appeals On the taxability of deposit on subscription, the CTA, citing First Southern Philippines Enterprises, Inc. v. Commissioner of Internal Revenue,20 pointed out that deposit on subscription is not subject to DST in the absence of proof that an equivalent amount of shares was subscribed or issued in consideration for the deposit. Expressed otherwise, deposit on stock subscription is not subject to DST if: (1) there is no agreement to subscribe; (2) there are no shares issued or any additional subscription in the restructuring plan; and (3) there is no proof that the issued shares can be considered as issued certificates of stock.21 The CTA ruled that Section 17522 of the Tax Code contemplates a subscription agreement. The CTA explained that there can be subscription only with reference to shares of stock which have been unissued, in the following cases: (a) the original issuance from authorized capital stock at the time of incorporation; (b) the opening, during the life of the corporation, of the portion of the original authorized capital stock previously unissued; or (c) the increase of authorized capital stock achieved through a formal amendment of the articles of incorporation and registration of the articles of incorporation with the Securities and Exchange Commission.23

The CTA held that in this case, there was no subscription or any contract for the acquisition of unissued stock for P800,000 in the taxable year assessed. The General Information Sheet (GIS) of respondent showed only a capital structure of P500,000 as Subscribed Capital Stock and P250,000 as Paid-up Capital Stock and did not include the assessed amount. Mere reliance on the presumption that the assessment was correct and done in good faith was unavailing vis--vis the evidence presented by respondent. Thus, the CTA ruled that the assessment for deficiency DST on deposit on subscription has not become final.24 The Issue Petitioner submits this sole issue for our consideration: whether the CTA erred on a question of law in disregarding the rule on finality of assessments prescribed under Section 228 of the Tax Code. Corollarily, petitioner raises the issue on whether respondent is liable to pay P12,328.45 as DST on deposit on subscription of capital stock. The Ruling of the Court Petitioner contends that the CTA erred in disregarding the rule on the finality of assessments prescribed under Section 228 of the Tax Code.25 Petitioner asserts that even if respondent filed a protest, it did not offer evidence to prove its claim that the deposit on subscription was an "advance" made by respondents stockholders.26 Petitioner alleges that respondents failure to submit supporting documents within 60 days from the filing of its protest as required under Section 228 of the Tax Code caused the assessment of P12,328.45 for deposit on subscription to become final and unassailable.27 Petitioner alleges that revenue officers are afforded the presumption of regularity in the performance of their official functions, since they have the distinct opportunity, aside from competence, to peruse records of the assessments. Petitioner invokes the principle that by reason of the expertise of administrative agencies over matters falling under their jurisdiction, they are in a better position to pass judgment thereon; thus, their findings of fact are generally accorded great respect, if not finality, by the courts. Hence, without the supporting documents to establish the non-inclusion from DST of the deposit on subscription, petitioners assessment pursuant to Section 228 of the Tax Code had become final and unassailable.28 Respondent, citing Standard Chartered Bank-Philippine Branches v. Commissioner of Internal Revenue,29 asserts that the submission of all the relevant supporting documents within the 60-day period from filing of the protest is directory. Respondent claims that petitioner requested for additional documents in petitioners letter dated 12 March 2002, to wit: (1) loan agreement from lender banks; (2) official receipts of interest payments issued to respondent; (3) documentary evidence to substantiate donations claimed; and (4) proof of payment of DST on subscription.30 It must be noted that the only document requested in connection with respondents DST assessment on deposit on subscription is proof of DST payment. However, respondent could not produce any proof of DST payment because it was not required to pay the same under the law considering that the deposit on subscription was an advance made by its stockholders for future subscription, and no stock certificates were

issued.31 Respondent insists that petitioner could have issued a subpoena requiring respondent to submit other documents to determine if the latter is liable for DST on deposit on subscription pursuant to Section 5(c) of the Tax Code.32 Respondent argues that deposit on future subscription is not subject to DST under Section 175 of the Tax Code. Respondent explains: It must be noted that deposits on subscription represent advances made by the stockholders and are in the nature of liabilities for which stocks may be issued in the future. Absent any express agreement between the stockholders and petitioner to convert said advances/deposits to capital stock, either through a subscription agreement or any other document, these deposits remain as liabilities owed by respondent to its stockholders. For these deposits to be subject to DST, it is necessary that a conversion/subscription agreement be made by First Express and its stockholders. Absent such conversion, no DST can be imposed on said deposits under Section 175 of the Tax Code.33 (Underscoring in the original) Respondent contends that by presenting its GIS and financial statements, it had already sufficiently proved that the amount sought to be taxed is deposit on future subscription, which is not subject to DST.34 Respondent claims that it cannot be required to submit proof of DST payment on subscription because such payment is non-existent. Thus, the burden of proving that there was an agreement to subscribe and that certificates of stock were issued for the deposit on subscription rests on petitioner and his examiners. Respondent states that absent any proof, the deficiency assessment has no basis and should be cancelled.35 On the Taxability of Deposit on Stock Subscription DST is a tax on documents, instruments, loan agreements, and papers evidencing the acceptance, assignment, sale or transfer of an obligation, right or property incident thereto. DST is actually an excise tax because it is imposed on the transaction rather than on the document.36 DST is also levied on the exercise by persons of certain privileges conferred by law for the creation, revision, or termination of specific legal relationships through the execution of specific instruments.37 The Tax Code provisions on DST relating to shares or certificates of stock state: Section 175. Stamp Tax on Original Issue of Shares of Stock. - On every original issue, whether on organization, reorganization or for any lawful purpose, of shares of stock by any association, company or corporation, there shall be collected a documentary stamp tax of Two pesos (P2.00) on each Two hundred pesos (P200), or fractional part thereof, of the par value, of such shares of stock: Provided, That in the case of the original issue of shares of stock without par value the amount of the documentary stamp tax herein prescribed shall be based upon the actual consideration for the issuance of such shares of stock: Provided, further, That in the case of stock dividends, on the actual value represented by each share.38 Section 176. Stamp Tax on Sales, Agreements to Sell, Memoranda of Sales, Deliveries or Transfer of Due-bills, Certificates of Obligation, or Shares or Certificates of Stock. - On all sales, or agreements to sell, or memoranda of sales, or deliveries, or transfer of due-bills, certificates of obligation, or shares or certificates of stock in any association, company or

corporation, or transfer of such securities by assignment in blank, or by delivery, or by any paper or agreement, or memorandum or other evidences of transfer or sale whether entitling the holder in any manner to the benefit of such due-bills, certificates of obligation or stock, or to secure the future payment of money, or for the future transfer of any due-bill, certificate of obligation or stock, there shall be collected a documentary stamp tax of One peso and fifty centavos (P1.50) on each Two hundred pesos (P200), or fractional part thereof, of the par value of such due-bill, certificate of obligation or stock: Provided, That only one tax shall be collected on each sale or transfer of stock or securities from one person to another, regardless of whether or not a certificate of stock or obligation is issued, indorsed, or delivered in pursuance of such sale or transfer: And provided, further, That in the case of stock without par value the amount of the documentary stamp tax herein prescribed shall be equivalent to twenty-five percent (25%) of the documentary stamp tax paid upon the original issue of said stock.39 In Section 175 of the Tax Code, DST is imposed on the original issue of shares of stock. The DST, as an excise tax, is levied upon the privilege, the opportunity and the facility of issuing shares of stock. In Commissioner of Internal Revenue v. Construction Resources of Asia, Inc.,40 this Court explained that the DST attaches upon acceptance of the stockholders subscription in the corporations capital stock regardless of actual or constructive delivery of the certificates of stock. Citing Philippine Consolidated Coconut Ind., Inc. v. Collector of Internal Revenue,41 the Court held: The documentary stamp tax under this provision of the law may be levied only once, that is upon the original issue of the certificate. The crucial point therefore, in the case before Us is the proper interpretation of the word issue. In other words, when is the certificate of stock deemed issued for the purpose of imposing the documentary stamp tax? Is it at the time the certificates of stock are printed, at the time they are filled up (in whose name the stocks represented in the certificate appear as certified by the proper officials of the corporation), at the time they are released by the corporation, or at the time they are in the possession (actual or constructive) of the stockholders owning them? xxx Ordinarily, when a corporation issues a certificate of stock (representing the ownership of stocks in the corporation to fully paid subscription) the certificate of stock can be utilized for the exercise of the attributes of ownership over the stocks mentioned on its face. The stocks can be alienated; the dividends or fruits derived therefrom can be enjoyed, and they can be conveyed, pledged or encumbered. The certificate as issued by the corporation, irrespective of whether or not it is in the actual or constructive possession of the stockholder, is considered issued because it is with value and hence the documentary stamp tax must be paid as imposed by Section 212 of the National Internal Revenue Code, as amended. In Section 176 of the Tax Code, DST is imposed on the sales, agreements to sell, memoranda of sales, deliveries or transfer of shares or certificates of stock in any association, company, or corporation, or transfer of such securities by assignment in blank, or by delivery, or by any paper or agreement, or memorandum or other evidences of transfer or sale whether entitling the holder in any manner to the benefit of such certificates of stock, or to secure the future payment of

money, or for the future transfer of certificates of stock. In Compagnie Financiere Sucres et Denrees v. Commissioner of Internal Revenue, this Court held that under Section 176 of the Tax Code, sales to secure the future transfer of due-bills, certificates of obligation or certificates of stock are subject to documentary stamp tax.42 Revenue Memorandum Order No. 08-98 (RMO 08-98) provides the guidelines on the corporate stock documentary stamp tax program. RMO 08-98 states that: 1. All existing corporations shall file the Corporation Stock DST Declaration, and the DST Return, if applicable when DST is still due on the subscribed share issued by the corporation, on or before the tenth day of the month following publication of this Order. xxx 3. All existing corporations with authorization for increased capital stock shall file their Corporate Stock DST Declaration, together with the DST Return, if applicable when DST is due on subscriptions made after the authorization, on or before the tenth day of the month following the date of authorization. (Boldfacing supplied) RMO 08-98, reiterating Revenue Memorandum Circular No. 47-97 (RMC 47-97), also states that what is being taxed is the privilege of issuing shares of stock, and, therefore, the taxes accrue at the time the shares are issued. RMC 47-97 also defines issuance as the point in which the stockholder acquires and may exercise attributes of ownership over the stocks. As pointed out by the CTA, Sections 175 and 176 of the Tax Code contemplate a subscription agreement in order for a taxpayer to be liable to pay the DST. A subscription contract is defined as any contract for the acquisition of unissued stocks in an existing corporation or a corporation still to be formed.43 A stock subscription is a contract by which the subscriber agrees to take a certain number of shares of the capital stock of a corporation, paying for the same or expressly or impliedly promising to pay for the same.44 In this case, respondents Stockholders Equity section of its Balance Sheet as of 31 December 199845 shows: Stockholders Equity Authorized Capital Stock Paid-up Capital Stock Deposit on Subscription Retained Earnings Net Income Total 1998 P 2,000,000.00 250,000.00 800,000.00 62,820.34 (858,498.38) P 254,321.96 209,607.20 (146,786.86) P 312,820.34 1997 P 2,000,000.00 250,000.00

The GIS submitted to the Securities and Exchange Commission on 31 March 1999 shows the following Capital Structure:46 B. Financial Profile 1. Capital Structure : AUTHORIZED - P2,000,000.00 SUBSCRIBED PAID-UP - 500,000.00 - 250,000.00

These entries were explained by Miguel Rosario, Jr. (Rosario), respondents external auditor, during the hearing before the CTA on 11 June 2003. Rosario testified in this wise: Atty. Napiza Q. Mr. Rosario, I refer you to the balance sheet of First Express for the year 1998 particularly the entry of deposit on subscription in the amount of P800 thousand, will you please tell us what is (sic) this entry represents? Mr. Rosario Jr. A. This amount of P800 thousand represents the case given by the stockholders to the company but does not necessarily made (sic) payment to subscribed portion. Atty. Napiza Q. What is (sic) that payment stands for? Mr. Rosario Jr. A. This payment stands as (sic) for the deposit for future subscription. Atty. Napiza Q. Would you know if First Express issued corresponding shares pertinent to the amount being deposited? Mr. Rosario Jr. A. No. Atty. Napiza

Q. What do you mean by no? Did they or they did not? Mr. Rosario Jr. A. They did not issue any shares because that is not the payment of subscription. That is just a mere deposit. Atty. Napiza Q. Would you know, Mr. Rosario, how much is the Subscribed Capital of First Express Pawnshop? Mr. Rosario Jr. A. The Subscribed Capital of First Express Pawnshop Company, Inc. for the year 1998 is P500 thousand. Atty. Napiza Q. How about the Paid Up Capital? Mr. Rosario Jr. A. The Paid Up Capital is P250 thousand. Atty. Napiza Q. Are (sic) all those figures appear in the balance sheet? Mr. Rosario Jr. A. The Paid Up Capital appeared here but the Subscribed Portion was not stated. (Boldfacing supplied) Based on Rosarios testimony and respondents financial statements as of 1998, there was no agreement to subscribe to the unissued shares. Here, the deposit on stock subscription refers to an amount of money received by the corporation as a deposit with the possibility of applying the same as payment for the future issuance of capital stock.47 In Commissioner of Internal Revenue v. Construction Resources of Asia, Inc.,48 we held: We are firmly convinced that the Government stands to lose nothing in imposing the documentary stamp tax only on those stock certificates duly issued, or wherein the stockholders can freely exercise the attributes of ownership and with value at the time they are originally issued. As regards those certificates of stocks temporarily subject to suspensive conditions they shall be liable for said tax only when released from said conditions, for then and only

then shall they truly acquire any practical value for their owners.lavvphil (Boldfacing supplied) Clearly, the deposit on stock subscription as reflected in respondents Balance Sheet as of 1998 is not a subscription agreement subject to the payment of DST. There is no P800,000 worth of subscribed capital stock that is reflected in respondents GIS. The deposit on stock subscription is merely an amount of money received by a corporation with a view of applying the same as payment for additional issuance of shares in the future, an event which may or may not happen. The person making a deposit on stock subscription does not have the standing of a stockholder and he is not entitled to dividends, voting rights or other prerogatives and attributes of a stockholder. Hence, respondent is not liable for the payment of DST on its deposit on subscription for the reason that there is yet no subscription that creates rights and obligations between the subscriber and the corporation. On the Finality of Assessment as Prescribed under Section 228 of the Tax Code Section 228 of the Tax Code provides: SEC. 228. Protesting of Assessment. - When the Commissioner or his duly authorized representative finds that proper taxes should be assessed, he shall first notify the taxpayer of his findings: Provided, however, That a preassessment notice shall not be required in the following cases: (a) When the finding for any deficiency tax is the result of mathematical error in the computation of the tax as appearing on the face of the return; or (b) When a discrepancy has been determined between the tax withheld and the amount actually remitted by the withholding agent; or (c) When a taxpayer who opted to claim a refund or tax credit of excess creditable withholding tax for a taxable period was determined to have carried over and automatically applied the same amount claimed against the estimated tax liabilities for the taxable quarter or quarters of the succeeding taxable year; or (d) When the excise tax due on excisable articles has not been paid; or (e) When an article locally purchased or imported by an exempt person, such as, but not limited to, vehicles, capital equipment, machineries and spare parts, has been sold, traded or transferred to non-exempt persons. The taxpayer shall be informed in writing of the law and the facts on which the assessment is made; otherwise, the assessment shall be void.

Within a period to be prescribed by implementing rules and regulations, the taxpayer shall be required to respond to said notice. If the taxpayer fails to respond, the Commissioner or his duly authorized representative shall issue an assessment based on his findings. Such assessment may be protested administratively by filing a request for reconsideration or reinvestigation within thirty (30) days from receipt of the assessment in such form and manner as may be prescribed by implementing rules and regulations. Within sixty (60) days from filing of the protest, all relevant supporting documents shall have been submitted; otherwise, the assessment shall become final. If the protest is denied in whole or in part, or is not acted upon within one hundred eighty (180) days from submission of documents, the taxpayer adversely affected by the decision or inaction may appeal to the Court of Tax Appeals within thirty (30) days from receipt of the said decision, or from the lapse of the one hundred eighty (180)-day period; otherwise, the decision shall become final, executory and demandable. (Boldfacing supplied) Section 228 of the Tax Code49 provides the remedy to dispute a tax assessment within a certain period of time. It states that an assessment may be protested by filing a request for reconsideration or reinvestigation within 30 days from receipt of the assessment by the taxpayer. Within 60 days from filing of the protest, all relevant supporting documents shall have been submitted; otherwise, the assessment shall become final. In this case, respondent received the tax assessment on 3 January 2002 and it had until 2 February 2002 to submit its protest. On 1 February 2002, respondent submitted its protest and attached the GIS and Balance Sheet as of 31 December 1998. Respondent explained that it received P800,000 as a deposit with the possibility of applying the same as payment for the future issuance of capital stock. Within 60 days from the filing of protest or until 2 April 2002, respondent should submit relevant supporting documents. Respondent, having submitted the supporting documents together with its protest, did not present additional documents anymore. In a letter dated 12 March 2002, petitioner requested respondent to present proof of payment of DST on subscription. In a letter-reply, respondent stated that it could not produce any proof of DST payment because it was not required to pay DST under the law considering that the deposit on subscription was an advance made by its stockholders for future subscription, and no stock certificates were issued. Since respondent has not allegedly submitted any relevant supporting documents, petitioner now claims that the assessment has become final, executory and demandable, hence, unappealable. We reject petitioners view that the assessment has become final and unappealable. It cannot be said that respondent failed to submit relevant supporting documents that would render the assessment final because when respondent submitted its protest, respondent attached the GIS and Balance Sheet. Further, petitioner cannot insist on the submission of proof of DST payment

because such document does not exist as respondent claims that it is not liable to pay, and has not paid, the DST on the deposit on subscription. The term "relevant supporting documents" should be understood as those documents necessary to support the legal basis in disputing a tax assessment as determined by the taxpayer. The BIR can only inform the taxpayer to submit additional documents. The BIR cannot demand what type of supporting documents should be submitted. Otherwise, a taxpayer will be at the mercy of the BIR, which may require the production of documents that a taxpayer cannot submit.1awphi1 After respondent submitted its letter-reply stating that it could not comply with the presentation of the proof of DST payment, no reply was received from petitioner. Section 228 states that if the protest is not acted upon within 180 days from submission of documents, the taxpayer adversely affected by the inaction may appeal to the CTA within 30 days from the lapse of the 180-day period. Respondent, having submitted its supporting documents on the same day the protest was filed, had until 31 July 2002 to wait for petitioners reply to its protest. On 28 August 2002 or within 30 days after the lapse of the 180-day period counted from the filing of the protest as the supporting documents were simultaneously filed, respondent filed a petition before the CTA. Respondent has complied with the requisites in disputing an assessment pursuant to Section 228 of the Tax Code. Hence, the tax assessment cannot be considered as final, executory and demandable. Further, respondents deposit on subscription is not subject to the payment of DST. Consequently, respondent is not liable to pay the deficiency DST of P12,328.45. Wherefore, we DENY the petition. We AFFIRM the Court of Tax Appeals Decision dated 24 March 2006 in the consolidated cases of C.T.A. EB Nos. 60 and 62. G.R. No. 139736 October 17, 2005 BANK OF THE PHILIPPINE ISLANDS, Petitioner, vs. COMMISSIONER OF INTERNAL REVENUE, Respondent. DECISION CHICO-NAZARIO, J.: This Petition for Review on Certiorari, under Rule 45 of the 1997 Rules of Civil Procedure, assails the Decision of the Court of Appeals in CA-G.R. SP No. 51271, dated 11 August 1999,1 which reversed and set aside the Decision of the Court of Tax Appeals (CTA), dated 02 February 1999,2 and which reinstated Assessment No. FAS-5-85-89-002054 requiring petitioner Bank of the Philippine Islands (BPI) to pay the amount of P28,020.00 as deficiency documentary stamp tax (DST) for the taxable year 1985, inclusive of the compromise penalty. There is hardly any controversy as to the factual antecedents of this Petition.

Petitioner BPI is a commercial banking corporation organized and existing under the laws of the Philippines. On two separate occasions, particularly on 06 June 1985 and 14 June 1985, it sold United States (US) $500,000.00 to the Central Bank of the Philippines (Central Bank), for the total sales amount of US$1,000,000.00. On 10 October 1989, the Bureau of Internal Revenue (BIR) issued Assessment No. FAS-5-8589-002054,3 finding petitioner BPI liable for deficiency DST on its afore-mentioned sales of foreign bills of exchange to the Central Bank, computed as follows 1985 Deficiency Documentary Stamp Tax Foreign Bills of Exchange.. P 18,480,000.00 Tax Due Thereon: P18,480,000.00 x P0.30 (Sec. 182 NIRC). P200.00 Add: Suggested compromise penalty. TOTAL AMOUNT DUE AND COLLECTIBLE.

27,720.00

300.00 P 28,020.00

Petitioner BPI received the Assessment, together with the attached Assessment Notice,4 on 20 October 1989. Petitioner BPI, through its counsel, protested the Assessment in a letter dated 16 November 1989, and filed with the BIR on 17 November 1989. The said protest letter is reproduced in full below November 16, 1989 The Commissioner of Internal Revenue Quezon City Attention of: Mr. Pedro C. Aguillon Asst. Commissioner for Collection Sir: On behalf of our client, Bank of the Philippine Islands (BPI), we have the honor to protest your assessment against it for deficiency documentary stamp tax for the year 1985 in the amount of P28,020.00, arising from its sale to the Central Bank of U.S. $500,000.00 on June 6, 1985 and another U.S. $500,000.00 on June 14, 1985. 1. Under established market practice, the documentary stamp tax on telegraphic transfers or sales of foreign exchange is paid by the buyer. Thus, when BPI sells to any party, the cost of

documentary stamp tax is added to the total price or charge to the buyer and the seller affixes the corresponding documentary stamp on the document. Similarly, when the Central Bank sells foreign exchange to BPI, it charges BPI for the cost of the documentary stamp on the transaction. 2. In the two transactions subject of your assessment, no documentary stamps were affixed because the buyer, Central Bank of the Philippines, was exempt from such tax. And while it is true that under P.D. 1994, a proviso was added to sec. 222 (now sec. 186) of the Tax Code "that whenever one party to a taxable document enjoys exemption from the tax herein imposed, the other party thereto who is not exempt shall be the one directly liable for the tax," this proviso (and the other amendments of P.D. 1994) took effect only on January 1, 1986, according to sec. 49 of P.D. 1994. Hence, the liability for the documentary stamp tax could not be shifted to the seller. In view of the foregoing, we request that the assessment be revoked and cancelled. Very truly yours, PADILLA LAW OFFICE By: (signed) SABINO PADILLA, JR.5 Petitioner BPI did not receive any immediate reply to its protest letter. However, on 15 October 1992, the BIR issued a Warrant of Distraint and/or Levy6 against petitioner BPI for the assessed deficiency DST for taxable year 1985, in the amount of P27,720.00 (excluding the compromise penalty of P300.00). It served the Warrant on petitioner BPI only on 23 October 1992.7 Then again, petitioner BPI did not hear from the BIR until 11 September 1997, when its counsel received a letter, dated 13 August 1997, signed by then BIR Commissioner Liwayway VinzonsChato, denying its "request for reconsideration," and addressing the points raised by petitioner BPI in its protest letter, dated 16 November 1989, thus In reply, please be informed that after a thorough and careful study of the facts of the case as well as the law and jurisprudence pertinent thereto, this Office finds the above argument to be legally untenable. It is admitted that while industry practice or market convention has the force of law between the members of a particular industry, it is not binding with the BIR since it is not a party thereto. The same should, therefore, not be allowed to prejudice the Bureau of its lawful task of collecting revenues necessary to defray the expenses of the government. (Art. 11 in relation to Art. 1306 of the New Civil Code.) Moreover, let it be stated that even before the amendment of Sec. 222 (now Sec. 173) of the Tax Code, as amended, the same was already interpreted to hold that the other party who is not

exempt from the payment of documentary stamp tax liable from the tax. This interpretation was further strengthened by the following BIR Rulings which in substance state: 1. BIR Unnumbered Ruling dated May 30, 1977 "x x x Documentary stamp taxes are payable by either person, signing, issuing, accepting, or transferring the instrument, document or paper. It is now settled that where one party to the instrument is exempt from said taxes, the other party who is not exempt should be liable." 2. BIR Ruling No. 144-84 dated September 3, 1984 "x x x Thus, where one party to the contract is exempt from said tax, the other party, who is not exempt, shall be liable therefore. Accordingly, since A.J.L. Construction Corporation, the other party to the contract and the one assuming the payment of the expenses incidental to the registration in the vendees name of the property sold, is not exempt from said tax, then it is the one liable therefore, pursuant to Sec. 245 (now Sec. 196), in relation to Sec. 222 (now Sec. 173), both of the Tax Code of 1977, as amended." Premised on all the foregoing considerations, your request for reconsideration is hereby DENIED.8 Upon receipt of the above-cited letter from the BIR, petitioner BPI proceeded to file a Petition for Review with the CTA on 10 October 1997;9 to which respondent BIR Commissioner, represented by the Office of the Solicitor General, filed an Answer on 08 December 1997.10 Petitioner BPI raised in its Petition for Review before the CTA, in addition to the arguments presented in its protest letter, dated 16 November 1989, the defense of prescription of the right of respondent BIR Commissioner to enforce collection of the assessed amount. It alleged that respondent BIR Commissioner only had three years to collect on Assessment No. FAS-5-85-89002054, but she waited for seven years and nine months to deny the protest. In her Answer and subsequent Memorandum, respondent BIR Commissioner merely reiterated her position, as stated in her letter to petitioner BPI, dated 13 August 1997, which denied the latters protest; and remained silent as to the expiration of the prescriptive period for collection of the assessed deficiency DST. After due trial, the CTA rendered a Decision on 02 February 1999, in which it identified two primary issues in the controversy between petitioner BPI and respondent BIR Commissioner: (1) whether or not the right of respondent BIR Commissioner to collect from petitioner BPI the alleged deficiency DST for taxable year 1985 had prescribed; and (2) whether or not the sales of US$1,000,000.00 on 06 June 1985 and 14 June 1985 by petitioner BPI to the Central Bank were subject to DST. The CTA answered the first issue in the negative and held that the statute of limitations for respondent BIR Commissioner to collect on the Assessment had not yet prescribed. In resolving the issue of prescription, the CTA reasoned that

In the case of Commissioner of Internal Revenue vs. Wyeth Suaco Laboratories, Inc., G.R. No. 76281, September 30, 1991, 202 SCRA 125, the Supreme Court laid to rest the first issue. It categorically ruled that a "protest" is to be treated as request for reinvestigation or reconsideration and a mere request for reexamination or reinvestigation tolls the prescriptive period of the Commissioner to collect on an assessment. . . ... In the case at bar, there being no dispute that petitioner filed its protest on the subject assessment on November 17, 1989, there can be no conclusion other than that said protest stopped the running of the prescriptive period of the Commissioner to collect. Section 320 (now 223) of the Tax Code, clearly states that a request for reinvestigation which is granted by the Commissioner, shall suspend the prescriptive period to collect. The underscored portion above does not mean that the Commissioner will cancel the subject assessment but should be construed as when the same was entertained by the Commissioner by not issuing any warrant of distraint or levy on the properties of the taxpayer or any action prejudicial to the latter unless and until the request for reinvestigation is finally given due course. Taking into consideration this provision of law and the aforementioned ruling of the Supreme Court in Wyeth Suaco which specifically and categorically states that a protest could be considered as a request for reinvestigation, We rule that prescription has not set in against the government.11 The CTA had likewise resolved the second issue in the negative. Referring to its own decision in an earlier case, Consolidated Bank & Trust Co. v. The Commissioner of Internal Revenue,12 the CTA reached the conclusion that the sales of foreign currency by petitioner BPI to the Central Bank in taxable year 1985 were not subject to DST From the abovementioned decision of this Court, it can be gleaned that the Central Bank, during the period June 11, 1984 to March 9, 1987 enjoyed tax exemption privilege, including the payment of documentary stamp tax (DST) pursuant to Resolution No. 35-85 dated May 3, 1985 of the Fiscal Incentive Review Board. As such, the Central Bank, as buyer of the foreign currency, is exempt from paying the documentary stamp tax for the period above-mentioned. This Court further expounded that said tax exemption of the Central Bank was modified beginning January 1, 1986 when Presidential Decree (P.D.) 1994 took effect. Under this decree, the liability for DST on sales of foreign currency to the Central Bank is shifted to the seller. Applying the above decision to the case at bar, petitioner cannot be held liable for DST on its 1985 sales of foreign currencies to the Central Bank, as the latter who is the purchaser of the subject currencies is the one liable thereof. However, since the Central Bank is exempt from all taxes during 1985 by virtue of Resolution No. 35-85 of the Fiscal Incentive Review Board dated March 3, 1985, neither the petitioner nor the Central Bank is liable for the payment of the documentary stamp tax for the formers 1985 sales of foreign currencies to the latter. This aforecited case of Consolidated Bank vs. Commissioner of Internal Revenue was affirmed by the Court of Appeals in its decision dated March 31, 1995, CA-GR Sp. No. 35930. Said decision was in turn affirmed by the Supreme Court in its resolution denying the petition filed by

Consolidated Bank dated November 20, 1995 with the Supreme Court under Entry of Judgment dated March 1, 1996.13 In sum, the CTA decided that the statute of limitations for respondent BIR Commissioner to collect on Assessment No. FAS-5-85-89-002054 had not yet prescribed; nonetheless, it still ordered the cancellation of the said Assessment because the sales of foreign currency by petitioner BPI to the Central Bank in taxable year 1985 were tax-exempt. Herein respondent BIR Commissioner appealed the Decision of the CTA to the Court of Appeals. In its Decision dated 11 August 1999,14 the Court of Appeals sustained the finding of the CTA on the first issue, that the running of the prescriptive period for collection on Assessment No. FAS-5-85-89-002054 was suspended when herein petitioner BPI filed a protest on 17 November 1989 and, therefore, the prescriptive period for collection on the Assessment had not yet lapsed. In the same Decision, however, the Court of Appeals reversed the CTA on the second issue and basically adopted the position of the respondent BIR Commissioner that the sales of foreign currency by petitioner BPI to the Central Bank in taxable year 1985 were subject to DST. The Court of Appeals, thus, ordered the reinstatement of Assessment No. FAS-5-85-89002054 which required petitioner BPI to pay the amount of P28,020.00 as deficiency DST for taxable year 1985, inclusive of the compromise penalty. Comes now petitioner BPI before this Court in this Petition for Review on Certiorari, seeking resolution of the same two legal issues raised and discussed in the courts below, to reiterate: (1) whether or not the right of respondent BIR Commissioner to collect from petitioner BPI the alleged deficiency DST for taxable year 1985 had prescribed; and (2) whether or not the sales of US$1,000,000.00 on 06 June 1985 and 14 June 1985 by petitioner BPI to the Central Bank were subject to DST. I The efforts of respondent Commissioner to collect on Assessment No. FAS-5-85-89-002054 were already barred by prescription. Anent the question of prescription, this Court disagrees in the Decisions of the CTA and the Court of Appeals, and herein determines the statute of limitations on collection of the deficiency DST in Assessment No. FAS-5-85-89-002054 had already prescribed. The period for the BIR to assess and collect an internal revenue tax is limited to three years by Section 203 of the Tax Code of 1977, as amended,15 which provides that SEC. 203. Period of limitation upon assessment and collection. Except as provided in the succeeding section, internal revenue taxes shall be assessed within three years after the last day prescribed by law for the filing of the return, and no proceeding in court without assessment for the collection of such taxes shall be begun after the expiration of such period: Provided, That in a case where a return is filed beyond the period prescribed by law, the three-year period shall be counted from the day the return was filed. For the purposes of this section, a return filed before the last day prescribed by law for the filing thereof shall be considered as filed on such last day.16

The three-year period of limitations on the assessment and collection of national internal revenue taxes set by Section 203 of the Tax Code of 1977, as amended, can be affected, adjusted, or suspended, in accordance with the following provisions of the same Code SEC. 223. Exceptions as to period of limitation of assessment and collection of taxes. (a) In the case of a false or fraudulent return with intent to evade tax or of failure to file a return, the tax may be assessed, or a proceeding in court for the collection of such tax may be begun without assessment, at any time within ten years after the discovery of the falsity, fraud, or omission: Provided, That in a fraud assessment which has become final and executory, the fact of fraud shall be judicially taken cognizance of in the civil or criminal action for the collection thereof. (b) If before the expiration of the time prescribed in the preceding section for the assessment of the tax, both the Commissioner and the taxpayer have agreed in writing to its assessment after such time the tax may be assessed within the period agreed upon. The period so agreed upon may be extended by subsequent written agreement made before the expiration of the period previously agreed upon. (c) Any internal revenue tax which has been assessed within the period of limitation aboveprescribed may be collected by distraint or levy or by a proceeding in court within three years following the assessment of the tax. (d) Any internal revenue tax which has been assessed within the period agreed upon as provided in paragraph (b) hereinabove may be collected by distraint or levy or by a proceeding in court within the period agreed upon in writing before the expiration of the three-year period. The period so agreed upon may be extended by subsequent written agreements made before the expiration of the period previously agreed upon. (e) Provided, however, That nothing in the immediately preceding section and paragraph (a) hereof shall be construed to authorize the examination and investigation or inquiry into any tax returns filed in accordance with the provisions of any tax amnesty law or decree.17 SEC. 224. Suspension of running of statute. The running of the statute of limitation provided in Section[s] 203 and 223 on the making of assessment and the beginning of distraint or levy or a proceeding in court for collection, in respect of any deficiency, shall be suspended for the period during which the Commissioner is prohibited from making the assessment or beginning distraint or levy or a proceeding in court and for sixty days thereafter; when the taxpayer requests for a reinvestigation which is granted by the Commissioner; when the taxpayer cannot be located in the address given by him in the return filed upon which a tax is being assessed or collected: Provided, That, if the taxpayer informs the Commissioner of any change in address, the running of the statute of limitations will not be suspended; when the warrant of distraint and levy is duly served upon the taxpayer, his authorized representative, or a member of his household with sufficient discretion, and no property could be located; and when the taxpayer is out of the Philippines.18 As enunciated in these statutory provisions, the BIR has three years, counted from the date of actual filing of the return or from the last date prescribed by law for the filing of such return,

whichever comes later, to assess a national internal revenue tax or to begin a court proceeding for the collection thereof without an assessment. In case of a false or fraudulent return with intent to evade tax or the failure to file any return at all, the prescriptive period for assessment of the tax due shall be 10 years from discovery by the BIR of the falsity, fraud, or omission. When the BIR validly issues an assessment, within either the three-year or ten-year period, whichever is appropriate, then the BIR has another three years19 after the assessment within which to collect the national internal revenue tax due thereon by distraint, levy, and/or court proceeding. The assessment of the tax is deemed made and the three-year period for collection of the assessed tax begins to run on the date the assessment notice had been released, mailed or sent by the BIR to the taxpayer.20 In the present Petition, there is no controversy on the timeliness of the issuance of the Assessment, only on the prescription of the period to collect the deficiency DST following its Assessment. While Assessment No. FAS-5-85-89-002054 and its corresponding Assessment Notice were both dated 10 October 1989 and were received by petitioner BPI on 20 October 1989, there was no showing as to when the said Assessment and Assessment Notice were released, mailed or sent by the BIR. Still, it can be granted that the latest date the BIR could have released, mailed or sent the Assessment and Assessment Notice to petitioner BPI was on the same date they were received by the latter, on 20 October 1989. Counting the three-year prescriptive period, for a total of 1,095 days,21 from 20 October 1989, then the BIR only had until 19 October 1992 within which to collect the assessed deficiency DST. The earliest attempt of the BIR to collect on Assessment No. FAS-5-85-89-002054 was its issuance and service of a Warrant of Distraint and/or Levy on petitioner BPI. Although the Warrant was issued on 15 October 1992, previous to the expiration of the period for collection on 19 October 1992, the same was served on petitioner BPI only on 23 October 1992. Under Section 223(c) of the Tax Code of 1977, as amended, it is not essential that the Warrant of Distraint and/or Levy be fully executed so that it can suspend the running of the statute of limitations on the collection of the tax. It is enough that the proceedings have validly began or commenced and that their execution has not been suspended by reason of the voluntary desistance of the respondent BIR Commissioner. Existing jurisprudence establishes that distraint and levy proceedings are validly begun or commenced by the issuance of the Warrant and service thereof on the taxpayer.22 It is only logical to require that the Warrant of Distraint and/or Levy be, at the very least, served upon the taxpayer in order to suspend the running of the prescriptive period for collection of an assessed tax, because it may only be upon the service of the Warrant that the taxpayer is informed of the denial by the BIR of any pending protest of the said taxpayer, and the resolute intention of the BIR to collect the tax assessed. If the service of the Warrant of Distraint and/or Levy on petitioner BPI on 23 October 1992 was already beyond the prescriptive period for collection of the deficiency DST, which had expired on 19 October 1992, then what more the letter of respondent BIR Commissioner, dated 13 August 1997 and received by the counsel of the petitioner BPI only on 11 September 1997, denying the protest of petitioner BPI and requesting payment of the deficiency DST? Even later and more unequivocally barred by prescription on collection was the demand made by

respondent BIR Commissioner for payment of the deficiency DST in her Answer to the Petition for Review of petitioner BPI before the CTA, filed on 08 December 1997.23 II There is no valid ground for the suspension of the running of the prescriptive period for collection of the assessed DST under the Tax Code of 1977, as amended. In their Decisions, both the CTA and the Court of Appeals found that the filing by petitioner BPI of a protest letter suspended the running of the prescriptive period for collecting the assessed DST. This Court, however, takes the opposing view, and, based on the succeeding discussion, concludes that there is no valid ground for suspending the running of the prescriptive period for collection of the deficiency DST assessed against petitioner BPI. A. The statute of limitations on assessment and collection of taxes is for the protection of the taxpayer and, thus, shall be construed liberally in his favor. Though the statute of limitations on assessment and collection of national internal revenue taxes benefits both the Government and the taxpayer, it principally intends to afford protection to the taxpayer against unreasonable investigation. The indefinite extension of the period for assessment is unreasonable because it deprives the said taxpayer of the assurance that he will no longer be subjected to further investigation for taxes after the expiration of a reasonable period of time.24 As aptly explained in Republic of the Philippines v. Ablaza25 The law prescribing a limitation of actions for the collection of the income tax is beneficial both to the Government and to its citizens; to the Government because tax officers would be obliged to act promptly in the making of assessment, and to citizens because after the lapse of the period of prescription citizens would have a feeling of security against unscrupulous tax agents who will always find an excuse to inspect the books of taxpayers, not to determine the latters real liability, but to take advantage of every opportunity to molest peaceful, law-abiding citizens. Without such a legal defense taxpayers would furthermore be under obligation to always keep their books and keep them open for inspection subject to harassment by unscrupulous tax agents. The law on prescription being a remedial measure should be interpreted in a way conducive to bringing about the beneficent purpose of affording protection to the taxpayer within the contemplation of the Commission which recommend the approval of the law. In order to provide even better protection to the taxpayer against unreasonable investigation, the Tax Code of 1977, as amended, identifies specifically in Sections 223 and 22426 thereof the circumstances when the prescriptive periods for assessing and collecting taxes could be suspended or interrupted. To give effect to the legislative intent, these provisions on the statute of limitations on assessment and collection of taxes shall be construed and applied liberally in favor of the taxpayer and strictly against the Government.

B. The statute of limitations on assessment and collection of national internal revenue taxes may be waived, subject to certain conditions, under paragraphs (b) and (d) of Section 223 of the Tax Code of 1977, as amended, respectively. Petitioner BPI, however, did not execute any such waiver in the case at bar. According to paragraphs (b) and (d) of Section 223 of the Tax Code of 1977, as amended, the prescriptive periods for assessment and collection of national internal revenue taxes, respectively, could be waived by agreement, to wit SEC. 223. Exceptions as to period of limitation of assessment and collection of taxes. ... (b) If before the expiration of the time prescribed in the preceding section for the assessment of the tax, both the Commissioner and the taxpayer have agreed in writing to its assessment after such time the tax may be assessed within the period agreed upon. The period so agreed upon may be extended by subsequent written agreement made before the expiration of the period previously agreed upon. ... (d) Any internal revenue tax which has been assessed within the period agreed upon as provided in paragraph (b) hereinabove may be collected by distraint or levy or by a proceeding in court within the period agreed upon in writing before the expiration of the three-year period. The period so agreed upon may be extended by subsequent written agreements made before the expiration of the period previously agreed upon.27 The agreements so described in the afore-quoted provisions are often referred to as waivers of the statute of limitations. The waiver of the statute of limitations, whether on assessment or collection, should not be construed as a waiver of the right to invoke the defense of prescription but, rather, an agreement between the taxpayer and the BIR to extend the period to a date certain, within which the latter could still assess or collect taxes due. The waiver does not mean that the taxpayer relinquishes the right to invoke prescription unequivocally.28 A valid waiver of the statute of limitations under paragraphs (b) and (d) of Section 223 of the Tax Code of 1977, as amended, must be: (1) in writing; (2) agreed to by both the Commissioner and the taxpayer; (3) before the expiration of the ordinary prescriptive periods for assessment and collection; and (4) for a definite period beyond the ordinary prescriptive periods for assessment and collection. The period agreed upon can still be extended by subsequent written agreement, provided that it is executed prior to the expiration of the first period agreed upon. The BIR had issued Revenue Memorandum Order (RMO) No. 20-90 on 04 April 1990 to lay down an even more detailed procedure for the proper execution of such a waiver. RMO No. 20-90 mandates that the procedure for execution of the waiver shall be strictly followed, and any revenue official who fails to comply therewith resulting in the prescription of the right to assess and collect shall be administratively dealt with.

This Court had consistently ruled in a number of cases that a request for reconsideration or reinvestigation by the taxpayer, without a valid waiver of the prescriptive periods for the assessment and collection of tax, as required by the Tax Code and implementing rules, will not suspend the running thereof.29 In the Petition at bar, petitioner BPI executed no such waiver of the statute of limitations on the collection of the deficiency DST per Assessment No. FAS-5-85-89-002054. In fact, an internal memorandum of the Chief of the Legislative, Ruling & Research Division of the BIR to her counterpart in the Collection Enforcement Division, dated 15 October 1992, expressly noted that, "The taxpayer fails to execute a Waiver of the Statute of Limitations extending the period of collection of the said tax up to December 31, 1993 pending reconsideration of its protest. . ."30 Without a valid waiver, the statute of limitations on collection by the BIR of the deficiency DST could not have been suspended under paragraph (d) of Section 223 of the Tax Code of 1977, as amended. C. The protest filed by petitioner BPI did not constitute a request for reinvestigation, granted by the respondent BIR Commissioner, which could have suspended the running of the statute of limitations on collection of the assessed deficiency DST under Section 224 of the Tax Code of 1977, as amended. The Tax Code of 1977, as amended, also recognizes instances when the running of the statute of limitations on the assessment and collection of national internal revenue taxes could be suspended, even in the absence of a waiver, under Section 224 thereof, which reads SEC. 224. Suspension of running of statute. The running of the statute of limitation provided in Section[s] 203 and 223 on the making of assessment and the beginning of distraint or levy or a proceeding in court for collection, in respect of any deficiency, shall be suspended for the period during which the Commissioner is prohibited from making the assessment or beginning distraint or levy or a proceeding in court and for sixty days thereafter; when the taxpayer requests for a reinvestigation which is granted by the Commissioner; when the taxpayer cannot be located in the address given by him in the return filed upon which a tax is being assessed or collected: Provided, That, if the taxpayer informs the Commissioner of any change in address, the running of the statute of limitations will not be suspended; when the warrant of distraint and levy is duly served upon the taxpayer, his authorized representative, or a member of his household with sufficient discretion, and no property could be located; and when the taxpayer is out of the Philippines.31 Of particular importance to the present case is one of the circumstances enumerated in Section 224 of the Tax Code of 1977, as amended, wherein the running of the statute of limitations on assessment and collection of taxes is considered suspended "when the taxpayer requests for a reinvestigation which is granted by the Commissioner." This Court gives credence to the argument of petitioner BPI that there is a distinction between a request for reconsideration and a request for reinvestigation. Revenue Regulations (RR) No. 1285, issued on 27 November 1985 by the Secretary of Finance, upon the recommendation of the

BIR Commissioner, governs the procedure for protesting an assessment and distinguishes between the two types of protest, as follows PROTEST TO ASSESSMENT SEC. 6. Protest. The taxpayer may protest administratively an assessment by filing a written request for reconsideration or reinvestigation. . . ... For the purpose of the protest herein (a) Request for reconsideration. refers to a plea for a re-evaluation of an assessment on the basis of existing records without need of additional evidence. It may involve both a question of fact or of law or both. (b) Request for reinvestigation. refers to a plea for re-evaluation of an assessment on the basis of newly-discovered or additional evidence that a taxpayer intends to present in the reinvestigation. It may also involve a question of fact or law or both. With the issuance of RR No. 12-85 on 27 November 1985 providing the above-quoted distinctions between a request for reconsideration and a request for reinvestigation, the two types of protest can no longer be used interchangeably and their differences so lightly brushed aside. It bears to emphasize that under Section 224 of the Tax Code of 1977, as amended, the running of the prescriptive period for collection of taxes can only be suspended by a request for reinvestigation, not a request for reconsideration. Undoubtedly, a reinvestigation, which entails the reception and evaluation of additional evidence, will take more time than a reconsideration of a tax assessment, which will be limited to the evidence already at hand; this justifies why the former can suspend the running of the statute of limitations on collection of the assessed tax, while the latter can not. The protest letter of petitioner BPI, dated 16 November 1989 and filed with the BIR the next day, on 17 November 1989, did not specifically request for either a reconsideration or reinvestigation. A close review of the contents thereof would reveal, however, that it protested Assessment No. FAS-5-85-89-002054 based on a question of law, in particular, whether or not petitioner BPI was liable for DST on its sales of foreign currency to the Central Bank in taxable year 1985. The same protest letter did not raise any question of fact; neither did it offer to present any new evidence. In its own letter to petitioner BPI, dated 10 September 1992, the BIR itself referred to the protest of petitioner BPI as a request for reconsideration.32 These considerations would lead this Court to deduce that the protest letter of petitioner BPI was in the nature of a request for reconsideration, rather than a request for reinvestigation and, consequently, Section 224 of the Tax Code of 1977, as amended, on the suspension of the running of the statute of limitations should not apply. Even if, for the sake of argument, this Court glosses over the distinction between a request for reconsideration and a request for reinvestigation, and considers the protest of petitioner BPI as a

request for reinvestigation, the filing thereof could not have suspended at once the running of the statute of limitations. Article 224 of the Tax Code of 1977, as amended, very plainly requires that the request for reinvestigation had been granted by the BIR Commissioner to suspend the running of the prescriptive periods for assessment and collection. That the BIR Commissioner must first grant the request for reinvestigation as a requirement for suspension of the statute of limitations is even supported by existing jurisprudence. In the case of Republic of the Philippines v. Gancayco,33 taxpayer Gancayco requested for a thorough reinvestigation of the assessment against him and placed at the disposal of the Collector of Internal Revenue all the evidences he had for such purpose; yet, the Collector ignored the request, and the records and documents were not at all examined. Considering the given facts, this Court pronounced that . . .The act of requesting a reinvestigation alone does not suspend the period. The request should first be granted, in order to effect suspension. (Collector vs. Suyoc Consolidated, supra; also Republic vs. Ablaza, supra). Moreover, the Collector gave appellee until April 1, 1949, within which to submit his evidence, which the latter did one day before. There were no impediments on the part of the Collector to file the collection case from April 1, 1949. . . .34 In Republic of the Philippines v. Acebedo,35 this Court similarly found that . . . [T]he defendant, after receiving the assessment notice of September 24, 1949, asked for a reinvestigation thereof on October 11, 1949 (Exh. A). There is no evidence that this request was considered or acted upon. In fact, on October 23, 1950 the then Collector of Internal Revenue issued a warrant of distraint and levy for the full amount of the assessment (Exh. D), but there was no follow-up of this warrant. Consequently, the request for reinvestigation did not suspend the running of the period for filing an action for collection. The burden of proof that the taxpayers request for reinvestigation had been actually granted shall be on respondent BIR Commissioner. The grant may be expressed in communications with the taxpayer or implied from the actions of the respondent BIR Commissioner or his authorized BIR representatives in response to the request for reinvestigation. In Querol v. Collector of Internal Revenue,36 the BIR, after receiving the protest letters of taxpayer Querol, sent a tax examiner to San Fernando, Pampanga, to conduct the reinvestigation; as a result of which, the original assessment against taxpayer Querol was revised by permitting him to deduct reasonable depreciation. In another case, Republic of the Philippines v. Lopez,37 taxpayer Lopez filed a total of four petitions for reconsideration and reinvestigation. The first petition was denied by the BIR. The second and third petitions were granted by the BIR and after each reinvestigation, the assessed amount was reduced. The fourth petition was again denied and, thereafter, the BIR filed a collection suit against taxpayer Lopez. When the taxpayers spouses Sison, in Commissioner of Internal Revenue v. Sison,38 contested the assessment against them and asked for a reinvestigation, the BIR ordered the reinvestigation resulting in the issuance of an amended assessment. Lastly, in Republic of the Philippines v. Oquias,39 the BIR granted taxpayer Oquiass request for reinvestigation and duly notified him of the date when such

reinvestigation would be held; only, neither taxpayer Oquias nor his counsel appeared on the given date. In all these cases, the request for reinvestigation of the assessment filed by the taxpayer was evidently granted and actual reinvestigation was conducted by the BIR, which eventually resulted in the issuance of an amended assessment. On the basis of these facts, this Court ruled in the same cases that the period between the request for reinvestigation and the revised assessment should be subtracted from the total prescriptive period for the assessment of the tax; and, once the assessment had been reconsidered at the taxpayers instance, the period for collection should begin to run from the date of the reconsidered or modified assessment.40 The rulings of the foregoing cases do not apply to the present Petition because: (1) the protest filed by petitioner BPI was a request for reconsideration, not a reinvestigation, of the assessment against it; and (2) even granting that the protest of petitioner BPI was a request for reinvestigation, there was no showing that it was granted by respondent BIR Commissioner and that actual reinvestigation had been conducted. Going back to the administrative records of the present case, it would seem that the BIR, after receiving a copy of the protest letter of petitioner BPI on 17 November 1989, did not attempt to communicate at all with the latter until 10 September 1992, less than a month before the prescriptive period for collection on Assessment No. FAS-5-85-89-002054 was due to expire. There were internal communications, mostly indorsements of the docket of the case from one BIR division to another; but these hardly fall within the same sort of acts in the previously discussed cases that satisfactorily demonstrated the grant of the taxpayers request for reinvestigation. Petitioner BPI, in the meantime, was left in the dark as to the status of its protest in the absence of any word from the BIR. Besides, in its letter to petitioner BPI, dated 10 September 1992, the BIR unwittingly admitted that it had not yet acted on the protest of the former This refers to your protest against and/or request for reconsideration of the assessment/s of this Office against you involving the amount of P28,020.00 under FAS-5-85-89-002054 dated October 23, 1989 as deficiency documentary stamp tax inclusive of compromise penalty for the year 1985. In this connection, it is requested that the enclosed waiver of the statute of limitations extending the period of collection of the said tax/es to December 31, 1993 be executed by you as a condition precedent of our giving due course to your protest41 When the BIR stated in its letter, dated 10 September 1992, that the waiver of the statute of limitations on collection was a condition precedent to its giving due course to the request for reconsideration of petitioner BPI, then it was understood that the grant of such request for reconsideration was being held off until compliance with the given condition. When petitioner BPI failed to comply with the condition precedent, which was the execution of the waiver, the logical inference would be that the request was not granted and was not given due course at all. III

The suspension of the statute of limitations on collection of the assessed deficiency DST from petitioner BPI does not find support in jurisprudence. It is the position of respondent BIR Commissioner, affirmed by the CTA and the Court of Appeals, that the three-year prescriptive period for collecting on Assessment No. FAS-5-85-89002054 had not yet prescribed, because the said prescriptive period was suspended, invoking the case of Commissioner of Internal Revenue v. Wyeth Suaco Laboratories, Inc.42 It was in this case in which this Court ruled that the prescriptive period provided by law to make a collection is interrupted once a taxpayer requests for reinvestigation or reconsideration of the assessment. Petitioner BPI, on the other hand, is requesting this Court to revisit the Wyeth Suaco case contending that it had unjustifiably expanded the grounds for suspending the prescriptive period for collection of national internal revenue taxes. This Court finds that although there is no compelling reason to abandon its decision in the Wyeth Suaco case, the said case cannot be applied to the particular facts of the Petition at bar. A. The only exception to the statute of limitations on collection of taxes, other than those already provided in the Tax Code, was recognized in the Suyoc case. As had been previously discussed herein, the statute of limitations on assessment and collection of national internal revenue taxes may be suspended if the taxpayer executes a valid waiver thereof, as provided in paragraphs (b) and (d) of Section 223 of the Tax Code of 1977, as amended; and in specific instances enumerated in Section 224 of the same Code, which include a request for reinvestigation granted by the BIR Commissioner. Outside of these statutory provisions, however, this Court also recognized one other exception to the statute of limitations on collection of taxes in the case of Collector of Internal Revenue v. Suyoc Consolidated Mining Co.43 In the said case, the Collector of Internal Revenue issued an assessment against taxpayer Suyoc Consolidated Mining Co. on 11 February 1947 for deficiency income tax for the taxable year 1941. Taxpayer Suyoc requested for at least a year within which to pay the amount assessed, but at the same time, reserving its right to question the correctness of the assessment before actual payment. The Collector granted taxpayer Suyoc an extension of only three months to pay the assessed tax. When taxpayer Suyoc failed to pay the assessed tax within the extended period, the Collector sent it a demand letter, dated 28 November 1950. Upon receipt of the demand letter, taxpayer Suyoc asked for a reinvestigation and reconsideration of the assessment, but the Collector denied the request. Taxpayer Suyoc reiterated its request for reconsideration on 25 April 1952, which was denied again by the Collector on 06 May 1953. Taxpayer Suyoc then appealed the denial to the Conference Staff. The Conference Staff heard the appeal from 02 September 1952 to 16 July 1955, and the negotiations resulted in the reduction of the assessment on 26 July 1955. It was the collection of the reduced assessment that was questioned before this Court for being enforced beyond the prescriptive period.44 In resolving the issue on prescription, this Court ratiocinated thus

It is obvious from the foregoing that petitioner refrained from collecting the tax by distraint or levy or by proceeding in court within the 5-year period from the filing of the second amended final return due to the several requests of respondent for extension to which petitioner yielded to give it every opportunity to prove its claim regarding the correctness of the assessment. Because of such requests, several reinvestigations were made and a hearing was even held by the Conference Staff organized in the collection office to consider claims of such nature which, as the record shows, lasted for several months. After inducing petitioner to delay collection as he in fact did, it is most unfair for respondent to now take advantage of such desistance to elude his deficiency income tax liability to the prejudice of the Government invoking the technical ground of prescription. While we may agree with the Court of Tax Appeals that a mere request for reexamination or reinvestigation may not have the effect of suspending the running of the period of limitation for in such case there is need of a written agreement to extend the period between the Collector and the taxpayer, there are cases however where a taxpayer may be prevented from setting up the defense of prescription even if he has not previously waived it in writing as when by his repeated requests or positive acts the Government has been, for good reasons, persuaded to postpone collection to make him feel that the demand was not unreasonable or that no harassment or injustice is meant by the Government. And when such situation comes to pass there are authorities that hold, based on weighty reasons, that such an attitude or behavior should not be countenanced if only to protect the interest of the Government.45 By the principle of estoppel, taxpayer Suyoc was not allowed to raise the defense of prescription against the efforts of the Government to collect the tax assessed against it. This Court adopted the following principle from American jurisprudence: "He who prevents a thing from being done may not avail himself of the nonperformance which he has himself occasioned, for the law says to him in effect this is your own act, and therefore you are not damnified."46 In the Suyoc case, this Court expressly conceded that a mere request for reconsideration or reinvestigation of an assessment may not suspend the running of the statute of limitations. It affirmed the need for a waiver of the prescriptive period in order to effect suspension thereof. However, even without such waiver, the taxpayer may be estopped from raising the defense of prescription because by his repeated requests or positive acts, he had induced Government authorities to delay collection of the assessed tax. Based on the foregoing, petitioner BPI contends that the declaration made in the later case of Wyeth Suaco, that the statute of limitations on collection is suspended once the taxpayer files a request for reconsideration or reinvestigation, runs counter to the ruling made by this Court in the Suyoc case. B. Although this Court is not compelled to abandon its decision in the Wyeth Suaco case, it finds that Wyeth Suaco is not applicable to the Petition at bar because of the distinct facts involved herein. In the case of Wyeth Suaco, taxpayer Wyeth Suaco was assessed for failing to remit withholding taxes on royalties and dividend declarations, as well as, for deficiency sales tax. The BIR issued

two assessments, dated 16 December 1974 and 17 December 1974, both received by taxpayer Wyeth Suaco on 19 December 1974. Taxpayer Wyeth Suaco, through its tax consultant, SGV & Co., sent to the BIR two letters, dated 17 January 1975 and 08 February 1975, protesting the assessments and requesting their cancellation or withdrawal on the ground that said assessments lacked factual or legal basis. On 12 September 1975, the BIR Commissioner advised taxpayer Wyeth Suaco to avail itself of the compromise settlement being offered under Letter of Instruction No. 308. Taxpayer Wyeth Suaco manifested its conformity to paying a compromise amount, but subject to certain conditions; though, apparently, the said compromise amount was never paid. On 10 December 1979, the BIR Commissioner rendered a decision reducing the assessment for deficiency withholding tax against taxpayer Wyeth Suaco, but maintaining the assessment for deficiency sales tax. It was at this point when taxpayer Wyeth Suaco brought its case before the CTA to enjoin the BIR from enforcing the assessments by reason of prescription. Although the CTA decided in favor of taxpayer Wyeth Suaco, it was reversed by this Court when the case was brought before it on appeal. According to the decision of this Court Settled is the rule that the prescriptive period provided by law to make a collection by distraint or levy or by a proceeding in court is interrupted once a taxpayer requests for reinvestigation or reconsideration of the assessment. . . ... Although the protest letters prepared by SGV & Co. in behalf of private respondent did not categorically state or use the words "reinvestigation" and "reconsideration," the same are to be treated as letters of reinvestigation and reconsideration These letters of Wyeth Suaco interrupted the running of the five-year prescriptive period to collect the deficiency taxes. The Bureau of Internal Revenue, after having reviewed the records of Wyeth Suaco, in accordance with its request for reinvestigation, rendered a final assessment It was only upon receipt by Wyeth Suaco of this final assessment that the five-year prescriptive period started to run again.47 The foremost criticism of petitioner BPI of the Wyeth Suaco decision is directed at the statement made therein that, "settled is the rule that the prescriptive period provided by law to make a collection by distraint or levy or by a proceeding in court is interrupted once a taxpayer requests for reinvestigation or reconsideration of the assessment."48 It would seem that both petitioner BPI and respondent BIR Commissioner, as well as, the CTA and Court of Appeals, take the statement to mean that the filing alone of the request for reconsideration or reinvestigation can already interrupt or suspend the running of the prescriptive period on collection. This Court therefore takes this opportunity to clarify and qualify this statement made in the Wyeth Suaco case. While it is true that, by itself, such statement would appear to be a generalization of the exceptions to the statute of limitations on collection, it is best interpreted in consideration of the particular facts of the Wyeth Suaco case and previous jurisprudence. The Wyeth Suaco case cannot be in conflict with the Suyoc case because there are substantial differences in the factual backgrounds of the two cases. The Suyoc case refers to a situation where there were repeated requests or positive acts performed by the taxpayer that convinced the

BIR to delay collection of the assessed tax. This Court pronounced therein that the repeated requests or positive acts of the taxpayer prevented or estopped it from setting up the defense of prescription against the Government when the latter attempted to collect the assessed tax. In the Wyeth Suaco case, taxpayer Wyeth Suaco filed a request for reinvestigation, which was apparently granted by the BIR and, consequently, the prescriptive period was indeed suspended as provided under Section 224 of the Tax Code of 1977, as amended.49 To reiterate, Section 224 of the Tax Code of 1977, as amended, identifies specific circumstances when the statute of limitations on assessment and collection may be interrupted or suspended, among which is a request for reinvestigation that is granted by the BIR Commissioner. The act of filing a request for reinvestigation alone does not suspend the period; such request must be granted.50 The grant need not be express, but may be implied from the acts of the BIR Commissioner or authorized BIR officials in response to the request for reinvestigation.51 This Court found in the Wyeth Suaco case that the BIR actually conducted a reinvestigation, in accordance with the request of the taxpayer Wyeth Suaco, which resulted in the reduction of the assessment originally issued against it. Taxpayer Wyeth Suaco was also aware that its request for reinvestigation was granted, as written by its Finance Manager in a letter dated 01 July 1975, addressed to the Chief of the Tax Accounts Division, wherein he admitted that, "[a]s we understand, the matter is now undergoing review and consideration by your Manufacturing Audit Division" The statute of limitations on collection, then, started to run only upon the issuance and release of the reduced assessment. The Wyeth Suaco case, therefore, is correct in declaring that the prescriptive period for collection is interrupted or suspended when the taxpayer files a request for reinvestigation, provided that, as clarified and qualified herein, such request is granted by the BIR Commissioner. Thus, this Court finds no compelling reason to abandon its decision in the Wyeth Suaco case. It also now rules that the said case is not applicable to the Petition at bar because of the distinct facts involved herein. As already heretofore determined by this Court, the protest filed by petitioner BPI was a request for reconsideration, which merely required a review of existing evidence and the legal basis for the assessment. Respondent BIR Commissioner did not require, neither did petitioner BPI offer, additional evidence on the matter. After petitioner BPI filed its request for reconsideration, there was no other communication between it and respondent BIR Commissioner or any of the authorized representatives of the latter. There was no showing that petitioner BPI was informed or aware that its request for reconsideration was granted or acted upon by the BIR. IV Conclusion To summarize all the foregoing discussion, this Court lays down the following rules on the exceptions to the statute of limitations on collection.

The statute of limitations on collection may only be interrupted or suspended by a valid waiver executed in accordance with paragraph (d) of Section 223 of the Tax Code of 1977, as amended, and the existence of the circumstances enumerated in Section 224 of the same Code, which include a request for reinvestigation granted by the BIR Commissioner. Even when the request for reconsideration or reinvestigation is not accompanied by a valid waiver or there is no request for reinvestigation that had been granted by the BIR Commissioner, the taxpayer may still be held in estoppel and be prevented from setting up the defense of prescription of the statute of limitations on collection when, by his own repeated requests or positive acts, the Government had been, for good reasons, persuaded to postpone collection to make the taxpayer feel that the demand is not unreasonable or that no harassment or injustice is meant by the Government, as laid down by this Court in the Suyoc case. Applying the given rules to the present Petition, this Court finds that (a) The statute of limitations for collection of the deficiency DST in Assessment No. FAS-5-8589-002054, issued against petitioner BPI, had already expired; and (b) None of the conditions and requirements for exception from the statute of limitations on collection exists herein: Petitioner BPI did not execute any waiver of the prescriptive period on collection as mandated by paragraph (d) of Section 223 of the Tax Code of 1977, as amended; the protest filed by petitioner BPI was a request for reconsideration, not a request for reinvestigation that was granted by respondent BIR Commissioner which could have suspended the prescriptive period for collection under Section 224 of the Tax Code of 1977, as amended; and, petitioner BPI, other than filing a request for reconsideration of Assessment No. FAS-5-8589-002054, did not make repeated requests or performed positive acts that could have persuaded the respondent BIR Commissioner to delay collection, and that would have prevented or estopped petitioner BPI from setting up the defense of prescription against collection of the tax assessed, as required in the Suyoc case. This is a simple case wherein respondent BIR Commissioner and other BIR officials failed to act promptly in resolving and denying the request for reconsideration filed by petitioner BPI and in enforcing collection on the assessment. They presented no reason or explanation as to why it took them almost eight years to address the protest of petitioner BPI. The statute on limitations imposed by the Tax Code precisely intends to protect the taxpayer from such prolonged and unreasonable assessment and investigation by the BIR. Considering that the right of the respondent BIR Commissioner to collect from petitioner BPI the deficiency DST in Assessment No. FAS-5-85-89-002054 had already prescribed, then, there is no more need for this Court to make a determination on the validity and correctness of the said Assessment for the latter would only be unenforceable. Wherefore, based on the foregoing, the instant Petition is GRANTED. The Decision of the Court of Appeals in CA-G.R. SP No. 51271, dated 11 August 1999, which reinstated Assessment No. FAS-5-85-89-002054 requiring petitioner BPI to pay the amount of P28,020.00 as deficiency documentary stamp tax for the taxable year 1985, inclusive of the compromise penalty, is

REVERSED and SET ASIDE. Assessment No. FAS-5-85-89-002054 is hereby ordered CANCELED. G.R. No. 178788 September 29, 2010

UNITED AIRLINES, INC., Petitioner, vs. COMMISSIONER OF INTERNAL REVENUE, Respondent. DECISION VILLARAMA, JR., J.: Before us is a petition for review on certiorari under Rule 45 of the 1997 Rules of Civil Procedure, as amended, of the Decision1 dated July 5, 2007 of the Court of Tax Appeals En Banc (CTA En Banc) in C.T.A. EB No. 227 denying petitioners claim for tax refund of P5.03 million. The undisputed facts are as follows: Petitioner United Airlines, Inc. is a foreign corporation organized and existing under the laws of the State of Delaware, U.S.A., engaged in the international airline business. Petitioner used to be an online international carrier of passenger and cargo, i.e., it used to operate passenger and cargo flights originating in the Philippines. Upon cessation of its passenger flights in and out of the Philippines beginning February 21, 1998, petitioner appointed a sales agent in the Philippines -- Aerotel Ltd. Corp., an independent general sales agent acting as such for several international airline companies.2 Petitioner continued operating cargo flights from the Philippines until January 31, 2001.3 On April 12, 2002, petitioner filed with respondent Commissioner a claim for income tax refund, pursuant to Section 28(A)(3)(a)4 of the National Internal Revenue Code of 1997 (NIRC) in relation to Article 4(7)5 of the Convention between the Government of the Republic of the Philippines and the Government of the United States of America with respect to Income Taxes (RP-US Tax Treaty). Petitioner sought to refund the total amount of P15,916,680.69 pertaining to income taxes paid on gross passenger and cargo revenues for the taxable years 1999 to 2001, which included the amount of P5,028,813.23 allegedly representing income taxes paid in 1999 on passenger revenue from tickets sold in the Philippines, the uplifts of which did not originate in the Philippines. Citing the change in definition of Gross Philippine Billings (GPB) in the NIRC, petitioner argued that since it no longer operated passenger flights originating from the Philippines beginning February 21, 1998, its passenger revenue for 1999, 2000 and 2001 cannot be considered as income from sources within the Philippines, and hence should not be subject to Philippine income tax under Article 96 of the RP-US Tax Treaty.7 As no resolution on its claim for refund had yet been made by the respondent and in view of the two (2)-year prescriptive period (from the time of filing the Final Adjustment Return for the

taxable year 1999) which was about to expire on April 15, 2002, petitioner filed on said date a petition for review with the Court of Tax Appeals (CTA).8 Petitioner asserted that under the new definition of GPB under the 1997 NIRC and Article 4(7) of the RP-US Tax Treaty, Philippine tax authorities have jurisdiction to tax only the gross revenue derived by US air and shipping carriers from outgoing traffic in the Philippines. Since the Bureau of Internal Revenue (BIR) erroneously imposed and collected income tax in 1999 based on petitioners gross passenger revenue, as beginning 1998 petitioner no longer flew passenger flights to and from the Philippines, petitioner is entitled to a refund of such erroneously collected income tax in the amount of P5,028,813.23.9 In its Decision10 dated May 18, 2006, the CTAs First Division11 ruled that no excess or erroneously paid tax may be refunded to petitioner because the income tax on GPB under Section 28(A)(3)(a) of the NIRC applies as well to gross revenue from carriage of cargoes originating from the Philippines. It agreed that petitioner cannot be taxed on its 1999 passenger revenue from flights originating outside the Philippines. However, in reporting a cargo revenue of P740.33 million in 1999, it was found that petitioner deducted two (2) items from its gross cargo revenue of P2.84 billion: P141.79 million as commission and P1.98 billion as other incentives of its agent. These deductions were erroneous because the gross revenue referred to in Section 28(A)(3)(a) of the NIRC was total revenue before any deduction of commission and incentives. Petitioners gross cargo revenue in 1999, being P2.84 billion, the GPB tax thereon was P42.54 million and not P11.1 million, the amount petitioner paid for the reported net cargo revenue of P740.33 million. The CTA First Division further noted that petitioner even underpaid its taxes on cargo revenue by P31.43 million, which amount was much higher than the P5.03 million it asked to be refunded. A motion for reconsideration was filed by petitioner but the First Division denied the same. It held that petitioners claim for tax refund was not offset with its tax liability; that petitioners tax deficiency was due to erroneous deductions from its gross cargo revenue; that it did not make an assessment against petitioner; and that it merely determined if petitioner was entitled to a refund based on the undisputed facts and whether petitioner had paid the correct amount of tax.12 Petitioner elevated the case to the CTA En Banc which affirmed the decision of the First Division. Hence, this petition anchored on the following grounds: I. THE CTA EN BANC GROSSLY ERRED IN DENYING THE PETITIONERS CLAIM FOR REFUND OF ERRONEOUSLY PAID INCOME TAX ON GROSS PHILIPPINE BILLINGS [GPB] BASED ON ITS FINDING THAT PETITIONERS UNDERPAYMENT OF [P31.43 MILLION] GPB TAX ON CARGO REVENUES IS A LOT HIGHER THAN THE GPB TAX OF [P5.03 MILLION] ON PASSENGER REVENUES, WHICH IS THE SUBJECT OF THE INSTANT CLAIM FOR REFUND. THE DENIAL OF PETITIONERS CLAIM ON SUCH GROUND CLEARLY AMOUNTS TO AN OFF-SETTING OF TAX LIABILITIES, CONTRARY TO WELLSETTLED JURISPRUDENCE.

II. THE DECISION OF THE CTA EN BANC VIOLATED PETITIONERS RIGHT TO DUE PROCESS. III. THE CTA EN BANC ACTED IN EXCESS OF ITS JURISDICTION BY DENYING PETITIONERS CLAIM FOR REFUND OF ERRONEOUSLY PAID INCOME TAX ON GROSS PHILIPPINE BILLINGS BASED ON ITS FINDING THAT PETITIONER UNDERPAID GPB TAX ON CARGO REVENUES IN THE AMOUNT OF [P31.43 MILLION] FOR THE TAXABLE YEAR 1999. IV. THE CTA EN BANC HAS NO AUTHORITY UNDER THE LAW TO MAKE ANY ASSESSMENTS FOR DEFICIENCY TAXES. THE AUTHORITY TO MAKE ASSESSMENTS FOR DEFICIENCY NATIONAL INTERNAL REVENUE TAXES IS VESTED BY THE 1997 NIRC UPON RESPONDENT. V. ANY ASSESSMENT AGAINST PETITIONER FOR DEFICIENCY INCOME TAX FOR THE TAXABLE YEAR 1999 IS ALREADY BARRED BY PRESCRIPTION.13 The main issue to be resolved is whether the petitioner is entitled to a refund of the amount of P5,028,813.23 it paid as income tax on its passenger revenues in 1999. Petitioner argues that its claim for refund of erroneously paid GPB tax on off-line passenger revenues cannot be denied based on the finding of the CTA that petitioner allegedly underpaid the GPB tax on cargo revenues by P31,431,171.09, which underpayment is allegedly higher than the GPB tax of P5,028,813.23 on passenger revenues, the amount of the instant claim. The denial of petitioners claim for refund on such ground is tantamount to an offsetting of petitioners claim for refund of erroneously paid GPB against its alleged tax liability. Petitioner thus cites the well-entrenched rule in taxation cases that internal revenue taxes cannot be the subject of set-off or compensation.14 According to petitioner, the offsetting of the liabilities is very clear in the instant case because the amount of petitioners claim for refund of erroneously paid GPB tax of P5,028,813.23 for the taxable year 1999 is being offset against petitioners alleged deficiency GPB tax liability on cargo revenues for the same year, which was not even the subject of an investigation nor any valid assessment issued by respondent against the petitioner. Under Section 22815 of the NIRC, the "taxpayer shall be informed in writing of the law and the facts on which the assessment is made; otherwise, the assessment shall be void." This administrative process of issuing an assessment is part of procedural due process enshrined in the 1987 Constitution. Records do not show that petitioner has been assessed by the BIR for any deficiency GBP tax for 1999, nor was there any finding or investigation being conducted by respondent of any liability of petitioner for GPB tax for the said taxable period. Clearly, petitioners right to due process was violated.16 Petitioner further argues that the CTA acted in excess of its jurisdiction because the exclusive appellate jurisdiction of the CTA covers only decisions or inactions of the respondent in cases involving disputed assessments. The CTA has effectively assessed petitioner with a P31.43 million tax deficiency when it concluded that petitioner underpaid its GPB tax on cargo revenue. Since respondent did not issue an assessment for any deficiency tax, the alleged deficiency tax

on its cargo revenue in 1999 cannot be considered a disputed assessment that may be passed upon by the CTA. Petitioner stresses that the authority to issue an assessment for deficiency internal revenue taxes is vested by law on respondent, not with the CTA.17 Lastly, petitioner argues that any assessment against it for deficiency income tax for taxable year 1999 is barred by prescription. Petitioner claims that the prescriptive period within which an assessment for deficiency income tax may be made has prescribed on April 17, 2003, three (3) years after it filed its 1999 tax return.18 Respondent Commissioner maintains that the CTA acted within its jurisdiction in denying petitioners claim for tax refund. It points out that the objective of the CTAs determination of whether petitioner correctly paid its GPB tax for the taxable year 1999 was to ascertain the latters entitlement to the claimed refund and not for the purpose of imposing any deficiency tax. Hence, petitioners arguments regarding the propriety of the CTAs determination of its deficiency tax on its GPB for gross cargo revenues for 1999 are clearly misplaced.19 The petition has no merit. As correctly pointed out by petitioner, inasmuch as it ceased operating passenger flights to or from the Philippines in 1998, it is not taxable under Section 28(A)(3)(a) of the NIRC for gross passenger revenues. This much was also found by the CTA. In South African Airways v. Commissioner of Internal Revenue,20 we ruled that the correct interpretation of the said provisions is that, if an international air carrier maintains flights to and from the Philippines, it shall be taxed at the rate of 2% of its GPB, while international air carriers that do not have flights to and from the Philippines but nonetheless earn income from other activities in the country will be taxed at the rate of 32% of such income. Here, the subject of claim for tax refund is the tax paid on passenger revenue for taxable year 1999 at the time when petitioner was still operating cargo flights originating from the Philippines although it had ceased passenger flight operations. The CTA found that petitioner had underpaid its GPB tax for 1999 because petitioner had made deductions from its gross cargo revenues in the income tax return it filed for the taxable year 1999, the amount of underpayment even greater than the refund sought for erroneously paid GPB tax on passenger revenues for the same taxable period. Hence, the CTA ruled petitioner is not entitled to a tax refund. Petitioners arguments regarding the propriety of such determination by the CTA are misplaced. Under Section 72 of the NIRC, the CTA can make a valid finding that petitioner made erroneous deductions on its gross cargo revenue; that because of the erroneous deductions, petitioner reported a lower cargo revenue and paid a lower income tax thereon; and that petitioner's underpayment of the income tax on cargo revenue is even higher than the income tax it paid on passenger revenue subject of the claim for refund, such that the refund cannot be granted. Section 72 of the NIRC reads:

SEC. 72. Suit to Recover Tax Based on False or Fraudulent Returns. - When an assessment is made in case of any list, statement or return, which in the opinion of the Commissioner was false or fraudulent or contained any understatement or undervaluation, no tax collected under such assessment shall be recovered by any suit, unless it is proved that the said list, statement or return was not false nor fraudulent and did not contain any understatement or undervaluation; but this provision shall not apply to statements or returns made or to be made in good faith regarding annual depreciation of oil or gas wells and mines. In the afore-cited case of South African Airways, this Court rejected similar arguments on the denial of claim for tax refund, as follows: Precisely, petitioner questions the offsetting of its payment of the tax under Sec. 28(A)(3)(a) with their liability under Sec. 28(A)(1), considering that there has not yet been any assessment of their obligation under the latter provision. Petitioner argues that such offsetting is in the nature of legal compensation, which cannot be applied under the circumstances present in this case. Article 1279 of the Civil Code contains the elements of legal compensation, to wit: Art. 1279. In order that compensation may be proper, it is necessary: (1) That each one of the obligors be bound principally, and that he be at the same time a principal creditor of the other; (2) That both debts consist in a sum of money, or if the things due are consumable, they be of the same kind, and also of the same quality if the latter has been stated; (3) That the two debts be due; (4) That they be liquidated and demandable; (5) That over neither of them there be any retention or controversy, commenced by third persons and communicated in due time to the debtor. And we ruled in Philex Mining Corporation v. Commissioner of Internal Revenue, thus: In several instances prior to the instant case, we have already made the pronouncement that taxes cannot be subject to compensation for the simple reason that the government and the taxpayer are not creditors and debtors of each other. There is a material distinction between a tax and debt. Debts are due to the Government in its corporate capacity, while taxes are due to the Government in its sovereign capacity. We find no cogent reason to deviate from the aforementioned distinction. Prescinding from this premise, in Francia v. Intermediate Appellate Court, we categorically held that taxes cannot be subject to set-off or compensation, thus:

We have consistently ruled that there can be no off-setting of taxes against the claims that the taxpayer may have against the government. A person cannot refuse to pay a tax on the ground that the government owes him an amount equal to or greater than the tax being collected. The collection of a tax cannot await the results of a lawsuit against the government. The ruling in Francia has been applied to the subsequent case of Caltex Philippines, Inc. v. Commission on Audit, which reiterated that: . . . a taxpayer may not offset taxes due from the claims that he may have against the government. Taxes cannot be the subject of compensation because the government and taxpayer are not mutually creditors and debtors of each other and a claim for taxes is not such a debt, demand, contract or judgment as is allowed to be set-off. Verily, petitioners argument is correct that the offsetting of its tax refund with its alleged tax deficiency is unavailing under Art. 1279 of the Civil Code. Commissioner of Internal Revenue v. Court of Tax Appeals, however, granted the offsetting of a tax refund with a tax deficiency in this wise: Further, it is also worth noting that the Court of Tax Appeals erred in denying petitioners supplemental motion for reconsideration alleging bringing to said courts attention the existence of the deficiency income and business tax assessment against Citytrust. The fact of such deficiency assessment is intimately related to and inextricably intertwined with the right of respondent bank to claim for a tax refund for the same year. To award such refund despite the existence of that deficiency assessment is an absurdity and a polarity in conceptual effects. Herein private respondent cannot be entitled to refund and at the same time be liable for a tax deficiency assessment for the same year.1avvphi1 The grant of a refund is founded on the assumption that the tax return is valid, that is, the facts stated therein are true and correct. The deficiency assessment, although not yet final, created a doubt as to and constitutes a challenge against the truth and accuracy of the facts stated in said return which, by itself and without unquestionable evidence, cannot be the basis for the grant of the refund. Section 82, Chapter IX of the National Internal Revenue Code of 1977, which was the applicable law when the claim of Citytrust was filed, provides that "(w)hen an assessment is made in case of any list, statement, or return, which in the opinion of the Commissioner of Internal Revenue was false or fraudulent or contained any understatement or undervaluation, no tax collected under such assessment shall be recovered by any suits unless it is proved that the said list, statement, or return was not false nor fraudulent and did not contain any understatement or undervaluation; but this provision shall not apply to statements or returns made or to be made in good faith regarding annual depreciation of oil or gas wells and mines." Moreover, to grant the refund without determination of the proper assessment and the tax due would inevitably result in multiplicity of proceedings or suits. If the deficiency assessment should subsequently be upheld, the Government will be forced to institute anew a proceeding for

the recovery of erroneously refunded taxes which recourse must be filed within the prescriptive period of ten years after discovery of the falsity, fraud or omission in the false or fraudulent return involved. This would necessarily require and entail additional efforts and expenses on the part of the Government, impose a burden on and a drain of government funds, and impede or delay the collection of much-needed revenue for governmental operations. Thus, to avoid multiplicity of suits and unnecessary difficulties or expenses, it is both logically necessary and legally appropriate that the issue of the deficiency tax assessment against Citytrust be resolved jointly with its claim for tax refund, to determine once and for all in a single proceeding the true and correct amount of tax due or refundable. In fact, as the Court of Tax Appeals itself has heretofore conceded, it would be only just and fair that the taxpayer and the Government alike be given equal opportunities to avail of remedies under the law to defeat each others claim and to determine all matters of dispute between them in one single case. It is important to note that in determining whether or not petitioner is entitled to the refund of the amount paid, it would [be] necessary to determine how much the Government is entitled to collect as taxes. This would necessarily include the determination of the correct liability of the taxpayer and, certainly, a determination of this case would constitute res judicata on both parties as to all the matters subject thereof or necessarily involved therein. (Emphasis supplied.) Sec. 82, Chapter IX of the 1977 Tax Code is now Sec. 72, Chapter XI of the 1997 NIRC. The above pronouncements are, therefore, still applicable today. Here, petitioners similar tax refund claim assumes that the tax return that it filed was correct. Given, however, the finding of the CTA that petitioner, although not liable under Sec. 28(A)(3)(a) of the 1997 NIRC, is liable under Sec. 28(A)(1), the correctness of the return filed by petitioner is now put in doubt. As such, we cannot grant the prayer for a refund.21 (Additional emphasis supplied.) In the case at bar, the CTA explained that it merely determined whether petitioner is entitled to a refund based on the facts. On the assumption that petitioner filed a correct return, it had the right to file a claim for refund of GPB tax on passenger revenues it paid in 1999 when it was not operating passenger flights to and from the Philippines. However, upon examination by the CTA, petitioners return was found erroneous as it understated its gross cargo revenue for the same taxable year due to deductions of two (2) items consisting of commission and other incentives of its agent. Having underpaid the GPB tax due on its cargo revenues for 1999, petitioner is not entitled to a refund of its GPB tax on its passenger revenue, the amount of the former being even much higher (P31.43 million) than the tax refund sought (P5.2 million). The CTA therefore correctly denied the claim for tax refund after determining the proper assessment and the tax due. Obviously, the matter of prescription raised by petitioner is a non-issue. The prescriptive periods under Sections 20322 and 22223 of the NIRC find no application in this case. We must emphasize that tax refunds, like tax exemptions, are construed strictly against the taxpayer and liberally in favor of the taxing authority.24 In any event, petitioner has not discharged its burden of proof in establishing the factual basis for its claim for a refund and we

find no reason to disturb the ruling of the CTA. It has been a long-standing policy and practice of the Court to respect the conclusions of quasi-judicial agencies such as the CTA, a highly specialized body specifically created for the purpose of reviewing tax cases.25 WHEREFORE, we DENY the petition for lack of merit and AFFIRM the Decision dated July 5, 2007 of the Court of Tax Appeals En Banc in C.T.A. EB No. 227. With costs against the petitioner.

G.R. No. 146941

August 9, 2007

FILINVEST DEVELOPMENT CORPORATION, Petitioner, vs. COMMISSIONER OF INTERNAL REVENUE and COURT OF TAX APPEALS, Respondents. DECISION NACHURA, J.: Before us is a Petition for Review on Certiorari under Rule 45 of the Revised Rules of Civil Procedure filed by Filinvest Development Corporation (Filinvest) assailing the Decision1 of the Court of Appeals (CA), dated August 18, 2000, and its Resolution2 dated January 25, 2001 in CA-G.R. SP No. 56800. The case stems from the claim for refund, or in the alternative, the issuance of a tax credit certificate (TCC), filed by petitioner Filinvest with respondent Commissioner of Internal Revenue (CIR) in the amount of P4,178,134.00 representing excess creditable withholding taxes for taxable years 1994, 1995, and 1996.3 When the CIR had not resolved petitioners claim for refund and the two-year prescriptive period was about to lapse, the latter filed a Petition for Review4 with the Court of Tax Appeals. In the petition before the CTA, docketed as CTA Case No. 5603, petitioner prayed for refund, or in the alternative, the issuance of a TCC, in the amount of P3,173,868.00. The amount of P1,004,236.00 representing excess/unutilized creditable withholding taxes for 1994 was no longer included as it was already barred by the two-year prescriptive period. On August 13, 1999, the CTA rendered a Decision5 dismissing the petition for review for insufficiency of evidence because petitioner failed to present in evidence its 1997 income tax return. The CTA held that since petitioner indicated in its 1996 Income Tax Return that it has opted to carry over any excess income tax paid to the following year, there was no way for the court to determine with particular certainty if petitioner Filinvest indeed applied or credited the refundable amount to its 1997 tax liability, if there were any. Petitioner filed a motion for reconsideration, which was denied on December 23, 1999.6

Subsequently, petitioner filed a Petition for Review7 before the CA on January 21, 2000. The CA dismissed the petition on the ground of failure to attach the proof of authority of Efren M. Reyes, who executed the certification of non-forum shopping, to sign for the corporation.8 On motion for reconsideration, the CA set aside the January 26, 2000 Resolution and reinstated the case.9 On August 18, 2000, the CA issued the assailed Decision10 denying Filinvests petition for review, thus: Petitioner fails to discharge the burden of being entitled to the tax refund sought for considering that evidence on hand shows that although petitioner was able to comply with the requirements which a taxpayer must have to comply before a claim for a refund would be sustained, yet, it has failed to present vital documents (sic), its Income Tax Return for the year 1997, which would show whether or not petitioner has applied or credited the refundable amount sought for in its 1997 liability, if there be any, since per its 1996 Income Tax Return, it readily revealed that petitioner opted to carry over the excess income tax paid to the succeeding year and it is only from petitioners Income Tax Return for the year 1997 that this fact can be determined with certainty and the non-presentation of this vital document proved fatal to the petitioners cause of action. xxxx WHEREFORE, FOREGOING PREMISES CONSIDERED, the petition is hereby DENIED for lack of merit. The assailed Decision dated August 13, 1999 of the Court of Tax Appeals is affirmed. Costs against petitioner. SO ORDERED. Petitioner filed a motion for reconsideration, which the CA denied in the assailed Resolution11 dated January 25, 2001. Petitioner filed a petition for review before this Court but the same was denied on April 18, 2001 for failure to show that the appellate court committed reversible error, and for failure to comply with the requirements of Section 4, Rule 7 of the 1997 Rules of Civil Procedure in the execution of the verification.12 Petitioner filed a motion for reconsideration, which the Court granted on April 3, 2002.13 Hence, this petition for review. In this petition for review, petitioner Filinvest alleges that the CA erred in (1) denying its claim for tax refund on the sole ground that it failed to present in evidence its Annual Income Tax Return for Corporations for 1997 despite holding that it had complied with all the requirements to sustain a claim for tax refund; (2) relying on CTA cases cited in its Decision as jurisprudential basis to support its ruling; (3) not ruling that Section 34, Rule 132 of the Revised Rules of Court, being a procedural rule, should be liberally construed in order that substantial justice due petitioner shall have been served; and (4) not ruling that, petitioner having proved that it paid excess taxes for taxable years 1995 and 1996, has shifted the burden of evidence to respondent CIR to show the factual basis to deny petitioners claim.14

On the other hand, respondent CIR argues that in claims for tax refund, the burden of proof of refundability rests with claimant, and considering the rules on formal offer of evidence, the CA did not err in ruling against petitioner due to its failure to present evidence vital to sustain its claim. Likewise, respondent maintains that the CA did not err in relying on CTA cases because the latter is an authority on matters of taxation and therefore its resolutions carry great weight.15 The main issue for our resolution is whether petitioner is entitled to the tax refund or tax credit it seeks. We rule in the affirmative. It is settled that the factual findings of the CTA, as affirmed by the Court of Appeals, are entitled to the highest respect16 and will not be disturbed on appeal unless it is shown that the lower courts committed gross error in the appreciation of facts.17 In the case at bench, the CA erred in ruling that petitioner failed to discharge the burden of proving that it is entitled to the refund because of the latters failure to attach its 1997 Income Tax Return. The appellate court itself acknowledges that petitioner had complied with the requirements to sustain a claim for tax refund or credit.18 Yet it held that "petitioner fail[ed] to discharge the burden of being entitled to the tax refund sought for considering the evidence on hand shows that x x x it has failed to present [a] vital document[], its Income Tax Return for the year 1997 x x x."19 Both the CTA and the CA, citing the case of F. Jacinto Group, Inc. v. CIR20 and Citibank N.A. v. Court of Appeals, et al.,21 determined the requisites to sustain a claim for refund, thus: (1) That the claim for refund was filed within two years as prescribed under Section 230 of the National Internal Revenue Code; (2) That the income upon which the taxes were withheld were included in the return of the recipient; and (3) That the fact of withholding is established by a copy of a statement duly issued by the payor (withholding agent) to the payee showing the amount paid and the amount of tax withheld therefrom.22 In the proceedings before the CTA, petitioner presented in evidence its letter of claim for refund before the BIR to show that it was made within the two-year reglementary period;23 its Income Tax Returns for the years 1995 and 1996 to prove its total creditable withholding tax and the fact that the amounts were declared as part of its gross income;24 and several certificates of income tax withheld at source corresponding to the period of claim to prove the total amount of the taxes erroneously withheld.25 More importantly, petitioner attached its 1997 Income Tax Return to its Motion for Reconsideration, making the same part of the records of the case. The CTA cannot simply ignore this document.

Thus, we hold that petitioner has complied with all the requirements to prove its claim for tax refund. The CA, therefore, erred in denying the petition for review of the CTAs denial of petitioners claim for tax refund on the ground that it failed to present its 1997 Income Tax Return. The CAs reliance on Rule 132, Section 3426 of the Rules on Evidence is misplaced. This provision must be taken in the light of Republic Act No. 1125, as amended, the law creating the CTA, which provides that proceedings therein shall not be governed strictly by technical rules of evidence.27 Moreover, this Court has held time and again that technicalities should not be used to defeat substantive rights, especially those that have been established as a matter of fact. The CA, likewise, erred in relying on CTA decisions as jurisprudential basis for its decision. As this Court has held in the past: [B]y tradition and in our system of judicial administration this Court has the last word on what the law is, and that its decisions applying or interpreting the laws or the Constitution form part of the legal system of the country, all other courts should take their bearings from the decisions of this Court, ever mindful of what this Court said fifty-seven years ago in People vs. Vera that "[a] becoming modesty of inferior courts demands conscious realization of the position that they occupy in the interrelation and operation of the integrated judicial system of the nation."28 The principle of stare decisis et non quieta movere, as embodied in Article 8 of the Civil Code of the Philippines,29 enjoins adherence to judicial precedents. It requires our courts to follow a rule already established in a final decision of the Supreme Court. That decision becomes a judicial precedent to be followed in subsequent cases by all courts in the land. 30 This is not the first time this issue has come before this Court. The case of BPI-Family Savings Bank v. Court of Appeals,31 involves factual antecedents similar to the present case. BPI Family Bank involves a claim for tax refund representing therein petitioner's taxes withheld for the year 1989. In petitioners 1989 Income Tax Return, petitioner had a total refundable amount of P297,492.00 inclusive of the P112,491.00 being claimed as tax refund. However, petitioner declared in the same 1989 Income Tax Return that the said total refundable amount will be applied as tax credit to the succeeding taxable year. On October 11, 1990, petitioner filed a written claim for refund in the amount of P112,491.00 before the CIR alleging that it did not apply the 1989 refundable amount to its 1990 Annual Income Tax Return or other tax liabilities due to alleged business losses it incurred for the same year. Without waiting for the CIR to act on the claim for refund, petitioner filed a petition for review with the CTA, seeking the refund of P112,491.00. The CTA dismissed the petition on the ground that petitioner failed to present as evidence its Corporate Annual Income Tax Return for 1990 to establish the fact that petitioner had not yet credited the refundable amount. Petitioner filed a motion for reconsideration. However, the same was denied on May 6, 1994. The CA affirmed the CTA decision, ruling that it was incumbent upon petitioner to show proof that it had not credited the amount of P297,492.00 to its 1990 Annual Income Tax Return as it had previously declared in its 1989 Income Tax Return that the

amount would be applied as a tax credit in 1990. Petitioner having failed to submit such requirement, the CA said there is no basis to grant the claim for refund, because tax refunds are in the nature of tax exemptions and are regarded as in derogation of sovereign authority to be construed strictissimi juris against the person or entity claiming the exemption. In other words, the burden of proof rests upon the taxpayer, according to the CA. In reversing the CA and ruling that petitioner was entitled to the refund, this Court held: More important, a copy of the Final Adjustment Return for 1990 was attached to petitioner's Motion for Reconsideration filed before the CTA. A final adjustment return shows whether a corporation incurred a loss or gained a profit during the taxable year. In this case, that Return clearly showed that petitioner incurred P52,480,173 as net loss in 1990. Clearly, it could not have applied the amount in dispute as a tax credit. Again, the BIR did not controvert the veracity of the said return. It did not even file an opposition to petitioner's Motion and the 1990 Final Adjustment Return attached thereto. In denying the Motion for Reconsideration, however, the CTA ignored the said Return. In the same vein, the CA did not pass upon that significant document. True, strict procedural rules generally frown upon the submission of the Return after the trial. The law creating the Court of Tax Appeals, however, specifically provides that proceedings before it "shall not be governed strictly by the technical rules of evidence." The paramount consideration remains the ascertainment of truth. Verily, the quest for orderly presentation of issues is not an absolute. It should not bar courts from considering undisputed facts to arrive at a just determination of a controversy.1avvphi1 In the present case, the Return attached to the Motion for Reconsideration clearly showed that petitioner suffered a net loss in 1990. Contrary to the holding of the CA and the CTA, petitioner could not have applied the amount as a tax credit. In failing to consider the said Return, as well as the other documentary evidence presented during the trial, the appellate court committed a reversible error. It should be stressed that the rationale of the rules of procedure is to secure a just determination of every action. They are tools designed to facilitate the attainment of justice. But there can be no just determination of the present action if we ignore, on grounds of strict technicality, the Return submitted before the CTA and even before this Court. To repeat, the undisputed fact is that petitioner suffered a net loss in 1990; accordingly, it incurred no tax liability to which the tax credit could be applied. Consequently, there is no reason for the BIR and this Court to withhold the tax refund which rightfully belongs to the petitioner.32 We find the foregoing disquisition applicable to the present case. As in the BPI Family Bank case, herein petitioners claim for refund is anchored on the following provisions of the National Internal Revenue Code (NIRC) then in effect: SEC. 69. Final Adjustment Return. Every corporation liable to tax under Section 24 shall file a final adjustment return covering the total taxable income for the preceding calendar or fiscal

year. If the sum of the quarterly tax payments made during the said taxable year is not equal to the total [tax] due on the entire taxable net income of that year the corporation shall either: (a) Pay the excess tax still due; or (b) Be refunded the excess amount paid, as the case may be. In case the corporation is entitled to a refund of the excess estimated quarterly income taxes paid, the refundable amount shown on its final adjustment return may be credited against the estimated quarterly income tax liabilities for the taxable quarters of the succeeding taxable year. SEC. 230. Recovery of tax erroneously or illegally collected. No suit or proceeding shall be maintained in any court for the recovery of any national internal revenue tax hereafter alleged to have been erroneously or illegally assessed or collected, or of any penalty claimed to have been collected without authority or of any sum alleged to have been excessive or in any manner wrongfully collected, until a claim for refund or credit has been duly filed with the Commissioner; but such suit or proceeding may be maintained, whether or not such tax, penalty, or sum has been paid under protest or duress. In any case, no such suit or proceeding shall be begun after the expiration of two years from the date of payment of the tax or penalty regardless of any supervening cause that may arise after payment: Provided, however, That the Commissioner may, even without a written claim therefor, refund or credit any tax, where on the face of the return upon which payment was made, such payment appears clearly to have been erroneously paid. (Emphasis supplied) On the other hand, Revenue Regulation No. 12-94, Section 10 provides for the requirements to claim for tax credit or refund, to wit: Section 10. Claim for Tax Credit or Refund. (a) Claims for Tax Credit or Refund of income tax deducted and withheld on income payments shall be given due course only when it is shown on the return that the income payment received has been declared as part of the gross income and the fact of withholding is established by a copy of the Withholding Tax Statement duly issued by the payor to the payee showing the amount paid and the amount of tax withheld therefrom. (b) Excess Credits. A taxpayer's excess expanded withholding tax credits for the taxable quarter/taxable year shall automatically be allowed as a credit for purposes of filing his income tax return for the taxable quarter/taxable year immediately succeeding the taxable quarter/taxable year in which the aforesaid excess credit arose, provided, however, he submits with his income tax return a copy of his income tax return for the aforesaid previous taxable period showing the amount of his aforementioned excess withholding tax credits. If the taxpayer, in lieu of the aforesaid automatic application of his excess credit, wants a cash refund or a tax credit certificate for use in payment of his other national internal tax liabilities, he

shall make a written request therefor. Upon filing of his request, the taxpayer's income tax return showing the excess expanded withholding tax credits shall be examined. The excess expanded withholding tax, if any, shall be determined and refunded/credited to the taxpayer-applicant. The refund/credit shall be made within a period of sixty (60) days from date of the taxpayer's request provided, however, that the taxpayer-applicant submitted for audit all his pertinent accounting records and that the aforesaid records established the veracity of his claim for a refund/credit of his excess expanded withholding tax credits. (Emphasis supplied) It is true that herein petitioner has the burden of proving that it is entitled to refund. However, we have already held that once the claimant has submitted all the required documents, it is the function of the BIR to assess these documents with purposeful dispatch.33 In proving the inclusion of the income payments which formed the basis of the withholding taxes and the fact of withholding, this Court has held that: [D]etailed proof of the truthfulness of each and every item in the income tax return is not required. That function is lodged in the Commissioner of Internal Revenue by the NIRC which requires the Commissioner to assess internal revenue taxes within three years after the last day prescribed by law for the filing of the return. x x x The grant of a refund is founded on the assumption that the tax return is valid; that is, the facts stated therein are true and correct. In fact, even without petitioner's tax claim, the Commissioner can proceed to examine the books, records of the petitioner-bank, or any data which may be relevant or material in accordance with Section 16 of the present NIRC.34 It is worth noting that under Section 230 of the NIRC and Section 10 of Revenue Regulation No. 12-84, the CIR is given the power to grant a tax credit or refund even without a written claim therefor, if the former determines from the face of the return that payment had clearly been erroneously made. Evidently, the CIRs function is not merely to receive the claims for refund but it is also given the positive duty to determine the veracity of such claim. In another case, the Court held that while a taxpayer is given the choice whether to claim for refund or have its excess taxes applied as tax credit for the succeeding taxable year, such election is not final. Prior verification and approval by the Commissioner of Internal Revenue is required. The availment of the remedy of tax credit is not absolute and mandatory. It does not confer an absolute right on the taxpayer to avail of the tax credit scheme if it so chooses. Neither does it impose a duty on the part of the government to sit back and allow an important facet of tax collection to be at the sole control and discretion of the taxpayer.35 In the case of San Carlos Milling Co., Inc. v. CIR,36 the Court struck down therein petitioners attempt to unilaterally declare as tax credit its excess estimated quarterly income taxes from the previous year. The Court explained, thus: The respondent Court held that the choice of a corporate taxpayer for an automatic tax credit does not ipso facto confer on it the right to immediately avail of the same. Respondent court went on to emphasize the need for an investigation to ascertain the correctness of the corporate returns and the amount sought to be credited. We agree.

It is difficult to see by what process of ratiocination petitioner insists on the literal interpretation of the word "automatic." Such literal interpretation has been discussed and precluded by the respondent court in its decision of 23 December 1991 where, as aforestated, it ruled that "once a taxpayer opts for either a refund or the automatic tax credit scheme, and signified his option in accordance with the regulation, this does not ipso facto confer on him the right to avail of the same immediately. An investigation, as a matter of procedure, is necessary to enable the Commissioner to determine the correctness of the petitioner's returns, and the tax amount to be credited." Prior approval by the Commissioner of Internal Revenue of the tax credit under then section 86 (now section 69) of the Tax Code would appear to be the most reasonable interpretation to be given to said section. An opportunity must be given the internal revenue branch of the government to investigate and confirm the veracity of the claims of the taxpayer. The absolute freedom that petitioner seeks to automatically credit tax payments against tax liabilities for a succeeding taxable year, can easily give rise to confusion and abuse, depriving the government of authority and control over the manner by which the taxpayers credit and offset their tax liabilities, not to mention the resultant loss of revenue to the government under such a scheme. Hence we do not agree with respondents contention that "the actual carry-over of the excess withholding tax to the next quarter virtually negates a refund of the excess since it is considered to have been automatically applied to any income of that period." However, even assuming that petitioner had the power to automatically apply its excess withholding taxes to subsequent payments, the fact remains that, in this particular case, it could not have done so given its business losses. We must also point out that, simply by exercising the CIRs power to examine and verify petitioners claim for tax exemption as granted by law, respondent CIR could have easily verified petitioners claim by presenting the latters 1997 Income Tax Return, the original of which it has in its files. However, records show that in the proceedings before the CTA, respondent CIR failed to comment on petitioners formal offer of evidence,37 waived its right to present its own evidence,38 and failed to file its memorandum.39 Neither did it file an opposition to petitioners motion to reconsider the CTA decision to which the 1997 Income Tax Return was appended. That no one shall unjustly enrich oneself at the expense of another is a long-standing principle prevailing in our legal system. This applies not only to individuals but to the State as well. In the field of taxation where the State exacts strict compliance upon its citizens, the State must likewise deal with taxpayers with fairness and honesty. The harsh power of taxation must be tempered with evenhandedness. Hence, under the principle of solutio indebiti,40 the Government has to restore to petitioner the sums representing erroneous payments of taxes. WHEREFORE, premises considered, the petition is GRANTED. The CA decision and the CTA decision are REVERSED and SET ASIDE. Respondent Commissioner of Internal Revenue is ORDERED to refund, or in the alternative, issue a Tax Credit Certificate to petitioner Filinvest Development Corporation in the amount of P3,173,868.00.

G.R. No. 184823

October 6, 2010

COMMISSIONER OF INTERNAL REVENUE, Petitioner, vs. AICHI FORGING COMPANY OF ASIA, INC., Respondent. DECISION DEL CASTILLO, J.: A taxpayer is entitled to a refund either by authority of a statute expressly granting such right, privilege, or incentive in his favor, or under the principle of solutio indebiti requiring the return of taxes erroneously or illegally collected. In both cases, a taxpayer must prove not only his entitlement to a refund but also his compliance with the procedural due process as nonobservance of the prescriptive periods within which to file the administrative and the judicial claims would result in the denial of his claim. This Petition for Review on Certiorari under Rule 45 of the Rules of Court seeks to set aside the July 30, 2008 Decision1 and the October 6, 2008 Resolution2 of the Court of Tax Appeals (CTA) En Banc. Factual Antecedents Respondent Aichi Forging Company of Asia, Inc., a corporation duly organized and existing under the laws of the Republic of the Philippines, is engaged in the manufacturing, producing, and processing of steel and its by-products.3 It is registered with the Bureau of Internal Revenue (BIR) as a Value-Added Tax (VAT) entity4 and its products, "close impression die steel forgings" and "tool and dies," are registered with the Board of Investments (BOI) as a pioneer status.5 On September 30, 2004, respondent filed a claim for refund/credit of input VAT for the period July 1, 2002 to September 30, 2002 in the total amount of P3,891,123.82 with the petitioner Commissioner of Internal Revenue (CIR), through the Department of Finance (DOF) One-Stop Shop Inter-Agency Tax Credit and Duty Drawback Center.6 Proceedings before the Second Division of the CTA On even date, respondent filed a Petition for Review7 with the CTA for the refund/credit of the same input VAT. The case was docketed as CTA Case No. 7065 and was raffled to the Second Division of the CTA. In the Petition for Review, respondent alleged that for the period July 1, 2002 to September 30, 2002, it generated and recorded zero-rated sales in the amount of P131,791,399.00,8 which was paid pursuant to Section 106(A) (2) (a) (1), (2) and (3) of the National Internal Revenue Code of 1997 (NIRC);9 that for the said period, it incurred and paid input VAT amounting to P3,912,088.14 from purchases and importation attributable to its zero-rated sales;10 and that in its

application for refund/credit filed with the DOF One-Stop Shop Inter-Agency Tax Credit and Duty Drawback Center, it only claimed the amount of P3,891,123.82.11 In response, petitioner filed his Answer12 raising the following special and affirmative defenses, to wit: 4. Petitioners alleged claim for refund is subject to administrative investigation by the Bureau; 5. Petitioner must prove that it paid VAT input taxes for the period in question; 6. Petitioner must prove that its sales are export sales contemplated under Sections 106(A) (2) (a), and 108(B) (1) of the Tax Code of 1997; 7. Petitioner must prove that the claim was filed within the two (2) year period prescribed in Section 229 of the Tax Code; 8. In an action for refund, the burden of proof is on the taxpayer to establish its right to refund, and failure to sustain the burden is fatal to the claim for refund; and 9. Claims for refund are construed strictly against the claimant for the same partake of the nature of exemption from taxation.13 Trial ensued, after which, on January 4, 2008, the Second Division of the CTA rendered a Decision partially granting respondents claim for refund/credit. Pertinent portions of the Decision read: For a VAT registered entity whose sales are zero-rated, to validly claim a refund, Section 112 (A) of the NIRC of 1997, as amended, provides: SEC. 112. Refunds or Tax Credits of Input Tax. (A) Zero-rated or Effectively Zero-rated Sales. Any VAT-registered person, whose sales are zero-rated or effectively zero-rated may, within two (2) years after the close of the taxable quarter when the sales were made, apply for the issuance of a tax credit certificate or refund of creditable input tax due or paid attributable to such sales, except transitional input tax, to the extent that such input tax has not been applied against output tax: x x x Pursuant to the above provision, petitioner must comply with the following requisites: (1) the taxpayer is engaged in sales which are zero-rated or effectively zero-rated; (2) the taxpayer is VAT-registered; (3) the claim must be filed within two years after the close of the taxable quarter when such sales were made; and (4) the creditable input tax due or paid must be attributable to such sales, except the transitional input tax, to the extent that such input tax has not been applied against the output tax. The Court finds that the first three requirements have been complied [with] by petitioner.

With regard to the first requisite, the evidence presented by petitioner, such as the Sales Invoices (Exhibits "II" to "II-262," "JJ" to "JJ-431," "KK" to "KK-394" and "LL") shows that it is engaged in sales which are zero-rated. The second requisite has likewise been complied with. The Certificate of Registration with OCN 1RC0000148499 (Exhibit "C") with the BIR proves that petitioner is a registered VAT taxpayer. In compliance with the third requisite, petitioner filed its administrative claim for refund on September 30, 2004 (Exhibit "N") and the present Petition for Review on September 30, 2004, both within the two (2) year prescriptive period from the close of the taxable quarter when the sales were made, which is from September 30, 2002. As regards, the fourth requirement, the Court finds that there are some documents and claims of petitioner that are baseless and have not been satisfactorily substantiated. xxxx In sum, petitioner has sufficiently proved that it is entitled to a refund or issuance of a tax credit certificate representing unutilized excess input VAT payments for the period July 1, 2002 to September 30, 2002, which are attributable to its zero-rated sales for the same period, but in the reduced amount of P3,239,119.25, computed as follows: Amount of Claimed Input VAT Less: Exceptions as found by the ICPA Net Creditable Input VAT Less: Output VAT Due Excess Creditable Input VAT P 3,891,123.82 41,020.37 P 3,850,103.45 610,984.20 P 3,239,119.25

WHEREFORE, premises considered, the present Petition for Review is PARTIALLY GRANTED. Accordingly, respondent is hereby ORDERED TO REFUND OR ISSUE A TAX CREDIT CERTIFICATE in favor of petitioner [in] the reduced amount of THREE MILLION TWO HUNDRED THIRTY NINE THOUSAND ONE HUNDRED NINETEEN AND 25/100 PESOS (P3,239,119.25), representing the unutilized input VAT incurred for the months of July to September 2002. SO ORDERED.14 Dissatisfied with the above-quoted Decision, petitioner filed a Motion for Partial Reconsideration,15 insisting that the administrative and the judicial claims were filed beyond the two-year period to claim a tax refund/credit provided for under Sections 112(A) and 229 of the NIRC. He reasoned that since the year 2004 was a leap year, the filing of the claim for tax refund/credit on September 30, 2004 was beyond the two-year period, which expired on

September 29, 2004.16 He cited as basis Article 13 of the Civil Code,17 which provides that when the law speaks of a year, it is equivalent to 365 days. In addition, petitioner argued that the simultaneous filing of the administrative and the judicial claims contravenes Sections 112 and 229 of the NIRC.18 According to the petitioner, a prior filing of an administrative claim is a "condition precedent"19 before a judicial claim can be filed. He explained that the rationale of such requirement rests not only on the doctrine of exhaustion of administrative remedies but also on the fact that the CTA is an appellate body which exercises the power of judicial review over administrative actions of the BIR. 20 The Second Division of the CTA, however, denied petitioners Motion for Partial Reconsideration for lack of merit. Petitioner thus elevated the matter to the CTA En Banc via a Petition for Review.21 Ruling of the CTA En Banc On July 30, 2008, the CTA En Banc affirmed the Second Divisions Decision allowing the partial tax refund/credit in favor of respondent. However, as to the reckoning point for counting the two-year period, the CTA En Banc ruled: Petitioner argues that the administrative and judicial claims were filed beyond the period allowed by law and hence, the honorable Court has no jurisdiction over the same. In addition, petitioner further contends that respondent's filing of the administrative and judicial [claims] effectively eliminates the authority of the honorable Court to exercise jurisdiction over the judicial claim. We are not persuaded. Section 114 of the 1997 NIRC, and We quote, to wit: SEC. 114. Return and Payment of Value-added Tax. (A) In General. Every person liable to pay the value-added tax imposed under this Title shall file a quarterly return of the amount of his gross sales or receipts within twenty-five (25) days following the close of each taxable quarter prescribed for each taxpayer: Provided, however, That VAT-registered persons shall pay the value-added tax on a monthly basis. [x x x x ] Based on the above-stated provision, a taxpayer has twenty five (25) days from the close of each taxable quarter within which to file a quarterly return of the amount of his gross sales or receipts. In the case at bar, the taxable quarter involved was for the period of July 1, 2002 to September 30, 2002. Applying Section 114 of the 1997 NIRC, respondent has until October 25, 2002 within which to file its quarterly return for its gross sales or receipts [with] which it complied when it filed its VAT Quarterly Return on October 20, 2002. In relation to this, the reckoning of the two-year period provided under Section 229 of the 1997 NIRC should start from the payment of tax subject claim for refund. As stated above, respondent

filed its VAT Return for the taxable third quarter of 2002 on October 20, 2002. Thus, respondent's administrative and judicial claims for refund filed on September 30, 2004 were filed on time because AICHI has until October 20, 2004 within which to file its claim for refund. In addition, We do not agree with the petitioner's contention that the 1997 NIRC requires the previous filing of an administrative claim for refund prior to the judicial claim. This should not be the case as the law does not prohibit the simultaneous filing of the administrative and judicial claims for refund. What is controlling is that both claims for refund must be filed within the twoyear prescriptive period. In sum, the Court En Banc finds no cogent justification to disturb the findings and conclusion spelled out in the assailed January 4, 2008 Decision and March 13, 2008 Resolution of the CTA Second Division. What the instant petition seeks is for the Court En Banc to view and appreciate the evidence in their own perspective of things, which unfortunately had already been considered and passed upon. WHEREFORE, the instant Petition for Review is hereby DENIED DUE COURSE and DISMISSED for lack of merit. Accordingly, the January 4, 2008 Decision and March 13, 2008 Resolution of the CTA Second Division in CTA Case No. 7065 entitled, "AICHI Forging Company of Asia, Inc. petitioner vs. Commissioner of Internal Revenue, respondent" are hereby AFFIRMED in toto. SO ORDERED.22 Petitioner sought reconsideration but the CTA En Banc denied23 his Motion for Reconsideration. Issue Hence, the present recourse where petitioner interposes the issue of whether respondents judicial and administrative claims for tax refund/credit were filed within the two-year prescriptive period provided in Sections 112(A) and 229 of the NIRC.24 Petitioners Arguments Petitioner maintains that respondents administrative and judicial claims for tax refund/credit were filed in violation of Sections 112(A) and 229 of the NIRC.25 He posits that pursuant to Article 13 of the Civil Code,26 since the year 2004 was a leap year, the filing of the claim for tax refund/credit on September 30, 2004 was beyond the two-year period, which expired on September 29, 2004.27 Petitioner further argues that the CTA En Banc erred in applying Section 114(A) of the NIRC in determining the start of the two-year period as the said provision pertains to the compliance requirements in the payment of VAT.28 He asserts that it is Section 112, paragraph (A), of the

same Code that should apply because it specifically provides for the period within which a claim for tax refund/ credit should be made.29 Petitioner likewise puts in issue the fact that the administrative claim with the BIR and the judicial claim with the CTA were filed on the same day.30 He opines that the simultaneous filing of the administrative and the judicial claims contravenes Section 229 of the NIRC, which requires the prior filing of an administrative claim.31 He insists that such procedural requirement is based on the doctrine of exhaustion of administrative remedies and the fact that the CTA is an appellate body exercising judicial review over administrative actions of the CIR.32 Respondents Arguments For its part, respondent claims that it is entitled to a refund/credit of its unutilized input VAT for the period July 1, 2002 to September 30, 2002 as a matter of right because it has substantially complied with all the requirements provided by law.33 Respondent likewise defends the CTA En Banc in applying Section 114(A) of the NIRC in computing the prescriptive period for the claim for tax refund/credit. Respondent believes that Section 112(A) of the NIRC must be read together with Section 114(A) of the same Code.34 As to the alleged simultaneous filing of its administrative and judicial claims, respondent contends that it first filed an administrative claim with the One-Stop Shop Inter-Agency Tax Credit and Duty Drawback Center of the DOF before it filed a judicial claim with the CTA.35 To prove this, respondent points out that its Claimant Information Sheet No. 4970236 and BIR Form No. 1914 for the third quarter of 2002,37 which were filed with the DOF, were attached as Annexes "M" and "N," respectively, to the Petition for Review filed with the CTA.38 Respondent further contends that the non-observance of the 120-day period given to the CIR to act on the claim for tax refund/credit in Section 112(D) is not fatal because what is important is that both claims are filed within the two-year prescriptive period.39 In support thereof, respondent cites Commissioner of Internal Revenue v. Victorias Milling Co., Inc.40 where it was ruled that "[i]f, however, the [CIR] takes time in deciding the claim, and the period of two years is about to end, the suit or proceeding must be started in the [CTA] before the end of the two-year period without awaiting the decision of the [CIR]."41 Lastly, respondent argues that even if the period had already lapsed, it may be suspended for reasons of equity considering that it is not a jurisdictional requirement.42 Our Ruling The petition has merit. Unutilized input VAT must be claimed within two years after the close of the taxable quarter when the sales were made In computing the two-year prescriptive period for claiming a refund/credit of unutilized input VAT, the Second Division of the CTA applied Section 112(A) of the NIRC, which states: SEC. 112. Refunds or Tax Credits of Input Tax.

(A) Zero-rated or Effectively Zero-rated Sales Any VAT-registered person, whose sales are zero-rated or effectively zero-rated may, within two (2) years after the close of the taxable quarter when the sales were made, apply for the issuance of a tax credit certificate or refund of creditable input tax due or paid attributable to such sales, except transitional input tax, to the extent that such input tax has not been applied against output tax: Provided, however, That in the case of zero-rated sales under Section 106(A)(2)(a)(1), (2) and (B) and Section 108 (B)(1) and (2), the acceptable foreign currency exchange proceeds thereof had been duly accounted for in accordance with the rules and regulations of the Bangko Sentral ng Pilipinas (BSP): Provided, further, That where the taxpayer is engaged in zero-rated or effectively zero-rated sale and also in taxable or exempt sale of goods or properties or services, and the amount of creditable input tax due or paid cannot be directly and entirely attributed to any one of the transactions, it shall be allocated proportionately on the basis of the volume of sales. (Emphasis supplied.) The CTA En Banc, on the other hand, took into consideration Sections 114 and 229 of the NIRC, which read: SEC. 114. Return and Payment of Value-Added Tax. (A) In General. Every person liable to pay the value-added tax imposed under this Title shall file a quarterly return of the amount of his gross sales or receipts within twenty-five (25) days following the close of each taxable quarter prescribed for each taxpayer: Provided, however, That VAT-registered persons shall pay the value-added tax on a monthly basis. Any person, whose registration has been cancelled in accordance with Section 236, shall file a return and pay the tax due thereon within twenty-five (25) days from the date of cancellation of registration: Provided, That only one consolidated return shall be filed by the taxpayer for his principal place of business or head office and all branches. xxxx SEC. 229. Recovery of tax erroneously or illegally collected. No suit or proceeding shall be maintained in any court for the recovery of any national internal revenue tax hereafter alleged to have been erroneously or illegally assessed or collected, or of any penalty claimed to have been collected without authority, or of any sum alleged to have been excessively or in any manner wrongfully collected, until a claim for refund or credit has been duly filed with the Commissioner; but such suit or proceeding may be maintained, whether or not such tax, penalty or sum has been paid under protest or duress. In any case, no such suit or proceeding shall be filed after the expiration of two (2) years from the date of payment of the tax or penalty regardless of any supervening cause that may arise after payment: Provided, however, That the Commissioner may, even without written claim therefor, refund or credit any tax, where on the face of the return upon which payment was made, such payment appears clearly to have been erroneously paid. (Emphasis supplied.)

Hence, the CTA En Banc ruled that the reckoning of the two-year period for filing a claim for refund/credit of unutilized input VAT should start from the date of payment of tax and not from the close of the taxable quarter when the sales were made.43 The pivotal question of when to reckon the running of the two-year prescriptive period, however, has already been resolved in Commissioner of Internal Revenue v. Mirant Pagbilao Corporation,44 where we ruled that Section 112(A) of the NIRC is the applicable provision in determining the start of the two-year period for claiming a refund/credit of unutilized input VAT, and that Sections 204(C) and 229 of the NIRC are inapplicable as "both provisions apply only to instances of erroneous payment or illegal collection of internal revenue taxes."45 We explained that: The above proviso [Section 112 (A) of the NIRC] clearly provides in no uncertain terms that unutilized input VAT payments not otherwise used for any internal revenue tax due the taxpayer must be claimed within two years reckoned from the close of the taxable quarter when the relevant sales were made pertaining to the input VAT regardless of whether said tax was paid or not. As the CA aptly puts it, albeit it erroneously applied the aforequoted Sec. 112 (A), "[P]rescriptive period commences from the close of the taxable quarter when the sales were made and not from the time the input VAT was paid nor from the time the official receipt was issued." Thus, when a zero-rated VAT taxpayer pays its input VAT a year after the pertinent transaction, said taxpayer only has a year to file a claim for refund or tax credit of the unutilized creditable input VAT. The reckoning frame would always be the end of the quarter when the pertinent sales or transaction was made, regardless when the input VAT was paid. Be that as it may, and given that the last creditable input VAT due for the period covering the progress billing of September 6, 1996 is the third quarter of 1996 ending on September 30, 1996, any claim for unutilized creditable input VAT refund or tax credit for said quarter prescribed two years after September 30, 1996 or, to be precise, on September 30, 1998. Consequently, MPCs claim for refund or tax credit filed on December 10, 1999 had already prescribed. Reckoning for prescriptive period under Secs. 204(C) and 229 of the NIRC inapplicable To be sure, MPC cannot avail itself of the provisions of either Sec. 204(C) or 229 of the NIRC which, for the purpose of refund, prescribes a different starting point for the two-year prescriptive limit for the filing of a claim therefor. Secs. 204(C) and 229 respectively provide: Sec. 204. Authority of the Commissioner to Compromise, Abate and Refund or Credit Taxes. The Commissioner may xxxx (c) Credit or refund taxes erroneously or illegally received or penalties imposed without authority, refund the value of internal revenue stamps when they are returned in good condition by the purchaser, and, in his discretion, redeem or change unused stamps that have been rendered unfit for use and refund their value upon proof of destruction. No credit or refund of taxes or penalties shall be allowed unless the taxpayer files in writing with the Commissioner a claim for

credit or refund within two (2) years after the payment of the tax or penalty: Provided, however, That a return filed showing an overpayment shall be considered as a written claim for credit or refund. xxxx Sec. 229. Recovery of Tax Erroneously or Illegally Collected. No suit or proceeding shall be maintained in any court for the recovery of any national internal revenue tax hereafter alleged to have been erroneously or illegally assessed or collected, or of any penalty claimed to have been collected without authority, of any sum alleged to have been excessively or in any manner wrongfully collected without authority, or of any sum alleged to have been excessively or in any manner wrongfully collected, until a claim for refund or credit has been duly filed with the Commissioner; but such suit or proceeding may be maintained, whether or not such tax, penalty, or sum has been paid under protest or duress. In any case, no such suit or proceeding shall be filed after the expiration of two (2) years from the date of payment of the tax or penalty regardless of any supervening cause that may arise after payment: Provided, however, That the Commissioner may, even without a written claim therefor, refund or credit any tax, where on the face of the return upon which payment was made, such payment appears clearly to have been erroneously paid. Notably, the above provisions also set a two-year prescriptive period, reckoned from date of payment of the tax or penalty, for the filing of a claim of refund or tax credit. Notably too, both provisions apply only to instances of erroneous payment or illegal collection of internal revenue taxes. MPCs creditable input VAT not erroneously paid For perspective, under Sec. 105 of the NIRC, creditable input VAT is an indirect tax which can be shifted or passed on to the buyer, transferee, or lessee of the goods, properties, or services of the taxpayer. The fact that the subsequent sale or transaction involves a wholly-tax exempt client, resulting in a zero-rated or effectively zero-rated transaction, does not, standing alone, deprive the taxpayer of its right to a refund for any unutilized creditable input VAT, albeit the erroneous, illegal, or wrongful payment angle does not enter the equation. xxxx Considering the foregoing discussion, it is clear that Sec. 112 (A) of the NIRC, providing a two-year prescriptive period reckoned from the close of the taxable quarter when the relevant sales or transactions were made pertaining to the creditable input VAT, applies to the instant case, and not to the other actions which refer to erroneous payment of taxes.46 (Emphasis supplied.) In view of the foregoing, we find that the CTA En Banc erroneously applied Sections 114(A) and 229 of the NIRC in computing the two-year prescriptive period for claiming refund/credit of unutilized input VAT. To be clear, Section 112 of the NIRC is the pertinent provision for the

refund/credit of input VAT. Thus, the two-year period should be reckoned from the close of the taxable quarter when the sales were made. The administrative claim was timely filed Bearing this in mind, we shall now proceed to determine whether the administrative claim was timely filed. Relying on Article 13 of the Civil Code,47 which provides that a year is equivalent to 365 days, and taking into account the fact that the year 2004 was a leap year, petitioner submits that the two-year period to file a claim for tax refund/ credit for the period July 1, 2002 to September 30, 2002 expired on September 29, 2004.48 We do not agree. In Commissioner of Internal Revenue v. Primetown Property Group, Inc.,49 we said that as between the Civil Code, which provides that a year is equivalent to 365 days, and the Administrative Code of 1987, which states that a year is composed of 12 calendar months, it is the latter that must prevail following the legal maxim, Lex posteriori derogat priori.50 Thus: Both Article 13 of the Civil Code and Section 31, Chapter VIII, Book I of the Administrative Code of 1987 deal with the same subject matter the computation of legal periods. Under the Civil Code, a year is equivalent to 365 days whether it be a regular year or a leap year. Under the Administrative Code of 1987, however, a year is composed of 12 calendar months. Needless to state, under the Administrative Code of 1987, the number of days is irrelevant. There obviously exists a manifest incompatibility in the manner of computing legal periods under the Civil Code and the Administrative Code of 1987. For this reason, we hold that Section 31, Chapter VIII, Book I of the Administrative Code of 1987, being the more recent law, governs the computation of legal periods. Lex posteriori derogat priori. Applying Section 31, Chapter VIII, Book I of the Administrative Code of 1987 to this case, the two-year prescriptive period (reckoned from the time respondent filed its final adjusted return on April 14, 1998) consisted of 24 calendar months, computed as follows: Year 1 1st calendar month 2nd calendar month 3rd calendar month 4th calendar month 5th calendar month 6th calendar month 7th calendar month April 15, 1998 to May 14, 1998 May 15, 1998 to June 14, 1998 June 15, 1998 to July 14, 1998 July 15, 1998 to August 14, 1998 August 15, 1998 to September 14, 1998 September 15, 1998 to October 14, 1998 October 15, 1998 to November 14, 1998

8th calendar month 9th calendar month 10th calendar month 11th calendar month 12th calendar month Year 2 13th calendar month 14th calendar month 15th calendar month 16th calendar month 17th calendar month 18th calendar month 19th calendar month 20th calendar month 21st calendar month 22nd calendar month 23rd calendar month 24th calendar month

November 15, 1998 to December 14, 1998 December 15, 1998 to January 14, 1999 January 15, 1999 to February 14, 1999 February 15, 1999 to March 14, 1999 March 15, 1999 to April 14, 1999 April 15, 1999 to May 14, 1999 May 15, 1999 to June 14, 1999 June 15, 1999 to July 14, 1999 July 15, 1999 to August 14, 1999 August 15, 1999 to September 14, 1999 September 15, 1999 to October 14, 1999 October 15, 1999 to November 14, 1999 November 15, 1999 to December 14, 1999 December 15, 1999 to January 14, 2000 January 15, 2000 to February 14, 2000 February 15, 2000 to March 14, 2000 March 15, 2000 to April 14, 2000

We therefore hold that respondent's petition (filed on April 14, 2000) was filed on the last day of the 24th calendar month from the day respondent filed its final adjusted return. Hence, it was filed within the reglementary period.51 Applying this to the present case, the two-year period to file a claim for tax refund/credit for the period July 1, 2002 to September 30, 2002 expired on September 30, 2004. Hence, respondents administrative claim was timely filed. The filing of the judicial claim was premature However, notwithstanding the timely filing of the administrative claim, we are constrained to deny respondents claim for tax refund/credit for having been filed in violation of Section 112(D) of the NIRC, which provides that: SEC. 112. Refunds or Tax Credits of Input Tax. xxxx

(D) Period within which Refund or Tax Credit of Input Taxes shall be Made. In proper cases, the Commissioner shall grant a refund or issue the tax credit certificate for creditable input taxes within one hundred twenty (120) days from the date of submission of complete documents in support of the application filed in accordance with Subsections (A) and (B) hereof. In case of full or partial denial of the claim for tax refund or tax credit, or the failure on the part of the Commissioner to act on the application within the period prescribed above, the taxpayer affected may, within thirty (30) days from the receipt of the decision denying the claim or after the expiration of the one hundred twenty day-period, appeal the decision or the unacted claim with the Court of Tax Appeals. (Emphasis supplied.) Section 112(D) of the NIRC clearly provides that the CIR has "120 days, from the date of the submission of the complete documents in support of the application [for tax refund/credit]," within which to grant or deny the claim. In case of full or partial denial by the CIR, the taxpayers recourse is to file an appeal before the CTA within 30 days from receipt of the decision of the CIR. However, if after the 120-day period the CIR fails to act on the application for tax refund/credit, the remedy of the taxpayer is to appeal the inaction of the CIR to CTA within 30 days. In this case, the administrative and the judicial claims were simultaneously filed on September 30, 2004. Obviously, respondent did not wait for the decision of the CIR or the lapse of the 120day period. For this reason, we find the filing of the judicial claim with the CTA premature. Respondents assertion that the non-observance of the 120-day period is not fatal to the filing of a judicial claim as long as both the administrative and the judicial claims are filed within the two-year prescriptive period52 has no legal basis. There is nothing in Section 112 of the NIRC to support respondents view. Subsection (A) of the said provision states that "any VAT-registered person, whose sales are zero-rated or effectively zero-rated may, within two years after the close of the taxable quarter when the sales were made, apply for the issuance of a tax credit certificate or refund of creditable input tax due or paid attributable to such sales." The phrase "within two (2) years x x x apply for the issuance of a tax credit certificate or refund" refers to applications for refund/credit filed with the CIR and not to appeals made to the CTA. This is apparent in the first paragraph of subsection (D) of the same provision, which states that the CIR has "120 days from the submission of complete documents in support of the application filed in accordance with Subsections (A) and (B)" within which to decide on the claim. In fact, applying the two-year period to judicial claims would render nugatory Section 112(D) of the NIRC, which already provides for a specific period within which a taxpayer should appeal the decision or inaction of the CIR. The second paragraph of Section 112(D) of the NIRC envisions two scenarios: (1) when a decision is issued by the CIR before the lapse of the 120-day period; and (2) when no decision is made after the 120-day period. In both instances, the taxpayer has 30 days within which to file an appeal with the CTA. As we see it then, the 120-day period is crucial in filing an appeal with the CTA.

With regard to Commissioner of Internal Revenue v. Victorias Milling, Co., Inc.53 relied upon by respondent, we find the same inapplicable as the tax provision involved in that case is Section 306, now Section 229 of the NIRC. And as already discussed, Section 229 does not apply to refunds/credits of input VAT, such as the instant case. In fine, the premature filing of respondents claim for refund/credit of input VAT before the CTA warrants a dismissal inasmuch as no jurisdiction was acquired by the CTA. WHEREFORE, the Petition is hereby GRANTED. The assailed July 30, 2008 Decision and the October 6, 2008 Resolution of the Court of Tax Appeals are hereby REVERSED and SET ASIDE. The Court of Tax Appeals Second Division is DIRECTED to dismiss CTA Case No. 7065 for having been prematurely filed.

G.R. No. 178090

February 8, 2010

PANASONIC COMMUNICATIONS IMAGING CORPORATION OF THE PHILIPPINES (formerly MATSUSHITA BUSINESS MACHINE CORPORATION OF THE PHILIPPINES), Petitioner, vs. COMMISSIONER OF INTERNAL REVENUE, Respondent. DECISION ABAD, J.: This petition for review puts in issue the May 23, 2007 Decision1 of the Court of Tax Appeals (CTA) en banc in CTA EB 239, entitled "Panasonic Communications Imaging Corporation of the Philippines v. Commissioner of Internal Revenue," which affirmed the denial of petitioners claim for refund. The Facts and the Case Petitioner Panasonic Communications Imaging Corporation of the Philippines (Panasonic) produces and exports plain paper copiers and their sub-assemblies, parts, and components. It is registered with the Board of Investments as a preferred pioneer enterprise under the Omnibus Investments Code of 1987. It is also a registered value-added tax (VAT) enterprise. From April 1 to September 30, 1998 and from October 1, 1998 to March 31, 1999, petitioner Panasonic generated export sales amounting to US$12,819,475.15 and US$11,859,489.78, respectively, for a total of US$24,678,964.93. Believing that these export sales were zero-rated for VAT under Section 106(A)(2)(a)(1) of the 1997 National Internal Revenue Code as amended by Republic Act (R.A.) 8424 (1997 NIRC),2 Panasonic paid input VAT of P4,980,254.26 and P4,388,228.14 for the two periods or a total of P9,368,482.40 attributable to its zero-rated sales.

Claiming that the input VAT it paid remained unutilized or unapplied, on March 12, 1999 and July 20, 1999 petitioner Panasonic filed with the Bureau of Internal Revenue (BIR) two separate applications for refund or tax credit of what it paid. When the BIR did not act on the same, Panasonic filed on December 16, 1999 a petition for review with the CTA, averring the inaction of the respondent Commissioner of Internal Revenue (CIR) on its applications. After trial or on August 22, 2006 the CTAs First Division rendered judgment,3 denying the petition for lack of merit. The First Division said that, while petitioner Panasonics export sales were subject to 0% VAT under Section 106(A)(2)(a)(1) of the 1997 NIRC, the same did not qualify for zero-rating because the word "zero-rated" was not printed on Panasonics export invoices. This omission, said the First Division, violates the invoicing requirements of Section 4.108-1 of Revenue Regulations (RR) 7-95.4 Its motion for reconsideration having been denied, on January 5, 2007 petitioner Panasonic appealed the First Divisions decision to the CTA en banc. On May 23, 2007 the CTA en banc upheld the First Divisions decision and resolution and dismissed the petition. Panasonic filed a motion for reconsideration of the en banc decision but this was denied. Thus, petitioner filed the present petition in accordance with R.A. 9282.5 The Issue Presented The sole issue presented in this case is whether or not the CTA en banc correctly denied petitioner Panasonics claim for refund of the VAT it paid as a zero-rated taxpayer on the ground that its sales invoices did not state on their faces that its sales were "zero-rated." The Courts Ruling The VAT is a tax on consumption, an indirect tax that the provider of goods or services may pass on to his customers. Under the VAT method of taxation, which is invoice-based, an entity can subtract from the VAT charged on its sales or outputs the VAT it paid on its purchases, inputs and imports.6 For example, when a seller charges VAT on its sale, it issues an invoice to the buyer, indicating the amount of VAT he charged. For his part, if the buyer is also a seller subjected to the payment of VAT on his sales, he can use the invoice issued to him by his supplier to get a reduction of his own VAT liability. The difference in tax shown on invoices passed and invoices received is the tax paid to the government. In case the tax on invoices received exceeds that on invoices passed, a tax refund may be claimed. Under the 1997 NIRC, if at the end of a taxable quarter the seller charges output taxes7 equal to the input taxes8 that his suppliers passed on to him, no payment is required of him. It is when his output taxes exceed his input taxes that he has to pay the excess to the BIR. If the input taxes exceed the output taxes, however, the excess payment shall be carried over to the succeeding quarter or quarters. Should the input taxes result from zero-rated or effectively zero-rated transactions or from the acquisition of capital goods, any excess over the output taxes shall instead be refunded to the taxpayer.9

Zero-rated transactions generally refer to the export sale of goods and services. The tax rate in this case is set at zero. When applied to the tax base or the selling price of the goods or services sold, such zero rate results in no tax chargeable against the foreign buyer or customer. But, although the seller in such transactions charges no output tax, he can claim a refund of the VAT that his suppliers charged him. The seller thus enjoys automatic zero rating, which allows him to recover the input taxes he paid relating to the export sales, making him internationally competitive.10 For the effective zero rating of such transactions, however, the taxpayer has to be VATregistered and must comply with invoicing requirements.11 Interpreting these requirements, respondent CIR ruled that under Revenue Memorandum Circular (RMC) 42-2003, the taxpayers failure to comply with invoicing requirements will result in the disallowance of his claim for refund. RMC 42-2003 provides: A-13. Failure by the supplier to comply with the invoicing requirements on the documents supporting the sale of goods and services will result to the disallowance of the claim for input tax by the purchaser-claimant.1avvphi1 If the claim for refund/TCC is based on the existence of zero-rated sales by the taxpayer but it fails to comply with the invoicing requirements in the issuance of sales invoices (e.g., failure to indicate the TIN), its claim for tax credit/refund of VAT on its purchases shall be denied considering that the invoice it is issuing to its customers does not depict its being a VATregistered taxpayer whose sales are classified as zero-rated sales. Nonetheless, this treatment is without prejudice to the right of the taxpayer to charge the input taxes to the appropriate expense account or asset account subject to depreciation, whichever is applicable. Moreover, the case shall be referred by the processing office to the concerned BIR office for verification of other tax liabilities of the taxpayer. Petitioner Panasonic points out, however, that in requiring the printing on its sales invoices of the word "zero-rated," the Secretary of Finance unduly expanded, amended, and modified by a mere regulation (Section 4.108-1 of RR 7-95) the letter and spirit of Sections 113 and 237 of the 1997 NIRC, prior to their amendment by R.A. 9337.12 Panasonic argues that the 1997 NIRC, which applied to its paymentsspecifically Sections 113 and 237required the VAT-registered taxpayers receipts or invoices to indicate only the following information: (1) A statement that the seller is a VAT-registered person, followed by his taxpayer's identification number (TIN); (2) The total amount which the purchaser pays or is obligated to pay to the seller with the indication that such amount includes the value-added tax; (3) The date of transaction, quantity, unit cost and description of the goods or properties or nature of the service; and (4) The name, business style, if any, address and taxpayers identification number (TIN) of the purchaser, customer or client.

Petitioner Panasonic points out that Sections 113 and 237 did not require the inclusion of the word "zero-rated" for zero-rated sales covered by its receipts or invoices. The BIR incorporated this requirement only after the enactment of R.A. 9337 on November 1, 2005, a law that did not yet exist at the time it issued its invoices. But when petitioner Panasonic made the export sales subject of this case, i.e., from April 1998 to March 1999, the rule that applied was Section 4.108-1 of RR 7-95, otherwise known as the Consolidated Value-Added Tax Regulations, which the Secretary of Finance issued on December 9, 1995 and took effect on January 1, 1996. It already required the printing of the word "zero-rated" on the invoices covering zero-rated sales. When R.A. 9337 amended the 1997 NIRC on November 1, 2005, it made this particular revenue regulation a part of the tax code. This conversion from regulation to law did not diminish the binding force of such regulation with respect to acts committed prior to the enactment of that law. Section 4.108-1 of RR 7-95 proceeds from the rule-making authority granted to the Secretary of Finance under Section 245 of the 1977 NIRC (Presidential Decree 1158) for the efficient enforcement of the tax code and of course its amendments.13 The requirement is reasonable and is in accord with the efficient collection of VAT from the covered sales of goods and services. As aptly explained by the CTAs First Division, the appearance of the word "zero-rated" on the face of invoices covering zero-rated sales prevents buyers from falsely claiming input VAT from their purchases when no VAT was actually paid. If, absent such word, a successful claim for input VAT is made, the government would be refunding money it did not collect.14 Further, the printing of the word "zero-rated" on the invoice helps segregate sales that are subject to 10% (now 12%) VAT from those sales that are zero-rated.15 Unable to submit the proper invoices, petitioner Panasonic has been unable to substantiate its claim for refund. Petitioner Panasonics citation of Intel Technology Philippines, Inc. v. Commissioner of Internal Revenue16 is misplaced. Quite the contrary, it strengthens the position taken by respondent CIR. In that case, the CIR denied the claim for tax refund on the ground of the taxpayers failure to indicate on its invoices the "BIR authority to print." But Sec. 4.108-1 required only the following to be reflected on the invoice: 1. The name, taxpayers identification number (TIN) and address of seller; 2. Date of transaction; 3. Quantity, unit cost and description of merchandise or nature of service; 4. The name, TIN, business style, if any, and address of the VAT-registered purchaser, customer or client; 5. The word "zero-rated" imprinted on the invoice covering zero-rated sales; and 6. The invoice value or consideration.

This Court held that, since the "BIR authority to print" is not one of the items required to be indicated on the invoices or receipts, the BIR erred in denying the claim for refund. Here, however, the ground for denial of petitioner Panasonics claim for tax refundthe absence of the word "zero-rated" on its invoicesis one which is specifically and precisely included in the above enumeration. Consequently, the BIR correctly denied Panasonics claim for tax refund. This Court will not set aside lightly the conclusions reached by the CTA which, by the very nature of its functions, is dedicated exclusively to the resolution of tax problems and has accordingly developed an expertise on the subject, unless there has been an abuse or improvident exercise of authority.17 Besides, statutes that grant tax exemptions are construed strictissimi juris against the taxpayer and liberally in favor of the taxing authority. Tax refunds in relation to the VAT are in the nature of such exemptions. The general rule is that claimants of tax refunds bear the burden of proving the factual basis of their claims. Taxes are the lifeblood of the nation. Therefore, statutes that allow exemptions are construed strictly against the grantee and liberally in favor of the government.18 WHEREFORE, the petition is DENIED for lack of merit.

G.R. No. 181298

January 10, 2011

BELLE CORPORATION, Petitioner, vs. COMMISSIONER OF INTERNAL REVENUE, Respondent. DECISION DEL CASTILLO, J.: Section 69 of the old National Internal Revenue Code (NIRC) allows unutilized tax credits to be refunded as long as the claim is filed within the prescriptive period. This, however, no longer holds true under Section 76 of the 1997 NIRC as the option to carry-over excess income tax payments to the succeeding taxable year is now irrevocable. This Petition for Review on Certiorari1 under Rule 45 of the Rules of Court seeks to set aside the January 25, 2007 Decision2 and the January 21, 2008 Resolution3 of the Court of Appeals (CA). Factual Antecedents Petitioner Belle Corporation is a domestic corporation engaged in the real estate and property business.4 On May 30, 1997, petitioner filed with the Bureau of Internal Revenue (BIR) its Income Tax Return (ITR) for the first quarter of 1997, showing a gross income of P741,607,495.00, a deduction of P65,381,054.00, a net taxable income of P676,226,441.00 and an income tax due of

P236,679,254.00, which petitioner paid on even date through PCI Bank, Tektite Tower Branch, an Authorized Agent Bank of the BIR.5 On August 14, 1997, petitioner filed with the BIR its second quarter ITR, declaring an overpayment of income taxes in the amount of P66,634,290.00. The computation of which is reproduced below: Gross Income Less: Deductions Taxable Income Tax Rate Tax Due Less: Tax Credits/Payments (a) Prior Years Excess Tax Credit (b) 1st Quarter Payment (c) Creditable Withholding Tax P236,679,254.00 ________________ (P 66,634,290.00)6 P 833,186,319.00 347,343,565.00 P 485,842,754.00 x 35% P 170,044,964.00

In view of the overpayment, no taxes were paid for the second and third quarters of 1997.7 Petitioners ITR for the taxable year ending December 31, 1997 thereby reflected an overpayment of income taxes in the amount of P132,043,528.00, computed as follows: Gross Income Less: Deductions Taxable Income Tax Rate Tax Due Less: Tax Credits/Payments (a) Prior Years Excess Tax Credit (b) 1st Quarter Payment (c) Creditable Withholding Tax P236,679,254.00 (1,410,295.00) (238,089,549.00) P 1,182,473,910.00 879,485,278.00 P 302,988,362.00 x 35% P 106,046,021.00

REFUNDABLE AMOUNT

(P 132,043,528.00)8

Instead of claiming the amount as a tax refund, petitioner decided to apply it as a tax credit to the succeeding taxable year by marking the tax credit option box in its 1997 ITR.9 For the taxable year 1998, petitioners amended ITR showed an overpayment of P106,447,318.00, computed as follows: Gross Income Less: Deduction Taxable Income (Loss) Tax Rate Tax Due (Regular Income Tax) Minimum Corporate Income Tax Tax Due Less: Tax Credits/Payments (a) Prior years excess Tax Credits (P 132,041,528.00) (b) Quarterly payment (c) Creditable tax withheld Tax Payable/Overpayment (P 106,447,318.00)10 P 1,279,810,489.00 1,346,553,546.00 (P 66,743,057.00) 34% - NIL P 25,596,210.00 25,596,210.00

On April 12, 2000, petitioner filed with the BIR an administrative claim for refund of its unutilized excess income tax payments for the taxable year 1997 in the amount of P106,447,318.00.11 Notwithstanding the filing of the administrative claim for refund, petitioner carried over the amount of P106,447,318.00 to the taxable year 1999 and applied a portion thereof to its 1999 Minimum Corporate Income Tax (MCIT) liability, as evidenced by its 1999 ITR.12 Thus: Gross Income Less: Deduction Taxable Income Tax Due P 708,888,638.00 1,328,101,776.00 (P 619,213,138.00)

Minimum Corporate Income Tax Less: Tax Credits/Payments (a) Prior years excess Credit P 106,447,318.00

P 14,185,874.00

(b) Tax Payments for the 1st & 3rd Qtrs. 0 (c) Creditable tax withheld TAX PAYABLE/REFUNDABLE Proceedings before the Court of Tax Appeals (CTA) On April 14, 2000, due to the inaction of the respondent Commissioner of Internal Revenue (CIR) and in order to toll the running of the two-year prescriptive period, petitioner appealed its claim for refund of unutilized excess income tax payments for the taxable year 1997 in the amount of P106,447,318.00 with the CTA via a Petition for Review,14 docketed as CTA Case No. 6070. In answer thereto, respondent interposed that: 4. Petitioners alleged claim for refund/tax credit is subject to administrative routinary investigation/examination by respondents Bureau; 5. Petitioner failed miserably to show that the total amount of P106,447,318.00 claimed as overpaid or excess income tax is refundable; 6. Taxes paid and collected are presumed to have been paid in accordance with law; hence, not refundable; 7. In an action for tax refund, the burden is on the taxpayer to establish its right to refund, and failure to sustain the burden is fatal to the claim for refund; 8. It is incumbent upon petitioner to show that it has complied with the provisions of Section 204 (c) in relation to Section 229 of the tax Code; 9. Well-established is the rule that refunds/tax credits are construed strictly against the taxpayer as they partake the nature of tax exemptions.15 To prove entitlement to the refund, petitioner submitted, among others, the following documents: its ITR for the first quarter of taxable year 1997 (Exhibit "B"),16 its tentative ITRs for taxable years 1997 (Exhibit "D")17 and 1998 (Exhibit "H"),18 its final ITRs for taxable years 1997 (Exhibit "E"),19 1998 (Exhibit "I")20 and 1999 (Exhibit "J"),21 its Letter Claim for Refund filed with the BIR (Exhibit "K")22 and the Official Receipt issued by PCI Bank showing the income 0 P 106,447,318.00 (P 92,261,444.00)13

tax payment made by petitioner in the amount of P236,679,254.00 for the first quarter of 1997 (Exhibit "C").23 On April 10, 2001, the CTA rendered a Decision24 denying petitioners claim for refund. It found: [T]hat all the allegations made by the Petitioner as well as the figures accompanying Petitioners claim are substantiated by documentary evidence but noticed some flaws in Petitioners application of the pertinent laws involved. It bears stressing that the applicable provision in the case at bar is Section 69 of the old Tax Code and not Section 76 of the 1997 Tax Code. Settled is the rule that under Section 69 of the old Tax Code, the carrying forward of any excess/overpaid income tax for a given taxable year is limited only up to the succeeding taxable year. A painstaking scrutiny of Petitioners income tax returns would show that Petitioner carried over its 1997 refundable tax of P132,043,528.00 to the succeeding year of 1998 yielding an overpayment of P106,447,318.00 (Exhibit I-1) after deducting therefrom the minimum Corporate Income tax of P25,596,210.00. However, Petitioner even went further to the taxable year 1999 and applied the Prior Years (1998) Excess Credit of P106,447,318.00 to its income tax liability.1avvphi1 True enough, upon verification of Petitioners 1999 Corporate Annual Income Tax Return (Exh. I), this Court found that the whole amount of P106,447,318.00 representing its prior year's excess credit (subject of this claim) was carried forward to its 1999 income tax liability, details of the 1999 Income Tax Return are shown below as follows: Gross Income Less: Deduction Taxable Income Tax Due Minimum Corporate Income Tax Less: Tax Credits/Payments (a) Prior year's excess Credit P 106,447,318.00 P 14,185,874.00 P 708,888,638.00 1,328,101,776.00 (P 619,213,138.00)

(b) Tax Payments for the 1st & 3rd Qtrs. 0 (c) Creditable tax withheld TAX PAYABLE/REFUNDABLE 0 P106,447,318.00 (P 92,261,444.00)

It is an elementary rule in taxation that an automatic carry over of an excess income tax payment should only be made for the succeeding year. (Paseo Realty and Devt. Corp. vs. CIR, CTA Case No. 4528, April 30, 1993) True enough, implicit from the provisions of Section 69 of the NIRC, as amended, (supra) is the fact that the refundable amount may be credited against the income tax liabilities for the taxable quarters of the succeeding taxable year not succeeding years; and that the carry-over is only limited to the quarters of the succeeding taxable year. (citing ANSCOR Hagedorn Securities Inc. vs. CIR, CA-GR SP 38177, December 21, 1999) To allow the application of excess taxes paid for two successive years would run counter to the specific provision of the law above-mentioned.25 (Emphasis supplied.) Petitioner sought reconsideration26 of the CTAs denial of its claim for refund, but the same was denied in a Resolution27 dated June 5, 2001, prompting petitioner to elevate the matter to the CA via a Petition for Review28 under Rule 43 of the Rules of Court. Ruling of the Court of Appeals On January 25, 2007, the CA, applying Philippine Bank of Communications v. Commissioner of Internal Revenue,29 denied the petition. The CA explained that the overpayment for taxable year 1997 can no longer be carried over to taxable year 1999 because excess income payments can only be credited against the income tax liabilities of the succeeding taxable year, in this case up to 1998 only and not beyond.30 Neither can the overpayment be refunded as the remedies of automatic tax crediting and tax refund are alternative remedies.31 Thus, the CA ruled: [W]hile BELLE may not have fully enjoyed the complete utilization of its option and the sum of Php106,447,318 still remained after it opted for a tax carry over of its excess payment for the taxable year 1998, but be that as it may, BELLE has only itself to blame for making such useless and damaging option, and BELLE may no longer opt to claim for a refund considering that the remedy of refund is barred after the corporation has previously opted for the tax carry over remedy. As a matter of fact, the CTA even made the factual findings that BELLE committed an aberration to exhaust its unutilized overpaid income tax by carrying it over further to the taxable year 1999, which is a blatant transgression of the "succeeding taxable year limit" provided for under Section 69 of the old NIRC.32 (Emphasis supplied) Hence, the fallo of the Decision reads: WHEREFORE, premises considered, the instant Petition for Review is DENIED, and accordingly, the herein impugned April 10, 2001 Decision and June 5, 2001 Resolution of the CTA are hereby affirmed. SO ORDERED.33 Petitioner moved for reconsideration.34 The CA, however, denied the same in a Resolution35 dated January 21, 2008.

Issues Aggrieved, petitioner availed of the present recourse, raising the following assignment of errors: A. THE CA COMMITTED SERIOUS ERROR OF LAW IN APPLYING THE PBCOM CASE. A.1. THE [DECISION IN THE] PBCOM CASE HAS ALREADY BEEN REPEALED. A.2. ASSUMING ARGUENDO THAT THE [DECISION IN THE] PBCOM CASE HAS NOT BEEN REPEALED, IT HAS NO APPLICATION TO BELLE. B. THE CA COMMITTED SERIOUS ERROR OF LAW IN FINDING THAT BELLES REFUND CLAIM IS NOT ON ALL FOURS WITH THE CASES OF BPI FAMILY AND AB LEASING. B.1. BELLES CARRYING-OVER OF ITS EXCESS INCOME TAX PAID FOR 1997 TO 1999 (BEYOND THE SUBSEQUENT YEAR) IS IMMATERIAL. B.2. BELLES PARTIAL USE OF ITS EXCESS INCOME TAX PAID IN 1998 (THE SUBSEQUENT YEAR) DOES NOT PRECLUDE BELLE FROM ASKING FOR A REFUND.36 In a nutshell, the issue boils down to whether petitioner is entitled to a refund of its excess income tax payments for the taxable year 1997 in the amount of P106,447,318.00. Petitioners Arguments Petitioner insists that it is entitled to a refund as the ruling in Philippine Bank of Communications v. Commissioner of Internal Revenue37 relied upon by the CA in denying its claim has been overturned by BPI-Family Savings Bank, Inc. v. Court of Appeals,38 AB Leasing and Finance Corporation v. Commissioner of Internal Revenue,39 Calamba Steel Center, Inc. v. Commissioner of Internal Revenue,40 and State Land Investment Corporation v. Commissioner of Internal Revenue.41 In these cases, the taxpayers were allowed to claim refund of unutilized tax credits.42 Similarly, in this case, petitioner asserts that it may still recover unutilized tax credits via a claim for refund.43 And while petitioner admits that it has committed a "blatant transgression" of the "succeeding taxable year limit" when it carried over its 1997 excess income tax payments beyond the taxable year 1998, petitioner believes that this should not result in the denial of its claim for refund but should only invalidate the application of its 1997 unutilized excess income tax payments to its 1999 income tax liabilities.44 Hence, petitioner postulates that a claim for refund of its unutilized tax credits for the taxable year 1997 may still be made because the carry-over thereof to the

taxable year 1999 produced no legal effect, and is, therefore, immaterial to the resolution of its claim for refund.45 Respondents Arguments Respondent, on the other hand, maintains that the cases of BPI-Family Savings Bank46 and AB Leasing47 are inapplicable as the facts obtaining therein are different from those of the present case.48 What is controlling, therefore, is the ruling in Philippine Bank of Communications,49 that tax refund and tax credit are alternative remedies; thus, "the choice of one precludes the other."50 Respondent, therefore, submits that since petitioner has already applied its 1997 excess income tax payments to its liabilities for taxable year 1998, it is precluded from carrying over the same to taxable year 1999, or from filing a claim for refund.51 Our Ruling The petition has no merit. Both the CTA and the CA erred in applying Section 6952 of the old NIRC. The law applicable is Section 76 of the NIRC. Unutilized excess income tax payments may be refunded within two years from the date of payment under Section 69 of the old NIRC Under Section 69 of the old NIRC, in case of overpayment of income taxes, a corporation may either file a claim for refund or carry-over the excess payments to the succeeding taxable year. Availment of one remedy, however, precludes the other.53 Although these remedies are mutually exclusive, we have in several cases allowed corporations, which have previously availed of the tax credit option, to file a claim for refund of their unutilized excess income tax payments. In BPI-Family Savings Bank,54 the bank availed of the tax credit option but since it suffered a net loss the succeeding year, the tax credit could not be applied; thus, the bank filed a claim for refund to recover its excess creditable taxes. Brushing aside technicalities, we granted the claim for refund. Likewise, in Calamba Steel Center, Inc.,55 we allowed the refund of excess income taxes paid in 1995 since these could not be credited to taxable year 1996 due to business losses. In that case, we declared that "a tax refund may be claimed even beyond the taxable year following that in which the tax credit arises x x x provided that the claim for such a refund is made within two years after payment of said tax."56 In State Land Investment Corporation,57 we reiterated that "if the excess income taxes paid in a given taxable year have not been entirely used by a x x x corporation against its quarterly income tax liabilities for the next taxable year, the unused amount of the excess may still be refunded, provided that the claim for such a refund is made within two years after payment of the tax."58

Thus, under Section 69 of the old NIRC, unutilized tax credits may be refunded as long as the claim is filed within the two-year prescriptive period. The option to carry over excess income tax payments is irrevocable under Section 76 of the 1997 NIRC This rule, however, no longer applies as Section 76 of the 1997 NIRC now reads: Section 76. Final Adjustment Return. Every corporation liable to tax under Section 24 shall file a final adjustment return covering the total net income for the preceding calendar or fiscal year. If the sum of the quarterly tax payments made during the said taxable year is not equal to the total tax due on the entire taxable net income of that year the corporation shall either: (a) Pay the excess tax still due; or (b) Be refunded the excess amount paid, as the case may be. In case the corporation is entitled to a refund of the excess estimated quarterly income taxes paid, the refundable amount shown on its final adjustment return may be credited against the estimated quarterly income tax liabilities for the taxable quarters of the succeeding taxable years. Once the option to carry over and apply the excess quarterly income tax against income tax due for the taxable quarters of the succeeding taxable years has been made, such option shall be considered irrevocable for that taxable period and no application for tax refund or issuance of a tax credit certificate shall be allowed therefor. (Emphasis supplied) Under the new law, in case of overpayment of income taxes, the remedies are still the same; and the availment of one remedy still precludes the other. But unlike Section 69 of the old NIRC, the carry-over of excess income tax payments is no longer limited to the succeeding taxable year. Unutilized excess income tax payments may now be carried over to the succeeding taxable years until fully utilized. In addition, the option to carry-over excess income tax payments is now irrevocable. Hence, unutilized excess income tax payments may no longer be refunded. In the instant case, both the CTA and the CA applied Section 69 of the old NIRC in denying the claim for refund. We find, however, that the applicable provision should be Section 76 of the 1997 NIRC because at the time petitioner filed its 1997 final ITR, the old NIRC was no longer in force. In Commissioner of Internal Revenue v. McGeorge Food Industries, Inc.,59 we explained that: Section 76 and its companion provisions in Title II, Chapter XII should be applied following the general rule on the prospective application of laws such that they operate to govern the conduct of corporate taxpayers the moment the 1997 NIRC took effect on 1 January 1998. There is no quarrel that at the time respondent filed its final adjustment return for 1997 on 15 April 1998, the deadline under Section 77 (B) of the 1997 NIRC (formerly Section 70(b) of the 1977 NIRC), the 1997 NIRC was already in force, having gone into effect a few months earlier on 1 January 1998. Accordingly, Section 76 is controlling.

The lower courts grounded their contrary conclusion on the fact that respondents overpayment in 1997 was based on transactions occurring before 1 January 1998. This analysis suffers from the twin defects of missing the gist of the present controversy and misconceiving the nature and purpose of Section 76. None of respondents corporate transactions in 1997 is disputed here. Nor can it be argued that Section 76 determines the taxability of corporate transactions. To sustain the rulings below is to subscribe to the untenable proposition that, had Congress in the 1997 NIRC moved the deadline for the filing of final adjustment returns from 15 April to 15 March of each year, taxpayers filing returns after 15 March 1998 can excuse their tardiness by invoking the 1977 NIRC because the transactions subject of the returns took place before 1 January 1998. A keener appreciation of the nature and purpose of the varied provisions of the 1997 NIRC cautions against sanctioning this reasoning.60 Accordingly, since petitioner already carried over its 1997 excess income tax payments to the succeeding taxable year 1998, it may no longer file a claim for refund of unutilized tax credits for taxable year 1997. To repeat, under the new law, once the option to carry-over excess income tax payments to the succeeding years has been made, it becomes irrevocable. Thus, applications for refund of the unutilized excess income tax payments may no longer be allowed. WHEREFORE, the petition is hereby DENIED. The Decision dated January 25, 2007 and the Resolution dated January 21, 2008 of the Court of Appeals are hereby AFFIRMED only insofar as the denial of petitioners claim for refund is concerned. G.R. No. 136975 March 31, 2005

COMMISSION OF INTERNAL REVENUE, Petitioner, vs. HANTEX TRADING CO., INC., respondent. DECISION CALLEJO, SR., J.: Before us is a petition for review of the Decision1 of the Court of Appeals (CA) which reversed the Decision2 of the Court of Tax Appeals (CTA) in CTA Case No. 5126, upholding the deficiency income and sales tax assessments against respondent Hantex Trading Co., Inc. The Antecedents The respondent is a corporation duly organized and existing under the laws of the Philippines. Being engaged in the sale of plastic products, it imports synthetic resin and other chemicals for the manufacture of its products. For this purpose, it is required to file an Import Entry and Internal Revenue Declaration (Consumption Entry) with the Bureau of Customs under Section 1301 of the Tariff and Customs Code.

Sometime in October 1989, Lt. Vicente Amoto, Acting Chief of Counter-Intelligence Division of the Economic Intelligence and Investigation Bureau (EIIB), received confidential information that the respondent had imported synthetic resin amounting to P115,599,018.00 but only declared P45,538,694.57.3 According to the informer, based on photocopies of 77 Consumption Entries furnished by another informer, the 1987 importations of the respondent were understated in its accounting records.4 Amoto submitted a report to the EIIB Commissioner recommending that an inventory audit of the respondent be conducted by the Internal Inquiry and Prosecution Office (IIPO) of the EIIB.5 Acting on the said report, Jose T. Almonte, then Commissioner of the EIIB, issued Mission Order No. 398-896 dated November 14, 1989 for the audit and investigation of the importations of Hantex for 1987. The IIPO issued subpoena duces tecum and ad testificandum for the president and general manager of the respondent to appear in a hearing and bring the following: 1. Books of Accounts for the year 1987; 2. Record of Importations of Synthetic Resin and Calcium Carbonate for the year 1987; 3. Income tax returns & attachments for 1987; and 4. Record of tax payments.7 However, the respondents president and general manager refused to comply with the subpoena, contending that its books of accounts and records of importation of synthetic resin and calcium bicarbonate had been investigated repeatedly by the Bureau of Internal Revenue (BIR) on prior occasions.8 The IIPO explained that despite such previous investigations, the EIIB was still authorized to conduct an investigation pursuant to Section 26-A of Executive Order No. 127. Still, the respondent refused to comply with the subpoena issued by the IIPO. The latter forthwith secured certified copies of the Profit and Loss Statements for 1987 filed by the respondent with the Securities and Exchange Commission (SEC).9 However, the IIPO failed to secure certified copies of the respondents 1987 Consumption Entries from the Bureau of Customs since, according to the custodian thereof, the original copies had been eaten by termites.10 In a Letter dated June 28, 1990, the IIPO requested the Chief of the Collection Division, Manila International Container Port, and the Acting Chief of the Collection Division, Port of Manila, to authenticate the machine copies of the import entries supplied by the informer. However, Chief of the Collection Division Merlita D. Tomas could not do so because the Collection Division did not have the original copies of the entries. Instead, she wrote the IIPO that, as gleaned from the records, the following entries had been duly processed and released after the payment of duties and taxes: IMPORTER HANTEX TRADING CO., INC. SERIES OF 1987 ENTRY NO. DATE ENTRY NO. DATE RELEASED RELEASED

03058-87 09120-87 18089-87 19439-87 19441-87 11667-87 23294-87 45478-87 45691-87 25464-87 26483-87 29950-87

1/30/87 3/20/87 5/21/87 6/2/87 6/3/87 4/15/87 7/7/87 11/16/87 12/2/87 7/16/87 7/23/87 8/11/87

50265-87 46427-87 30764-87 30833-87 34690-87 34722-87 43234-87 44850-87 44851-87 46461-87 46467-87 48091-87

12/9/87 11/27/87 8/21/87 8/20/87 9/16/87 9/11/87 11/2/87 11/16/87 11/16/87 11/19/87 11/18/87 11-27-8711

Acting Chief of the Collection Division of the Bureau of Customs Augusto S. Danganan could not authenticate the machine copies of the import entries as well, since the original copies of the said entries filed with the Bureau of Customs had apparently been eaten by termites. However, he issued a certification that the following enumerated entries were filed by the respondent which were processed and released from the Port of Manila after payment of duties and taxes, to wit: Hantex Trading Co., Inc. Entry No. Date Released Entry No. Date Released 3903 1/29/87 22869 4/8/87 4414 1/20/87 19441 3/31/87 10683 2/17/87 24189 4/21/87 12611 2/24/87 26431 4/20/87 12989 2/26/87 45478 7/3/87 17050 3/13/87 26796 4/23/87 17169 3/13/87 28827 4/30/87 18089 3/16/87 31617 5/14/87 19439 4/1/87 39068 6/5/87 21189 4/3/87 42581 6/21/87 43451 6/29/87 42793 6/23/87 42795 6/23/87 45477 7/3/87 35582 not received 85830 11/13/87 45691 7/3/87 86650 not received 46187 7/8/87 87647 11/18/87 46427 7/3/87 88829 11/23/87 57669 8/12/87 92293 12/3/87

62471 63187 66859 67890 68115 69974 72213 77688 84253 85534

8/28/87 9/2/87 9/15/87 9/17/87 9/15/87 9/24/87 10/2/87 10/16/87 11/10/87 11/11/87

93292 96357 96822 98823 99428 99429 99441 101406 101407 3118

12/7/87 12/16/87 12/15/87 not received 12/28/87 12/28/87 12/28/87 1/5/87 1/8/87 1-19-8712

Bienvenido G. Flores, Chief of the Investigation Division, and Lt. Leo Dionela, Lt. Vicente Amoto and Lt. Rolando Gatmaitan conducted an investigation. They relied on the certified copies of the respondents Profit and Loss Statement for 1987 and 1988 on file with the SEC, the machine copies of the Consumption Entries, Series of 1987, submitted by the informer, as well as excerpts from the entries certified by Tomas and Danganan. Based on the documents/records on hand, inclusive of the machine copies of the Consumption Entries, the EIIB found that for 1987, the respondent had importations totaling P105,716,527.00 (inclusive of advance sales tax). Compared with the declared sales based on the Profit and Loss Statements filed with the SEC, the respondent had unreported sales in the amount of P63,032,989.17, and its corresponding income tax liability was P41,916,937.78, inclusive of penalty charge and interests. EIIB Commissioner Almonte transmitted the entire docket of the case to the BIR and recommended the collection of the total tax assessment from the respondent.13 On February 12, 1991, Deputy Commissioner Deoferio, Jr. issued a Memorandum to the BIR Assistant Commissioner for Special Operations Service, directing the latter to prepare a conference letter advising the respondent of its deficiency taxes.14 Meanwhile, as ordered by the Regional Director, Revenue Enforcement Officers Saturnino D. Torres and Wilson Filamor conducted an investigation on the 1987 importations of the respondent, in the light of the records elevated by the EIIB to the BIR, inclusive of the photocopies of the Consumption Entries. They were to ascertain the respondents liability for deficiency sales and income taxes for 1987, if any. Per Torres and Filamors Report dated March 6, 1991 which was based on the report of the EIIB and the documents/records appended thereto, there was a prima facie case of fraud against the respondent in filing its 1987 Consumption Entry reports with the Bureau of Customs. They found that the respondent had unrecorded importation in the total amount of P70,661,694.00, and that the amount was not declared in its income tax return for 1987. The District Revenue Officer and the Regional Director of the BIR concurred with the report.15

Based on the said report, the Acting Chief of the Special Investigation Branch wrote the respondent and invited its representative to a conference at 10:00 a.m. of March 14, 1991 to discuss its deficiency internal revenue taxes and to present whatever documentary and other evidence to refute the same.16 Appended to the letter was a computation of the deficiency income and sales tax due from the respondent, inclusive of increments: B. Computations: 1. Cost of Sales Ratio A2/A1 2. Undeclared Sales Imported A3/B1 3. Undeclared Gross Profit B2-A3 C. Deficiency Taxes Due: 1. Deficiency Income Tax B3 x 35% 50% Surcharge C1 x 50% Interest to 2/28/91 C1 x 57.5% Total 2. Deficiency Sales Tax at 10% at 20% Total Due Less: Advanced Sales Taxes Paid Deficiency Sales Tax 50% Surcharge C2 x 50% Interest to 2/28/91 Total

85.492923% 110,079,491.61 15,969,316.61 5,589,261.00 2,794,630.50 3,213,825.08 11,597,825.58 7,290,082.72 10,493,312.31 17,783,395.03 11,636,352.00 6,147,043.03 3,073,521.52 5,532,338.73 14,752,903.2817

The invitation was reiterated in a Letter dated March 15, 1991. In his Reply dated March 15, 1991, Mariano O. Chua, the President and General Manager of the respondent, requested that the report of Torres and Filamor be set aside on the following claim: [W]e had already been investigated by RDO No. 23 under Letters of Authority Nos. 0322988 RR dated Oct. 1, 1987, 0393561 RR dated Aug. 17, 1988 and 0347838 RR dated March 2, 1988, and re-investigated by the Special Investigation Team on Aug. 17, 1988 under Letter of Authority No. 0357464 RR, and the Intelligence and Investigation Office on Sept. 27, 1988 under Letter of Authority No. 0020188 NA, all for income and business tax liabilities for 1987. The Economic Intelligence and Investigation Bureau on Nov. 20, 1989, likewise, confronted us on the same information for the same year. In all of these investigations, save your request for an informal conference, we welcomed them and proved the contrary of the allegation. Now, with your new inquiry, we think that there will be no end to the problem.

Madam, we had been subjected to so many investigations and re-investigations for 1987 and nothing came out except the payment of deficiency taxes as a result of oversight. Tax evasion through underdeclaration of income had never been proven.18 Invoking Section 23519 of the 1977 National Internal Revenue Code (NIRC), as amended, Chua requested that the inquiry be set aside. The petitioner, the Commissioner of Internal Revenue, through Assistant Commissioner for Collection Jaime M. Maza, sent a Letter dated April 15, 1991 to the respondent demanding payment of its deficiency income tax of P13,414,226.40 and deficiency sales tax of P14,752,903.25, inclusive of surcharge and interest.20 Appended thereto were the Assessment Notices of Tax Deficiency Nos. FAS-1-87-91-001654 and FAS-4-87-91-001655.21 On February 12, 1992, the Chief of the Accounts Receivables/Billing Division of the BIR sent a letter to the respondent demanding payment of its tax liability due for 1987 within ten (10) days from notice, on pain of the collection tax due via a warrant of distraint and levy and/or judicial action.22 The Warrant of Distraint and/or Levy23 was actually served on the respondent on January 21, 1992. On September 7, 1992, it wrote the Commissioner of Internal Revenue protesting the assessment on the following grounds: I. THAT THE ASSESSMENT HAS NO FACTUAL AS WELL AS LEGAL BASIS, THE FACT THAT NO INVESTIGATION OF OUR RECORDS WAS EVER MADE BY THE EIIB WHICH RECOMMENDED ITS ISSUANCE.24 II. THAT GRANTING BUT WITHOUT ADMITTING THAT OUR PURCHASES FOR 1987 AMOUNTED TO P105,716,527.00 AS CLAIMED BY THE EIIB, THE ASSESSMENT OF A DEFICIENCY INCOME TAX IS STILL DEFECTIVE FOR IT FAILED TO CONSIDER OUR REAL PURCHASES OF P45,538,694.57.25 III. THAT THE ASSESSMENT OF A DEFICIENCY SALES TAX IS ALSO BASELESS AND UNFOUNDED CONSIDERING THAT WE HAVE DUTIFULLY PAID THE SALES TAX DUE FROM OUR BUSINESS.26 In view of the impasse, administrative hearings were conducted on the respondents protest to the assessment. During the hearing of August 20, 1993, the IIPO representative presented the photocopies of the Consumption and Import Entries and the Certifications issued by Tomas and Danganan of the Bureau of Customs. The IIPO representative testified that the Bureau of Customs failed to furnish the EIIB with certified copies of the Consumption and Import Entries; hence, the EIIB relied on the machine copies from their informer.27 The respondent wrote the BIR Commissioner on July 12, 1993 questioning the assessment on the ground that the EIIB representative failed to present the original, or authenticated, or duly certified copies of the Consumption and Import Entry Accounts, or excerpts thereof if the original copies were not readily available; or, if the originals were in the official custody of a public officer, certified copies thereof as provided for in Section 12, Chapter 3, Book VII, Administrative Procedure, Administrative Order of 1987. It stated that the only copies of the

Consumption Entries submitted to the Hearing Officer were mere machine copies furnished by an informer of the EIIB. It asserted that the letters of Tomas and Danganan were unreliable because of the following: In the said letters, the two collection officers merely submitted a listing of alleged import entry numbers and dates released of alleged importations by Hantex Trading Co., Inc. of merchandise in 1987, for which they certified that the corresponding duties and taxes were paid after being processed in their offices. In said letters, no amounts of the landed costs and advance sales tax and duties were stated, and no particulars of the duties and taxes paid per import entry document was presented. The contents of the two letters failed to indicate the particulars of the importations per entry number, and the said letters do not constitute as evidence of the amounts of importations of Hantex Trading Co., Inc. in 1987.28 The respondent cited the following findings of the Hearing Officer: [T]hat the import entry documents do not constitute evidence only indicate that the tax assessments in question have no factual basis, and must, at this point in time, be withdrawn and cancelled. Any new findings by the IIPO representative who attended the hearing could not be used as evidence in this hearing, because all the issues on the tax assessments in question have already been raised by the herein taxpayer.29 The respondent requested anew that the income tax deficiency assessment and the sales tax deficiency assessment be set aside for lack of factual and legal basis. The BIR Commissioner30 wrote the respondent on December 10, 1993, denying its letter-request for the dismissal of the assessments.31 The BIR Commissioner admitted, in the said letter, the possibility that the figures appearing in the photocopies of the Consumption Entries had been tampered with. She averred, however, that she was not proscribed from relying on other admissible evidence, namely, the Letters of Torres and Filamor dated August 7 and 22, 1990 on their investigation of the respondents tax liability. The Commissioner emphasized that her decision was final.32 The respondent forthwith filed a petition for review in the CTA of the Commissioners Final Assessment Letter dated December 10, 1993 on the following grounds: First. The alleged 1987 deficiency income tax assessment (including increments) and the alleged 1987 deficiency sales tax assessment (including increments) are void ab initio, since under Sections 16(a) and 49(b) of the Tax Code, the Commissioner shall examine a return after it is filed and, thereafter, assess the correct amount of tax. The following facts obtaining in this case, however, are indicative of the incorrectness of the tax assessments in question: the deficiency interests imposed in the income and percentage tax deficiency assessment notices were computed in violation of the provisions of Section 249(b) of the NIRC of 1977, as amended; the percentage tax deficiency was computed on an annual basis for the year 1987 in accordance with the provision of Section 193, which should have been computed in accordance with Section 162 of

the 1977 NIRC, as amended by Pres. Decree No. 1994 on a quarterly basis; and the BIR official who signed the deficiency tax assessments was the Assistant Commissioner for Collection, who had no authority to sign the same under the NIRC. Second. Even granting arguendo that the deficiency taxes and increments for 1987 against the respondent were correctly computed in accordance with the provisions of the Tax Code, the facts indicate that the above-stated assessments were based on alleged documents which are inadmissible in either administrative or judicial proceedings. Moreover, the alleged bases of the tax computations were anchored on mere presumptions and not on actual facts. The alleged undeclared purchases for 1987 were based on mere photocopies of alleged import entry documents, not the original ones, and which had never been duly certified by the public officer charged with the custody of such records in the Bureau of Customs. According to the respondent, the alleged undeclared sales were computed based on mere presumptions as to the alleged gross profit contained in its 1987 financial statement. Moreover, even the alleged financial statement of the respondent was a mere machine copy and not an official copy of the 1987 income and business tax returns. Finally, the respondent was following the accrual method of accounting in 1987, yet, the BIR investigator who computed the 1987 income tax deficiency failed to allow as a deductible item the alleged sales tax deficiency for 1987 as provided for under Section 30(c) of the NIRC of 1986.33 The Commissioner did not adduce in evidence the original or certified true copies of the 1987 Consumption Entries on file with the Commission on Audit. Instead, she offered in evidence as proof of the contents thereof, the photocopies of the Consumption Entries which the respondent objected to for being inadmissible in evidence.34 She also failed to present any witness to prove the correct amount of tax due from it. Nevertheless, the CTA provisionally admitted the said documents in evidence, subject to its final evaluation of their relevancy and probative weight to the issues involved.35 On December 11, 1997, the CTA rendered a decision, the dispositive portion of which reads: IN THE LIGHT OF ALL THE FOREGOING, judgment is hereby rendered DENYING the herein petition. Petitioner is hereby ORDERED TO PAY the respondent Commissioner of Internal Revenue its deficiency income and sales taxes for the year 1987 in the amounts of P11,182,350.26 and P12,660,382.46, respectively, plus 20% delinquency interest per annum on both deficiency taxes from April 15, 1991 until fully paid pursuant to Section 283(c)(3) of the 1987 Tax Code, with costs against the petitioner. SO ORDERED.36 The CTA ruled that the respondent was burdened to prove not only that the assessment was erroneous, but also to adduce the correct taxes to be paid by it. The CTA declared that the respondent failed to prove the correct amount of taxes due to the BIR. It also ruled that the respondent was burdened to adduce in evidence a certification from the Bureau of Customs that the Consumption Entries in question did not belong to it.

On appeal, the CA granted the petition and reversed the decision of the CTA. The dispositive portion of the decision reads: FOREGOING PREMISES CONSIDERED, the Petition for Review is GRANTED and the December 11, 1997 decision of the CTA in CTA Case No. 5162 affirming the 1987 deficiency income and sales tax assessments and the increments thereof, issued by the BIR is hereby REVERSED. No costs.37 The Ruling of the Court of Appeals The CA held that the income and sales tax deficiency assessments issued by the petitioner were unlawful and baseless since the copies of the import entries relied upon in computing the deficiency tax of the respondent were not duly authenticated by the public officer charged with their custody, nor verified under oath by the EIIB and the BIR investigators.38 The CA also noted that the public officer charged with the custody of the import entries was never presented in court to lend credence to the alleged loss of the originals.39 The CA pointed out that an import entry is a public document which falls within the provisions of Section 19, Rule 132 of the Rules of Court, and to be admissible for any legal purpose, Section 24, Rule 132 of the Rules of Court should apply.40 Citing the ruling of this Court in Collector of Internal Revenue v. Benipayo,41 the CA ruled that the assessments were unlawful because they were based on hearsay evidence. The CA also ruled that the respondent was deprived of its right to due process of law. The CA added that the CTA should not have just brushed aside the legal requisites provided for under the pertinent provisions of the Rules of Court in the matter of the admissibility of public documents, considering that substantive rules of evidence should not be disregarded. It also ruled that the certifications made by the two Customs Collection Chiefs under the guise of supporting the respondents alleged tax deficiency assessments invoking the best evidence obtainable rule under the Tax Code should not be permitted to supplant the best evidence rule under Section 7, Rule 130 of the Rules of Court. Finally, the CA noted that the tax deficiency assessments were computed without the tax returns. The CA opined that the use of the tax returns is indispensable in the computation of a tax deficiency; hence, this essential requirement must be complied with in the preparation and issuance of valid tax deficiency assessments.42 The Present Petition The Commissioner of Internal Revenue, the petitioner herein, filed the present petition for review under Rule 45 of the Rules of Court for the reversal of the decision of the CA and for the reinstatement of the ruling of the CTA. As gleaned from the pleadings of the parties, the threshold issues for resolution are the following: (a) whether the petition at bench is proper and complies with Sections 4 and 5, Rule 7 of the Rules of Court; (b) whether the December 10, 1991 final assessment of the petitioner against the respondent for deficiency income tax and sales tax for the latters 1987 importation of

resins and calcium bicarbonate is based on competent evidence and the law; and (c) the total amount of deficiency taxes due from the respondent for 1987, if any. On the first issue, the respondent points out that the petition raises both questions of facts and law which cannot be the subject of an appeal by certiorari under Rule 45 of the Rules of Court. The respondent notes that the petition is defective because the verification and the certification against forum shopping were not signed by the petitioner herself, but only by the Regional Director of the BIR. The respondent submits that the petitioner should have filed a motion for reconsideration with the CA before filing the instant petition for review.43 We find and so rule that the petition is sufficient in form. A verification and certification against forum shopping signed by the Regional Director constitutes sufficient compliance with the requirements of Sections 4 and 5, Rule 7 of the Rules of Court. Under Section 10 of the NIRC of 1997,44 the Regional Director has the power to administer and enforce internal revenue laws, rules and regulations, including the assessment and collection of all internal revenue taxes, charges and fees. Such power is broad enough to vest the Revenue Regional Director with the authority to sign the verification and certification against forum shopping in behalf of the Commissioner of Internal Revenue. There is no other person in a better position to know the collection cases filed under his jurisdiction than the Revenue Regional Director. Moreover, under Revenue Administrative Order No. 5-83,45 the Regional Director is authorized to sign all pleadings filed in connection with cases referred to the Revenue Regions by the National Office which, otherwise, require the signature of the petitioner. We do not agree with the contention of the respondent that a motion for reconsideration ought to have been filed before the filing of the instant petition. A motion for reconsideration of the decision of the CA is not a condition sine qua non for the filing of a petition for review under Rule 45. As we held in Almora v. Court of Appeals:46 Rule 45, Sec. 1 of the Rules of Court, however, distinctly provides that: A party may appeal by certiorari from a judgment of the Court of Appeals, by filing with the Supreme Court a petition for certiorari within fifteen (15) days from notice of judgment, or of the denial of his motion for reconsideration filed in due time. (Emphasis supplied) The conjunctive "or" clearly indicates that the 15-day reglementary period for the filing of a petition for certiorari under Rule 45 commences either from notice of the questioned judgment or from notice of denial of the appellants motion for reconsideration. A prior motion for reconsideration is not indispensable for a petition for review on certiorari under Rule 45 to prosper. 47 While Rule 45 of the Rules of Court provides that only questions of law may be raised by the petitioner and resolved by the Court, under exceptional circumstances, the Court may take cognizance thereof and resolve questions of fact. In this case, the findings and conclusion of the CA are inconsistent with those of the CTA, not to mention those of the Commissioner of Internal

Revenue. The issues raised in this case relate to the propriety and the correctness of the tax assessments made by the petitioner against the respondent, as well as the propriety of the application of Section 16, paragraph (b) of the 1977 NIRC, as amended by Pres. Decree Nos. 1705, 1773, 1994 and Executive Order No. 273, in relation to Section 3, Rule 132 of the Rules of Evidence. There is also an imperative need for the Court to resolve the threshold factual issues to give justice to the parties, and to determine whether the CA capriciously ignored, misunderstood or misinterpreted cogent facts and circumstances which, if considered, would change the outcome of the case. On the second issue, the petitioner asserts that since the respondent refused to cooperate and show its 1987 books of account and other accounting records, it was proper for her to resort to the best evidence obtainable the photocopies of the import entries in the Bureau of Customs and the respondents financial statement filed with the SEC.48 The petitioner maintains that these import entries were admissible as secondary evidence under the best evidence obtainable rule, since they were duly authenticated by the Bureau of Customs officials who processed the documents and released the cargoes after payment of the duties and taxes due.49 Further, the petitioner points out that under the best evidence obtainable rule, the tax return is not important in computing the tax deficiency.50 The petitioner avers that the best evidence obtainable rule under Section 16 of the 1977 NIRC, as amended, legally cannot be equated to the best evidence rule under the Rules of Court; nor can the best evidence rule, being procedural law, be made strictly operative in the interpretation of the best evidence obtainable rule which is substantive in character.51 The petitioner posits that the CTA is not strictly bound by technical rules of evidence, the reason being that the quantum of evidence required in the said court is merely substantial evidence.52 Finally, the petitioner avers that the respondent has the burden of proof to show the correct assessments; otherwise, the presumption in favor of the correctness of the assessments made by it stands.53 Since the respondent was allowed to explain its side, there was no violation of due process.54 The respondent, for its part, maintains that the resort to the best evidence obtainable method was illegal. In the first place, the respondent argues, the EIIB agents are not duly authorized to undertake examination of the taxpayers accounting records for internal revenue tax purposes. Hence, the respondents failure to accede to their demands to show its books of accounts and other accounting records cannot justify resort to the use of the best evidence obtainable method.55 Secondly, when a taxpayer fails to submit its tax records upon demand by the BIR officer, the remedy is not to assess him and resort to the best evidence obtainable rule, but to punish the taxpayer according to the provisions of the Tax Code.56 In any case, the respondent argues that the photocopies of import entries cannot be used in making the assessment because they were not properly authenticated, pursuant to the provisions of Sections 2457 and 2558 of Rule 132 of the Rules of Court. It avers that while the CTA is not bound by the technical rules of evidence, it is bound by substantial rules.59 The respondent points out that the petitioner did not even secure a certification of the fact of loss of the original documents from the custodian of the import entries. It simply relied on the report of the EIIB

agents that the import entry documents were no longer available because they were eaten by termites. The respondent posits that the two collectors of the Bureau of Customs never authenticated the xerox copies of the import entries; instead, they only issued certifications stating therein the import entry numbers which were processed by their office and the date the same were released.60 The respondent argues that it was not necessary for it to show the correct assessment, considering that it is questioning the assessments not only because they are erroneous, but because they were issued without factual basis and in patent violation of the assessment procedures laid down in the NIRC of 1977, as amended.61 It is also pointed out that the petitioner failed to use the tax returns filed by the respondent in computing the deficiency taxes which is contrary to law;62 as such, the deficiency assessments constituted deprivation of property without due process of law.63 Central to the second issue is Section 16 of the NIRC of 1977, as amended,64 which provides that the Commissioner of Internal Revenue has the power to make assessments and prescribe additional requirements for tax administration and enforcement. Among such powers are those provided in paragraph (b) thereof, which we quote: (b) Failure to submit required returns, statements, reports and other documents. When a report required by law as a basis for the assessment of any national internal revenue tax shall not be forthcoming within the time fixed by law or regulation or when there is reason to believe that any such report is false, incomplete or erroneous, the Commissioner shall assess the proper tax on the best evidence obtainable. In case a person fails to file a required return or other document at the time prescribed by law, or willfully or otherwise files a false or fraudulent return or other document, the Commissioner shall make or amend the return from his own knowledge and from such information as he can obtain through testimony or otherwise, which shall be prima facie correct and sufficient for all legal purposes.65 This provision applies when the Commissioner of Internal Revenue undertakes to perform her administrative duty of assessing the proper tax against a taxpayer, to make a return in case of a taxpayers failure to file one, or to amend a return already filed in the BIR. The petitioner may avail herself of the best evidence or other information or testimony by exercising her power or authority under paragraphs (1) to (4) of Section 7 of the NIRC: (1) To examine any book, paper, record or other data which may be relevant or material to such inquiry; (2) To obtain information from any office or officer of the national and local governments, government agencies or its instrumentalities, including the Central Bank of the Philippines and government owned or controlled corporations;

(3) To summon the person liable for tax or required to file a return, or any officer or employee of such person, or any person having possession, custody, or care of the books of accounts and other accounting records containing entries relating to the business of the person liable for tax, or any other person, to appear before the Commissioner or his duly authorized representative at a time and place specified in the summons and to produce such books, papers, records, or other data, and to give testimony; (4) To take such testimony of the person concerned, under oath, as may be relevant or material to such inquiry; 66 The "best evidence" envisaged in Section 16 of the 1977 NIRC, as amended, includes the corporate and accounting records of the taxpayer who is the subject of the assessment process, the accounting records of other taxpayers engaged in the same line of business, including their gross profit and net profit sales.67 Such evidence also includes data, record, paper, document or any evidence gathered by internal revenue officers from other taxpayers who had personal transactions or from whom the subject taxpayer received any income; and record, data, document and information secured from government offices or agencies, such as the SEC, the Central Bank of the Philippines, the Bureau of Customs, and the Tariff and Customs Commission. The law allows the BIR access to all relevant or material records and data in the person of the taxpayer. It places no limit or condition on the type or form of the medium by which the record subject to the order of the BIR is kept. The purpose of the law is to enable the BIR to get at the taxpayers records in whatever form they may be kept. Such records include computer tapes of the said records prepared by the taxpayer in the course of business.68 In this era of developing information-storage technology, there is no valid reason to immunize companies with computerbased, record-keeping capabilities from BIR scrutiny. The standard is not the form of the record but where it might shed light on the accuracy of the taxpayers return. In Campbell, Jr. v. Guetersloh,69 the United States (U.S.) Court of Appeals (5th Circuit) declared that it is the duty of the Commissioner of Internal Revenue to investigate any circumstance which led him to believe that the taxpayer had taxable income larger than reported. Necessarily, this inquiry would have to be outside of the books because they supported the return as filed. He may take the sworn testimony of the taxpayer; he may take the testimony of third parties; he may examine and subpoena, if necessary, traders and brokers accounts and books and the taxpayers book accounts. The Commissioner is not bound to follow any set of patterns. The existence of unreported income may be shown by any practicable proof that is available in the circumstances of the particular situation. Citing its ruling in Kenney v. Commissioner,70 the U.S. appellate court declared that where the records of the taxpayer are manifestly inaccurate and incomplete, the Commissioner may look to other sources of information to establish income made by the taxpayer during the years in question.71 We agree with the contention of the petitioner that the best evidence obtainable may consist of hearsay evidence, such as the testimony of third parties or accounts or other records of other taxpayers similarly circumstanced as the taxpayer subject of the investigation, hence, inadmissible in a regular proceeding in the regular courts.72 Moreover, the general rule is that administrative agencies such as the BIR are not bound by the technical rules of evidence. It can

accept documents which cannot be admitted in a judicial proceeding where the Rules of Court are strictly observed. It can choose to give weight or disregard such evidence, depending on its trustworthiness. However, the best evidence obtainable under Section 16 of the 1977 NIRC, as amended, does not include mere photocopies of records/documents. The petitioner, in making a preliminary and final tax deficiency assessment against a taxpayer, cannot anchor the said assessment on mere machine copies of records/documents. Mere photocopies of the Consumption Entries have no probative weight if offered as proof of the contents thereof. The reason for this is that such copies are mere scraps of paper and are of no probative value as basis for any deficiency income or business taxes against a taxpayer. Indeed, in United States v. Davey,73 the U.S. Court of Appeals (2nd Circuit) ruled that where the accuracy of a taxpayers return is being checked, the government is entitled to use the original records rather than be forced to accept purported copies which present the risk of error or tampering.74 In Collector of Internal Revenue v. Benipayo,75 the Court ruled that the assessment must be based on actual facts. The rule assumes more importance in this case since the xerox copies of the Consumption Entries furnished by the informer of the EIIB were furnished by yet another informer. While the EIIB tried to secure certified copies of the said entries from the Bureau of Customs, it was unable to do so because the said entries were allegedly eaten by termites. The Court can only surmise why the EIIB or the BIR, for that matter, failed to secure certified copies of the said entries from the Tariff and Customs Commission or from the National Statistics Office which also had copies thereof. It bears stressing that under Section 1306 of the Tariff and Customs Code, the Consumption Entries shall be the required number of copies as prescribed by regulations.76 The Consumption Entry is accomplished in sextuplicate copies and quadruplicate copies in other places. In Manila, the six copies are distributed to the Bureau of Customs, the Tariff and Customs Commission, the Declarant (Importer), the Terminal Operator, and the Bureau of Internal Revenue. Inexplicably, the Commissioner and the BIR personnel ignored the copy of the Consumption Entries filed with the BIR and relied on the photocopies supplied by the informer of the EIIB who secured the same from another informer. The BIR, in preparing and issuing its preliminary and final assessments against the respondent, even ignored the records on the investigation made by the District Revenue officers on the respondents importations for 1987. The original copies of the Consumption Entries were of prime importance to the BIR. This is so because such entries are under oath and are presumed to be true and correct under penalty of falsification or perjury. Admissions in the said entries of the importers documents are admissions against interest and presumptively correct.77 In fine, then, the petitioner acted arbitrarily and capriciously in relying on and giving weight to the machine copies of the Consumption Entries in fixing the tax deficiency assessments against the respondent. The rule is that in the absence of the accounting records of a taxpayer, his tax liability may be determined by estimation. The petitioner is not required to compute such tax liabilities with mathematical exactness. Approximation in the calculation of the taxes due is justified. To hold

otherwise would be tantamount to holding that skillful concealment is an invincible barrier to proof.78 However, the rule does not apply where the estimation is arrived at arbitrarily and capriciously.79 We agree with the contention of the petitioner that, as a general rule, tax assessments by tax examiners are presumed correct and made in good faith. All presumptions are in favor of the correctness of a tax assessment. It is to be presumed, however, that such assessment was based on sufficient evidence. Upon the introduction of the assessment in evidence, a prima facie case of liability on the part of the taxpayer is made.80 If a taxpayer files a petition for review in the CTA and assails the assessment, the prima facie presumption is that the assessment made by the BIR is correct, and that in preparing the same, the BIR personnel regularly performed their duties. This rule for tax initiated suits is premised on several factors other than the normal evidentiary rule imposing proof obligation on the petitioner-taxpayer: the presumption of administrative regularity; the likelihood that the taxpayer will have access to the relevant information; and the desirability of bolstering the record-keeping requirements of the NIRC.81 However, the prima facie correctness of a tax assessment does not apply upon proof that an assessment is utterly without foundation, meaning it is arbitrary and capricious. Where the BIR has come out with a "naked assessment," i.e., without any foundation character, the determination of the tax due is without rational basis.82 In such a situation, the U.S. Court of Appeals ruled83 that the determination of the Commissioner contained in a deficiency notice disappears. Hence, the determination by the CTA must rest on all the evidence introduced and its ultimate determination must find support in credible evidence. The issue that now comes to fore is whether the tax deficiency assessment against the respondent based on the certified copies of the Profit and Loss Statement submitted by the respondent to the SEC in 1987 and 1988, as well as certifications of Tomas and Danganan, is arbitrary, capricious and illegal. The CTA ruled that the respondent failed to overcome the prima facie correctness of the tax deficiency assessment issued by the petitioner, to wit: The issue should be ruled in the affirmative as petitioner has failed to rebut the validity or correctness of the aforementioned tax assessments. It is incongruous for petitioner to prove its cause by simply drawing an inference unfavorable to the respondent by attacking the source documents (Consumption Entries) which were the bases of the assessment and which were certified by the Chiefs of the Collection Division, Manila International Container Port and the Port of Manila, as having been processed and released in the name of the petitioner after payment of duties and taxes and the duly certified copies of Financial Statements secured from the Securities and Exchange Commission. Any such inference cannot operate to relieve petitioner from bearing its burden of proof and this Court has no warrant of absolution. The Court should have been persuaded to grant the reliefs sought by the petitioner should it have presented any evidence of relevance and competence required, like that of a certification from the Bureau of Customs or from any other agencies, attesting to the fact that those consumption entries did not really belong to them.

The burden of proof is on the taxpayer contesting the validity or correctness of an assessment to prove not only that the Commissioner of Internal Revenue is wrong but the taxpayer is right (Tan Guan v. CTA, 19 SCRA 903), otherwise, the presumption in favor of the correctness of tax assessment stands (Sy Po v. CTA, 164 SCRA 524). The burden of proving the illegality of the assessment lies upon the petitioner alleging it to be so. In the case at bar, petitioner miserably failed to discharge this duty.84 We are not in full accord with the findings and ratiocination of the CTA. Based on the letter of the petitioner to the respondent dated December 10, 1993, the tax deficiency assessment in question was based on (a) the findings of the agents of the EIIB which was based, in turn, on the photocopies of the Consumption Entries; (b) the Profit and Loss Statements of the respondent for 1987 and 1988; and (c) the certifications of Tomas and Danganan dated August 7, 1990 and August 22, 1990: In reply, please be informed that after a thorough evaluation of the attending facts, as well as the laws and jurisprudence involved, this Office holds that you are liable to the assessed deficiency taxes. The conclusion was arrived at based on the findings of agents of the Economic Intelligence & Investigation Bureau (EIIB) and of our own examiners who have painstakingly examined the records furnished by the Bureau of Customs and the Securities & Exchange Commission (SEC). The examination conducted disclosed that while your actual sales for 1987 amounted to P110,731,559.00, you declared for taxation purposes, as shown in the Profit and Loss Statements, the sum of P47,698,569.83 only. The difference, therefore, of P63,032,989.17 constitutes as undeclared or unrecorded sales which must be subjected to the income and sales taxes. You also argued that our assessment has no basis since the alleged amount of underdeclared importations were lifted from uncertified or unauthenticated xerox copies of consumption entries which are not admissible in evidence. On this issue, it must be considered that in letters dated August 7 and 22, 1990, the Chief and Acting Chief of the Collection Division of the Manila International Container Port and Port of Manila, respectively, certified that the enumerated consumption entries were filed, processed and released from the port after payment of duties and taxes. It is noted that the certification does not touch on the genuineness, authenticity and correctness of the consumption entries which are all xerox copies, wherein the figures therein appearing may have been tampered which may render said documents inadmissible in evidence, but for tax purposes, it has been held that the Commissioner is not required to make his determination (assessment) on the basis of evidence legally admissible in a formal proceeding in Court (Mertens, Vol. 9, p. 214, citing Cohen v. Commissioner). A statutory notice may be based in whole or in part upon admissible evidence (Llorente v. Commissioner, 74 TC 260 (1980); Weimerskirch v. Commissioner, 67 TC 672 (1977); and Rosano v. Commissioner, 46 TC 681 (1966). In the case also of Weimerskirch v. Commissioner (1977), the assessment was given due course in the presence of admissible evidence as to how the Commissioner arrived at his determination, although there was no admissible evidence with respect to the substantial issue of whether the taxpayer had unreported or undeclared income from narcotics sale. 85

Based on a Memorandum dated October 23, 1990 of the IIPO, the source documents for the actual cost of importation of the respondent are the machine copies of the Consumption Entries from the informer which the IIPO claimed to have been certified by Tomas and Danganan: The source documents for the total actual cost of importations, abovementioned, were the different copies of Consumption Entries, Series of 1987, filed by subject with the Bureau of Customs, marked Annexes "F-1" to "F-68." The total cost of importations is the sum of the Landed Costs and the Advance Sales Tax as shown in the annexed entries. These entries were duly authenticated as having been processed and released, after payment of the duties and taxes due thereon, by the Chief, Collection Division, Manila International Container Port, dated August 7, 1990, "Annex-G," and the Port of Manila, dated August 22, 1990, "Annex-H." So, it was established that subject-importations, mostly resins, really belong to HANTEX TRADING CO., INC.86 It also appears on the worksheet of the IIPO, as culled from the photocopies of the Consumption Entries from its informer, that the total cost of the respondents importation for 1987 was P105,761,527.00. Per the report of Torres and Filamor, they also relied on the photocopies of the said Consumption Entries: The importations made by taxpayer verified by us from the records of the Bureau of Customs and xerox copies of which are hereto attached shows the big volume of importations made and not declared in the income tax return filed by taxpayer. Based on the above findings, it clearly shows that a prima facie case of fraud exists in the herein transaction of the taxpayer, as a consequence of which, said transaction has not been possibly entered into the books of accounts of the subject taxpayer.87 In fine, the petitioner based her finding that the 1987 importation of the respondent was underdeclared in the amount of P105,761,527.00 on the worthless machine copies of the Consumption Entries. Aside from such copies, the petitioner has no other evidence to prove that the respondent imported goods costing P105,761,527.00. The petitioner cannot find solace on the certifications of Tomas and Danganan because they did not authenticate the machine copies of the Consumption Entries, and merely indicated therein the entry numbers of Consumption Entries and the dates when the Bureau of Customs released the same. The certifications of Tomas and Danganan do not even contain the landed costs and the advance sales taxes paid by the importer, if any. Comparing the certifications of Tomas and Danganan and the machine copies of the Consumption Entries, only 36 of the entry numbers of such copies are included in the said certifications; the entry numbers of the rest of the machine copies of the Consumption Entries are not found therein. Even if the Court would concede to the petitioners contention that the certification of Tomas and Danganan authenticated the machine copies of the Consumption Entries referred to in the certification, it appears that the total cost of importations inclusive of advance sales tax is only P64,324,953.00 far from the amount of P105,716,527.00 arrived at by the EIIB and the BIR,88 or even the amount of P110,079,491.61 arrived at by Deputy Commissioner Deoferio, Jr.89 As gleaned from the certifications of Tomas and Danganan, the goods covered by the Consumption

Entries were released by the Bureau of Customs, from which it can be presumed that the respondent must have paid the taxes due on the said importation. The petitioner did not adduce any documentary evidence to prove otherwise. Thus, the computations of the EIIB and the BIR on the quantity and costs of the importations of the respondent in the amount of P105,761,527.00 for 1987 have no factual basis, hence, arbitrary and capricious. The petitioner cannot rely on the presumption that she and the other employees of the BIR had regularly performed their duties. As the Court held in Collector of Internal Revenue v. Benipayo,90 in order to stand judicial scrutiny, the assessment must be based on facts. The presumption of the correctness of an assessment, being a mere presumption, cannot be made to rest on another presumption. Moreover, the uncontroverted fact is that the BIR District Revenue Office had repeatedly examined the 1987 books of accounts of the respondent showing its importations, and found that the latter had minimal business tax liability. In this case, the presumption that the District Revenue officers performed their duties in accordance with law shall apply. There is no evidence on record that the said officers neglected to perform their duties as mandated by law; neither is there evidence aliunde that the contents of the 1987 and 1988 Profit and Loss Statements submitted by the respondent with the SEC are incorrect. Admittedly, the respondent did not adduce evidence to prove its correct tax liability. However, considering that it has been established that the petitioners assessment is barren of factual basis, arbitrary and illegal, such failure on the part of the respondent cannot serve as a basis for a finding by the Court that it is liable for the amount contained in the said assessment; otherwise, the Court would thereby be committing a travesty. On the disposition of the case, the Court has two options, namely, to deny the petition for lack of merit and affirm the decision of the CA, without prejudice to the petitioners issuance of a new assessment against the respondent based on credible evidence; or, to remand the case to the CTA for further proceedings, to enable the petitioner to adduce in evidence certified true copies or duplicate original copies of the Consumption Entries for the respondents 1987 importations, if there be any, and the correct tax deficiency assessment thereon, without prejudice to the right of the respondent to adduce controverting evidence, so that the matter may be resolved once and for all by the CTA. In the higher interest of justice to both the parties, the Court has chosen the latter option. After all, as the Tax Court of the United States emphasized in Harbin v. Commissioner of Internal Revenue,91 taxation is not only practical; it is vital. The obligation of good faith and fair dealing in carrying out its provision is reciprocal and, as the government should never be overreaching or tyrannical, neither should a taxpayer be permitted to escape payment by the concealment of material facts. IN LIGHT OF ALL THE FOREGOING, the petition is GRANTED. The Decision of the Court of Appeals is SET ASIDE. The records are REMANDED to the Court of Tax Appeals for further proceedings, conformably with the decision of this Court. No costs.
CIR vs. HANTEX TRADING CO., INC. G.R. No. 136975; March 31, 2005

Facts: Hantex Trading Co is a company organized under the Philippines. It is engaged in the sale of plastic products, it imports synthetic resin and other chemicals for the manufacture of its products. For this purpose, it is required to file an Import Entry and Internal Revenue Declaration (Consumption Entry) with the Bureau of Customs under Section 1301 of the Tariff and Customs Code. Sometime in October 1989, Lt. Vicente Amoto, Acting Chief of Counter-Intelligence Division of the Economic Intelligence and Investigation Bureau (EIIB), received confidential information that the respondent had imported synthetic resin amounting to P115,599,018.00 but only declared P45,538,694.57. Thus, Hentex receive a subpoena to present its books of account which it failed to do. The bureau cannot find any original copies of the products Hentex imported since the originals were eaten by termites. Thus, the Bureau relied on the certified copies of the respondents Profit and Loss Statement for 1987 and 1988 on file with the SEC, the machine copies of the Consumption Entries, Series of 1987, submitted by the informer, as well as excerpts from the entries certified by Tomas and Danganan. The case was submitted to the CTA which ruled that Hentex have tax deficiency and is ordered to pay, per investigation of the Bureau. The CA ruled that the income and sales tax deficiency assessments issued by the petitioner were unlawful and baseless since the copies of the import entries relied upon in computing the deficiency tax of the respondent were not duly authenticated by the public officer charged with their custody, nor verified under oath by the EIIB and the BIR investigators. Issue: Whether or not the final assessment of the petitioner against the respondent for deficiency income tax and sales tax for the latters 1987 importation of resins and calcium bicarbonate is based on competent evidence and the law. Held: Central to the second issue is Section 16 of the NIRC of 1977, as amended which provides that the Commissioner of Internal Revenue has the power to make assessments and prescribe additional requirements for tax administration and enforcement. Among such powers are those provided in paragraph (b), which provides that Failure to submit required returns, statements, reports and other documents. When a report required by law as a basis for the assessment of any national internal revenue tax shall not be forthcoming within the time fixed by law or regulation or when there is reason to believe that any such report is false, incomplete or erroneous, the Commissioner shall assess the proper tax on the best evidence obtainable. This provision applies when the Commissioner of Internal Revenue undertakes to perform her administrative duty of assessing the proper tax against a taxpayer, to make a return in case of a taxpayers failure to file one, or to amend a return already filed in the BIR. The best evidence envisaged in Section 16 of the 1977 NIRC, as amended, includes the corpor ate and accounting records of the taxpayer who is the subject of the assessment process, the accounting records of other taxpayers engaged in the same line of business, including their gross profit and net profit sales. Such evidence also includes data, record, paper, document or any evidence gathered by internal revenue officers from other taxpayers who had personal transactions or from whom the subject taxpayer received any income; and record, data, document and information secured from government offices or agencies, such as the SEC, the Central Bank of the Philippines, the Bureau of Customs, and the Tariff and Customs Commission. However, the best evidence obtainable under Section 16 of the 1977 NIRC, as amended, does not include mere photocopies of records/documents. The petitioner, in making a preliminary and final tax deficiency assessment against a taxpayer, cannot anchor the said assessment on mere machine copies of records/documents. Mere photocopies of the Consumption Entries have no probative weight if offered as proof of the contents thereof. The reason for this is that such copies are mere scraps of paper and are of no probative value as basis for any deficiency income or business taxes against a taxpayer.

G.R. No. 185371

December 8, 2010

COMMISSIONER OF INTERNAL REVENUE, Petitioner, vs. METRO STAR SUPERAMA, INC., Respondent.

DECISION MENDOZA, J.: This petition for review on certiorari under Rule 45 of the Rules of Court filed by the petitioner Commissioner of Internal Revenue (CIR) seeks to reverse and set aside the 1] September 16, 2008 Decision1 of the Court of Tax Appeals En Banc (CTA-En Banc), in C.T.A. EB No. 306 and 2] its November 18, 2008 Resolution2 denying petitioners motion for reconsideration. The CTA-En Banc affirmed in toto the decision of its Second Division (CTA-Second Division) in CTA Case No. 7169 reversing the February 8, 2005 Decision of the CIR which assessed respondent Metro Star Superama, Inc. (Metro Star) of deficiency value-added tax and withholding tax for the taxable year 1999. Based on a Joint Stipulation of Facts and Issues3 of the parties, the CTA Second Division summarized the factual and procedural antecedents of the case, the pertinent portions of which read: Petitioner is a domestic corporation duly organized and existing by virtue of the laws of the Republic of the Philippines, x x x. On January 26, 2001, the Regional Director of Revenue Region No. 10, Legazpi City, issued Letter of Authority No. 00006561 for Revenue Officer Daisy G. Justiniana to examine petitioners books of accounts and other accounting records for income tax and other internal revenue taxes for the taxable year 1999. Said Letter of Authority was revalidated on August 10, 2001 by Regional Director Leonardo Sacamos. For petitioners failure to comply with several requests for the presentation of records and Subpoena Duces Tecum, [the] OIC of BIR Legal Division issued an Indorsement dated September 26, 2001 informing Revenue District Officer of Revenue Region No. 67, Legazpi City to proceed with the investigation based on the best evidence obtainable preparatory to the issuance of assessment notice. On November 8, 2001, Revenue District Officer Socorro O. Ramos-Lafuente issued a Preliminary 15-day Letter, which petitioner received on November 9, 2001. The said letter stated that a post audit review was held and it was ascertained that there was deficiency value-added and withholding taxes due from petitioner in the amount of P 292,874.16. On April 11, 2002, petitioner received a Formal Letter of Demand dated April 3, 2002 from Revenue District No. 67, Legazpi City, assessing petitioner the amount of Two Hundred Ninety Two Thousand Eight Hundred Seventy Four Pesos and Sixteen Centavos (P292,874.16.) for deficiency value-added and withholding taxes for the taxable year 1999, computed as follows: ASSESSMENT NOTICE NO. 067-99-003-579-072 VALUE ADDED TAX

Gross Sales Output Tax Less: Input Tax VAT Payable Add: 25% Surcharge 20% Interest Compromise Penalty Late Payment Failure to File VAT returns TOTAL WITHHOLDING TAX Compensation Expanded Total Tax Due Less: Tax Withheld Deficiency Withholding Tax Add: 20% Interest p.a. Compromise Penalty TOTAL *Expanded Withholding Tax P1,949,334.25 x 5% Film Rental Audit Fee Rental Expense Security Service Service Contractor Total SUMMARIES OF DEFICIENCIES VALUE ADDED TAX 10,000.25 193,261.20 41,272.73 156,142.01 x 10% x 5% x 1% x 1% P16,000.00 2,400.00 18,400.00 P 38,584.54 79,746.49

P1,697,718.90 P 154,338.08 _____________ P 154,338.08

136,731.01 P 291,069.09

2,772.91 110,103.92 P 112,876.83 111,848.27 P 1,028.56 576.51 200.00 P 1,805.07 97,466.71 1,000.00 9,663.00 412.73 1,561.42 P 110,103.92

P 291,069.09

WITHHOLDING TAX TOTAL

1,805.07 P 292,874.16

Subsequently, Revenue District Office No. 67 sent a copy of the Final Notice of Seizure dated May 12, 2003, which petitioner received on May 15, 2003, giving the latter last opportunity to settle its deficiency tax liabilities within ten (10) [days] from receipt thereof, otherwise respondent BIR shall be constrained to serve and execute the Warrants of Distraint and/or Levy and Garnishment to enforce collection. On February 6, 2004, petitioner received from Revenue District Office No. 67 a Warrant of Distraint and/or Levy No. 67-0029-23 dated May 12, 2003 demanding payment of deficiency value-added tax and withholding tax payment in the amount of P292,874.16. On July 30, 2004, petitioner filed with the Office of respondent Commissioner a Motion for Reconsideration pursuant to Section 3.1.5 of Revenue Regulations No. 12-99. On February 8, 2005, respondent Commissioner, through its authorized representative, Revenue Regional Director of Revenue Region 10, Legaspi City, issued a Decision denying petitioners Motion for Reconsideration. Petitioner, through counsel received said Decision on February 18, 2005. x x x. Denying that it received a Preliminary Assessment Notice (PAN) and claiming that it was not accorded due process, Metro Star filed a petition for review4 with the CTA. The parties then stipulated on the following issues to be decided by the tax court: 1. Whether the respondent complied with the due process requirement as provided under the National Internal Revenue Code and Revenue Regulations No. 12-99 with regard to the issuance of a deficiency tax assessment; 1.1 Whether petitioner is liable for the respective amounts of P291,069.09 and P1,805.07 as deficiency VAT and withholding tax for the year 1999; 1.2. Whether the assessment has become final and executory and demandable for failure of petitioner to protest the same within 30 days from its receipt thereof on April 11, 2002, pursuant to Section 228 of the National Internal Revenue Code; 2. Whether the deficiency assessments issued by the respondent are void for failure to state the law and/or facts upon which they are based. 2.2 Whether petitioner was informed of the law and facts on which the assessment is made in compliance with Section 228 of the National Internal Revenue Code;

3. Whether or not petitioner, as owner/operator of a movie/cinema house, is subject to VAT on sales of services under Section 108(A) of the National Internal Revenue Code; 4. Whether or not the assessment is based on the best evidence obtainable pursuant to Section 6(b) of the National Internal Revenue Code. The CTA-Second Division found merit in the petition of Metro Star and, on March 21, 2007, rendered a decision, the decretal portion of which reads: WHEREFORE, premises considered, the Petition for Review is hereby GRANTED. Accordingly, the assailed Decision dated February 8, 2005 is hereby REVERSED and SET ASIDE and respondent is ORDERED TO DESIST from collecting the subject taxes against petitioner. The CTA-Second Division opined that "[w]hile there [is] a disputable presumption that a mailed letter [is] deemed received by the addressee in the ordinary course of mail, a direct denial of the receipt of mail shifts the burden upon the party favored by the presumption to prove that the mailed letter was indeed received by the addressee."5 It also found that there was no clear showing that Metro Star actually received the alleged PAN, dated January 16, 2002. It, accordingly, ruled that the Formal Letter of Demand dated April 3, 2002, as well as the Warrant of Distraint and/or Levy dated May 12, 2003 were void, as Metro Star was denied due process.6 The CIR sought reconsideration7 of the decision of the CTA-Second Division, but the motion was denied in the latters July 24, 2007 Resolution.8 Aggrieved, the CIR filed a petition for review9 with the CTA-En Banc, but the petition was dismissed after a determination that no new matters were raised. The CTA-En Banc disposed: WHEREFORE, the instant Petition for Review is hereby DENIED DUE COURSE and DISMISSED for lack of merit. Accordingly, the March 21, 2007 Decision and July 27, 2007 Resolution of the CTA Second Division in CTA Case No. 7169 entitled, "Metro Star Superama, Inc., petitioner vs. Commissioner of Internal Revenue, respondent" are hereby AFFIRMED in toto. SO ORDERED. The motion for reconsideration10 filed by the CIR was likewise denied by the CTA-En Banc in its November 18, 2008 Resolution.11 The CIR, insisting that Metro Star received the PAN, dated January 16, 2002, and that due process was served nonetheless because the latter received the Final Assessment Notice (FAN), comes now before this Court with the sole issue of whether or not Metro Star was denied due process. The general rule is that the Court will not lightly set aside the conclusions reached by the CTA which, by the very nature of its functions, has accordingly developed an exclusive expertise on

the resolution unless there has been an abuse or improvident exercise of authority.12 In Barcelon, Roxas Securities, Inc. (now known as UBP Securities, Inc.) v. Commissioner of Internal Revenue,13 the Court wrote: Jurisprudence has consistently shown that this Court accords the findings of fact by the CTA with the highest respect. In Sea-Land Service Inc. v. Court of Appeals [G.R. No. 122605, 30 April 2001, 357 SCRA 441, 445-446], this Court recognizes that the Court of Tax Appeals, which by the very nature of its function is dedicated exclusively to the consideration of tax problems, has necessarily developed an expertise on the subject, and its conclusions will not be overturned unless there has been an abuse or improvident exercise of authority. Such findings can only be disturbed on appeal if they are not supported by substantial evidence or there is a showing of gross error or abuse on the part of the Tax Court. In the absence of any clear and convincing proof to the contrary, this Court must presume that the CTA rendered a decision which is valid in every respect. On the matter of service of a tax assessment, a further perusal of our ruling in Barcelon is instructive, viz: Jurisprudence is replete with cases holding that if the taxpayer denies ever having received an assessment from the BIR, it is incumbent upon the latter to prove by competent evidence that such notice was indeed received by the addressee. The onus probandi was shifted to respondent to prove by contrary evidence that the Petitioner received the assessment in the due course of mail. The Supreme Court has consistently held that while a mailed letter is deemed received by the addressee in the course of mail, this is merely a disputable presumption subject to controversion and a direct denial thereof shifts the burden to the party favored by the presumption to prove that the mailed letter was indeed received by the addressee (Republic vs. Court of Appeals, 149 SCRA 351). Thus as held by the Supreme Court in Gonzalo P. Nava vs. Commissioner of Internal Revenue, 13 SCRA 104, January 30, 1965: "The facts to be proved to raise this presumption are (a) that the letter was properly addressed with postage prepaid, and (b) that it was mailed. Once these facts are proved, the presumption is that the letter was received by the addressee as soon as it could have been transmitted to him in the ordinary course of the mail. But if one of the said facts fails to appear, the presumption does not lie. (VI, Moran, Comments on the Rules of Court, 1963 ed, 56-57 citing Enriquez vs. Sunlife Assurance of Canada, 41 Phil 269)." x x x. What is essential to prove the fact of mailing is the registry receipt issued by the Bureau of Posts or the Registry return card which would have been signed by the Petitioner or its authorized representative. And if said documents cannot be located, Respondent at the very least, should have submitted to the Court a certification issued by the Bureau of Posts and any other pertinent document which is executed with the intervention of the Bureau of Posts. This Court does not put much credence to the self serving documentations made by the BIR personnel especially if they are unsupported by substantial evidence establishing the fact of mailing. Thus: "While we have held that an assessment is made when sent within the prescribed period, even if received by the taxpayer after its expiration (Coll. of Int. Rev. vs. Bautista, L-12250 and L-

12259, May 27, 1959), this ruling makes it the more imperative that the release, mailing or sending of the notice be clearly and satisfactorily proved. Mere notations made without the taxpayers intervention, notice or control, without adequate supporting evidence cannot suffice; otherwise, the taxpayer would be at the mercy of the revenue offices, without adequate protection or defense." (Nava vs. CIR, 13 SCRA 104, January 30, 1965). x x x. The failure of the respondent to prove receipt of the assessment by the Petitioner leads to the conclusion that no assessment was issued. Consequently, the governments right to issue an assessment for the said period has already prescribed. (Industrial Textile Manufacturing Co. of the Phils., Inc. vs. CIR CTA Case 4885, August 22, 1996). (Emphases supplied.) The Court agrees with the CTA that the CIR failed to discharge its duty and present any evidence to show that Metro Star indeed received the PAN dated January 16, 2002. It could have simply presented the registry receipt or the certification from the postmaster that it mailed the PAN, but failed. Neither did it offer any explanation on why it failed to comply with the requirement of service of the PAN. It merely accepted the letter of Metro Stars chairman dated April 29, 2002, that stated that he had received the FAN dated April 3, 2002, but not the PAN; that he was willing to pay the tax as computed by the CIR; and that he just wanted to clarify some matters with the hope of lessening its tax liability. This now leads to the question: Is the failure to strictly comply with notice requirements prescribed under Section 228 of the National Internal Revenue Code of 1997 and Revenue Regulations (R.R.) No. 12-99 tantamount to a denial of due process? Specifically, are the requirements of due process satisfied if only the FAN stating the computation of tax liabilities and a demand to pay within the prescribed period was sent to the taxpayer? The answer to these questions require an examination of Section 228 of the Tax Code which reads: SEC. 228. Protesting of Assessment. - When the Commissioner or his duly authorized representative finds that proper taxes should be assessed, he shall first notify the taxpayer of his findings: provided, however, that a preassessment notice shall not be required in the following cases: (a) When the finding for any deficiency tax is the result of mathematical error in the computation of the tax as appearing on the face of the return; or (b) When a discrepancy has been determined between the tax withheld and the amount actually remitted by the withholding agent; or (c) When a taxpayer who opted to claim a refund or tax credit of excess creditable withholding tax for a taxable period was determined to have carried over and automatically applied the same amount claimed against the estimated tax liabilities for the taxable quarter or quarters of the succeeding taxable year; or

(d) When the excise tax due on exciseable articles has not been paid; or (e) When the article locally purchased or imported by an exempt person, such as, but not limited to, vehicles, capital equipment, machineries and spare parts, has been sold, traded or transferred to non-exempt persons. The taxpayers shall be informed in writing of the law and the facts on which the assessment is made; otherwise, the assessment shall be void. Within a period to be prescribed by implementing rules and regulations, the taxpayer shall be required to respond to said notice. If the taxpayer fails to respond, the Commissioner or his duly authorized representative shall issue an assessment based on his findings. Such assessment may be protested administratively by filing a request for reconsideration or reinvestigation within thirty (30) days from receipt of the assessment in such form and manner as may be prescribed by implementing rules and regulations. Within sixty (60) days from filing of the protest, all relevant supporting documents shall have been submitted; otherwise, the assessment shall become final. If the protest is denied in whole or in part, or is not acted upon within one hundred eighty (180) days from submission of documents, the taxpayer adversely affected by the decision or inaction may appeal to the Court of Tax Appeals within thirty (30) days from receipt of the said decision, or from the lapse of one hundred eighty (180)-day period; otherwise, the decision shall become final, executory and demandable. (Emphasis supplied). Indeed, Section 228 of the Tax Code clearly requires that the taxpayer must first be informed that he is liable for deficiency taxes through the sending of a PAN. He must be informed of the facts and the law upon which the assessment is made. The law imposes a substantive, not merely a formal, requirement. To proceed heedlessly with tax collection without first establishing a valid assessment is evidently violative of the cardinal principle in administrative investigations - that taxpayers should be able to present their case and adduce supporting evidence.14 This is confirmed under the provisions R.R. No. 12-99 of the BIR which pertinently provide: SECTION 3. Due Process Requirement in the Issuance of a Deficiency Tax Assessment. 3.1 Mode of procedures in the issuance of a deficiency tax assessment: 3.1.1 Notice for informal conference. The Revenue Officer who audited the taxpayer's records shall, among others, state in his report whether or not the taxpayer agrees with his findings that the taxpayer is liable for deficiency tax or taxes. If the taxpayer is not amenable, based on the said Officer's submitted report of investigation, the taxpayer shall be informed, in writing, by the Revenue District Office or by the Special Investigation Division, as the case may be (in the case Revenue Regional Offices) or by the Chief of Division concerned (in the case of the BIR National Office) of the discrepancy or discrepancies in the taxpayer's payment of his internal revenue taxes, for the purpose of

"Informal Conference," in order to afford the taxpayer with an opportunity to present his side of the case. If the taxpayer fails to respond within fifteen (15) days from date of receipt of the notice for informal conference, he shall be considered in default, in which case, the Revenue District Officer or the Chief of the Special Investigation Division of the Revenue Regional Office, or the Chief of Division in the National Office, as the case may be, shall endorse the case with the least possible delay to the Assessment Division of the Revenue Regional Office or to the Commissioner or his duly authorized representative, as the case may be, for appropriate review and issuance of a deficiency tax assessment, if warranted. 3.1.2 Preliminary Assessment Notice (PAN). If after review and evaluation by the Assessment Division or by the Commissioner or his duly authorized representative, as the case may be, it is determined that there exists sufficient basis to assess the taxpayer for any deficiency tax or taxes, the said Office shall issue to the taxpayer, at least by registered mail, a Preliminary Assessment Notice (PAN) for the proposed assessment, showing in detail, the facts and the law, rules and regulations, or jurisprudence on which the proposed assessment is based (see illustration in ANNEX A hereof). If the taxpayer fails to respond within fifteen (15) days from date of receipt of the PAN, he shall be considered in default, in which case, a formal letter of demand and assessment notice shall be caused to be issued by the said Office, calling for payment of the taxpayer's deficiency tax liability, inclusive of the applicable penalties. 3.1.3 Exceptions to Prior Notice of the Assessment. The notice for informal conference and the preliminary assessment notice shall not be required in any of the following cases, in which case, issuance of the formal assessment notice for the payment of the taxpayer's deficiency tax liability shall be sufficient: (i) When the finding for any deficiency tax is the result of mathematical error in the computation of the tax appearing on the face of the tax return filed by the taxpayer; or (ii) When a discrepancy has been determined between the tax withheld and the amount actually remitted by the withholding agent; or (iii) When a taxpayer who opted to claim a refund or tax credit of excess creditable withholding tax for a taxable period was determined to have carried over and automatically applied the same amount claimed against the estimated tax liabilities for the taxable quarter or quarters of the succeeding taxable year; or (iv) When the excise tax due on excisable articles has not been paid; or (v) When an article locally purchased or imported by an exempt person, such as, but not limited to, vehicles, capital equipment, machineries and spare parts, has been sold, traded or transferred to non-exempt persons.

3.1.4 Formal Letter of Demand and Assessment Notice. The formal letter of demand and assessment notice shall be issued by the Commissioner or his duly authorized representative. The letter of demand calling for payment of the taxpayer's deficiency tax or taxes shall state the facts, the law, rules and regulations, or jurisprudence on which the assessment is based, otherwise, the formal letter of demand and assessment notice shall be void (see illustration in ANNEX B hereof). The same shall be sent to the taxpayer only by registered mail or by personal delivery. If sent by personal delivery, the taxpayer or his duly authorized representative shall acknowledge receipt thereof in the duplicate copy of the letter of demand, showing the following: (a) His name; (b) signature; (c) designation and authority to act for and in behalf of the taxpayer, if acknowledged received by a person other than the taxpayer himself; and (d) date of receipt thereof. x x x. From the provision quoted above, it is clear that the sending of a PAN to taxpayer to inform him of the assessment made is but part of the "due process requirement in the issuance of a deficiency tax assessment," the absence of which renders nugatory any assessment made by the tax authorities. The use of the word "shall" in subsection 3.1.2 describes the mandatory nature of the service of a PAN. The persuasiveness of the right to due process reaches both substantial and procedural rights and the failure of the CIR to strictly comply with the requirements laid down by law and its own rules is a denial of Metro Stars right to due process.15 Thus, for its failure to send the PAN stating the facts and the law on which the assessment was made as required by Section 228 of R.A. No. 8424, the assessment made by the CIR is void. The case of CIR v. Menguito16 cited by the CIR in support of its argument that only the nonservice of the FAN is fatal to the validity of an assessment, cannot apply to this case because the issue therein was the non-compliance with the provisions of R. R. No. 12-85 which sought to interpret Section 229 of the old tax law. RA No. 8424 has already amended the provision of Section 229 on protesting an assessment. The old requirement of merely notifying the taxpayer of the CIRs findings was changed in 1998 to informing the taxpayer of not only the law, but also of the facts on which an assessment would be made. Otherwise, the assessment itself would be invalid.17 The regulation then, on the other hand, simply provided that a notice be sent to the respondent in the form prescribed, and that no consequence would ensue for failure to comply with that form.1avvphi1 The Court need not belabor to discuss the matter of Metro Stars failure to file its protest, for it is well-settled that a void assessment bears no fruit.18 It is an elementary rule enshrined in the 1987 Constitution that no person shall be deprived of property without due process of law.19 In balancing the scales between the power of the State to tax and its inherent right to prosecute perceived transgressors of the law on one side, and the constitutional rights of a citizen to due process of law and the equal protection of the laws on the other, the scales must tilt in favor of the individual, for a citizens right is amply protected by the

Bill of Rights under the Constitution. Thus, while "taxes are the lifeblood of the government," the power to tax has its limits, in spite of all its plenitude. Hence in Commissioner of Internal Revenue v. Algue, Inc.,20 it was said Taxes are the lifeblood of the government and so should be collected without unnecessary hindrance. On the other hand, such collection should be made in accordance with law as any arbitrariness will negate the very reason for government itself. It is therefore necessary to reconcile the apparently conflicting interests of the authorities and the taxpayers so that the real purpose of taxation, which is the promotion of the common good, may be achieved. xxx xxx xxx

It is said that taxes are what we pay for civilized society. Without taxes, the government would be paralyzed for the lack of the motive power to activate and operate it. Hence, despite the natural reluctance to surrender part of ones hard-earned income to taxing authorities, every person who is able to must contribute his share in the running of the government. The government for its part is expected to respond in the form of tangible and intangible benefits intended to improve the lives of the people and enhance their moral and material values. This symbiotic relationship is the rationale of taxation and should dispel the erroneous notion that it is an arbitrary method of exaction by those in the seat of power. But even as we concede the inevitability and indispensability of taxation, it is a requirement in all democratic regimes that it be exercised reasonably and in accordance with the prescribed procedure. If it is not, then the taxpayer has a right to complain and the courts will then come to his succor. For all the awesome power of the tax collector, he may still be stopped in his tracks if the taxpayer can demonstrate x x x that the law has not been observed.21 (Emphasis supplied). WHEREFORE, the petition is DENIED. G.R. No. 178087 May 5, 2010

COMMISSIONER OF INTERNAL REVENUE, Petitioner, vs. KUDOS METAL CORPORATION, Respondent. DECISION DEL CASTILLO, J.: The prescriptive period on when to assess taxes benefits both the government and the taxpayer.1 Exceptions extending the period to assess must, therefore, be strictly construed. This Petition for Review on Certiorari seeks to set aside the Decision2 dated March 30, 2007 of the Court of Tax Appeals (CTA) affirming the cancellation of the assessment notices for having been issued beyond the prescriptive period and the Resolution3 dated May 18, 2007 denying the motion for reconsideration.

Factual Antecedents On April 15, 1999, respondent Kudos Metal Corporation filed its Annual Income Tax Return (ITR) for the taxable year 1998. Pursuant to a Letter of Authority dated September 7, 1999, the Bureau of Internal Revenue (BIR) served upon respondent three Notices of Presentation of Records. Respondent failed to comply with these notices, hence, the BIR issued a Subpeona Duces Tecum dated September 21, 2006, receipt of which was acknowledged by respondents President, Mr. Chan Ching Bio, in a letter dated October 20, 2000. A review and audit of respondents records then ensued. On December 10, 2001, Nelia Pasco (Pasco), respondents accountant, executed a Waiver of the Defense of Prescription,4 which was notarized on January 22, 2002, received by the BIR Enforcement Service on January 31, 2002 and by the BIR Tax Fraud Division on February 4, 2002, and accepted by the Assistant Commissioner of the Enforcement Service, Percival T. Salazar (Salazar). This was followed by a second Waiver of Defense of Prescription5 executed by Pasco on February 18, 2003, notarized on February 19, 2003, received by the BIR Tax Fraud Division on February 28, 2003 and accepted by Assistant Commissioner Salazar. On August 25, 2003, the BIR issued a Preliminary Assessment Notice for the taxable year 1998 against the respondent. This was followed by a Formal Letter of Demand with Assessment Notices for taxable year 1998, dated September 26, 2003 which was received by respondent on November 12, 2003. Respondent challenged the assessments by filing its "Protest on Various Tax Assessments" on December 3, 2003 and its "Legal Arguments and Documents in Support of Protests against Various Assessments" on February 2, 2004. On June 22, 2004, the BIR rendered a final Decision6 on the matter, requesting the immediate payment of the following tax liabilities: Kind of Tax Income Tax VAT EWT Amount P 9,693,897.85 13,962,460.90 1,712,336.76

Withholding Tax-Compensation 247,353.24 Penalties Total 8,000.00

P25,624,048.76

Ruling of the Court of Tax Appeals, Second Division Believing that the governments right to assess taxes had prescribed, respondent filed on August 27, 2004 a Petition for Review7 with the CTA. Petitioner in turn filed his Answer.8 On April 11, 2005, respondent filed an "Urgent Motion for Preferential Resolution of the Issue on Prescription."9 On October 4, 2005, the CTA Second Division issued a Resolution10 canceling the assessment notices issued against respondent for having been issued beyond the prescriptive period. It found the first Waiver of the Statute of Limitations incomplete and defective for failure to comply with the provisions of Revenue Memorandum Order (RMO) No. 20-90. Thus: First, the Assistant Commissioner is not the revenue official authorized to sign the waiver, as the tax case involves more than P1,000,000.00. In this regard, only the Commissioner is authorized to enter into agreement with the petitioner in extending the period of assessment; Secondly, the waiver failed to indicate the date of acceptance. Such date of acceptance is necessary to determine whether the acceptance was made within the prescriptive period; Third, the fact of receipt by the taxpayer of his file copy was not indicated on the original copy. The requirement to furnish the taxpayer with a copy of the waiver is not only to give notice of the existence of the document but also of the acceptance by the BIR and the perfection of the agreement.1avvphi1 The subject waiver is therefore incomplete and defective. As such, the three-year prescriptive period was not tolled or extended and continued to run. x x x11 Petitioner moved for reconsideration but the CTA Second Division denied the motion in a Resolution12 dated April 18, 2006. Ruling of the Court of Tax Appeals, En Banc On appeal, the CTA En Banc affirmed the cancellation of the assessment notices. Although it ruled that the Assistant Commissioner was authorized to sign the waiver pursuant to Revenue Delegation Authority Order (RDAO) No. 05-01, it found that the first waiver was still invalid based on the second and third grounds stated by the CTA Second Division. Pertinent portions of the Decision read as follows: While the Court En Banc agrees with the second and third grounds for invalidating the first waiver, it finds that the Assistant Commissioner of the Enforcement Service is authorized to sign the waiver pursuant to RDAO No. 05-01, which provides in part as follows:

A. For National Office cases Designated Revenue Official 1. Assistant Commissioner (ACIR), For tax fraud and policy Enforcement Service cases 2. ACIR, Large Taxpayers Service For large taxpayers cases other than those cases falling under Subsection B hereof 3. ACIR, Legal Service For cases pending verification and awaiting resolution of certain legal issues prior to prescription and for issuance/compliance of Subpoena Duces Tecum 4. ACIR, Assessment Service (AS) For cases which are pending in or subject to review or approval by the ACIR, AS Based on the foregoing, the Assistant Commissioner, Enforcement Service is authorized to sign waivers in tax fraud cases. A perusal of the records reveals that the investigation of the subject deficiency taxes in this case was conducted by the National Investigation Division of the BIR, which was formerly named the Tax Fraud Division. Thus, the subject assessment is a tax fraud case. Nevertheless, the first waiver is still invalid based on the second and third grounds stated by the Court in Division. Hence, it did not extend the prescriptive period to assess. Moreover, assuming arguendo that the first waiver is valid, the second waiver is invalid for violating Section 222(b) of the 1997 Tax Code which mandates that the period agreed upon in a waiver of the statute can still be extended by subsequent written agreement, provided that it is executed prior to the expiration of the first period agreed upon. As previously discussed, the exceptions to the law on prescription must be strictly construed. In the case at bar, the period agreed upon in the subject first waiver expired on December 31, 2002. The second waiver in the instant case which was supposed to extend the period to assess to December 31, 2003 was executed on February 18, 2003 and was notarized on February 19, 2003. Clearly, the second waiver was executed after the expiration of the first period agreed upon. Consequently, the same could not have tolled the 3-year prescriptive period to assess.13 Petitioner sought reconsideration but the same was unavailing. Issue Hence, the present recourse where petitioner interposes that: THE COURT OF TAX APPEALS EN BANC ERRED IN RULING THAT THE GOVERNMENTS RIGHT TO ASSESS UNPAID TAXES OF RESPONDENT PRESCRIBED.14

Petitioners Arguments Petitioner argues that the governments right to assess taxes is not barred by prescription as the two waivers executed by respondent, through its accountant, effectively tolled or extended the period within which the assessment can be made. In disputing the conclusion of the CTA that the waivers are invalid, petitioner claims that respondent is estopped from adopting a position contrary to what it has previously taken. Petitioner insists that by acquiescing to the audit during the period specified in the waivers, respondent led the government to believe that the "delay" in the process would not be utilized against it. Thus, respondent may no longer repudiate the validity of the waivers and raise the issue of prescription. Respondents Arguments Respondent maintains that prescription had set in due to the invalidity of the waivers executed by Pasco, who executed the same without any written authority from it, in clear violation of RDAO No. 5-01. As to the doctrine of estoppel by acquiescence relied upon by petitioner, respondent counters that the principle of equity comes into play only when the law is doubtful, which is not present in the instant case. Our Ruling The petition is bereft of merit. Section 20315 of the National Internal Revenue Code of 1997 (NIRC) mandates the government to assess internal revenue taxes within three years from the last day prescribed by law for the filing of the tax return or the actual date of filing of such return, whichever comes later. Hence, an assessment notice issued after the three-year prescriptive period is no longer valid and effective. Exceptions however are provided under Section 22216 of the NIRC. The waivers executed by respondents accountant did not extend the period within which the assessment can be made Petitioner does not deny that the assessment notices were issued beyond the three-year prescriptive period, but claims that the period was extended by the two waivers executed by respondents accountant. We do not agree. Section 222 (b) of the NIRC provides that the period to assess and collect taxes may only be extended upon a written agreement between the CIR and the taxpayer executed before the expiration of the three-year period. RMO 20-9017 issued on April 4, 1990 and RDAO 05-0118 issued on August 2, 2001 lay down the procedure for the proper execution of the waiver, to wit: 1. The waiver must be in the proper form prescribed by RMO 20-90. The phrase "but not after ______ 19 ___", which indicates the expiry date of the period agreed upon to

assess/collect the tax after the regular three-year period of prescription, should be filled up. 2. The waiver must be signed by the taxpayer himself or his duly authorized representative. In the case of a corporation, the waiver must be signed by any of its responsible officials. In case the authority is delegated by the taxpayer to a representative, such delegation should be in writing and duly notarized. 3. The waiver should be duly notarized. 4. The CIR or the revenue official authorized by him must sign the waiver indicating that the BIR has accepted and agreed to the waiver. The date of such acceptance by the BIR should be indicated. However, before signing the waiver, the CIR or the revenue official authorized by him must make sure that the waiver is in the prescribed form, duly notarized, and executed by the taxpayer or his duly authorized representative. 5. Both the date of execution by the taxpayer and date of acceptance by the Bureau should be before the expiration of the period of prescription or before the lapse of the period agreed upon in case a subsequent agreement is executed. 6. The waiver must be executed in three copies, the original copy to be attached to the docket of the case, the second copy for the taxpayer and the third copy for the Office accepting the waiver. The fact of receipt by the taxpayer of his/her file copy must be indicated in the original copy to show that the taxpayer was notified of the acceptance of the BIR and the perfection of the agreement.19 A perusal of the waivers executed by respondents accountant reveals the following infirmities: 1. The waivers were executed without the notarized written authority of Pasco to sign the waiver in behalf of respondent. 2. The waivers failed to indicate the date of acceptance. 3. The fact of receipt by the respondent of its file copy was not indicated in the original copies of the waivers. Due to the defects in the waivers, the period to assess or collect taxes was not extended. Consequently, the assessments were issued by the BIR beyond the three-year period and are void. Estoppel does not apply in this case We find no merit in petitioners claim that respondent is now estopped from claiming prescription since by executing the waivers, it was the one which asked for additional time to submit the required documents.

In Collector of Internal Revenue v. Suyoc Consolidated Mining Company,20 the doctrine of estoppel prevented the taxpayer from raising the defense of prescription against the efforts of the government to collect the assessed tax. However, it must be stressed that in the said case, estoppel was applied as an exception to the statute of limitations on collection of taxes and not on the assessment of taxes, as the BIR was able to make an assessment within the prescribed period. More important, there was a finding that the taxpayer made several requests or positive acts to convince the government to postpone the collection of taxes, viz: It appears that the first assessment made against respondent based on its second final return filed on November 28, 1946 was made on February 11, 1947. Upon receipt of this assessment respondent requested for at least one year within which to pay the amount assessed although it reserved its right to question the correctness of the assessment before actual payment. Petitioner granted an extension of only three months. When it failed to pay the tax within the period extended, petitioner sent respondent a letter on November 28, 1950 demanding payment of the tax as assessed, and upon receipt of the letter respondent asked for a reinvestigation and reconsideration of the assessment. When this request was denied, respondent again requested for a reconsideration on April 25, 1952, which was denied on May 6, 1953, which denial was appealed to the Conference Staff. The appeal was heard by the Conference Staff from September 2, 1953 to July 16, 1955, and as a result of these various negotiations, the assessment was finally reduced on July 26, 1955. This is the ruling which is now being questioned after a protracted negotiation on the ground that the collection of the tax has already prescribed. It is obvious from the foregoing that petitioner refrained from collecting the tax by distraint or levy or by proceeding in court within the 5-year period from the filing of the second amended final return due to the several requests of respondent for extension to which petitioner yielded to give it every opportunity to prove its claim regarding the correctness of the assessment. Because of such requests, several reinvestigations were made and a hearing was even held by the Conference Staff organized in the collection office to consider claims of such nature which, as the record shows, lasted for several months. After inducing petitioner to delay collection as he in fact did, it is most unfair for respondent to now take advantage of such desistance to elude his deficiency income tax liability to the prejudice of the Government invoking the technical ground of prescription. While we may agree with the Court of Tax Appeals that a mere request for reexamination or reinvestigation may not have the effect of suspending the running of the period of limitation for in such case there is need of a written agreement to extend the period between the Collector and the taxpayer, there are cases however where a taxpayer may be prevented from setting up the defense of prescription even if he has not previously waived it in writing as when by his repeated requests or positive acts the Government has been, for good reasons, persuaded to postpone collection to make him feel that the demand was not unreasonable or that no harassment or injustice is meant by the Government. And when such situation comes to pass there are authorities that hold, based on weighty reasons, that such an attitude or behavior should not be countenanced if only to protect the interest of the Government. This case has no precedent in this jurisdiction for it is the first time that such has risen, but there are several precedents that may be invoked in American jurisprudence. As Mr. Justice Cardozo

has said: "The applicable principle is fundamental and unquestioned. He who prevents a thing from being done may not avail himself of the nonperformance which he has himself occasioned, for the law says to him in effect "this is your own act, and therefore you are not damnified." "(R. H. Stearns Co. vs. U.S., 78 L. ed., 647). Or, as was aptly said, "The tax could have been collected, but the government withheld action at the specific request of the plaintiff. The plaintiff is now estopped and should not be permitted to raise the defense of the Statute of Limitations." [Newport Co. vs. U.S., (DC-WIS), 34 F. Supp. 588].21 Conversely, in this case, the assessments were issued beyond the prescribed period. Also, there is no showing that respondent made any request to persuade the BIR to postpone the issuance of the assessments. The doctrine of estoppel cannot be applied in this case as an exception to the statute of limitations on the assessment of taxes considering that there is a detailed procedure for the proper execution of the waiver, which the BIR must strictly follow. As we have often said, the doctrine of estoppel is predicated on, and has its origin in, equity which, broadly defined, is justice according to natural law and right.22 As such, the doctrine of estoppel cannot give validity to an act that is prohibited by law or one that is against public policy.23 It should be resorted to solely as a means of preventing injustice and should not be permitted to defeat the administration of the law, or to accomplish a wrong or secure an undue advantage, or to extend beyond them requirements of the transactions in which they originate.24 Simply put, the doctrine of estoppel must be sparingly applied. Moreover, the BIR cannot hide behind the doctrine of estoppel to cover its failure to comply with RMO 20-90 and RDAO 05-01, which the BIR itself issued. As stated earlier, the BIR failed to verify whether a notarized written authority was given by the respondent to its accountant, and to indicate the date of acceptance and the receipt by the respondent of the waivers. Having caused the defects in the waivers, the BIR must bear the consequence. It cannot shift the blame to the taxpayer. To stress, a waiver of the statute of limitations, being a derogation of the taxpayers right to security against prolonged and unscrupulous investigations, must be carefully and strictly construed.25 As to the alleged delay of the respondent to furnish the BIR of the required documents, this cannot be taken against respondent. Neither can the BIR use this as an excuse for issuing the assessments beyond the three-year period because with or without the required documents, the CIR has the power to make assessments based on the best evidence obtainable.26 WHEREFORE, the petition is DENIED. The assailed Decision dated March 30, 2007 and Resolution dated May 18, 2007 of the Court of Tax Appeals are hereby AFFIRMED. G.R. No. 178087 May 5, 2010

COMMISSIONER OF INTERNAL REVENUE, Petitioner, vs. KUDOS METAL CORPORATION, Respondent.

DECISION DEL CASTILLO, J.: The prescriptive period on when to assess taxes benefits both the government and the taxpayer.1 Exceptions extending the period to assess must, therefore, be strictly construed. This Petition for Review on Certiorari seeks to set aside the Decision2 dated March 30, 2007 of the Court of Tax Appeals (CTA) affirming the cancellation of the assessment notices for having been issued beyond the prescriptive period and the Resolution3 dated May 18, 2007 denying the motion for reconsideration. Factual Antecedents On April 15, 1999, respondent Kudos Metal Corporation filed its Annual Income Tax Return (ITR) for the taxable year 1998. Pursuant to a Letter of Authority dated September 7, 1999, the Bureau of Internal Revenue (BIR) served upon respondent three Notices of Presentation of Records. Respondent failed to comply with these notices, hence, the BIR issued a Subpeona Duces Tecum dated September 21, 2006, receipt of which was acknowledged by respondents President, Mr. Chan Ching Bio, in a letter dated October 20, 2000. A review and audit of respondents records then ensued. On December 10, 2001, Nelia Pasco (Pasco), respondents accountant, executed a Waiver of the Defense of Prescription,4 which was notarized on January 22, 2002, received by the BIR Enforcement Service on January 31, 2002 and by the BIR Tax Fraud Division on February 4, 2002, and accepted by the Assistant Commissioner of the Enforcement Service, Percival T. Salazar (Salazar). This was followed by a second Waiver of Defense of Prescription5 executed by Pasco on February 18, 2003, notarized on February 19, 2003, received by the BIR Tax Fraud Division on February 28, 2003 and accepted by Assistant Commissioner Salazar. On August 25, 2003, the BIR issued a Preliminary Assessment Notice for the taxable year 1998 against the respondent. This was followed by a Formal Letter of Demand with Assessment Notices for taxable year 1998, dated September 26, 2003 which was received by respondent on November 12, 2003. Respondent challenged the assessments by filing its "Protest on Various Tax Assessments" on December 3, 2003 and its "Legal Arguments and Documents in Support of Protests against Various Assessments" on February 2, 2004. On June 22, 2004, the BIR rendered a final Decision6 on the matter, requesting the immediate payment of the following tax liabilities:

Kind of Tax Income Tax VAT EWT

Amount P 9,693,897.85 13,962,460.90 1,712,336.76

Withholding Tax-Compensation 247,353.24 Penalties Total 8,000.00 P25,624,048.76

Ruling of the Court of Tax Appeals, Second Division Believing that the governments right to assess taxes had prescribed, respondent filed on August 27, 2004 a Petition for Review7 with the CTA. Petitioner in turn filed his Answer.8 On April 11, 2005, respondent filed an "Urgent Motion for Preferential Resolution of the Issue on Prescription."9 On October 4, 2005, the CTA Second Division issued a Resolution10 canceling the assessment notices issued against respondent for having been issued beyond the prescriptive period. It found the first Waiver of the Statute of Limitations incomplete and defective for failure to comply with the provisions of Revenue Memorandum Order (RMO) No. 20-90. Thus: First, the Assistant Commissioner is not the revenue official authorized to sign the waiver, as the tax case involves more than P1,000,000.00. In this regard, only the Commissioner is authorized to enter into agreement with the petitioner in extending the period of assessment; Secondly, the waiver failed to indicate the date of acceptance. Such date of acceptance is necessary to determine whether the acceptance was made within the prescriptive period; Third, the fact of receipt by the taxpayer of his file copy was not indicated on the original copy. The requirement to furnish the taxpayer with a copy of the waiver is not only to give notice of the existence of the document but also of the acceptance by the BIR and the perfection of the agreement.1avvphi1 The subject waiver is therefore incomplete and defective. As such, the three-year prescriptive period was not tolled or extended and continued to run. x x x11 Petitioner moved for reconsideration but the CTA Second Division denied the motion in a Resolution12 dated April 18, 2006. Ruling of the Court of Tax Appeals, En Banc

On appeal, the CTA En Banc affirmed the cancellation of the assessment notices. Although it ruled that the Assistant Commissioner was authorized to sign the waiver pursuant to Revenue Delegation Authority Order (RDAO) No. 05-01, it found that the first waiver was still invalid based on the second and third grounds stated by the CTA Second Division. Pertinent portions of the Decision read as follows: While the Court En Banc agrees with the second and third grounds for invalidating the first waiver, it finds that the Assistant Commissioner of the Enforcement Service is authorized to sign the waiver pursuant to RDAO No. 05-01, which provides in part as follows: A. For National Office cases Designated Revenue Official 1. Assistant Commissioner (ACIR), For tax fraud and policy Enforcement Service cases 2. ACIR, Large Taxpayers Service For large taxpayers cases other than those cases falling under Subsection B hereof 3. ACIR, Legal Service For cases pending verification and awaiting resolution of certain legal issues prior to prescription and for issuance/compliance of Subpoena Duces Tecum 4. ACIR, Assessment Service (AS) For cases which are pending in or subject to review or approval by the ACIR, AS Based on the foregoing, the Assistant Commissioner, Enforcement Service is authorized to sign waivers in tax fraud cases. A perusal of the records reveals that the investigation of the subject deficiency taxes in this case was conducted by the National Investigation Division of the BIR, which was formerly named the Tax Fraud Division. Thus, the subject assessment is a tax fraud case. Nevertheless, the first waiver is still invalid based on the second and third grounds stated by the Court in Division. Hence, it did not extend the prescriptive period to assess. Moreover, assuming arguendo that the first waiver is valid, the second waiver is invalid for violating Section 222(b) of the 1997 Tax Code which mandates that the period agreed upon in a waiver of the statute can still be extended by subsequent written agreement, provided that it is executed prior to the expiration of the first period agreed upon. As previously discussed, the exceptions to the law on prescription must be strictly construed. In the case at bar, the period agreed upon in the subject first waiver expired on December 31, 2002. The second waiver in the instant case which was supposed to extend the period to assess to December 31, 2003 was executed on February 18, 2003 and was notarized on February 19, 2003. Clearly, the second waiver was executed after the expiration of the first period agreed upon. Consequently, the same could not have tolled the 3-year prescriptive period to assess.13

Petitioner sought reconsideration but the same was unavailing. Issue Hence, the present recourse where petitioner interposes that: THE COURT OF TAX APPEALS EN BANC ERRED IN RULING THAT THE GOVERNMENTS RIGHT TO ASSESS UNPAID TAXES OF RESPONDENT PRESCRIBED.14 Petitioners Arguments Petitioner argues that the governments right to assess taxes is not barred by prescription as the two waivers executed by respondent, through its accountant, effectively tolled or extended the period within which the assessment can be made. In disputing the conclusion of the CTA that the waivers are invalid, petitioner claims that respondent is estopped from adopting a position contrary to what it has previously taken. Petitioner insists that by acquiescing to the audit during the period specified in the waivers, respondent led the government to believe that the "delay" in the process would not be utilized against it. Thus, respondent may no longer repudiate the validity of the waivers and raise the issue of prescription. Respondents Arguments Respondent maintains that prescription had set in due to the invalidity of the waivers executed by Pasco, who executed the same without any written authority from it, in clear violation of RDAO No. 5-01. As to the doctrine of estoppel by acquiescence relied upon by petitioner, respondent counters that the principle of equity comes into play only when the law is doubtful, which is not present in the instant case. Our Ruling The petition is bereft of merit. Section 20315 of the National Internal Revenue Code of 1997 (NIRC) mandates the government to assess internal revenue taxes within three years from the last day prescribed by law for the filing of the tax return or the actual date of filing of such return, whichever comes later. Hence, an assessment notice issued after the three-year prescriptive period is no longer valid and effective. Exceptions however are provided under Section 22216 of the NIRC. The waivers executed by respondents accountant did not extend the period within which the assessment can be made Petitioner does not deny that the assessment notices were issued beyond the three-year prescriptive period, but claims that the period was extended by the two waivers executed by respondents accountant.

We do not agree. Section 222 (b) of the NIRC provides that the period to assess and collect taxes may only be extended upon a written agreement between the CIR and the taxpayer executed before the expiration of the three-year period. RMO 20-9017 issued on April 4, 1990 and RDAO 05-0118 issued on August 2, 2001 lay down the procedure for the proper execution of the waiver, to wit: 1. The waiver must be in the proper form prescribed by RMO 20-90. The phrase "but not after ______ 19 ___", which indicates the expiry date of the period agreed upon to assess/collect the tax after the regular three-year period of prescription, should be filled up. 2. The waiver must be signed by the taxpayer himself or his duly authorized representative. In the case of a corporation, the waiver must be signed by any of its responsible officials. In case the authority is delegated by the taxpayer to a representative, such delegation should be in writing and duly notarized. 3. The waiver should be duly notarized. 4. The CIR or the revenue official authorized by him must sign the waiver indicating that the BIR has accepted and agreed to the waiver. The date of such acceptance by the BIR should be indicated. However, before signing the waiver, the CIR or the revenue official authorized by him must make sure that the waiver is in the prescribed form, duly notarized, and executed by the taxpayer or his duly authorized representative. 5. Both the date of execution by the taxpayer and date of acceptance by the Bureau should be before the expiration of the period of prescription or before the lapse of the period agreed upon in case a subsequent agreement is executed. 6. The waiver must be executed in three copies, the original copy to be attached to the docket of the case, the second copy for the taxpayer and the third copy for the Office accepting the waiver. The fact of receipt by the taxpayer of his/her file copy must be indicated in the original copy to show that the taxpayer was notified of the acceptance of the BIR and the perfection of the agreement.19 A perusal of the waivers executed by respondents accountant reveals the following infirmities: 1. The waivers were executed without the notarized written authority of Pasco to sign the waiver in behalf of respondent. 2. The waivers failed to indicate the date of acceptance. 3. The fact of receipt by the respondent of its file copy was not indicated in the original copies of the waivers.

Due to the defects in the waivers, the period to assess or collect taxes was not extended. Consequently, the assessments were issued by the BIR beyond the three-year period and are void. Estoppel does not apply in this case We find no merit in petitioners claim that respondent is now estopped from claiming prescription since by executing the waivers, it was the one which asked for additional time to submit the required documents. In Collector of Internal Revenue v. Suyoc Consolidated Mining Company,20 the doctrine of estoppel prevented the taxpayer from raising the defense of prescription against the efforts of the government to collect the assessed tax. However, it must be stressed that in the said case, estoppel was applied as an exception to the statute of limitations on collection of taxes and not on the assessment of taxes, as the BIR was able to make an assessment within the prescribed period. More important, there was a finding that the taxpayer made several requests or positive acts to convince the government to postpone the collection of taxes, viz: It appears that the first assessment made against respondent based on its second final return filed on November 28, 1946 was made on February 11, 1947. Upon receipt of this assessment respondent requested for at least one year within which to pay the amount assessed although it reserved its right to question the correctness of the assessment before actual payment. Petitioner granted an extension of only three months. When it failed to pay the tax within the period extended, petitioner sent respondent a letter on November 28, 1950 demanding payment of the tax as assessed, and upon receipt of the letter respondent asked for a reinvestigation and reconsideration of the assessment. When this request was denied, respondent again requested for a reconsideration on April 25, 1952, which was denied on May 6, 1953, which denial was appealed to the Conference Staff. The appeal was heard by the Conference Staff from September 2, 1953 to July 16, 1955, and as a result of these various negotiations, the assessment was finally reduced on July 26, 1955. This is the ruling which is now being questioned after a protracted negotiation on the ground that the collection of the tax has already prescribed. It is obvious from the foregoing that petitioner refrained from collecting the tax by distraint or levy or by proceeding in court within the 5-year period from the filing of the second amended final return due to the several requests of respondent for extension to which petitioner yielded to give it every opportunity to prove its claim regarding the correctness of the assessment. Because of such requests, several reinvestigations were made and a hearing was even held by the Conference Staff organized in the collection office to consider claims of such nature which, as the record shows, lasted for several months. After inducing petitioner to delay collection as he in fact did, it is most unfair for respondent to now take advantage of such desistance to elude his deficiency income tax liability to the prejudice of the Government invoking the technical ground of prescription. While we may agree with the Court of Tax Appeals that a mere request for reexamination or reinvestigation may not have the effect of suspending the running of the period of limitation for in such case there is need of a written agreement to extend the period between the Collector and

the taxpayer, there are cases however where a taxpayer may be prevented from setting up the defense of prescription even if he has not previously waived it in writing as when by his repeated requests or positive acts the Government has been, for good reasons, persuaded to postpone collection to make him feel that the demand was not unreasonable or that no harassment or injustice is meant by the Government. And when such situation comes to pass there are authorities that hold, based on weighty reasons, that such an attitude or behavior should not be countenanced if only to protect the interest of the Government. This case has no precedent in this jurisdiction for it is the first time that such has risen, but there are several precedents that may be invoked in American jurisprudence. As Mr. Justice Cardozo has said: "The applicable principle is fundamental and unquestioned. He who prevents a thing from being done may not avail himself of the nonperformance which he has himself occasioned, for the law says to him in effect "this is your own act, and therefore you are not damnified." "(R. H. Stearns Co. vs. U.S., 78 L. ed., 647). Or, as was aptly said, "The tax could have been collected, but the government withheld action at the specific request of the plaintiff. The plaintiff is now estopped and should not be permitted to raise the defense of the Statute of Limitations." [Newport Co. vs. U.S., (DC-WIS), 34 F. Supp. 588].21 Conversely, in this case, the assessments were issued beyond the prescribed period. Also, there is no showing that respondent made any request to persuade the BIR to postpone the issuance of the assessments. The doctrine of estoppel cannot be applied in this case as an exception to the statute of limitations on the assessment of taxes considering that there is a detailed procedure for the proper execution of the waiver, which the BIR must strictly follow. As we have often said, the doctrine of estoppel is predicated on, and has its origin in, equity which, broadly defined, is justice according to natural law and right.22 As such, the doctrine of estoppel cannot give validity to an act that is prohibited by law or one that is against public policy.23 It should be resorted to solely as a means of preventing injustice and should not be permitted to defeat the administration of the law, or to accomplish a wrong or secure an undue advantage, or to extend beyond them requirements of the transactions in which they originate.24 Simply put, the doctrine of estoppel must be sparingly applied. Moreover, the BIR cannot hide behind the doctrine of estoppel to cover its failure to comply with RMO 20-90 and RDAO 05-01, which the BIR itself issued. As stated earlier, the BIR failed to verify whether a notarized written authority was given by the respondent to its accountant, and to indicate the date of acceptance and the receipt by the respondent of the waivers. Having caused the defects in the waivers, the BIR must bear the consequence. It cannot shift the blame to the taxpayer. To stress, a waiver of the statute of limitations, being a derogation of the taxpayers right to security against prolonged and unscrupulous investigations, must be carefully and strictly construed.25 As to the alleged delay of the respondent to furnish the BIR of the required documents, this cannot be taken against respondent. Neither can the BIR use this as an excuse for issuing the assessments beyond the three-year period because with or without the required documents, the CIR has the power to make assessments based on the best evidence obtainable.26

WHEREFORE, the petition is DENIED. The assailed Decision dated March 30, 2007 and Resolution dated May 18, 2007 of the Court of Tax Appeals are hereby AFFIRMED. G.R. No. 177279 October 13, 2010

COMMISSIONER OF INTERNAL REVENUE, Petitioner, vs. HON. RAUL M. GONZALEZ, Secretary of Justice, L. M. CAMUS ENGINEERING CORPORATION (represented by LUIS M. CAMUS and LINO D. MENDOZA), Respondents. DECISION VILLARAMA, JR., J.: This is a petition for review on certiorari under Rule 45 of the 1997 Rules of Civil Procedure, as amended, assailing the Decision1 dated October 31, 2006 and Resolution2 dated March 6, 2007 of the Court of Appeals (CA) in CA-G.R. SP No. 93387 which affirmed the Resolution3 dated December 13, 2005 of respondent Secretary of Justice in I.S. No. 2003-774 for violation of Sections 254 and 255 of the National Internal Revenue Code of 1997 (NIRC). The facts as culled from the records: Pursuant to Letter of Authority (LA) No. 00009361 dated August 25, 2000 issued by then Commissioner of Internal Revenue (petitioner) Dakila B. Fonacier, Revenue Officers Remedios C. Advincula, Jr., Simplicio V. Cabantac, Jr., Ricardo L. Suba, Jr. and Aurelio Agustin T. Zamora supervised by Section Chief Sixto C. Dy, Jr. of the Tax Fraud Division (TFD), National Office, conducted a fraud investigation for all internal revenue taxes to ascertain/determine the tax liabilities of respondent L. M. Camus Engineering Corporation (LMCEC) for the taxable years 1997, 1998 and 1999.4 The audit and investigation against LMCEC was precipitated by the information provided by an "informer" that LMCEC had substantial underdeclared income for the said period. For failure to comply with the subpoena duces tecum issued in connection with the tax fraud investigation, a criminal complaint was instituted by the Bureau of Internal Revenue (BIR) against LMCEC on January 19, 2001 for violation of Section 266 of the NIRC (I.S. No. 00-956 of the Office of the City Prosecutor of Quezon City).5 Based on data obtained from an "informer" and various clients of LMCEC,6 it was discovered that LMCEC filed fraudulent tax returns with substantial underdeclarations of taxable income for the years 1997, 1998 and 1999. Petitioner thus assessed the company of total deficiency taxes amounting to P430,958,005.90 (income tax - P318,606,380.19 and value-added tax [VAT] P112,351,625.71) covering the said period. The Preliminary Assessment Notice (PAN) was received by LMCEC on February 22, 2001.7 LMCECs alleged underdeclared income was summarized by petitioner as follows: Year Income Income Undeclared Percentage of

Per ITR 1997 1998 1999 96,638,540.00 86,793,913.00 88,287,792.00

Per Investigation 283,412,140.84 236,863,236.81 251,507,903.13

Income 186,733,600.84 150,069,323.81 163,220,111.13

Underdeclaration 193.30% 172.90% 184.90%8

In view of the above findings, assessment notices together with a formal letter of demand dated August 7, 2002 were sent to LMCEC through personal service on October 1, 2002.9 Since the company and its representatives refused to receive the said notices and demand letter, the revenue officers resorted to constructive service10 in accordance with Section 3, Revenue Regulations (RR) No. 12-9911. On May 21, 2003, petitioner, through then Commissioner Guillermo L. Parayno, Jr., referred to the Secretary of Justice for preliminary investigation its complaint against LMCEC, Luis M. Camus and Lino D. Mendoza, the latter two were sued in their capacities as President and Comptroller, respectively. The case was docketed as I.S. No. 2003-774. In the Joint Affidavit executed by the revenue officers who conducted the tax fraud investigation, it was alleged that despite the receipt of the final assessment notice and formal demand letter on October 1, 2002, LMCEC failed and refused to pay the deficiency tax assessment in the total amount of P630,164,631.61, inclusive of increments, which had become final and executory as a result of the said taxpayers failure to file a protest thereon within the thirty (30)-day reglementary period.12 Camus and Mendoza filed a Joint Counter-Affidavit contending that LMCEC cannot be held liable whatsoever for the alleged tax deficiency which had become due and demandable. Considering that the complaint and its annexes all showed that the suit is a simple civil action for collection and not a tax evasion case, the Department of Justice (DOJ) is not the proper forum for BIRs complaint. They also assail as invalid the assessment notices which bear no serial numbers and should be shown to have been validly served by an Affidavit of Constructive Service executed and sworn to by the revenue officers who served the same. As stated in LMCECs letter-protest dated December 12, 2002 addressed to Revenue District Officer (RDO) Clavelina S. Nacar of RD No. 40, Cubao, Quezon City, the company had already undergone a series of routine examinations for the years 1997, 1998 and 1999; under the NIRC, only one examination of the books of accounts is allowed per taxable year.13 LMCEC further averred that it had availed of the Bureaus Tax Amnesty Programs (Economic Recovery Assistance Payment [ERAP] Program and the Voluntary Assessment Program [VAP]) for 1998 and 1999; for 1997, its tax liability was terminated and closed under Letter of Termination14 dated June 1, 1999 issued by petitioner and signed by the Chief of the Assessment Division.15 LMCEC claimed it made payments of income tax, VAT and expanded withholding tax (EWT), as follows: YEAR AMOUNT OF TAXES

PAID 1997 Termination Letter Under Letter of Authority No. 174600 Dated November 4, 1998 ERAP Program pursuant to RR #2-99 VAP Program pursuant to RR #8-2001 EWT - P 6,000.00 VAT - 540,605.02 IT - 3,000.00 WC - 38,404.55 VAT - 61,635.40 IT - 878,495.28 VAT - 1,324,317.0016

1998 1999

LMCEC argued that petitioner is now estopped from further taking any action against it and its corporate officers concerning the taxable years 1997 to 1999. With the grant of immunity from audit from the companys availment of ERAP and VAP, which have a feature of a tax amnesty, the element of fraud is negated the moment the Bureau accepts the offer of compromise or payment of taxes by the taxpayer. The act of the revenue officers in finding justification under Section 6(B) of the NIRC (Best Evidence Obtainable) is misplaced and unavailing because they were not able to open the books of the company for the second time, after the routine examination, issuance of termination letter and the availment of ERAP and VAP. LMCEC thus maintained that unless there is a prior determination of fraud supported by documents not yet incorporated in the docket of the case, petitioner cannot just issue LAs without first terminating those previously issued. It emphasized the fact that the BIR officers who filed and signed the Affidavit-Complaint in this case were the same ones who appeared as complainants in an earlier case filed against Camus for his alleged "failure to obey summons in violation of Section 5 punishable under Section 266 of the NIRC of 1997" (I.S. No. 00-956 of the Office of the City Prosecutor of Quezon City). After preliminary investigation, said case was dismissed for lack of probable cause in a Resolution issued by the Investigating Prosecutor on May 2, 2001.17 LMCEC further asserted that it filed on April 20, 2001 a protest on the PAN issued by petitioner for having no basis in fact and law. However, until now the said protest remains unresolved. As to the alleged informant who purportedly supplied the "confidential information," LMCEC believes that such person is fictitious and his true identity and personality could not be produced. Hence, this case is another form of harassment against the company as what had been found by the Office of the City Prosecutor of Quezon City in I.S. No. 00-956. Said case and the present case both have something to do with the audit/examination of LMCEC for taxable years 1997, 1998 and 1999 pursuant to LA No. 00009361.18 In the Joint Reply-Affidavit executed by the Bureaus revenue officers, petitioner disagreed with the contention of LMCEC that the complaint filed is not criminal in nature, pointing out that LMCEC and its officers Camus and Mendoza were being charged for the criminal offenses defined and penalized under Sections 254 (Attempt to Evade or Defeat Tax) and 255 (Willful Failure to Pay Tax) of the NIRC. This finds support in Section 205 of the same Code which provides for administrative (distraint, levy, fine, forfeiture, lien, etc.) and judicial (criminal or civil action) remedies in order to enforce collection of taxes. Both remedies may be pursued either independently or simultaneously. In this case, the BIR decided to simultaneously pursue

both remedies and thus aside from this criminal action, the Bureau also initiated administrative proceedings against LMCEC.19 On the lack of control number in the assessment notice, petitioner explained that such is a mere office requirement in the Assessment Service for the purpose of internal control and monitoring; hence, the unnumbered assessment notices should not be interpreted as irregular or anomalous. Petitioner stressed that LMCEC already lost its right to file a protest letter after the lapse of the thirty (30)-day reglementary period. LMCECs protest-letter dated December 12, 2002 to RDO Clavelina S. Nacar, RD No. 40, Cubao, Quezon City was actually filed only on December 16, 2002, which was disregarded by the petitioner for being filed out of time. Even assuming for the sake of argument that the assessment notices were invalid, petitioner contended that such could not affect the present criminal action,20 citing the ruling in the landmark case of Ungab v. Cusi, Jr.21 As to the Letter of Termination signed by Ruth Vivian G. Gandia of the Assessment Division, Revenue Region No. 7, Quezon City, petitioner pointed out that LMCEC failed to mention that the undated Certification issued by RDO Pablo C. Cabreros, Jr. of RD No. 40, Cubao, Quezon City stated that the report of the 1997 Internal Revenue taxes of LMCEC had already been submitted for review and approval of higher authorities. LMCEC also cannot claim as excuse from the reopening of its books of accounts the previous investigations and examinations. Under Section 235 (a), an exception was provided in the rule on once a year audit examination in case of "fraud, irregularity or mistakes, as determined by the Commissioner". Petitioner explained that the distinction between a Regular Audit Examination and Tax Fraud Audit Examination lies in the fact that the former is conducted by the district offices of the Bureaus Regional Offices, the authority emanating from the Regional Director, while the latter is conducted by the TFD of the National Office only when instances of fraud had been determined by the petitioner.22 Petitioner further asserted that LMCECs claim that it was granted immunity from audit when it availed of the VAP and ERAP programs is misleading. LMCEC failed to state that its availment of ERAP under RR No. 2-99 is not a grant of absolute immunity from audit and investigation, aside from the fact that said program was only for income tax and did not cover VAT and withholding tax for the taxable year 1998. As for LMCECS availment of VAP in 1999 under RR No. 8-2001 dated August 1, 2001 as amended by RR No. 10-2001 dated September 3, 2001, the company failed to state that it covers only income tax and VAT, and did not include withholding tax. However, LMCEC is not actually entitled to the benefits of VAP under Section 1 (1.1 and 1.2) of RR No. 10-2001. As to the principle of estoppel invoked by LMCEC, estoppel clearly does not lie against the BIR as this involved the exercise of an inherent power by the government to collect taxes.23 Petitioner also pointed out that LMCECs assertion correlating this case with I.S. No. 00-956 is misleading because said case involves another violation and offense (Sections 5 and 266 of the NIRC). Said case was filed by petitioner due to the failure of LMCEC to submit or present its books of accounts and other accounting records for examination despite the issuance of subpoena duces tecum against Camus in his capacity as President of LMCEC. While indeed a Resolution was issued by Asst. City Prosecutor Titus C. Borlas on May 2, 2001 dismissing the complaint, the same is still on appeal and pending resolution by the DOJ. The determination of probable

cause in said case is confined to the issue of whether there was already a violation of the NIRC by Camus in not complying with the subpoena duces tecum issued by the BIR.24 Petitioner contended that precisely the reason for the issuance to the TFD of LA No. 00009361 by the Commissioner is because the latter agreed with the findings of the investigating revenue officers that fraud exists in this case. In the conduct of their investigation, the revenue officers observed the proper procedure under Revenue Memorandum Order (RMO) No. 49-2000 wherein it is required that before the issuance of a Letter of Authority against a particular taxpayer, a preliminary investigation should first be conducted to determine if a prima facie case for tax fraud exists. As to the allegedly unresolved protest filed on April 20, 2001 by LMCEC over the PAN, this has been disregarded by the Bureau for being pro forma and having been filed beyond the 15-day reglementary period. A subsequent letter dated April 20, 2001 was filed with the TFD and signed by a certain Juan Ventigan. However, this was disregarded and considered a mere scrap of paper since the said signatory had not shown any prior authorization to represent LMCEC. Even assuming said protest letter was validly filed on behalf of the company, the issuance of a Formal Demand Letter and Assessment Notice through constructive service on October 1, 2002 is deemed an implied denial of the said protest. Lastly, the details regarding the "informer" being confidential, such information is entitled to some degree of protection, including the identity of the informant against LMCEC.25 In their Joint Rejoinder-Affidavit,26 Camus and Mendoza reiterated their argument that the identity of the alleged informant is crucial to determine if he/she is qualified under Section 282 of the NIRC. Moreover, there was no assessment that has already become final, the validity of its issuance and service has been put in issue being anomalous, irregular and oppressive. It is contended that for criminal prosecution to proceed before assessment, there must be a prima facie showing of a willful attempt to evade taxes. As to LMCECs availment of the VAP and ERAP programs, the certificate of immunity from audit issued to it by the BIR is plain and simple, but petitioner is now saying it has the right to renege with impunity from its undertaking. Though petitioner deems LMCEC not qualified to avail of the benefits of VAP, it must be noted that if it is true that at the time the petitioner filed I.S. No. 00-956 sometime in January 2001 it had already in its custody that "Confidential Information No. 29-2000 dated July 7, 2000", these revenue officers could have rightly filed the instant case and would not resort to filing said criminal complaint for refusal to comply with a subpoena duces tecum. On September 22, 2003, the Chief State Prosecutor issued a Resolution27 finding no sufficient evidence to establish probable cause against respondents LMCEC, Camus and Mendoza. It was held that since the payments were made by LMCEC under ERAP and VAP pursuant to the provisions of RR Nos. 2-99 and 8-2001 which were offered to taxpayers by the BIR itself, the latter is now in estoppel to insist on the criminal prosecution of the respondent taxpayer. The voluntary payments made thereunder are in the nature of a tax amnesty. The unnumbered assessment notices were found highly irregular and thus their validity is suspect; if the amounts indicated therein were collected, it is uncertain how these will be accounted for and if it would go to the coffers of the government or elsewhere. On the required prior determination of fraud, the Chief State Prosecutor declared that the Office of the City Prosecutor in I.S. No. 00-956 has already squarely ruled that (1) there was no prior determination of fraud, (2) there was indiscriminate issuance of LAs, and (3) the complaint was more of harassment. In view of such

findings, any ensuing LA is thus defective and allowing the collection on the assailed assessment notices would already be in the context of a "fishing expedition" or "witch-hunting." Consequently, there is nothing to speak of regarding the finality of assessment notices in the aggregate amount of P630,164,631.61. Petitioner filed a motion for reconsideration which was denied by the Chief State Prosecutor.28 Petitioner appealed to respondent Secretary of Justice but the latter denied its petition for review under Resolution dated December 13, 2005.29 The Secretary of Justice found that petitioners claim that there is yet no finality as to LMCECs payment of its 1997 taxes since the audit report was still pending review by higher authorities, is unsubstantiated and misplaced. It was noted that the Termination Letter issued by the Commissioner on June 1, 1999 is explicit that the matter is considered closed. As for taxable year 1998, respondent Secretary stated that the record shows that LMCEC paid VAT and withholding tax in the amount of P61,635.40 and P38,404.55, respectively. This eventually gave rise to the issuance of a certificate of immunity from audit for 1998 by the Office of the Commissioner of Internal Revenue. For taxable year 1999, respondent Secretary found that pursuant to earlier LA No. 38633 dated July 4, 2000, LMCECs 1999 tax liabilities were still pending investigation for which reason LMCEC assailed the subsequent issuance of LA No. 00009361 dated August 25, 2000 calling for a similar investigation of its alleged 1999 tax deficiencies when no final determination has yet been arrived on the earlier LA No. 38633.30 On the allegation of fraud, respondent Secretary ruled that petitioner failed to establish the existence of the following circumstances indicating fraud in the settlement of LMCECs tax liabilities: (1) there must be intentional and substantial understatement of tax liability by the taxpayer; (2) there must be intentional and substantial overstatement of deductions or exemptions; and (3) recurrence of the foregoing circumstances. First, petitioner miserably failed to explain why the assessment notices were unnumbered; second, the claim that the tax fraud investigation was precipitated by an alleged "informant" has not been corroborated nor was it clearly established, hence there is no other conclusion but that the Bureau engaged in a "fishing expedition"; and furthermore, petitioners course of action is contrary to Section 235 of the NIRC allowing only once in a given taxable year such examination and inspection of the taxpayers books of accounts and other accounting records. There was no convincing proof presented by petitioner to show that the case of LMCEC falls under the exceptions provided in Section 235. Respondent Secretary duly considered the issuance of Certificate of Immunity from Audit and Letter of Termination dated June 1, 1999 issued to LMCEC.31 Anent the earlier case filed against the same taxpayer (I.S. No. 00-956), the Secretary of Justice found petitioner to have engaged in forum shopping in view of the fact that while there is still pending an appeal from the Resolution of the City Prosecutor of Quezon City in said case, petitioner hurriedly filed the instant case, which not only involved the same parties but also similar substantial issues (the joint complaint-affidavit also alleged the issuance of LA No. 00009361 dated August 25, 2000). Clearly, the evidence of litis pendentia is present. Finally, respondent Secretary noted that if indeed LMCEC committed fraud in the settlement of its tax liabilities, then at the outset, it should have been discovered by the agents of petitioner, and

consequently petitioner should not have issued the Letter of Termination and the Certificate of Immunity From Audit. Petitioner thus should have been more circumspect in the issuance of said documents.32 Its motion for reconsideration having been denied, petitioner challenged the ruling of respondent Secretary via a certiorari petition in the CA. On October 31, 2006, the CA rendered the assailed decision33 denying the petition and concurred with the findings and conclusions of respondent Secretary. Petitioners motion for reconsideration was likewise denied by the appellate court.34 It appears that entry of judgment was issued by the CA stating that its October 31, 2006 Decision attained finality on March 25, 2007.35 However, the said entry of judgment was set aside upon manifestation by the petitioner that it has filed a petition for review before this Court subsequent to its receipt of the Resolution dated March 6, 2007 denying petitioners motion for reconsideration on March 20, 2007.36 The petition is anchored on the following grounds: I. The Honorable Court of Appeals erroneously sustained the findings of the Secretary of Justice who gravely abused his discretion by dismissing the complaint based on grounds which are not even elements of the offenses charged. II. The Honorable Court of Appeals erroneously sustained the findings of the Secretary of Justice who gravely abused his discretion by dismissing petitioners evidence, contrary to law. III. The Honorable Court of Appeals erroneously sustained the findings of the Secretary of Justice who gravely abused his discretion by inquiring into the validity of a Final Assessment Notice which has become final, executory and demandable pursuant to Section 228 of the Tax Code of 1997 for failure of private respondent to file a protest against the same.37 The core issue to be resolved is whether LMCEC and its corporate officers may be prosecuted for violation of Sections 254 (Attempt to Evade or Defeat Tax) and 255 (Willful Failure to Supply Correct and Accurate Information and Pay Tax). Petitioner filed the criminal complaint against the private respondents for violation of the following provisions of the NIRC, as amended: SEC. 254. Attempt to Evade or Defeat Tax. Any person who willfully attempts in any manner to evade or defeat any tax imposed under this Code or the payment thereof shall, in addition to other penalties provided by law, upon conviction thereof, be punished by a fine of not less than Thirty thousand pesos (P30,000) but not more than One hundred thousand pesos (P100,000) and

suffer imprisonment of not less than two (2) years but not more than four (4) years: Provided, That the conviction or acquittal obtained under this Section shall not be a bar to the filing of a civil suit for the collection of taxes. SEC. 255. Failure to File Return, Supply Correct and Accurate Information, Pay Tax, Withhold and Remit Tax and Refund Excess Taxes Withheld on Compensation. Any person required under this Code or by rules and regulations promulgated thereunder to pay any tax, make a return, keep any record, or supply any correct and accurate information, who willfully fails to pay such tax, make such return, keep such record, or supply such correct and accurate information, or withhold or remit taxes withheld, or refund excess taxes withheld on compensations at the time or times required by law or rules and regulations shall, in addition to other penalties provided by law, upon conviction thereof, be punished by a fine of not less than Ten thousand pesos (P10,000) and suffer imprisonment of not less than one (1) year but not more than ten (10) years. x x x x (Emphasis supplied.) Respondent Secretary concurred with the Chief State Prosecutors conclusion that there is insufficient evidence to establish probable cause to charge private respondents under the above provisions, based on the following findings: (1) the tax deficiencies of LMCEC for taxable years 1997, 1998 and 1999 have all been settled or terminated, as in fact LMCEC was issued a Certificate of Immunity and Letter of Termination, and availed of the ERAP and VAP programs; (2) there was no prior determination of the existence of fraud; (3) the assessment notices are unnumbered, hence irregular and suspect; (4) the books of accounts and other accounting records may be subject to audit examination only once in a given taxable year and there is no proof that the case falls under the exceptions provided in Section 235 of the NIRC; and (5) petitioner committed forum shopping when it filed the instant case even as the earlier criminal complaint (I.S. No. 00-956) dismissed by the City Prosecutor of Quezon City was still pending appeal. Petitioner argues that with the finality of the assessment due to failure of the private respondents to challenge the same in accordance with Section 228 of the NIRC, respondent Secretary has no jurisdiction and authority to inquire into its validity. Respondent taxpayer is thereby allowed to do indirectly what it cannot do directly to raise a collateral attack on the assessment when even a direct challenge of the same is legally barred. The rationale for dismissing the complaint on the ground of lack of control number in the assessment notice likewise betrays a lack of awareness of tax laws and jurisprudence, such circumstance not being an element of the offense. Worse, the final, conclusive and undisputable evidence detailing a crime under our taxation laws is swept under the rug so easily on mere conspiracy theories imputed on persons who are not even the subject of the complaint. We grant the petition. There is no dispute that prior to the filing of the complaint with the DOJ, the report on the tax fraud investigation conducted on LMCEC disclosed that it made substantial underdeclarations in its income tax returns for 1997, 1998 and 1999. Pursuant to RR No. 12-99,38 a PAN was sent to and received by LMCEC on February 22, 2001 wherein it was notified of the proposed

assessment of deficiency taxes amounting to P430,958,005.90 (income tax - P318,606,380.19 and VAT - P112,351,625.71) covering taxable years 1997, 1998 and 1999.39 In response to said PAN, LMCEC sent a letter-protest to the TFD, which denied the same on April 12, 2001 for lack of legal and factual basis and also for having been filed beyond the 15-day reglementary period.40 As mentioned in the PAN, the revenue officers were not given the opportunity to examine LMCECs books of accounts and other accounting records because its officers failed to comply with the subpoena duces tecum earlier issued, to verify its alleged underdeclarations of income reported by the Bureaus informant under Section 282 of the NIRC. Hence, a criminal complaint was filed by the Bureau against private respondents for violation of Section 266 which provides: SEC. 266. Failure to Obey Summons. Any person who, being duly summoned to appear to testify, or to appear and produce books of accounts, records, memoranda, or other papers, or to furnish information as required under the pertinent provisions of this Code, neglects to appear or to produce such books of accounts, records, memoranda, or other papers, or to furnish such information, shall, upon conviction, be punished by a fine of not less than Five thousand pesos (P5,000) but not more than Ten thousand pesos (P10,000) and suffer imprisonment of not less than one (1) year but not more than two (2) years. It is clear that I.S. No. 00-956 involves a separate offense and hence litis pendentia is not present considering that the outcome of I.S. No. 00-956 is not determinative of the issue as to whether probable cause exists to charge the private respondents with the crimes of attempt to evade or defeat tax and willful failure to supply correct and accurate information and pay tax defined and penalized under Sections 254 and 255, respectively. For the crime of tax evasion in particular, compliance by the taxpayer with such subpoena, if any had been issued, is irrelevant. As we held in Ungab v. Cusi, Jr.,41 "[t]he crime is complete when the [taxpayer] has x x x knowingly and willfully filed [a] fraudulent [return] with intent to evade and defeat x x x the tax." Thus, respondent Secretary erred in holding that petitioner committed forum shopping when it filed the present criminal complaint during the pendency of its appeal from the City Prosecutors dismissal of I.S. No. 00-956 involving the act of disobedience to the summons in the course of the preliminary investigation on LMCECs correct tax liabilities for taxable years 1997, 1998 and 1999. In the Details of Discrepancies attached as Annex B of the PAN,42 private respondents were already notified that inasmuch as the revenue officers were not given the opportunity to examine LMCECs books of accounts, accounting records and other documents, said revenue officers gathered information from third parties. Such procedure is authorized under Section 5 of the NIRC, which provides: SEC. 5. Power of the Commissioner to Obtain Information, and to Summon, Examine, and Take Testimony of Persons. In ascertaining the correctness of any return, or in making a return when none has been made, or in determining the liability of any person for any internal revenue tax, or in collecting any such liability, or in evaluating tax compliance, the Commissioner is authorized:

(A) To examine any book, paper, record or other data which may be relevant or material to such inquiry; (B) To obtain on a regular basis from any person other than the person whose internal revenue tax liability is subject to audit or investigation, or from any office or officer of the national and local governments, government agencies and instrumentalities, including the Bangko Sentral ng Pilipinas and government-owned or -controlled corporations, any information such as, but not limited to, costs and volume of production, receipts or sales and gross incomes of taxpayers, and the names, addresses, and financial statements of corporations, mutual fund companies, insurance companies, regional operating headquarters of multinational companies, joint accounts, associations, joint ventures or consortia and registered partnerships, and their members; (C) To summon the person liable for tax or required to file a return, or any officer or employee of such person, or any person having possession, custody, or care of the books of accounts and other accounting records containing entries relating to the business of the person liable for tax, or any other person, to appear before the Commissioner or his duly authorized representative at a time and place specified in the summons and to produce such books, papers, records, or other data, and to give testimony; (D) To take such testimony of the person concerned, under oath, as may be relevant or material to such inquiry; x x x x x x x (Emphasis supplied.) Private respondents assertions regarding the qualifications of the "informer" of the Bureau deserve scant consideration. We have held that the lack of consent of the taxpayer under investigation does not imply that the BIR obtained the information from third parties illegally or that the information received is false or malicious. Nor does the lack of consent preclude the BIR from assessing deficiency taxes on the taxpayer based on the documents.43 In the same vein, herein private respondents cannot be allowed to escape criminal prosecution under Sections 254 and 255 of the NIRC by mere imputation of a "fictitious" or disqualified informant under Section 282 simply because other than disclosure of the official registry number of the third party "informer," the Bureau insisted on maintaining the confidentiality of the identity and personal circumstances of said "informer." Subsequently, petitioner sent to LMCEC by constructive service allowed under Section 3 of RR No. 12-99, assessment notice and formal demand informing the said taxpayer of the law and the facts on which the assessment is made, as required by Section 228 of the NIRC. Respondent Secretary, however, fully concurred with private respondents contention that the assessment notices were invalid for being unnumbered and the tax liabilities therein stated have already been settled and/or terminated. We do not agree. A notice of assessment is:

[A] declaration of deficiency taxes issued to a [t]axpayer who fails to respond to a PreAssessment Notice (PAN) within the prescribed period of time, or whose reply to the PAN was found to be without merit. The Notice of Assessment shall inform the [t]axpayer of this fact, and that the report of investigation submitted by the Revenue Officer conducting the audit shall be given due course. The formal letter of demand calling for payment of the taxpayers deficiency tax or taxes shall state the fact, the law, rules and regulations or jurisprudence on which the assessment is based, otherwise the formal letter of demand and the notice of assessment shall be void .44 As it is, the formality of a control number in the assessment notice is not a requirement for its validity but rather the contents thereof which should inform the taxpayer of the declaration of deficiency tax against said taxpayer. Both the formal letter of demand and the notice of assessment shall be void if the former failed to state the fact, the law, rules and regulations or jurisprudence on which the assessment is based, which is a mandatory requirement under Section 228 of the NIRC. Section 228 of the NIRC provides that the taxpayer shall be informed in writing of the law and the facts on which the assessment is made. Otherwise, the assessment is void. To implement the provisions of Section 228 of the NIRC, RR No. 12-99 was enacted. Section 3.1.4 of the revenue regulation reads: 3.1.4. Formal Letter of Demand and Assessment Notice. The formal letter of demand and assessment notice shall be issued by the Commissioner or his duly authorized representative. The letter of demand calling for payment of the taxpayers deficiency tax or taxes shall state the facts, the law, rules and regulations, or jurisprudence on which the assessment is based, otherwise, the formal letter of demand and assessment notice shall be void. The same shall be sent to the taxpayer only by registered mail or by personal delivery. x x x.45 (Emphasis supplied.) The Formal Letter of Demand dated August 7, 2002 contains not only a detailed computation of LMCECs tax deficiencies but also details of the specified discrepancies, explaining the legal and factual bases of the assessment. It also reiterated that in the absence of accounting records and other documents necessary for the proper determination of the companys internal revenue tax liabilities, the investigating revenue officers resorted to the "Best Evidence Obtainable" as provided in Section 6(B) of the NIRC (third party information) and in accordance with the procedure laid down in RMC No. 23-2000 dated November 27, 2000. Annex "A" of the Formal Letter of Demand thus stated: Thus, to verify the validity of the information previously provided by the informant, the assigned revenue officers resorted to third party information. Pursuant to Section 5(B) of the NIRC of 1997, access letters requesting for information and the submission of certain documents (i.e., Certificate of Income Tax Withheld at Source and/or Alphabetical List showing the income payments made to L.M. Camus Engineering Corporation for the taxable years 1997 to 1999) were sent to the various clients of the subject corporation, including but not limited to the following:

1. Ayala Land Inc. 2. Filinvest Alabang Inc. 3. D.M. Consunji, Inc. 4. SM Prime Holdings, Inc. 5. Alabang Commercial Corporation 6. Philam Properties Corporation 7. SM Investments, Inc. 8. Shoemart, Inc. 9. Philippine Securities Corporation 10. Makati Development Corporation From the documents gathered and the data obtained therein, the substantial underdeclaration as defined under Section 248(B) of the NIRC of 1997 by your corporation of its income had been confirmed. x x x x46 (Emphasis supplied.) In the same letter, Assistant Commissioner Percival T. Salazar informed private respondents that the estimated tax liabilities arising from LMCECs underdeclaration amounted to P186,773,600.84 in 1997, P150,069,323.81 in 1998 and P163,220,111.13 in 1999. These figures confirmed that the non-declaration by LMCEC for the taxable years 1997, 1998 and 1999 of an amount exceeding 30% income47 declared in its return is considered a substantial underdeclaration of income, which constituted prima facie evidence of false or fraudulent return under Section 248(B)48 of the NIRC, as amended.49 On the alleged settlement of the assessed tax deficiencies by private respondents, respondent Secretary found the latters claim as meritorious on the basis of the Certificate of Immunity From Audit issued on December 6, 1999 pursuant to RR No. 2-99 and Letter of Termination dated June 1, 1999 issued by Revenue Region No. 7 Chief of Assessment Division Ruth Vivian G. Gandia. Petitioner, however, clarified that the certificate of immunity from audit covered only income tax for the year 1997 and does not include VAT and withholding taxes, while the Letter of Termination involved tax liabilities for taxable year 1997 (EWT, VAT and income taxes) but which was submitted for review of higher authorities as per the Certification of RD No. 40 District Officer Pablo C. Cabreros, Jr.50 For 1999, private respondents supposedly availed of the VAP pursuant to RR No. 8-2001. RR No. 2-99 issued on February 7, 1999 explained in its Policy Statement that considering the scarcity of financial and human resources as well as the time constraints within which the Bureau has to "clean the Bureaus backlog of unaudited tax returns in order to keep updated and be

focused with the most current accounts" in preparation for the full implementation of a computerized tax administration, the said revenue regulation was issued "providing for last priority in audit and investigation of tax returns" to accomplish the said objective "without, however, compromising the revenue collection that would have been generated from audit and enforcement activities." The program named as "Economic Recovery Assistance Payment (ERAP) Program" granted immunity from audit and investigation of income tax, VAT and percentage tax returns for 1998. It expressly excluded withholding tax returns (whether for income, VAT, or percentage tax purposes). Since such immunity from audit and investigation does not preclude the collection of revenues generated from audit and enforcement activities, it follows that the Bureau is likewise not barred from collecting any tax deficiency discovered as a result of tax fraud investigations. Respondent Secretarys opinion that RR No. 2-99 contains the feature of a tax amnesty is thus misplaced. Tax amnesty is a general pardon to taxpayers who want to start a clean tax slate. It also gives the government a chance to collect uncollected tax from tax evaders without having to go through the tedious process of a tax case.51 Even assuming arguendo that the issuance of RR No. 2-99 is in the nature of tax amnesty, it bears noting that a tax amnesty, much like a tax exemption, is never favored nor presumed in law and if granted by statute, the terms of the amnesty like that of a tax exemption must be construed strictly against the taxpayer and liberally in favor of the taxing authority.52 For the same reason, the availment by LMCEC of VAP under RR No. 8-2001 as amended by RR No. 10-2001, through payment supposedly made in October 29, 2001 before the said program ended on October 31, 2001, did not amount to settlement of its assessed tax deficiencies for the period 1997 to 1999, nor immunity from prosecution for filing fraudulent return and attempt to evade or defeat tax. As correctly asserted by petitioner, from the express terms of the aforesaid revenue regulations, LMCEC is not qualified to avail of the VAP granting taxpayers the privilege of last priority in the audit and investigation of all internal revenue taxes for the taxable year 2000 and all prior years under certain conditions, considering that first, it was issued a PAN on February 19, 2001, and second, it was the subject of investigation as a result of verified information filed by a Tax Informer under Section 282 of the NIRC duly recorded in the BIR Official Registry as Confidential Information (CI) No. 29-200053 even prior to the issuance of the PAN. Section 1 of RR No. 8-2001 provides: SECTION 1. COVERAGE. x x x Any person, natural or juridical, including estates and trusts, liable to pay any of the above-cited internal revenue taxes for the above specified period/s who, due to inadvertence or otherwise, erroneously paid his internal revenue tax liabilities or failed to file tax return/pay taxes may avail of the Voluntary Assessment Program (VAP), except those falling under any of the following instances: 1.1 Those covered by a Preliminary Assessment Notice (PAN), Final Assessment Notice (FAN), or Collection Letter issued on or before July 31, 2001; or

1.2 Persons under investigation as a result of verified information filed by a Tax Informer under Section 282 of the Tax Code of 1997, duly processed and recorded in the BIR Official Registry Book on or before July 31, 2001; 1.3 Tax fraud cases already filed and pending in courts for adjudication; and x x x x (Emphasis supplied.) Moreover, private respondents cannot invoke LMCECs availment of VAP to foreclose any subsequent audit of its account books and other accounting records in view of the strong finding of underdeclaration in LMCECs payment of correct income tax liability by more than 30% as supported by the written report of the TFD detailing the facts and the law on which such finding is based, pursuant to the tax fraud investigation authorized by petitioner under LA No. 00009361. This conclusion finds support in Section 2 of RR No. 8-2001 as amended by RR No. 10-2001 provides: SEC. 2. TAXPAYERS BENEFIT FROM AVAILMENT OF THE VAP. A taxpayer who has availed of the VAP shall not be audited except upon authorization and approval of the Commissioner of Internal Revenue when there is strong evidence or finding of understatement in the payment of taxpayers correct tax liability by more than thirty percent (30%) as supported by a written report of the appropriate office detailing the facts and the law on which such finding is based: Provided, however, that any VAP payment should be allowed as tax credit against the deficiency tax due, if any, in case the concerned taxpayer has been subjected to tax audit. xxxx Given the explicit conditions for the grant of immunity from audit under RR No. 2-99, RR No. 82001 and RR No. 10-2001, we hold that respondent Secretary gravely erred in declaring that petitioner is now estopped from assessing any tax deficiency against LMCEC after issuance of the aforementioned documents of immunity from audit/investigation and settlement of tax liabilities. It is axiomatic that the State can never be in estoppel, and this is particularly true in matters involving taxation. The errors of certain administrative officers should never be allowed to jeopardize the governments financial position.54 Respondent Secretarys other ground for assailing the course of action taken by petitioner in proceeding with the audit and investigation of LMCEC -- the alleged violation of the general rule in Section 235 of the NIRC allowing the examination and inspection of taxpayers books of accounts and other accounting records only once in a taxable year -- is likewise untenable. As correctly pointed out by petitioner, the discovery of substantial underdeclarations of income by LMCEC for taxable years 1997, 1998 and 1999 upon verified information provided by an "informer" under Section 282 of the NIRC, as well as the necessity of obtaining information from third parties to ascertain the correctness of the return filed or evaluation of tax compliance in collecting taxes (as a result of the disobedience to the summons issued by the Bureau against the private respondents), are circumstances warranting exception from the general rule in Section 235.55

As already stated, the substantial underdeclared income in the returns filed by LMCEC for 1997, 1998 and 1999 in amounts equivalent to more than 30% (the computation in the final assessment notice showed underdeclarations of almost 200%) constitutes prima facie evidence of fraudulent return under Section 248(B) of the NIRC. Prior to the issuance of the preliminary and final notices of assessment, the revenue officers conducted a preliminary investigation on the information and documents showing substantial understatement of LMCECs tax liabilities which were provided by the Informer, following the procedure under RMO No. 15-95.56 Based on the prima facie finding of the existence of fraud, petitioner issued LA No. 00009361 for the TFD to conduct a formal fraud investigation of LMCEC.57 Consequently, respondent Secretarys ruling that the filing of criminal complaint for violation of Sections 254 and 255 of the NIRC cannot prosper because of lack of prior determination of the existence of fraud, is bereft of factual basis and contradicted by the evidence on record. Tax assessments by tax examiners are presumed correct and made in good faith, and all presumptions are in favor of the correctness of a tax assessment unless proven otherwise.58 We have held that a taxpayers failure to file a petition for review with the Court of Tax Appeals within the statutory period rendered the disputed assessment final, executory and demandable, thereby precluding it from interposing the defenses of legality or validity of the assessment and prescription of the Governments right to assess.59 Indeed, any objection against the assessment should have been pursued following the avenue paved in Section 229 (now Section 228) of the NIRC on protests on assessments of internal revenue taxes.60 Records bear out that the assessment notice and Formal Letter of Demand dated August 7, 2002 were duly served on LMCEC on October 1, 2002. Private respondents did not file a motion for reconsideration of the said assessment notice and formal demand; neither did they appeal to the Court of Tax Appeals. Section 228 of the NIRC61 provides the remedy to dispute a tax assessment within a certain period of time. It states that an assessment may be protested by filing a request for reconsideration or reinvestigation within 30 days from receipt of the assessment by the taxpayer. No such administrative protest was filed by private respondents seeking reconsideration of the August 7, 2002 assessment notice and formal letter of demand. Private respondents cannot belatedly assail the said assessment, which they allowed to lapse into finality, by raising issues as to its validity and correctness during the preliminary investigation after the BIR has referred the matter for prosecution under Sections 254 and 255 of the NIRC. As we held in Marcos II v. Court of Appeals62: It is not the Department of Justice which is the government agency tasked to determine the amount of taxes due upon the subject estate, but the Bureau of Internal Revenue, whose determinations and assessments are presumed correct and made in good faith. The taxpayer has the duty of proving otherwise. In the absence of proof of any irregularities in the performance of official duties, an assessment will not be disturbed. Even an assessment based on estimates is prima facie valid and lawful where it does not appear to have been arrived at arbitrarily or capriciously. The burden of proof is upon the complaining party to show clearly that the assessment is erroneous. Failure to present proof of error in the assessment will justify the judicial affirmance of said assessment. x x x.

Moreover, these objections to the assessments should have been raised, considering the ample remedies afforded the taxpayer by the Tax Code, with the Bureau of Internal Revenue and the Court of Tax Appeals, as described earlier, and cannot be raised now via Petition for Certiorari, under the pretext of grave abuse of discretion. The course of action taken by the petitioner reflects his disregard or even repugnance of the established institutions for governance in the scheme of a well-ordered society. The subject tax assessments having become final, executory and enforceable, the same can no longer be contested by means of a disguised protest. In the main, Certiorari may not be used as a substitute for a lost appeal or remedy. This judicial policy becomes more pronounced in view of the absence of sufficient attack against the actuations of government. (Emphasis supplied.) The determination of probable cause is part of the discretion granted to the investigating prosecutor and ultimately, the Secretary of Justice. However, this Court and the CA possess the power to review findings of prosecutors in preliminary investigations. Although policy considerations call for the widest latitude of deference to the prosecutors findings, courts should never shirk from exercising their power, when the circumstances warrant, to determine whether the prosecutors findings are supported by the facts, or by the law. In so doing, courts do not act as prosecutors but as organs of the judiciary, exercising their mandate under the Constitution, relevant statutes, and remedial rules to settle cases and controversies.63 Clearly, the power of the Secretary of Justice to review does not preclude this Court and the CA from intervening and exercising our own powers of review with respect to the DOJs findings, such as in the exceptional case in which grave abuse of discretion is committed, as when a clear sufficiency or insufficiency of evidence to support a finding of probable cause is ignored.64 WHEREFORE, the petition is GRANTED. The Decision dated October 31, 2006 and Resolution dated March 6, 2007 of the Court of Appeals in CA-G.R. SP No. 93387 are hereby REVERSED and SET ASIDE. The Secretary of Justice is hereby DIRECTED to order the Chief State Prosecutor to file before the Regional Trial Court of Quezon City, National Capital Judicial Region, the corresponding Information against L. M. Camus Engineering Corporation, represented by its President Luis M. Camus and Comptroller Lino D. Mendoza, for Violation of Sections 254 and 255 of the National Internal Revenue Code of 1997. No costs.

G.R. No. 159694

January 27, 2006

COMMISSIONER OF INTERNAL REVENUE, Petitioner, vs. AZUCENA T. REYES, Respondent. x -- -- -- -- -- -- -- -- -- -- -- -- -- x G.R. No. 163581 January 27, 2006

AZUCENA T. REYES, Petitioner, vs. COMMISSIONER OF INTERNAL REVENUE, Respondent. DECISION PANGANIBAN, CJ.: Under the present provisions of the Tax Code and pursuant to elementary due process, taxpayers must be informed in writing of the law and the facts upon which a tax assessment is based; otherwise, the assessment is void. Being invalid, the assessment cannot in turn be used as a basis for the perfection of a tax compromise. The Case Before us are two consolidated1 Petitions for Review2 filed under Rule 45 of the Rules of Court, assailing the August 8, 2003 Decision3 of the Court of Appeals (CA) in CA-GR SP No. 71392. The dispositive portion of the assailed Decision reads as follows: "WHEREFORE, the petition is GRANTED. The assailed decision of the Court of Tax Appeals is ANNULLED and SET ASIDE without prejudice to the action of the National Evaluation Board on the proposed compromise settlement of the Maria C. Tancinco estates tax liability."4 The Facts The CA narrated the facts as follows: "On July 8, 1993, Maria C. Tancinco (or decedent) died, leaving a 1,292 square-meter residential lot and an old house thereon (or subject property) located at 4931 Pasay Road, Dasmarias Village, Makati City. "On the basis of a sworn information-for-reward filed on February 17, 1997 by a certain Raymond Abad (or Abad), Revenue District Office No. 50 (South Makati) conducted an investigation on the decedents estate (or estate). Subsequently, it issued a Return Verification Order. But without the required preliminary findings being submitted, it issued Letter of Authority No. 132963 for the regular investigation of the estate tax case. Azucena T. Reyes (or [Reyes]), one of the decedents heirs, received the Letter of Authority on March 14, 1997. "On February 12, 1998, the Chief, Assessment Division, Bureau of Internal Revenue (or BIR), issued a preliminary assessment notice against the estate in the amount of P14,580,618.67. On May 10, 1998, the heirs of the decedent (or heirs) received a final estate tax assessment notice and a demand letter, both dated April 22, 1998, for the amount of P14,912,205.47, inclusive of surcharge and interest.

"On June 1, 1998, a certain Felix M. Sumbillo (or Sumbillo) protested the assessment [o]n behalf of the heirs on the ground that the subject property had already been sold by the decedent sometime in 1990. "On November 12, 1998, the Commissioner of Internal Revenue (or [CIR]) issued a preliminary collection letter to [Reyes], followed by a Final Notice Before Seizure dated December 4, 1998. "On January 5, 1999, a Warrant of Distraint and/or Levy was served upon the estate, followed on February 11, 1999 by Notices of Levy on Real Property and Tax Lien against it. "On March 2, 1999, [Reyes] protested the notice of levy. However, on March 11, 1999, the heirs proposed a compromise settlement of P1,000,000.00. "In a letter to [the CIR] dated January 27, 2000, [Reyes] proposed to pay 50% of the basic tax due, citing the heirs inability to pay the tax assessment. On March 20, 2000, [the CIR] rejected [Reyess] offer, pointing out that since the estate tax is a charge on the estate and not on the heirs, the latters financial incapacity is immaterial as, in fact, the gross value of the estate amounting to P32,420,360.00 is more than sufficient to settle the tax liability. Thus, [the CIR] demanded payment of the amount of P18,034,382.13 on or before April 15, 2000[;] otherwise, the notice of sale of the subject property would be published. "On April 11, 2000, [Reyes] again wrote to [the CIR], this time proposing to pay 100% of the basic tax due in the amount of P5,313,891.00. She reiterated the proposal in a letter dated May 18, 2000. "As the estate failed to pay its tax liability within the April 15, 2000 deadline, the Chief, Collection Enforcement Division, BIR, notified [Reyes] on June 6, 2000 that the subject property would be sold at public auction on August 8, 2000. "On June 13, 2000, [Reyes] filed a protest with the BIR Appellate Division. Assailing the scheduled auction sale, she asserted that x x x the assessment, letter of demand[,] and the whole tax proceedings against the estate are void ab initio. She offered to file the corresponding estate tax return and pay the correct amount of tax without surcharge [or] interest. "Without acting on [Reyess] protest and offer, [the CIR] instructed the Collection Enforcement Division to proceed with the August 8, 2000 auction sale. Consequently, on June 28, 2000, [Reyes] filed a [P]etition for [R]eview with the Court of Tax Appeals (or CTA), docketed as CTA Case No. 6124. "On July 17, 2000, [Reyes] filed a Motion for the Issuance of a Writ of Preliminary Injunction or Status Quo Order, which was granted by the CTA on July 26, 2000. Upon [Reyess] filing of a surety bond in the amount of P27,000,000.00, the CTA issued a [R]esolution dated August 16, 2000 ordering [the CIR] to desist and refrain from proceeding with the auction sale of the subject property or from issuing a [W]arrant of [D]istraint or [G]arnishment of [B]ank [A]ccount[,] pending determination of the case and/or unless a contrary order is issued.

"[The CIR] filed a [M]otion to [D]ismiss the petition on the grounds (i) that the CTA no longer has jurisdiction over the case[,] because the assessment against the estate is already final and executory; and (ii) that the petition was filed out of time. In a [R]esolution dated November 23, 2000, the CTA denied [the CIRs] motion. "During the pendency of the [P]etition for [R]eview with the CTA, however, the BIR issued Revenue Regulation (or RR) No. 6-2000 and Revenue Memorandum Order (or RMO) No. 42-2000 offering certain taxpayers with delinquent accounts and disputed assessments an opportunity to compromise their tax liability. "On November 25, 2000, [Reyes] filed an application with the BIR for the compromise settlement (or compromise) of the assessment against the estate pursuant to Sec. 204(A) of the Tax Code, as implemented by RR No. 6-2000 and RMO No. 42-2000. "On December 26, 2000, [Reyes] filed an Ex-Parte Motion for Postponement of the hearing before the CTA scheduled on January 9, 2001, citing her pending application for compromise with the BIR. The motion was granted and the hearing was reset to February 6, 2001. "On January 29, 2001, [Reyes] moved for postponement of the hearing set on February 6, 2001, this time on the ground that she had already paid the compromise amount of P1,062,778.20 but was still awaiting approval of the National Evaluation Board (or NEB). The CTA granted the motion and reset the hearing to February 27, 2001. "On February 19, 2001, [Reyes] filed a Motion to Declare Application for the Settlement of Disputed Assessment as a Perfected Compromise. In said motion, she alleged that [the CIR] had not yet signed the compromise[,] because of procedural red tape requiring the initials of four Deputy Commissioners on relevant documents before the compromise is signed by the [CIR]. [Reyes] posited that the absence of the requisite initials and signature[s] on said documents does not vitiate the perfected compromise. "Commenting on the motion, [the CIR] countered that[,] without the approval of the NEB, [Reyess] application for compromise with the BIR cannot be considered a perfected or consummated compromise. "On March 9, 2001, the CTA denied [Reyess] motion, prompting her to file a Motion for Reconsideration Ad Cautelam. In a [R]esolution dated April 10, 2001, the CTA denied the [M]otion for [R]econsideration with the suggestion that[,] for an orderly presentation of her case and to prevent piecemeal resolutions of different issues, [Reyes] should file a [S]upplemental [P]etition for [R]eview[,] setting forth the new issue of whether there was already a perfected compromise. "On May 2, 2001, [Reyes] filed a Supplemental Petition for Review with the CTA, followed on June 4, 2001 by its Amplificatory Arguments (for the Supplemental Petition for Review), raising the following issues:

1. Whether or not an offer to compromise by the [CIR], with the acquiescence by the Secretary of Finance, of a tax liability pending in court, that was accepted and paid by the taxpayer, is a perfected and consummated compromise. 2. Whether this compromise is covered by the provisions of Section 204 of the Tax Code (CTRP) that requires approval by the BIR [NEB]. "Answering the Supplemental Petition, [the CIR] averred that an application for compromise of a tax liability under RR No. 6-2000 and RMO No. 42-2000 requires the evaluation and approval of either the NEB or the Regional Evaluation Board (or REB), as the case may be. "On June 14, 2001, [Reyes] filed a Motion for Judgment on the Pleadings; the motion was granted on July 11, 2001. After submission of memoranda, the case was submitted for [D]ecision. "On June 19, 2002, the CTA rendered a [D]ecision, the decretal portion of which pertinently reads: WHEREFORE, in view of all the foregoing, the instant [P]etition for [R]eview is hereby DENIED. Accordingly, [Reyes] is hereby ORDERED to PAY deficiency estate tax in the amount of Nineteen Million Five Hundred Twenty Four Thousand Nine Hundred Nine and 78/100 (P19,524,909.78), computed as follows: xxxxxxxxx [Reyes] is likewise ORDERED to PAY 20% delinquency interest on deficiency estate tax due of P17,934,382.13 from January 11, 2001 until full payment thereof pursuant to Section 249(c) of the Tax Code, as amended. "In arriving at its decision, the CTA ratiocinated that there can only be a perfected and consummated compromise of the estates tax liability[,] if the NEB has approved [Reyess] application for compromise in accordance with RR No. 6-2000, as implemented by RMO No. 42-2000. "Anent the validity of the assessment notice and letter of demand against the estate, the CTA stated that at the time the questioned assessment notice and letter of demand were issued, the heirs knew very well the law and the facts on which the same were based. It also observed that the petition was not filed within the 30-day reglementary period provided under Sec. 11 of Rep. Act No. 1125 and Sec. 228 of the Tax Code."5 Ruling of the Court of Appeals In partly granting the Petition, the CA said that Section 228 of the Tax Code and RR 12-99 were mandatory and unequivocal in their requirement. The assessment notice and the demand letter should have stated the facts and the law on which they were based; otherwise, they were deemed void.6 The appellate court held that while administrative agencies, like the BIR, were not bound

by procedural requirements, they were still required by law and equity to observe substantive due process. The reason behind this requirement, said the CA, was to ensure that taxpayers would be duly apprised of -- and could effectively protest -- the basis of tax assessments against them.7 Since the assessment and the demand were void, the proceedings emanating from them were likewise void, and any order emanating from them could never attain finality. The appellate court added, however, that it was premature to declare as perfected and consummated the compromise of the estates tax liability. It explained that, where the basic tax assessed exceeded P1 million, or where the settlement offer was less than the prescribed minimum rates, the National Evaluation Boards (NEB) prior evaluation and approval were the conditio sine qua non to the perfection and consummation of any compromise.8 Besides, the CA pointed out, Section 204(A) of the Tax Code applied to all compromises, whether governmentinitiated or not.9 Where the law did not distinguish, courts too should not distinguish. Hence, this Petition.10 The Issues In GR No. 159694, petitioner raises the following issues for the Courts consideration: "I. Whether petitioners assessment against the estate is valid. "II. Whether respondent can validly argue that she, as well as the other heirs, was not aware of the facts and the law on which the assessment in question is based, after she had opted to propose several compromises on the estate tax due, and even prematurely acting on such proposal by paying 20% of the basic estate tax due."11 The foregoing issues can be simplified as follows: first, whether the assessment against the estate is valid; and, second, whether the compromise entered into is also valid. The Courts Ruling The Petition is unmeritorious. First Issue: Validity of the Assessment Against the Estate The second paragraph of Section 228 of the Tax Code12 is clear and mandatory. It provides as follows: "Sec. 228. Protesting of Assessment. --

xxxxxxxxx "The taxpayers shall be informed in writing of the law and the facts on which the assessment is made: otherwise, the assessment shall be void." In the present case, Reyes was not informed in writing of the law and the facts on which the assessment of estate taxes had been made. She was merely notified of the findings by the CIR, who had simply relied upon the provisions of former Section 22913 prior to its amendment by Republic Act (RA) No. 8424, otherwise known as the Tax Reform Act of 1997. First, RA 8424 has already amended the provision of Section 229 on protesting an assessment. The old requirement of merely notifying the taxpayer of the CIRs findings was changed in 1998 to informing the taxpayer of not only the law, but also of the facts on which an assessment would be made; otherwise, the assessment itself would be invalid. It was on February 12, 1998, that a preliminary assessment notice was issued against the estate. On April 22, 1998, the final estate tax assessment notice, as well as demand letter, was also issued. During those dates, RA 8424 was already in effect. The notice required under the old law was no longer sufficient under the new law. To be simply informed in writing of the investigation being conducted and of the recommendation for the assessment of the estate taxes due is nothing but a perfunctory discharge of the tax function of correctly assessing a taxpayer. The act cannot be taken to mean that Reyes already knew the law and the facts on which the assessment was based. It does not at all conform to the compulsory requirement under Section 228. Moreover, the Letter of Authority received by respondent on March 14, 1997 was for the sheer purpose of investigation and was not even the requisite notice under the law. The procedure for protesting an assessment under the Tax Code is found in Chapter III of Title VIII, which deals with remedies. Being procedural in nature, can its provision then be applied retroactively? The answer is yes. The general rule is that statutes are prospective. However, statutes that are remedial, or that do not create new or take away vested rights, do not fall under the general rule against the retroactive operation of statutes.14 Clearly, Section 228 provides for the procedure in case an assessment is protested. The provision does not create new or take away vested rights. In both instances, it can surely be applied retroactively. Moreover, RA 8424 does not state, either expressly or by necessary implication, that pending actions are excepted from the operation of Section 228, or that applying it to pending proceedings would impair vested rights. Second, the non-retroactive application of Revenue Regulation (RR) No. 12-99 is of no moment, considering that it merely implements the law. A tax regulation is promulgated by the finance secretary to implement the provisions of the Tax Code.15 While it is desirable for the government authority or administrative agency to have one

immediately issued after a law is passed, the absence of the regulation does not automatically mean that the law itself would become inoperative. At the time the pre-assessment notice was issued to Reyes, RA 8424 already stated that the taxpayer must be informed of both the law and facts on which the assessment was based. Thus, the CIR should have required the assessment officers of the Bureau of Internal Revenue (BIR) to follow the clear mandate of the new law. The old regulation governing the issuance of estate tax assessment notices ran afoul of the rule that tax regulations -- old as they were -- should be in harmony with, and not supplant or modify, the law.16 It may be argued that the Tax Code provisions are not self-executory. It would be too wide a stretch of the imagination, though, to still issue a regulation that would simply require tax officials to inform the taxpayer, in any manner, of the law and the facts on which an assessment was based. That requirement is neither difficult to make nor its desired results hard to achieve. Moreover, an administrative rule interpretive of a statute, and not declarative of certain rights and corresponding obligations, is given retroactive effect as of the date of the effectivity of the statute.17 RR 12-99 is one such rule. Being interpretive of the provisions of the Tax Code, even if it was issued only on September 6, 1999, this regulation was to retroact to January 1, 1998 -- a date prior to the issuance of the preliminary assessment notice and demand letter. Third, neither Section 229 nor RR 12-85 can prevail over Section 228 of the Tax Code. No doubt, Section 228 has replaced Section 229. The provision on protesting an assessment has been amended. Furthermore, in case of discrepancy between the law as amended and its implementing but old regulation, the former necessarily prevails.18 Thus, between Section 228 of the Tax Code and the pertinent provisions of RR 12-85, the latter cannot stand because it cannot go beyond the provision of the law. The law must still be followed, even though the existing tax regulation at that time provided for a different procedure. The regulation then simply provided that notice be sent to the respondent in the form prescribed, and that no consequence would ensue for failure to comply with that form. Fourth, petitioner violated the cardinal rule in administrative law that the taxpayer be accorded due process. Not only was the law here disregarded, but no valid notice was sent, either. A void assessment bears no valid fruit. The law imposes a substantive, not merely a formal, requirement. To proceed heedlessly with tax collection without first establishing a valid assessment is evidently violative of the cardinal principle in administrative investigations: that taxpayers should be able to present their case and adduce supporting evidence.19 In the instant case, respondent has not been informed of the basis of the estate tax liability. Without complying with the unequivocal mandate of first informing the taxpayer of the governments claim, there can be no deprivation of property, because no effective protest can be made.20 The haphazard shot at slapping an assessment, supposedly based on estate taxations general provisions that are expected to be known by the taxpayer, is utter chicanery.

Even a cursory review of the preliminary assessment notice, as well as the demand letter sent, reveals the lack of basis for -- not to mention the insufficiency of -- the gross figures and details of the itemized deductions indicated in the notice and the letter. This Court cannot countenance an assessment based on estimates that appear to have been arbitrarily or capriciously arrived at. Although taxes are the lifeblood of the government, their assessment and collection "should be made in accordance with law as any arbitrariness will negate the very reason for government itself."21 Fifth, the rule against estoppel does not apply. Although the government cannot be estopped by the negligence or omission of its agents, the obligatory provision on protesting a tax assessment cannot be rendered nugatory by a mere act of the CIR . Tax laws are civil in nature.22 Under our Civil Code, acts executed against the mandatory provisions of law are void, except when the law itself authorizes the validity of those acts.23 Failure to comply with Section 228 does not only render the assessment void, but also finds no validation in any provision in the Tax Code. We cannot condone errant or enterprising tax officials, as they are expected to be vigilant and law-abiding. Second Issue: Validity of Compromise It would be premature for this Court to declare that the compromise on the estate tax liability has been perfected and consummated, considering the earlier determination that the assessment against the estate was void. Nothing has been settled or finalized. Under Section 204(A) of the Tax Code, where the basic tax involved exceeds one million pesos or the settlement offered is less than the prescribed minimum rates, the compromise shall be subject to the approval of the NEB composed of the petitioner and four deputy commissioners. Finally, as correctly held by the appellate court, this provision applies to all compromises, whether government-initiated or not. Ubi lex non distinguit, nec nos distinguere debemos. Where the law does not distinguish, we should not distinguish. WHEREFORE, the Petition is hereby DENIED and the assailed Decision AFFIRMED. No pronouncement as to costs.

G.R. No. 128315. June 29, 1999]


COMMISSIONER OF INTERNAL REVENUE, petitioner, vs. PASCOR REALTY AND DEVELOPMENT CORPORATION, ROGELIO A. DIO and VIRGINIA S. DIO, respondents. DECISION
PANGANIBAN, J.:

An assessment contains not only a computation of tax liabilities, but also a demand for payment within a prescribed period. It also signals the time when penalties and interests begin to accrue against the taxpayer. To enable the taxpayer to determine his remedies thereon, due process requires that it must be served on and received by the taxpayer. Accordingly, an affidavit, which was executed by revenue officers stating the tax liabilities of a taxpayer and attached to a criminal complaint for tax evasion, cannot be deemed an assessment that can be questioned before the Court of Tax Appeals. Statement of the Case Before this Court is a Petition for Review on Certiorari under Rule 45 of the Rules of Court praying for the nullification of the October 30, 1996 Decisioni[1] of the Court of Appealsii[2] in CA-GR SP No. 40853, which effectively affirmed the January 25, 1996 Resolutioniii[3] of the Court of Tax Appealsiv[4] in CTA Case No. 5271. The CTA disposed as follows: WHEREFORE, finding [the herein petitioners] Motion to Dismiss as UNMERITORIOUS, the same is hereby DENIED. [The CIR] is hereby given a period of thirty (30) days from receipt hereof to file her answer. Petitioner also seeks to nullify the February 13, 1997 Resolutionv[5] of the Court of Appeals denying reconsideration. The Facts As found by the Court of Appeals, the undisputed facts of the case are as follows: It appears that by virtue of Letter of Authority No. 001198, then BIR Commissioner Jose U. Ong authorized Revenue Officers Thomas T. Que, Sonia T. Estorco and Emmanuel M. Savellano to examine the books of accounts and other accounting records of Pascor Realty and Development Corporation. (PRDC) for the years ending 1986, 1987 and 1988. The said examination resulted in a recommendation for the issuance of an assessment in the amounts of P7,498,434.65 and P3,015,236.35 for the years 1986 and 1987, respectively. On March 1, 1995, the Commissioner of Internal Revenue filed a criminal complaint before the Department of Justice against the PRDC, its President Rogelio A. Dio, and its Treasurer Virginia S. Dio, alleging evasion of taxes in the total amount of P10,513,671.00. Private respondents PRDC, et. al. filed an Urgent Request for Reconsideration/Reinvestigation disputing the tax assessment and tax liability. On March 23, 1995, private respondents received a subpoena from the DOJ in connection with the criminal complaint filed by the Commissioner of Internal Revenue (BIR) against them. In a letter dated May 17, 1995, the CIR denied the urgent request for reconsideration/reinvestigation of the private respondents on the ground that no formal assessment has as yet been issued by the Commissioner.

Private respondents then elevated the Decision of the CIR dated May 17, 1995 to the Court of Tax Appeals on a petition for review docketed as CTA Case No. 5271 on July 21, 1995. On September 6, 1995, the CIR filed a Motion to Dismiss the petition on the ground that the CTA has no jurisdiction over the subject matter of the petition, as there was no formal assessment issued against the petitioners. The CTA denied the said motion to dismiss in a Resolution dated January 25, 1996 and ordered the CIR to file an answer within thirty (30) days from receipt of said resolution. The CIR received the resolution on January 31, 1996 but did not file an answer nor did she move to reconsider the resolution. Instead, the CIR filed this petition on June 7, 1996, alleging as grounds that: Respondent Court of Tax Appeals acted with grave abuse of discretion and without jurisdiction in considering the affidavit/report of the revenue officer and the indorsement of said report to the secretary of justice as assessment which may be appealed to the Court of Tax Appeals; Respondent Court of Tax Appeals acted with grave abuse of discretion in considering the denial by petitioner of private respondents Motion for Reconsideration as [a] final decision which may be appealed to the Court of Tax Appeals. In denying the motion to dismiss filed by the CIR, the Court of Tax Appeals stated: We agree with petitioners contentions, that the criminal complaint for tax evasion is the assessment issued, and that the letter denial of May 17, 1995 is the decision properly appealable to [u]s. Respondents ground of denial, therefore, that there was no formal assessment issued, is untenable. It is the Courts honest belief, that the criminal case for tax evasion is already an assessment. The complaint, more particularly, the Joint Affidavit of Revenue Examiners Lagmay and Savellano attached thereto, contains the details of the assessment like the kind and amount of tax due, and the period covered. Petitioners are right, in claiming that the provisions of Republic Act No. 1125, relating to exclusive appellate jurisdiction of this Court, do not, make any mention of formal assessment. The law merely states, that this Court has exclusive appellate jurisdiction over decisions of the Commissioner of Internal Revenue on disputed assessments, and other matters arising under the National Internal Revenue Code, other law or part administered by the Bureau of Internal Revenue Code. As far as this Court is concerned, the amount and kind of tax due, and the period covered, are sufficient details needed for an assessment. These details are more than complete, compared to the following definitions of the term as quoted hereunder. Thus: Assessment is laying a tax. Johnson City v. Clinchfield R. Co., 43 S.W. (2d) 386, 387, 163 Tenn. 332. (Words and Phrases, Permanent Edition, Vol. 4, p. 446)

The word assessment when used in connection with taxation, may have more than one meaning. The ultimate purpose of an assessment to such a connection is to ascertain the amount that each taxpayer is to pay. More commonly, the word assessment means the official valuation of a taxpayers property for purpose of taxation. State v. New York, N.H. and H.R. Co. 22 A. 765, 768, 60 Conn. 326, 325. (Ibid. p. 445) From the above, it can be gleaned that an assessment simply states how much tax is due from a taxpayer. Thus, based on these definitions, the details of the tax as given in the Joint Affidavit of respondents examiners, which was attached to the tax evasion complaint, more than suffice to qualify as an assessment. Therefore, this assessment having been disputed by petitioners, and there being a denial of their letter disputing such assessment, this Court unquestionably acquired jurisdiction over the instant petition for review.vi[6] As earlier observed, the Court of Appeals sustained the CTA and dismissed the petition. Hence, this recourse to this Court.vii[7] Ruling of the Court of Appeals The Court of Appeals held that the tax court committed no grave abuse of discretion in ruling that the Criminal Complaint for tax evasion filed by the Commissioner of Internal Revenue with the Department of Justice constituted an assessment of the tax due, and that the said assessment could be the subject of a protest. By definition, an assessment is simply the statement of the details and the amount of tax due from a taxpayer. Based on this definition, the details of the tax contained in the BIR examiners Joint Affidavit,viii[8] which was attached to the criminal Complaint, constituted an assessment. Since the assailed Order of the CTA was merely interlocutory and devoid of grave abuse of discretion, a petition for certiorari did not lie. Issues Petitioners submit for the consideration of this Court the following issues: (1) Whether or not the criminal complaint for tax evasion can be construed as an assessment.

(2) Whether or not an assessment is necessary before criminal charges for tax evasion may be instituted. (3) Whether or not the CTA can take cognizance of the case in the absence of an assessment.ix[9] In the main, the Court will resolve whether the revenue officers Affidavit-Report, which was attached to the criminal Complaint filed with the Department of Justice, constituted an assessment that could be questioned before the Court of Tax Appeals. The Courts Ruling

The petition is meritorious. Main Issue: Assessment Petitioner argues that the filing of the criminal complaint with the Department of Justice cannot in any way be construed as a formal assessment of private respondents tax liabilities. This position is based on Section 205 of the National Internal Revenue Codex[10] (NIRC), which provides that remedies for the collection of deficient taxes may be by either civil or criminal action. Likewise, petitioner cites Section 223(a) of the same Code, which states that in case of failure to file a return, the tax may be assessed or a proceeding in court may be begun without assessment. Respondents, on the other hand, maintain that an assessment is not an action or proceeding for the collection of taxes, but merely a notice that the amount stated therein is due as tax and that the taxpayer is required to pay the same. Thus, qualifying as an assessment was the BIR examiners Joint Affidavit, which contained the details of the supposed taxes due from respondent for taxable years ending 1987 and 1988, and which was attached to the tax evasion Complaint filed with the DOJ. Consequently, the denial by the BIR of private respondents request for reinvestigation of the disputed assessment is properly appealable to the CTA. We agree with petitioner. Neither the NIRC nor the revenue regulations governing the protest of assessmentsxi[11] provide a specific definition or form of an assessment. However, the NIRC defines the specific functions and effects of an assessment. To consider the affidavit attached to the Complaint as a proper assessment is to subvert the nature of an assessment and to set a bad precedent that will prejudice innocent taxpayers. True, as pointed out by the private respondents, an assessment informs the taxpayer that he or she has tax liabilities. But not all documents coming from the BIR containing a computation of the tax liability can be deemed assessments. To start with, an assessment must be sent to and received by a taxpayer, and must demand payment of the taxes described therein within a specific period. Thus, the NIRC imposes a 25 percent penalty, in addition to the tax due, in case the taxpayer fails to pay the deficiency tax within the time prescribed for its payment in the notice of assessment. Likewise, an interest of 20 percent per annum, or such higher rate as may be prescribed by rules and regulations, is to be collected from the date prescribed for its payment until the full payment.xii[12] The issuance of an assessment is vital in determining the period of limitation regarding its proper issuance and the period within which to protest it. Section 203xiii[13]of the NIRC provides that internal revenue taxes must be assessed within three years from the last day within which to file the return. Section 222,xiv[14] on the other hand, specifies a period of ten years in case a fraudulent return with intent to evade was submitted or in case of failure to file a return. Also, Section 228xv[15] of the same law states that said assessment may be protested only within thirty days from receipt thereof. Necessarily, the taxpayer must be certain that a specific document constitutes an assessment. Otherwise, confusion would arise regarding the period within which to make an assessment or to protest the same, or whether interest and penalty may accrue thereon.

It should also be stressed that the said document is a notice duly sent to the taxpayer. Indeed, an assessment is deemed made only when the collector of internal revenue releases, mails or sends such notice to the taxpayer.xvi[16] In the present case, the revenue officers Affidavit merely contained a computation of respondents tax liability. It did not state a demand or a period for payment. Worse, it was addressed to the justice secretary, not to the taxpayers. Respondents maintain that an assessment, in relation to taxation, is simply understood to mean: A notice to the effect that the amount therein stated is due as tax and a demand for payment thereof.xvii[17] Fixes the liability of the taxpayer and ascertains the facts and furnishes the data for the proper presentation of tax rolls.xviii[18] Even these definitions fail to advance private respondents case. That the BIR examiners Joint Affidavit attached to the Criminal Complaint contained some details of the tax liabilities of private respondents does not ipso facto make it an assessment. The purpose of the Joint Affidavit was merely to support and substantiate the Criminal Complaint for tax evasion. Clearly, it was not meant to be a notice of the tax due and a demand to the private respondents for payment thereof. The fact that the Complaint itself was specifically directed and sent to the Department of Justice and not to private respondents shows that the intent of the commissioner was to file a criminal complaint for tax evasion, not to issue an assessment. Although the revenue officers recommended the issuance of an assessment, the commissioner opted instead to file a criminal case for tax evasion. What private respondents received was a notice from the DOJ that a criminal case for tax evasion had been filed against them, not a notice that the Bureau of Internal Revenue had made an assessment. In addition, what private respondents sent to the commissioner was a motion for a reconsideration of the tax evasion charges filed, not of an assessment, as shown thus: This is to request for reconsideration of the tax evasion charges against my client, PASCOR Realty and Development Corporation and for the same to be referred to the Appellate Division in order to give my client the opportunity of a fair and objective hearingxix[19] Additional Issues: Assessment Not Necessary Before Filing of Criminal Complaint Private respondents maintain that the filing of a criminal complaint must be preceded by an assessment. This is incorrect, because Section 222 of the NIRC specifically states that in cases where a false or fraudulent return is submitted or in cases of failure to file a return such as this case, proceedings in court may be commenced without an assessment. Furthermore, Section 205 of the same Code clearly mandates that the civil and criminal aspects of the case may be pursued simultaneously. In Ungab v. Cusi,xx[20] petitioner therein sought the dismissal of the criminal

Complaints for being premature, since his protest to the CTA had not yet been resolved. The Court held that such protests could not stop or suspend the criminal action which was independent of the resolution of the protest in the CTA. This was because the commissioner of internal revenue had, in such tax evasion cases, discretion on whether to issue an assessment or to file a criminal case against the taxpayer or to do both. Private respondents insist that Section 222 should be read in relation to Section 255 of the NIRC,xxi[21] which penalizes failure to file a return. They add that a tax assessment should precede a criminal indictment. We disagree. To reiterate, said Section 222 states that an assessment is not necessary before a criminal charge can be filed. This is the general rule. Private respondents failed to show that they are entitled to an exception. Moreover, the criminal charge need only be supported by a prima facie showing of failure to file a required return. This fact need not be proven by an assessment. The issuance of an assessment must be distinguished from the filing of a complaint. Before an assessment is issued, there is, by practice, a pre-assessment notice sent to the taxpayer. The taxpayer is then given a chance to submit position papers and documents to prove that the assessment is unwarranted. If the commissioner is unsatisfied, an assessment signed by him or her is then sent to the taxpayer informing the latter specifically and clearly that an assessment has been made against him or her. In contrast, the criminal charge need not go through all these. The criminal charge is filed directly with the DOJ. Thereafter, the taxpayer is notified that a criminal case had been filed against him, not that the commissioner has issued an assessment. It must be stressed that a criminal complaint is instituted not to demand payment, but to penalize the taxpayer for violation of the Tax Code. WHEREFORE, the petition is hereby GRANTED. The assailed Decision is REVERSED and SET ASIDE. CTA Case No. 5271 is likewise DISMISSED. No costs.

G.R. Nos. 130371 &130855 August 4, 2009 REPUBLIC OF THE PHILIPPINES, Petitioner, vs. FERDINAND R. MARCOS II and IMELDA R. MARCOS, Respondents. DECISION DEL CASTILLO, J.: Before this Court is a Petition for Review on Certiorari1 under Rule 45 of the Rules of Court, seeking to set aside the March 13, 1997 Decision2 and August 27, 1997 Resolution3 of the Court of Appeals (CA) in CA-G.R. SP No. 43450. The facts of the case are as follows: On January 11, 1996, the Regional Trial Court (RTC) of Pasig City Branch 156, acting as a probate court, in Special Proceeding No. 10279, issued an Order4 granting letters testamentary in solidum to respondents Ferdinand R. Marcos II and Imelda Trinidad Romualdez-Marcos as executors of the last will and testament of the late Ferdinand E. Marcos. The dispositive portion of the January 11, 1996 Order reads: WHEREFORE, finding the Last Will and Testament of Ferdinand Edralin Marcos to have been duly executed in accordance with law, the same is hereby ALLOWED AND ADMITTED TO PROBATE. Upon the filing of a bond in the amount of P50,000.00, let letters testamentary be issued in solidum to Imelda Trinidad Romualdez-Marcos AND Ferdinand Romualdez Marcos II, named executors therein. Pending the filing of said bond and their oath, Commissioner Liwayway Vinzons-Chato of the Bureau of Internal Revenue is hereby authorized to continue her functions as Special Administrator of the Estate of Ferdinand Edralin Marcos. Let NOTICE be given to all known heirs and creditors of the decedent, and to any other persons having an interest in the estate for them to lay their claim against the Estate or forever hold their peace.

SO ORDERED.5 On January 15, 1996, the petitioner Republic of the Philippines filed a Motion for Partial Reconsideration6 in so far as the January 11, 1996 RTC Order granted letters testamentary to respondents. On the other hand, respondent Imelda Marcos filed her own motion for reconsideration on the ground that the will is lost and that petitioner has not proven its existence and validity. On February 5, 1996, respondent Ferdinand Marcos II filed a Compliance stating that he already filed a bond in the amount of P50,000.00 as directed by the January 11, 1996 RTC Order and that he took his oath as named executor of the will on January 30, 1996. On March 13, 1996, the RTC issued Letters of Administration7 to BIR Commissioner Liwayway Vinzons-Chato in accordance with an earlier Order dated September 9, 1994, appointing her as Special Administratrix of the Marcos Estate. On April 1, 1996, respondent Ferdinand Marcos II filed a Motion to Revoke the Letters of Administration issued by the RTC to BIR Commissioner Vinzons-Chato. On April 26, 1996, the RTC issued an Order8 denying the motion for partial reconsideration filed by petitioner as well as the motion for reconsideration filed by respondent Imelda Marcos, the penultimate portion of which reads: Under the Rules, a decedents testamentary privilege must be accorded utmost respect. Guided by this legal precept, therefore, in resolving the two (2) motions at hand, the Court is constrained to DENY both. Examining the arguments poised by the movants, the Court observed that these are but a mere rehash of issues already raised and passed upon by the Court. One has to review the previous orders issued by the Court in this case, e.g., the orders dated September 9, 1994, November 25, 1994, as well as October 3, 1995, to see that even as far back then, the Court has considered the matter of competency of the oppositors and of Commissioner Liwayway Vinzons-Chato as having been settled. It cannot be overstressed that the assailed January 11, 1996 Orders of the Court was arrived at only after extensive consideration of every legal facet available on the question of validity of the Will. WHEREFORE, for lack of merit, the motion for reconsideration filed separately by petitioner Republic and oppositor Imelda R. Marcos are both DENIED. SO ORDERED.9 On June 6, 1996, petitioner filed with this Court a Petition for Review on Certiorari, under Ruled 45 of the Rules of Court, questioning the aforementioned RTC Orders granting letters testamentary to respondents. On February 5, 1997, the First Division of this Court issued a Resolution referring the petition to the CA, to wit: xxxx The special civil action for certiorari as well as all the other pleadings filed herein are REFERRED to the Court of Appeals for consideration and adjudication on the merits or any other action as it may deem appropriate, the latter having jurisdiction concurrent with this Court over the Case, and this Court having been cited to no special and important reason for it to take cognizance of said case in the first instance.10 (Emphasis and Underscoring Supplied) On March 13, 1997, the CA issued a Decision,11 dismissing the referred petition for having taken the wrong mode of appeal, the pertinent portions of which reads: Consequently, for having taken the wrong mode of appeal, the present petition should be dismissed in accordance with the same Supreme Court Circular 2-90 which expressly provides that: 4. Erroneous Appeals An appeal taken to either the Supreme Court or the Court of Appeals by the wrong or inappropriate mode shall be dismissed. IN VIEW OF THE FOREGOING, the instant petition for review is hereby DISMISSED. SO ORDERED.12 Petitioner filed a Motion for Reconsideration,13 which was, however denied by the CA in a Resolution14 dated August 27, 1997. Hence, herein petition, with petitioner raising the following assignment of errors, to wit: I. THE COURT OF APPEALS GRAVELY ERRED IN DISMISSING THE PETITION ON TECHNICAL GROUNDS DESPITE THE SUPREME COURT RESOLUTION SPECIFICALLY REFERRING SAID PETITION FOR A DECISION ON THE MERITS. II. THE PROBATE COURT GRAVELY ERRED IN FAILING TO CONSIDER THAT RESPONDENTS IMELDA R. MARCOS AND FERDINAND R. MARCOS II SHOULD BE DISQUALIFIED TO ACT AND SERVE AS EXECUTORS. III. THE PROBATE COURT GRAVELY ERRED IN FAILING TO CONSIDER THAT SAID PRIVATE RESPONDENTS HAVE DENIED AND DISCLAIMED THE VERY EXISTENCE AND VALIDITY OF THE MARCOS WILL. IV. THE PROBATE COURT GRAVELY ERRED IN FAILING TO CONSIDER THAT ITS ORDER OF JANUARY 11, 1996, WHICH ADMITTED THE MARCOS WILL TO PROBATE AND WHICH DIRECTED THE ISSUANCE OF LETTERS TESTAMENTARY IN SOLIDUM TO PRIVATE RESPONDENTS AS EXECUTORS OF SAID MARCOS WILL, WAS BASED ON THE EVIDENCE OF THE REPUBLIC ALONE. V. THE PROBATE COURT GRAVELY ERRED IN FAILING TO CONSIDER THAT BOTH PRIVATE RESPONDENTS HAVE OBSTRUCTED THE TRANSFER TO THE PHILIPPINES OF THE MARCOS ASSETS DEPOSITED IN THE SWISS BANKS.15 In the meantime, on October 9, 2002, the RTC, acting on the pending unresolved motions before it, issued an Order16 which reads: WHEREFORE, the Court hereby appoints as joint special administrators of the estate of the late Ferdinand E. Marcos, the nominee of the Republic of the Philippines (the Undersecretary of the Department of Justice whom the Secretary of Justice will designate for this purpose) and Mrs. Imelda Romualdez Marcos and Mr. Ferdinand R. Marcos II, to serve as such until an executor is finally appointed. SO ORDERED. The petition is without merit.

When the assailed Orders granting letters testamentary in solidum to respondents were issued by the RTC, petitioner sought to question them by filing a petition for review on certiorari under Rule 45 of the Rules of Court. Supreme Court Circular No. 2-90,17 which was then in effect, reads: 2. Appeals from Regional Trial Courts to the Supreme Court. Except in criminal cases where the penalty imposed is life imprisonment to reclusion perpetua, judgments of regional trial courts may be appealed to the Supreme Court only by petition for review on certiorari in accordance with Rule 45 of the Rules of Court in relation to Section 17 of the Judiciary Act of 1948, as amended, this being the clear intendment of the provision of the Interim Rules that "(a)ppeals to the Supreme Court shall be taken by petition for certiorari which shall be governed by Rule 45 of the Rules of Court. (Emphasis and Underscoring Supplied) The pertinent portions of Section 1718 of the Judiciary Act of 1948 read: The Supreme Court shall further have exclusive jurisdiction to review, revise, reverse, modify or affirm on certiorari as the law or rules of court may provide, final judgments and decrees of inferior courts as herein provided, in (1) All cases in which the constitutionality or validity of any treaty, law, ordinance, or executive order or regulation is in question; (2) All cases involving the legality of any tax, impost, assessment or toll, or any penalty imposed in relation thereto; (3) All cases in which the jurisdiction of any inferior court is in issue; (4) All other cases in which only errors or questions of law are involved: Provided, however, That if, in addition to constitutional, tax or jurisdictional questions, the cases mentioned in the three next preceding paragraphs also involve questions of fact or mixed questions of fact and law, the aggrieved party shall appeal to the Court of Appeals; and the final judgment or decision of the latter may be reviewed, revised, reversed, modified or affirmed by the Supreme Court on writ of certiorari; and (5) Final awards, judgments, decision or orders of the Commission on Elections, Court of Tax Appeals, Court of Industrial Relations, the Public Service Commission, and the Workmens Compensation Commission. A reading of Supreme Court Circular 2-90, in relation to Section 17 of the Judiciary Act of 1948, clearly shows that the subject matter of therein petition, that is, the propriety of granting letters testamentary to respondents, do not fall within any ground which can be the subject of a direct appeal to this Court. The CA was thus correct in declaring that the "issues raised by petitioner do not fall within the purview of Section 17 of the Judiciary Act of 1948 such that the Supreme Court should take cognizance of the instant case."19 Moreover, the Courts pronouncement in Suarez v. Judge Villarama20 is instructive: Section 4 of Circular No. 2-90, in effect at the time of the antecedents, provides that an appeal taken to either the Supreme Court or the Court of Appeals by the wrong mode or inappropriate mode shall be dismissed. This rule is now incorporated in Section 5, Rule 56 of the 1997 Rules of Civil Procedure. Moreover, the filing of the case directly with this Court runs afoul of the doctrine of hierarchy of courts. Pursuant to this doctrine, direct resort from the lower courts to the Supreme Court will not be entertained unless the appropriate remedy cannot be obtained in the lower tribunals. This Court is a court of last resort, and must so remain if it is to satisfactorily perform the functions assigned to it by the Constitution and immemorial tradition. Thus, a petition for review on certiorari assailing the decision involving both questions of fact and law must first be brought before the Court of Appeals.21 Also, in Southern Negros Development Bank v. Court of Appeals,22 this Court ruled: It is incumbent upon private respondent qua appellants to utilize the correct mode of appeal of the decisions of trial courts to the appellate courts. In the mistaken choice of their remedy, they can blame no one but themselves (Jocson v. Baguio, 179 SCRA 550 [1989]; Yucuanseh Drug Co. v. National Labor Union, 101 Phil. 409 [1957]). xxxx Pursuant to Section 4 of Circular No. 2-90, which provides that "[a]n appeal taken to either the Supreme Court or the Court of Appeals by the wrong mode or inappropriate mode shall be dismissed," the only course of action of the Court to which an erroneous appeal is made is to dismiss the same. There is no longer any justification for allowing transfers of erroneous appeals from one court to another (Quesada v. Court of Appeals, G.R. No. 93869, November 12, 1990, First Division, Minute Resolution).23 Based on the foregoing, petitioner cannot deny that the determination of whether or not respondents should be disqualified to act as executors is a question of fact. Hence, the proper remedy was to appeal to the CA, not to this Court. Petitioner is adamant, however, that notwithstanding the improper remedy, the CA should not have dismissed therein petition. Petitioner argues in the wise: However, as can be seen in the Resolution of February 5, 1997, (Annex "H") this Honorable Court deemed it more proper to transmit the first Petition for Review to respondent appellate court for the reason that: This Court having been cited to no special and important reason for it to take cognizance of said case in the first instance. x x x It would appear then that even though this Honorable Court apparently considers the Republics petition as deserving to be given due course, it deemed it in the best interest of the parties concerned if the Court of Appeals would first take cognizance of said case, thereby preserving its stance as a court of last resort. Additionally, this Honorable Court itself plainly stated that the case under review is: .REFERRED to the Court of Appeals for consideration and adjudication on the merits. The latter having jurisdiction concurrent with this C ourt over the case24 Petitioners arguments are misplaced. To stress, the February 5, 1997 Resolution reads: The special civil action for certiorari as well as all the other pleadings filed herein are REFERRED to the Court of Appeals for consideration and adjudication on the merits or any other action as it may deem appropriate, the latter having jurisdiction concurrent with this Court over the Case, and this Court having been cited to no special and important reason for it to take cognizance of said case in the first instance.25 Based thereon, this Court agrees with the ruling of the CA that said resolution gave the CA discretion and latitude to decide the petition as it may deem proper. The resolution is clear that the petition was referred to the CA for consideration and adjudication on the merits or any other action as it may deem appropriate. Thus, no error can be attributed to the CA when the action it deemed appropriate was to dismiss the petition for having availed of an improper remedy. More importantly, the action of the CA was sanctioned under Section 4 of Supreme Court

Circular 2-90 which provides that "an appeal taken to either the Supreme Court or the Court of Appeals by the wrong mode or inappropriate mode shall be dismissed." Moreover, petitioner mistakenly relies in Oriental Media, Inc. v. Court of Appeals,26 in which this Court made the following pronouncements: In the case at bar, there was no urgency or need for Oriental to resort to the extraordinary remedy of certiorari for when it learned of the case and the judgment against it on July 25, 1986, due to its receipt of a copy of the decision by default; no execution had as yet been ordered by the trial court. As aforementioned, Oriental had still the time and the opportunity to file a motion for reconsideration, as was actually done. Upon the denial of its motion for reconsideration in the first case, or at the latest upon the denial of its petition for relief from judgment, Oriental should have appealed. Oriental should have followed the procedure set forth in the Rules of Court for Rules of procedure are intended to ensure the orderly administration of justice and the protection of substantive rights in judicial and extrajudicial proceedings. It is a mistake to purpose that substantive law and adjective law are contradictory to each other or, as has often been suggested, that enforcement of procedural rules should never be permitted if it will result in prejudice to the substantive rights of the litigants. This is not exactly true; the concept is much misunderstood. As a matter of fact, the policy of the courts is to give effect to both kinds of law, as complementing each other, in the just and speedy resolution of the dispute between the parties. Observance of both substantive rights is equally guaranteed by due process whatever the source of such rights, be it the Constitution itself or only a statute or a rule of court.27 In the case at bar, as found by this Court in its February 5, 1997 Resolution, therein petition offered no important or special reason for the Court to take cognizance of it at the first instance. Petitioner offered no plausible reason why it went straight to this Court when an adequate and proper remedy was still available. The CA was thus correct that the remedy that petitioner should have availed of was to file an appeal under Rule 109 of the Rules of Court which states: Section 1. Orders of judgments from which appeals taken. An interested person may appeal in special proceedings from an order or judgment rendered by a Court of First Instance or a Juvenile and Domestic Relations Court, where such order or judgment: (a) allows or disallows a will; Because of the preceding discussion, herein petition must necessarily fail. However, even if this Court were to set aside pe titioners procedural lapses, a careful review of the records of the case reveal that herein petition is without merit. At the crux of the controversy is a determination of whether or not respondents are incompetent to serve as executors of the will of Ferdinand Marcos. Ozeata v. Pecson28 is instructive: The choice of his executor is a precious prerogative of a testator, a necessary concomitant of his right to dispose of his property in the manner he wishes. It is natural that the testator should desire to appoint one of his confidence, one who can be trusted to carry out his wishes in the disposal of the estate. The curtailment of this right may be considered as a curtailment of the right to dispose. And as the rights granted by will take effect from the time of death (Article 777, Civil Code of the Philippines), the management of his estate by the administrator of his choice should be made as soon as practicable, when no reasonable objection to his assumption of the trust can be interposed any longer. It has been held that when a will has been admitted to probate, it is the duty of the court to issue letters testamentary to the person named as executor upon his application (23 C.J. 1023). xxxx The case of In re Erlanger's Estate, 242 N.Y.S. 249, also reiterates the same principle. The courts have always respected the right to which a testator enjoys to determine who is most suitable to settle his testamentary affairs, and his solemn selection should not lightly be disregarded. After the admission of a will to probate, the courts will not name a better executor for the testator nor disqualify, by a judicial veto, the widow or friend or other person selected in the will, except upon strict proof of the statutory grounds of incompetency. Matter of Leland's Will, 219 N.Y. 387, 393, 114 N.E. 854. x x x29 Section 1(c), Rule 78 of the Rules of Court defines who are incompetent to serve as executors, to wit: Section 1. Who are incompetent to serve as executors or administrators. No person is competent to serve as executor or administrator who: xxxx (c) Is in the opinion of the court unfit to execute the duties of trust by reason of drunkenness, improvidence, or want of understanding or integrity, or by reason of conviction of an offense involving moral turpitude. (Emphasis Supplied) In the case at bar, petitioner anchored its opposition to the grant of letters testamentary to respondents, specifically on the following grounds: (1) want of integrity, and (2) conviction of an offense involving moral turpitude. Petitioner contends that respondents have been convicted of a number of cases30 and, hence, should be characterized as one without integrity, or at the least, with questionable integrity. 31 The RTC, however, in its January 11, 1996 Order, made the following findings: However, except for petitioner Republics allegation of want of integrity on the part of Imelda Trinidad Romualdez-Marcos and Ferdinand Romualdez Marco II, named executors in the last will and testament, so as to render them "incompetent" to serve as executors, the Court sees at this time, no evidence on record, oral or documentary, to substantiate and support the said allegation. (Emphasis Supplied) Based on the foregoing, this Court stresses that an appellate court is disinclined to interfere with the action taken by the probate court in the matter of removal of an executor or administrator unless positive error or gross abuse of discretion is shown.32 The Rules of Court gives the lower court the duty and discretion to determine whether in its opinion an individual is unfit to serve as an executor. The sufficiency of any ground for removal should thus be determined by the said court, whose sensibilities are, in the first place, affected by any act or omission on the part of the administrator not conformable to or in disregard of the rules of orders of the court.33 Hence, in order to reverse the findings of the RTC, this Court must evaluate the evidence presented or alleged by petitioner in support of its petition for disqualification. However, after a painstaking review of the records and evidence on hand, this Court finds that the RTC committed no error or gross abuse of discretion when it ruled that petitioner failed to substantiate its allegation. Petitioner conveniently omits to state that the two cases against respondent Imelda Marcos have already been reversed by this Court. Her conviction in Criminal Case No. 17453 was reversed by this Court in Dans, Jr. v. People.34 Likewise, her conviction in Criminal Case No. 17450 was reversed by this Court in Marcos v. Sandiganbayan.35 Hence, the so-called "convictions" against respondent Imelda Marcos cannot serve as a ground for her disqualification to serve as an executor.

On the other hand, the eight cases filed against respondent Ferdinand Marcos II involve four charges for violation of Section 45 (failure to file income tax returns) and four charges for violation of Section 50 (non-payment of deficiency taxes) of the National Internal Revenue Code of 1977 (NIRC). It is a matter of record, that in CA-G.R. CR No. 18569,36 the CA acquitted respondent Ferdinand Marcos II of all the four charges for violation of Section 50 and sustained his conviction for all the four charges for violation of Section 45. It, however, bears to stress, that the CA only ordered respondent Marcos II to pay a fine for his failure to file his income tax return. Moreover, and as admitted by petitioner, 37 said decision is still pending appeal. Therefore, since respondent Ferdinand Marcos II has appealed his conviction relating to four violations of Section 45 of the NIRC, the same should not serve as a basis to disqualify him to be appointed as an executor of the will of his father. More importantly, even assuming arguendo that his conviction is later on affirmed, the same is still insufficient to disqualify him as the "failure to file an income tax return" is not a crime involving moral turpitude. In Villaber v. Commision on Elections,38 this Court held: As to the meaning of "moral turpitude," we have consistently adopted the definition in Black's Law Dictionary as "an act of baseness, vileness, or depravity in the private duties which a man owes his fellow men, or to society in general, contrary to the accepted and customary rule of right and duty between man and woman, or conduct contrary to justice, honesty, modesty, or good morals." In In re Vinzon, the term "moral turpitude" is considered as encompassing "everything which is done contrary to justice, honesty, or good morals." xxxx We, however, clarified in Dela Torre vs. Commission on Elections that "not every criminal act involves moral turpitude," and that ''as to what crime involves moral turpitude is for the Supreme Court to determine."39 Moreover, In De Jesus-Paras v. Vailoces:40 Indeed, it is well-settled that "embezzlement, forgery, robbery, and swindling are crimes which denote moral turpitude and, as a general rule, all crimes of which fraud is an element are looked on as involving moral turpitude" (58 C.J.S., 1206). The "failure to file an income tax return" is not a crime involving moral turpitude as the mere omission is already a violation regardless of the fraudulent intent or willfulness of the individual. This conclusion is supported by the provisions of the NIRC as well as previous Court decisions which show that with regard to the filing of an income tax return, the NIRC considers three distinct violations: (1) a false return, (2) a fraudulent return with intent to evade tax, and (3) failure to file a return. The same is illustrated in Section 51(b) of the NIRC which reads: (b) Assessment and payment of deficiency tax xxx In case a person fails to make and file a return or list at the time prescribed by law, or makes willfully or otherwise, false or fraudulent return or list x x x. (Emphasis Supplied) Likewise, in Aznar v. Court of Tax Appeals,41 this Court observed: To our minds we can dispense with these controversial arguments on facts, although we do not deny that the findings of facts by the Court of Tax Appeals, supported as they are by very substantial evidence, carry great weight, by resorting to a proper interpretation of Section 332 of the NIRC. We believe that the proper and reasonable interpretation of said provision should be that in the three different cases of (1) false return, (2) fraudulent return with intent to evade tax, (3) failure to file a return, the tax may be assessed, or a proceeding in court for the collection of such tax may be begun without assessment, at any time within ten years after the discovery of the (1) falsity, (2) fraud, and (3) omission. Our stand that the law should be interpreted to mean a separation of the three different situations of false return, fraudulent return with intent to evade tax, and failure to file a return is strengthened immeasurably by the last portion of the provision which segregates the situations into three different classes, namely, "falsity," "fraud" and "omission."42 (Emphasis Supplied) Applying the foregoing considerations to the case at bar, the filing of a "fraudulent return with intent to evade tax" is a crime involving moral turpitude as it entails willfulness and fraudulent intent on the part of the individual. The same, however, cannot be said for "failure to file a return" where the mere omission already constitutes a violation. Thus, this Court holds that even if the conviction of respondent Marcos II is affirmed, the same not being a crime involving moral turpitude cannot serve as a ground for his disqualification. Anent the third error raised by petitioner, the same has no merit. Petitioner contends that respondents denied the existence of the will, and are, therefore, estopped from claiming to be the rightful executors thereof. Petitioner further claims that said actions clearly show that respondents lack the competence and integrity to serve as officers of the court. This Court does not agree with the posture taken by petitioner, and instead, accepts the explanation given by respondents, to wit: Respondents opposed the petition for probate not because they are disclaiming the existence of the will, but because of certain legal grounds, to wit: (a) petitioner does not have the requisite interest to institute it; (b) the original copy of the will was not attached to the petition for probate as required by the rules; and (c) the Commissioner of the Bureau of Internal Revenue is not qualified to be appointed as administrator of the estate.43 Based on the foregoing, considering the nature of their opposition, respondents cannot be held guilty of estoppel as they merely acted within their rights when they put in issue legal grounds in opposing the probate proceedings. More importantly, even if said grounds were later on overruled by the RTC, said court was still of opinion that respondents were fit to serve as executors notwithstanding their earlier opposition. Again, in the absence of palpable error or gross abuse of discretion, this Court will not interfere with the RTCs discretion. As for the remaining errors assigned by petitioner, the same are bereft of merit. Petitioner contends that respondents have strongly objected to the transfer to the Philippines of the Marcos assets deposited in the Swiss Banks44 and thus the same should serve as a ground for their disqualification to act as executors. This Court does not agree. In the first place, the same are mere allegations which, without proof, deserve scant consideration. Time and again, this Court has stressed that this Court is a court of law and not a court of public opinion. Moreover, petitioner had already raised the same argument in its motion for partial reconsideration before the RTC.1avvphi1 Said court, however, still did not find the same as a sufficient ground to disqualify respondents. Again, in the absence of palpable error or gross abuse of discretion, this Court will not interfere with the RTCs discretion.

Lastly, petitioner argues that the assailed RTC Orders were based solely on their own evidence and that respondents offered no evidence to show that they were qualified to serve as executors.45 It is basic that one who alleges a fact has the burden of proving it and a mere allegation is not evidence.46 Consequently, it was the burden of petitioner (not respondents) to substantiate the grounds upon which it claims that respondents should be disqualified to serve as executors, and having failed in doing so, its petition must necessarily fail. WHEREFORE, premises considered, the March 13, 1997 Decision and August 27, 1997 Resolution of the Court of Appeals in CA-G.R. SP No. 43450 are hereby AFFIRMED. The Regional Trial Court of Pasig City, Branch 156, acting as a probate court in Special Proceeding No. 10279, is hereby ORDERED to issue letters testamentary, in solidum, to Imelda Romualdez-Marcos and Ferdinand Marcos II.

G.R. No. 167146 October 31, 2006 COMMISSIONER OF INTERNAL REVENUE, petitioner, vs. PHILIPPINE GLOBAL COMMUNICATION, INC., respondent. DECISION CHICO-NAZARIO, J.: This is a Petition for Review on Certiorari, under Rule 45 of the Rules of Court, seeking to set aside the en banc Decision of the Court of Tax Appeals (CTA) in CTA EB No. 37 dated 22 February 2005,1 ordering the petitioner to withdraw and cancel Assessment Notice No. 000688-807333 issued against respondent Philippine Global Communication, Inc. for its 1990 income tax deficiency. The CTA, in its assailed en banc Decision, affirmed the Decision of the First Division of the CTA dated 9 June 20042 and its Resolution dated 22 September 2004 in C.T.A. Case No. 6568. Respondent, a corporation engaged in telecommunications, filed its Annual Income Tax Return for taxable year 1990 on 15 April 1991. On 13 April 1992, the Commissioner of Internal Revenue (CIR) issued Letter of Authority No. 0002307, authorizing the appropriate Bureau of Internal Revenue (BIR) officials to examine the books of account and other accounting records of respondent, in connection with the investigation of respondents 1990 income tax liability. On 22 April 1992, the BIR sent a letter to respondent requesting the latter to present for examination certain records and documents, but respondent failed to present any document. On 21 April 1994, respondent received a Preliminary Assessment Notice dated 13 April 1994 for deficiency income tax in the amount of P118,271,672.00, inclusive of surcharge, interest, and compromise penalty, arising from deductions that were disallowed for failure to pay the withholding tax and interest expenses that were likewise disallowed. On the following day, 22 April 1994, respondent received a Formal Assessment Notice with Assessment Notice No. 00068880-7333, dated 14 April 1994, for deficiency income tax in the total amount of P118,271,672.00.3 On 6 May 1994, respondent, through its counsel Ponce Enrile Cayetano Reyes and Manalastas Law Offices, filed a formal protest letter against Assessment Notice No. 000688-80-7333. Respondent filed another protest letter on 23 May 1994, through another counsel Siguion Reyna Montecillo & Ongsiako Law Offices. In both letters, respondent requested for the cancellation of the tax assessment, which they alleged was invalid for lack of factual and legal basis.4 On 16 October 2002, more than eight years after the assessment was presumably issued, the Ponce Enrile Cayetano Reyes and Manalastas Law Offices received from the CIR a Final Decision dated 8 October 2002 denying the respondents protest against Assessment Notic e No. 00068880-7333, and affirming the said assessment in toto.5 On 15 November 2002, respondent filed a Petition for Review with the CTA. After due notice and hearing, the CTA rendered a Decision in favor of respondent on 9 June 2004.6 The CTA ruled on the primary issue of prescription and found it unnecessary to decide the issues on the validity and propriety of the assessment. It decided that the protest letters filed by the respondent cannot constitute a request for reinvestigation, hence, they cannot toll the running of the prescriptive period to collect the assessed deficiency income tax.7 Thus, since more than three years had lapsed from the time Assessment Notice No. 000688-80-7333 was issued in 1994, the CIRs right to collect the same has prescribed in conformity with Section 269 of the National Internal Revenue Code of 19778 (Tax Code of 1977). The dispositive portion of this decision reads: WHEREFORE, premises considered, judgment is hereby rendered in favor of the petitioner. Accordingly, respondents Final Decision dated October 8, 2002 is hereby REVERSED and SET ASIDE and respondent is hereby ORDERED to WITHDRAW and CANCEL Assessment Notice No. 000688-80-7333 issued against the petitioner for its 1990 income tax deficiency because respondents right to collect the same has prescribed.9 The CIR moved for reconsideration of the aforesaid Decision but was denied by the CTA in a Resolution dated 22 September 2004.10 Thereafter, the CIR filed a Petition for Review with the CTA en banc, questioning the aforesaid Decision and Resolution. In its en banc Decision, the CTA affirmed the Decision and Resolution in CTA Case No. 6568. The dispositive part reads: WHEREFORE, premises considered, the Petition for Review is hereby DISMISSED for lack of merit. Accordingly, the assailed Decision and Resolution in CTA Case No. 6568 are hereby AFFIRMED in toto.11 Hence, this Petition for Review on Certiorari raising the following grounds: THE COURT OF TAX APPEALS, SITTING EN BANC, COMMITTED REVERSIBLE ERROR IN AFFIRMING THE ASSAILED DECISION AND RESOLUTION IN CTA CASE NO. 6568 DECLARING THAT THE RIGHT OF THE GOVERNMENT TO COLLECT THE DEFICIENCY INCOME TAX FROM RESPONDENT FOR THE YEAR 1990 HAS PRESCRIBED A. THE PRESCRIPTIVE PERIOD WAS INTERUPTED WHEN RESPONDENT FILED TWO LETTERS OF PROTEST DISPUTING IN DETAIL THE DEFICIENCY ASSESSMENT IN QUESTION AND REQUESTING THE CANCELLATION OF SAID ASSESSMENT. THE TWO LETTERS OF PROTEST ARE, BY NATURE, REQUESTS FOR REINVESTIGATION OF THE DISPUTED ASSESSMENT. B. THE REQUESTS FOR REINVESTIGATION OF RESPONDENT WERE GRANTED BY THE BUREAU OF INTERNAL REVENUE. 12 This Court finds no merit in this Petition.

The main issue in this case is whether or not CIRs right to collect respondents alleged deficiency income tax is barred by prescription under Section 269(c) of the Tax Code of 1977, which reads: Section 269. Exceptions as to the period of limitation of assessment and collection of taxes. x x x xxxx c. Any internal revenue tax which has been assessed within the period of limitation above-prescribed may be collected by distraint or levy or by a proceeding in court within three years following the assessment of the tax. The law prescribed a period of three years from the date the return was actually filed or from the last date prescribed by law for the filing of such return, whichever came later, within which the BIR may assess a national internal revenue tax.13 However, the law increased the prescriptive period to assess or to begin a court proceeding for the collection without an assessment to ten years when a false or fraudulent return was filed with the intent of evading the tax or when no return was filed at all.14 In such cases, the ten-year period began to run only from the date of discovery by the BIR of the falsity, fraud or omission. If the BIR issued this assessment within the three-year period or the ten-year period, whichever was applicable, the law provided another three years after the assessment for the collection of the tax due thereon through the administrative process of distraint and/or levy or through judicial proceedings.15 The three-year period for collection of the assessed tax began to run on the date the assessment notice had been released, mailed or sent by the BIR.16 The assessment, in this case, was presumably issued on 14 April 1994 since the respondent did not dispute the CIRs claim. Therefore, the BIR had until 13 April 1997. However, as there was no Warrant of Distraint and/or Levy served on the respondents nor any judicial proceedings initiated by the BIR, the earliest attempt of the BIR to collect the tax due based on this assessment was when it filed its Answer in CTA Case No. 6568 on 9 January 2003, which was several years beyond the three-year prescriptive period. Thus, the CIR is now prescribed from collecting the assessed tax. The provisions on prescription in the assessment and collection of national internal revenue taxes became law upon the recommendation of the tax commissioner of the Philippines. The report submitted by the tax commission clearly states that these provisions on prescription should be enacted to benefit and protect taxpayers: Under the former law, the right of the Government to collect the tax does not prescribe. However, in fairness to the taxpayer, the Government should be estopped from collecting the tax where it failed to make the necessary investigation and assessment within 5 years after the filing of the return and where it failed to collect the tax within 5 years from the date of assessment thereof. Just as the government is interested in the stability of its collections, so also are the taxpayers entitled to an assurance that they will not be subjected to further investigation for tax purposes after the expiration of a reasonable period of time. (Vol. II, Report of the Tax Commission of the Philippines, pp. 321-322).17 In a number of cases, this Court has also clarified that the statute of limitations on the collection of taxes should benefit both the Government and the taxpayers. In these cases, the Court further illustrated the harmful effects that the delay in the assessment and collection of taxes inflicts upon taxpayers. In Collector of Internal Revenue v. Suyoc Consolidated Mining Company,18 Justice Montemayor, in his dissenting opinion, identified the potential loss to the taxpayer if the assessment and collection of taxes are not promptly made. Prescription in the assessment and in the collection of taxes is provided by the Legislature for the benefit of both the Government and the taxpayer; for the Government for the purpose of expediting the collection of taxes, so that the agency charged with the assessment and collection may not tarry too long or indefinitely to the prejudice of the interests of the Government, which needs taxes to run it; and for the taxpayer so that within a reasonable time after filing his return, he may know the amount of the assessment he is required to pay, whether or not such assessment is well founded and reasonable so that he may either pay the amount of the assessment or contest its validity in court x x x. It would surely be prejudicial to the interest of the taxpayer for the Government collecting agency to unduly delay the assessment and the collection because by the time the collecting agency finally gets around to making the assessment or making the collection, the taxpayer may then have lost his papers and books to support his claim and contest that of the Government, and what is more, the tax is in the meantime accumulating interest which the taxpayer eventually has to pay . In Republic of the Philippines v. Ablaza,19 this Court emphatically explained that the statute of limitations of actions for the collection of taxes is justified by the need to protect law-abiding citizens from possible harassment: The law prescribing a limitation of actions for the collection of the income tax is beneficial both to the Government and to its citizens; to the Government because tax officers would be obliged to act promptly in the making of assessment, and to citizens because after the lapse of the period of prescription citizens would have a feeling of security against unscrupulous tax agents who will always find an excuse to inspect the books of taxpayers, not to determine the latters real liability, but to take advantage of every opportunity to molest, peaceful, law-abiding citizens. Without such legal defense taxpayers would furthermore be under obligation to always keep their books and keep them open for inspection subject to harassment by unscrupulous tax agents. The law on prescription being a remedial measure should be interpreted in a way conducive to bringing about the beneficient purpose of affording protection to the taxpayer within the contemplation of the Commission which recommended the approval of the law. And again in the recent case Bank of the Philippine Islands v. Commissioner of Internal Revenue,20 this Court, in confirming these earlier rulings, pronounced that: Though the statute of limitations on assessment and collection of national internal revenue taxes benefits both the Government and the taxpayer, it principally intends to afford protection to the taxpayer against unreasonable investigation. The indefinite extension of the period for assessment is unreasonable because it deprives the said taxpayer of the assurance that he will no longer be subjected to further investigation for taxes after the expiration of a reasonable period of time. Thus, in Commissioner of Internal Revenue v. B.F. Goodrich,21 this Court affirmed that the law on prescription should be liberally construed in order to protect taxpayers and that, as a corollary, the exceptions to the law on prescription should be strictly construed. The Tax Code of 1977, as amended, provides instances when the running of the statute of limitations on the assessment and collection of national internal revenue taxes could be suspended, even in the absence of a waiver, under Section 271 thereof which reads:

Section 224. Suspension of running of statute. The running of the statute of limitation provided in Sections 268 and 269 on the making of assessments and the beginning of distraint or levy or a proceeding in court for collection in respect of any deficiency, shall be suspended for the period during which the Commissioner is prohibited from making the assessment or beginning distraint or levy or a proceeding in court and for sixty days thereafter; when the taxpayer requests for a reinvestigation which is granted by the Commissioner; when the taxpayer cannot be located in the address given by him in the return filed upon which a tax is being assessed or collected x x x. (Emphasis supplied.) Among the exceptions provided by the aforecited section, and invoked by the CIR as a ground for this petition, is the instance when the taxpayer requests for a reinvestigation which is granted by the Commissioner. However, this exception does not apply to this case since the respondent never requested for a reinvestigation. More importantly, the CIR could not have conducted a reinvestigation where, as admitted by the CIR in its Petition, the respondent refused to submit any new evidence. Revenue Regulations No. 12-85, the Procedure Governing Administrative Protests of Assessment of the Bureau of Internal Revenue, issued on 27 November 1985, defines the two types of protest, the request for reconsideration and the request for reinvestigation, and distinguishes one from the other in this manner: Section 6. Protest. - The taxpayer may protest administratively an assessment by filing a written request for reconsideration or reinvestigation specifying the following particulars: xxxx For the purpose of protest herein (a) Request for reconsideration-- refers to a plea for a re-evaluation of an assessment on the basis of existing records without need of additional evidence. It may involve both a question of fact or of law or both. (b) Request for reinvestigationrefers to a plea for re-evaluation of an assessment on the basis of newly-discovered evidence or additional evidence that a taxpayer intends to present in the investigation. It may also involve a question of fact or law or both. The main difference between these two types of protests lies in the records or evidence to be examined by internal revenue officers, whether these are existing records or newly discovered or additional evidence. A re-evaluation of existing records which results from a request for reconsideration does not toll the running of the prescription period for the collection of an assessed tax. Section 271 distinctly limits the suspension of the running of the statute of limitations to instances when reinvestigation is requested by a taxpayer and is granted by the CIR. The Court provided a clear-cut rationale in the case of Bank of the Philippine Islands v. Commissioner of Internal Revenue22 explaining why a request for reinvestigation, and not a request for reconsideration, interrupts the running of the statute of limitations on the collection of the assessed tax: Undoubtedly, a reinvestigation, which entails the reception and evaluation of additional evidence, will take more time than a reconsideration of a tax assessment, which will be limited to the evidence already at hand; this justifies why the former can suspend the running of the statute of limitations on collection of the assessed tax, while the latter cannot. In the present case, the separate letters of protest dated 6 May 1994 and 23 May 1994 are requests for reconsideration. The CIRs allegation that there was a request for reinvestigation is inconceivable since respondent consistently and categorically refused to submit new evidence and cooperate in any reinvestigation proceedings. This much was admitted in the Decision dated 8 October 2002 issued by then CIR Guillermo Payarno, Jr. In the said conference-hearing, Revenue Officer Alameda basically testified that Philcom, despite repeated demands, failed to submit documentary evidences in support of its claimed deductible expenses. Hence, except for the item of interest expense which was disallowed for being not ordinary and necessary, the rest of the claimed expenses were disallowed for non-withholding. In the same token, Revenue Officer Escober testified that upon his assignment to conduct the re-investigation, he immediately requested the taxpayer to present various accounting records for the year 1990, in addition to other documents in relation to the disallowed items (p.171). This was followed by other requests for submission of documents (pp.199 &217) but these were not heeded by the taxpayer. Essentially, he stated that Philcom did not cooperate in his reinvestigation of the case. In response to the testimonies of the Revenue Officers, Philcom thru Atty. Consunji, emphasized that it was denied due process because of the issuance of the Pre-Assessment Notice and the Assessment Notice on successive dates. x x x Counsel for the taxpayer even questioned the propriety of the conference-hearing inasmuch as the only question to resolved (sic) is the legality of the issuance of the assessment. On the disallowed items, Philcom thru counsel manifested that it has no intention to present documents and/or evidences allegedly because of the pending legal question on the validity of the assessment.23 Prior to the issuance of Revenue Regulations No. 12-85, which distinguishes a request for reconsideration and a request for reinvestigation, there have been cases wherein these two terms were used interchangeably. But upon closer examination, these cases all involved a reinvestigation that was requested by the taxpayer and granted by the BIR. In Collector of Internal Revenue v. Suyoc Consolidated Mining Company,24 the Court weighed the considerable time spent by the BIR to actually conduct the reinvestigations requested by the taxpayer in deciding that the prescription period was suspended during this time. Because of such requests, several reinvestigations were made and a hearing was even held by the Conference Staff organized in the collection office to consider claims of such nature which, as the record shows, lasted for several months. After inducing petitioner to delay collection as he in fact did, it is most unfair for respondent to now take advantage of such desistance to elude his deficiency income tax liability to the prejudice of the Government invoking the technical ground of prescription. Although the Court used the term "requests for reconsideration" in reference to the letters sent by the taxpayer in the case of Querol v. Collector of Internal Revenue,25 it took into account the reinvestigation conducted soon after these letters were received and the revised assessment that resulted from the reinvestigations. It is true that the Collector revised the original assessment on February 9, 1955; and appellant avers that this revision was invalid in that it was not made within the five-year prescriptive period provided by law (Collector vs. Pineda, 112 Phil. 321). But that fact is that the revised assessment was merely a result of petitioner Querols requests for reconsideration of the original assessment, contained in his letters of December 14, 1951 and May 25, 1953. The records of the Bureau of Internal Revenue show that after receiving the letters, the Bureau conducted a reinvestigation of petitioners tax liabilities, and, in fact, sent a tax examiner to San Fernando, La Union, for that purpose; that because of the examiners report, the Bureau revised the original assessment, x x x. In

other words, the reconsideration was granted in part, and the original assessment was altered. Consequently, the period between the petition for reconsideration and the revised assessment should be subtracted from the total prescriptive period (Republic vs. Ablaza, 108 Phil 1105). The Court, in Republic v. Lopez,26 even gave a detailed accounting of the time the BIR spent for each reinvestigation in order to deduct it from the five-year period set at that time in the statute of limitations: It is now a settled ruled in our jurisdiction that the five-year prescriptive period fixed by Section 332(c) of the Internal Revenue Code within which the Government may sue to collect an assessed tax is to be computed from the last revised assessment resulting from a reinvestigation asked for by the taxpayer and (2) that where a taxpayer demands a reinvestigation, the time employed in reinvestigating should be deducted from the total period of limitation. xxxx The first reinvestigation was granted, and a reduced assessment issued on 29 May 1954, from which date the Government had five years for bringing an action to collect. The second reinvestigation was asked on 16 January 1956, and lasted until it was decided on 22 April 1960, or a period of 4 years, 3 months, and 6 days, during which the limitation period was interrupted. The Court reiterated the ruling in Republic v. Lopez in the case of Commissioner of Internal Revenue v. Sison,27 "that where a taxpayer demands a reinvestigation, the time employed in reinvestigating should be deducted from the total period of limitation." Finally, in Republic v. Arcache,28 the Court enumerated the reasons why the taxpayer is barred from invoking the defense of prescription, one of which was that, "In the first place, it appears obvious that the delay in the collection of his 1946 tax liability was due to his own repeated requests for reinvestigation and similarly repeated requests for extension of time to pay." In this case, the BIR admitted that there was no new or additional evidence presented. Considering that the BIR issued its Preliminary Assessment Notice on 13 April 1994 and its Formal Assessment Notice on 14 April 1994, just one day before the three-year prescription period for issuing the assessment expired on 15 April 1994, it had ample time to make a factually and legally well-founded assessment. Added to the fact that the Final Decision that the CIR issued on 8 October 2002 merely affirmed its earlier findings, whatever examination that the BIR may have conducted cannot possibly outlast the entire three-year prescriptive period provided by law to collect the assessed tax, not to mention the eight years it actually took the BIR to decide the respondents protest. The factual and legal issues involved in the assessm ent are relatively simple, that is, whether certain income tax deductions should be disallowed, mostly for failure to pay withholding taxes. Thus, there is no reason to suspend the running of the statute of limitations in this case. The distinction between a request for reconsideration and a request for reinvestigation is significant. It bears repetition that a request for reconsideration, unlike a request for reinvestigation, cannot suspend the statute of limitations on the collection of an assessed tax. If both types of protest can effectively interrupt the running of the statute of limitations, an erroneous assessment may never prescribe. If the taxpayer fails to file a protest, then the erroneous assessment would become final and unappealable.29 On the other hand, if the taxpayer does file the protest on a patently erroneous assessment, the statute of limitations would automatically be suspended and the tax thereon may be collected long after it was assessed. Meanwhile the interest on the deficiencies and the surcharges continue to accumulate. And for an unrestricted number of years, the taxpayers remain uncertain and are burdened with the costs of preserving their books and records. This is the predicament that the law on the statute of limitations seeks to prevent. The Court, in sustaining for the first time the suspension of the running of the statute of limitations in cases where the taxpayer requested for a reinvestigation, gave this justification: A taxpayer may be prevented from setting up the defense of prescription even if he has not previously waived it in writing as when by his repeated requests or positive acts the Government has been, for good reasons, persuaded to postpone collection to make him feel that the demand was not unreasonable or that no harassment or injustice is meant by the Government. xxxx This case has no precedent in this jurisdiction for it is the first time that such has risen, but there are several precedents that may be invoked in American jurisprudence. As Mr. Justice Cardozo has said: "The applicable principle is fundamental and unquestioned. He who prevents a thing from being done may not avail himself of the nonperformance which he himself occasioned, for the law says to him in effect "this is your own act, and therefore you are not damnified." (R.H. Stearns Co. v. U.S., 78 L. ed., 647). (Emphasis supplied.)30 This rationale is not applicable to the present case where the respondent did nothing to prevent the BIR from collecting the tax. It did not present to the BIR any new evidence for its re-evaluation. At the earliest opportunity, respondent insisted that the assessment was invalid and made clear to the BIR its refusal to produce documents that the BIR requested. On the other hand, the BIR also communicated to the respondent its unwavering stance that its assessment is correct. Given that both parties were at a deadlock, the next logical step would have been for the BIR to issue a Decision denying the respondents protest and to initiate proceedings for the collection of the assessed tax and, thus, allow the respondent, should it so choose, to contest the assessment before the CTA. Postponing the collection for eight long years could not possibly make the taxpayer feel that the demand was not unreasonable or that no harassment or injustice is meant by the Government. There was no legal, or even a moral, obligation preventing the CIR from collecting the assessed tax. In a similar case, Cordero v. Conda,31 the Court did not suspend the running of the prescription period where the acts of the taxpayer did not prevent the government from collecting the tax. The government also urges that partial payment is "acknowledgement of the tax obligation", hence a "waiver on the defense of prescription." But partial payment would not prevent the government from suing the taxpayer. Because, by such act of payment, the government is not thereby "persuaded to postpone collection to make him feel that the demand was not unreasonable or that no harassment or injustice is meant." Which, as stated in Collector v. Suyoc Consolidated Mining Co., et al., L-11527, November 25, 1958, is the underlying reason behind the rule that prescriptive period is arrested by the taxpayers request for reexamination or reinvestigation even if "he has not previously waived it [prescription] in writing." The Court reminds us, in the case of Commissioner of Internal Revenue v. Algue, Inc., 32 of the need to balance the conflicting interests of the government and the taxpayers.

Taxes are the lifeblood of the government and so should be collected without unnecessary hindrance. On the other hand, such collection should be made in accordance with law as any arbitrariness will negate the very reason for government itself. It is therefore necessary to reconcile the apparently conflicting interest of the authorities and the taxpayers so that the real purpose of taxation, which is the promotion of common good, may be achieved. Thus, the three-year statute of limitations on the collection of an assessed tax provided under Section 269(c) of the Tax Code of 1977, a law enacted to protect the interests of the taxpayer, must be given effect. In providing for exceptions to such rule in Section 271, the law strictly limits the suspension of the running of the prescription period to, among other instances, protests wherein the taxpayer requests for a reinvestigation. In this case, where the taxpayer merely filed two protest letters requesting for a reconsideration, and where the BIR could not have conducted a reinvestigation because no new or additional evidence was submitted, the running of statute of limitations cannot be interrupted. The tax which is the subject of the Decision issued by the CIR on 8 October 2002 affirming the Formal Assessment issued on 14 April 1994 can no longer be the subject of any proceeding for its collection. Consequently, the right of the government to collect the alleged deficiency tax is barred by prescription. IN VIEW OF THE FOREGOING, the instant Petition is DENIED. The assailed en banc Decision of the CTA in CTA EB No. 37 dated 22 February 2005, cancelling Assessment Notice No. 000688-80-7333 issued against Philippine Global Communication, Inc. for its 1990 income tax deficiency for the reason that it is barred by prescription, is hereby AFFIRMED. No costs.

También podría gustarte